Está en la página 1de 209

 

1. UNA PACIENTE DE 13 AÑOS ACUDE A LA URGENCIA DE PEDIATRÍA POR DOLOR


HIPOGÁSTRICO SEVERO, QUE YA LE HABÍA SUCEDIDO OTRAS VECES EN EL ÚLTIMO
AÑO. SE EVIDENCIA UNA MASA DE 15 CM EN LA PELVIS MENOR. EL ULTRASONIDO LA
DESCRIBE COMO QUÍSTICA, CON CONTENIDO DE ECOGENICIDAD INTERMEDIA, Y NO
SE OBSERVA ÚTERO NI OVARIOS. LOS GENITALES EXTERNOS SON NORMALES, ASÍ
COMO EL VELLO GENITAL. ENTRE LOS LABIOS MENORES SE VE UNA ZONA
OSCURECIDA A TENSIÓN. ¿QUÉ PUEDE SER?:

1. 1. TUMOR DE ESTIRPE ECTODÉRMICA.


2. 2. SARCOMA BOTRIOIDE QUÍSTICO.
3. 3. QUISTE DE GARDNER.
4. 4. HIMEN IMPERFORADO.
Gráfico de respuestas
Comentario
Lo más importante de las amenorreas es conocer su diagnóstico diferencial, además debe conocer
unos datos básicos de las causas más frecuentes o más características. El diagnóstico del himen
imperforado se basa en la exploración genital que se debe realizar a toda niña prepúber en la
primera visita al pediatra. El acúmulo menstrual retenido puede producir dolor abdominal. Cura con
la incisión y evacuación del contenido vaginal.(R4)

2. ¿Cuándo se establece el diagnóstico de hipertensión arterial?:

1. 1. Mediante una determinación aislada de presión sistólica de 140 mmHg.


2. 2. Mediante dos determinaciones diarias de presión sistólica de 140 mmHg.
3. 3. Mediante tres determinaciones diarias de presión sistólica de 140 mmHg.
Mediante tres determinaciones separadas, como mínimo, por 1 semana, con promedio de
4. 4. presión arterial diastólica igual o superior a 90 mmHg y/o sistólica igual o superior a 140
mmHg.
Gráfico de respuestas
Comentario
Es importante que no falles esta pregunta sobre HTA porque es sencilla y sobre todo porque ha
sido preguntada en más de una ocasión en el MIR. Recuerda que para daignosticar a un paciente
de HTA, éste debe mostrar repetidamente cifras de presión arterial superiores a las
correspondientes a su edad y sexo, tomadas en varias ocasiones durante dos o tres semanas.
Además, en el diagnóstico de HTA debe descartarse la posibilidad de que se trate de una
?pseudohipertensión?, que consiste en una PA elevada como consecuencia de una arteria braquial
rígida.(R4)

3. Señale la relación FALSA sobre las infecciones connatales:

1. 1. La infección materna por parvovirus B19-hidrops fetal no inmune.


2. 2. Infección por enterovirus-sepsis con afectación sistémica.
3. 3. Infección por hepatitis B-ictericia y hepatomegalia.
Varicela en primer trimestre-afectación del SNC, hipoplasia de miembros y cicatrices
4. 4.
cutáneas.
Gráfico de respuestas
Comentario

 
 
 
 
La relación incorrecta es la número 3, ya que los hijos de madres con infección por VHB con Ag de
superficie y e positivo tendrán un alto riesgo de desarrollar hepatitis crónica, cirrosis y carcinoma
hepatocelular pero en un futuro, no al nacimiento. El resto de relaciones son correctas.(R3)

4. En las infecciones urinarias en niños:

1. 1. Es más frecuente en varones menores de 3 meses.


2. 2. Es más frecuente en mujeres de 3 meses.
3. 3. No es necesario urocultivo de seguimiento.
4. 4. Siempre produce fiebre.
Gráfico de respuestas
Comentario

Es más frecuente en niñas (3/1), salvo en el primer año de vida y, especialmente en el periodo
neonatal, donde es hasta cinco veces superior que en niños. Respuesta 1 correcta.(R1)

5. Acude a urgencias una paciente de 28 años con amenorrea de 9 semanas por


metrorragia menor que una regla, sin dolor. Le solicita un test de embarazo que es
positivo. A la exploración ginecológica se objetivan restos hemáticos escasos en vagina,
no sangrado activo, cérvix cerrado. En la ecografía trasvaginal se objetiva una vesícula
gestacional de 18 mm con un embrión de 3 mm en el que no se objetiva latido cardiaco.
Señale la respuesta INCORRECTA:

Se trata de una amenaza de aborto, por lo tanto trataremos con reposo relativo y control
1. 1.
ecográfico en 7-10 días.
2. 2. Este cuadro suele aparecer en el 20% de las pacientes.
3. 3. Practicaremos legrado evacuador al tratarse de un aborto diferido, ya que no hay latido.
4. 4. La amenorrea no coincide con la datación ecográfica.
Gráfico de respuestas
Comentario

En este caso clínico se plantea una gestación de 9 semanas en la que el embrión corresponde
como a 6 semanas y empieza a sangrar. En la ecografía se ve embrión de 3 mm sin latido. Para
que un embrión deba tener latido obligatoriamente debe ser mayor de 5 mm (en ese caso, si no
tiene latido y es >5 mm es un aborto diferido). Por tanto, al ser <3 mm, entra dentro de lo normal
que no tenga movimiento cardiaco. Así, lo etiquetaremos de amenaza de aborto, debiendo
recomendar reposo, se pueden pautar progestágenos y repetir el ultrasonido en unos días para ver
que el embrión es > 5 mm y presenta latido. Si al repetir el ultrasonido no fuera así, sería un aborto
diferido. Las amenazas de aborto aparecen en el 20- 25% de las pacientes.(R3)

6. El tratamiento más recomendable de la diver-ticulosis asintomática es:

1. 1. Laxantes salinos.
2. 2. Anticolinérgicos.
3. 3. Enemas jabonosos semanales.
4. 4. Dieta rica en residuos.
Gráfico de respuestas
Comentario

Cuando se habla de diverticulosis, el término se refiere a la presencia de divertículos, sin


complicación acompañante. La diverticulosis aumenta en frecuencia con la edad, y suele ser

 
 
 
 
asintomática. Por tanto, es evidente que, de inicio, no se indica ningún tratamiento quirúrgico. En
situación estable, sin clínica, no se precisa el uso de fármacos de ningún tipo. En cambio, sí parece
aconsejable una dieta rica en fibra y residuos, que “arrastre” el contenido del colon, lo que dificulta
el estancamiento de restos fecales intradiverticulares, lo que incrementa el riesgo de
diverticulitis.(R4)

7. Un paciente de 55 años de edad presenta una osteomielitis crónica a nivel femoral hace
más de 20 años que le surgió a raíz de una fractura abierta producida por un accidente
de tráfico que sufrió cuando estaba realizando el Servicio Militar. Presenta una fístula con
supuración crónica de material purulento. El paciente se niega a someterse a una
intervención quirúrgica, por lo que es tratado con curas locales y antibioterapia sistémica
en los períodos de mayor actividad. Ahora acude a consulta con disnea de medianos
esfuerzos y clínica de congestión venosa sistémica (hepatomegalia dolorosa, ascitis
leve...). En la exploración cardiovascular destacan la disminución en la intensidad de los
ruidos cardíacos, R3 y R4 presentes y signo de Kussmaul. El impulso apical es palpable.
El ECO demuestra un engrosamiento leve de las paredes cardíacas y una fracción de
eyección normal. ¿Qué patología cardíaca sospecharía?:

Endocarditis infecciosa por S. aureus (el germen más frecuentemente aislado en la


1. 1.
osteomielitis crónica).
2. 2. Fibrosis endomiocárdica.
3. 3. Miocardiopatía restrictiva por amiloidosis secundaria.
4. 4. Taponamiento cardíaco crónico.
Gráfico de respuestas
Comentario
Pregunta de elevada dificultad y de poca relevancia para el MIR. Se trata de un caso clínico de
disnea de esfuerzo y clínica de congestión venosa sistémica (hepatomegalia, ascitis?), con signo
de Kussmaul (que consiste en el aumento de la PVY con la inspiración). Ante estos datos debemos
sospechar principalmente dos enfermedades: la miocardiopatía restrictiva y la pericarditis
constrictiva. Sin embargo, hay otros datos que nos ayudan a concretar más el diagnóstico. El
impulso apical es palpable y el ecocardiograma muestra un engrosamiento leve de las paredes
cardíacas y una FE normal. Estos datos nos orientan hacia una Miocardiopatía restrictiva. Además
el antecedente de la osteomielitis crónica podría producir una amiloidosis secundaria, que
justificaría este tipo de afectación cardíaca.(R3)

8. Paciente de 72 años que presenta pérdida de peso y dolor epigástrico. Se le realiza una
endoscopia gástrica que demuestra la existencia de una masa en el cuerpo gástrico de 3
cm de diámetro. Se realiza una biopsia de la masa y el resultado anatomopatológico es
de un linfoma gástrico de tipo linfocítico. Tras un estudio de extensión se determina que
se trata de un linfoma gástrico aislado que no infiltra estructuras vecinas y que no tiene
compromiso ganglionar. Ante esta situación, el mejor tratamiento será:

1. 1. Sólo radioterapia.
2. 2. Sólo quimioterapia.
3. 3. Gastrectomía subtotal más radioterapia.
4. 4. Debe considerarse en primer lugar el tratamiento erradicador del Helicobacter pylori.
Gráfico de respuestas

 
 
 
 
Comentario

En los linfomas gástricos de bajo grado, MALT sin extensión tumoral, puede haber respuesta al
tratamiento erradicador del Helicobacter pylori (si previamente se ha comprobado infección). Este
tratamiento puede ser curativo. Si no hubiera respuesta, o inicialmente el tumor fuera de alto grado,
incluso con extensión tumoral a ganglios y/o estructuras vecinas el tratamiento sería más agresivo,
con cirugía y quimioterapia, o quimioterapia únicamente.(R4)

9. Le avisan para valorar una paciente de 25 años con antecedentes psiquiátricos que
acude a urgencias por mal estado general. La paciente no refiere vómitos ni alteraciones
digestivas ni ingesta de fármacos. En la analítica presenta Na 122 mmol/l, K 3,3 mmol/l,
bicarbonato 33 mmol/l, osmolaridad 265 mosm/Kg. En orina Na 12 mmol/l, Cl 5 mmol/l,
osmolaridad orina 250 mosm/Kg. ¿Cuál será su sospecha diagnóstica?

1. 1. Toma crónica de laxantes.


2. 2. Vómitos.
3. 3. Ingesta diuréticos.
4. 4. SIADH.
Gráfico de respuestas
Comentario
La paciente presenta una hiponatremia. La presencia de Na en orina 12 nos indica adecuada
compensación renal. La causa, por tanto deben ser pérdidas extrarrenales. Entre laxantes y
vómitos nos decantamos por la segunda por la alcalosis hipopotasémica que presenta.(R2)

10. Indique la respuesta INCORRECTA sobre la microtoma de sangre fetal:

La muestra de sangre se obtiene a través del cérvix mediante una pequeña incisión en el
1. 1.
cuero cabelludo del feto.
2. 2. El pH fetal tiene valor pronóstico sobre el estado del recién nacido.
La diferencia de pH materno fetal en caso de feto no hipóxico es de 0,10-0,15 unidades
3. 3.
de pH.
Cuando el pH se encuentra en valor prepatológico (7,15-7,25) se debe tratar a la madre
4. 4.
con bicarbonato y repetir la toma a los 15-20 minutos.
Gráfico de respuestas
Comentario

Pregunta sobre la microtoma de sangre fetal bastante fácil. La microtoma se obtiene realizando
una pequeña incisión en el cuero cabelludo del feto y, acto seguido, nos informan del valor del pH,
siendo normal entre 7,25 y 7,45; prepatológico entre 7,20 y 7,25, y patológico si es menor de 7,20.
Teniendo en cuenta todo esto, la respuesta incorrecta es la 5, ya que incluye como prepatológico el
intervalo de 7,15 a 7,20.

El resto de opciones son correctas: la acidosis materna puede hacernos variar el valor del pH fetal,
y habría que tenerlo en cuenta para los valores prepatológicos. El enunciado de la opción 3
también es cierto, pues el pH fetal es una muestra objetiva del estado del feto; de ahí que se utilice
como criterio para tomar determinadas decisiones (por ejemplo, cesárea cuando es inferior a
7.20).(R4)

 
 
 
 
11. Sólo una de las siguientes sustancias estimula la secreción ácida en estómago.
Señálela:

1. 1. VIP.
2. 2. Acetilcolina.
3. 3. Secretina.
4. 4. Enteroglucagón.
Gráfico de respuestas
Comentario
Gastrina y acetilcolina son las vías hormonales y como neurotransmisor comunmente implicadas
en la fisiología gástrica, estimulando la secreción, tanto de ácido como de pepsina. El VIP es el
principal inhibidor de la secreción ácido.(R2)

12. El primer signo de desarrollo puberal en las niñas es:

1. 1. La aparición del vello axilar.


2. 2. La telarquia.
3. 3. La aceleración del crecimiento.
4. 4. La menarquia.
Gráfico de respuestas
Comentario
La pubertad se produce por la activación del eje hipotálamo- hipofiso- gonadal (gonadarquia). En
las niñas el primer signo es la telarquia (aparición del botón mamario) entre los 8 y los 13 años (si
aparece antes de los 8 o la menarquia antes de los 9 hablamos de pubertad precoz). Meses
después se produce la pubarquia (aparición de vello pubiano) y más tarde el pico de empuje
puberal y la menarquia. Tanner diseñó un método de 5 estadíos para valorar la madurez sexual,
teniendo en cuenta el desarrollo de las mamas en las niñas, el de los genitales en los niños y la
distribución del vello pubiano en ambos. En los niños el primer signo puberal es el aumento del
volumen testicular y suele comenzar a los 9 años. Más tarde aparece el vello pubiano y el pico de
empuje puberal. La espermaquia no ocurrirá hasta los 13 años. La adrenarquia ocurre a los 6 a 8
años y precede a la gonadarquia en unos 2 años. Se manifiesta por un cambio en el olor del sudor
y menos habitualmente por la aparición de vello púbico y axilar. Aunque la adrenarquia tiene
relación temporal con la gonadarquia, estos eventos son independientes y regulados por
mecanismos diferentes.(R2)

13. ¿Cuál de las siguientes afirmaciones es cierta respecto a la enfermedad venooclusiva


pulmonar?:

1. 1. Cursa con hipertensión pulmonar postcapilar.


2. 2. Su tratamiento específico son los glucocorticoides.
3. 3. La radiografía de tórax suele ser normal.
4. 4. Está causada por obstrucción de las arteriolas pulmonares.
Gráfico de respuestas
Comentario

Esta pregunta puede resultar difícil si no tenemos claro que la enfermedad venooclusiva es una
variante de la forma plexógena de hipertensión pulmonar primaria. Esta se caracteriza por
presentar en al examen anatomopatológico: obstrucción de venas y vénulas pulmonares (en
ocasiones, también arteriolas, respuesta 4 falsa), por fibrosis de la íntima y septos fibrosos
intravasculares que sugieren trombos recanalizados (respuesta 1 correcta). El tratamiento de la
HAP no se realiza con corticoides, sino con vasodilatadores y anticoagulantes, y en pacientes

 
 
 
 
seleccionados que no responden al tratamiento médico con mala clase funcional, trasplante
pulmonar o cardiopulmonar (opción 2 falsa). La Rx tórax suele mostrar una protrusión de la arteria
pulmonar principal con aumento en la anchura de la rama descendente de arteria pulmonar
derecha., oligohemia periférica y cardiomegalia (opción 3 falsa).(R1)

14. Si un paciente presenta una oftalmoplejía supranuclear progresiva asociada a rigidez


distónica de los músculos del cuello y tronco, síndrome pseudobulbar y demencia leve,
probablemente está afecto de:

1. 1. Enfermedad de Parkinson.
2. 2. Enfermedad de Huntington.
3. 3. Parálisis supranuclear progresiva.
4. 4. Síndrome de Gilles de la Tourette.
Gráfico de respuestas
Comentario
Esta pregunta hace referencia a un tema muy importante en el MIR, los trastornos
extrapiramidales. No obstante, la enfermedad de Steele- Richardson- Olzewsky (parálisis
supranuclear progresiva) no se pregunta tanto como la enfemedad de Parkinson. Esta enfermedad
se caracteriza por demencia de aparición precoz , un síndrome parkinsoniano con escaso temblor
que se acompaña de parálisis de la infraversión de la mirada, rigidez de predomino axial (van
tiesos) aparte de otros síntomas extrapidamidales que pueden aparecer en otras entidades. No
olvides que la enfermedad de Parkinson produce lo contrario: parálisis de la SUPRAVERSION de
la mirada . Además la PSP no mejora con la administración ed L- dopa, mientras que en la
enfermedad de Parkinson es el tratamiento de elección. El síndrome de Guilles de la Tourette,
aparece en niños o adolescentes, y cursa con múltiples tics verbales y motores. La enfermedad de
Hungtington es la causa más frecuente de corea hereditario. La distonía consiste en movimientos
involuntarios sostenidos que producen desviación o torsión de un area corporal.(R3)

15. Un joven de 18 años acude por presentar rectorragia ocasional. Presenta


pigmentación melanótica en la mucosa bucal y en manos y pies. La inspección anal
evidenció pigmentación perianal, sin hemorroides externas. Se realizó una colonoscopía
que reveló pólipos en sigma y colon descendente. ¿Cuál de las siguientes afirmaciones
es FALSA respecto a la entidad que presenta su paciente?

1. 1. Se localizan con mayor frecuencia a nivel de colon distal.


2. 2. Puede haber pólipos en localizaciones extraintestinales.
3. 3. Existe riesgo de presentar cáncer intestinal.
4. 4. Sería conveniente realizar una exhaustiva exploración genital.
Gráfico de respuestas
Comentario

Lo más importante de esta pregunta es que seamos capaces de dar con el dato clave: melanosis
periorificial, que te haría pensar en un síndrome de Peutz-Jeghers. La pregunta, por lo demás, es
difícil, ya que exige conocer con cierto detalle esta enfermedad tan infrecuente.

El síndrome de PeutzJeghers se trasmite con herencia autosómica recesiva. El signo cutáneo


típico es la pigmentación mucocutánea. Durante la infancia, aparece una pigmentación melanótica
alrededor de la nariz, labios, mucosa bucal, manos y pies. Ocasionalmente, se acompaña de
hiperpigmentación genital y perianal. En la pubertad, pueden desaparecer las pigmentaciones,
excepto las de la mucosa oral.

 
 
 
 
Los pólipos pueden aparecer en estómago, intestino delgado y colon, siendo sobre todo frecuentes
en intestino delgado, y rara vez malignizan, pues son de tipo hamartomatoso. Por ello, no está
justificada la colectomía profiláctica. Se han descrito cánceres en duodeno, yeyuno, íleon y colon.
En un estudio, casi el 50% de los pacientes tuvieron cánceres intestinales o extraintestinales, con
una media de edad de 50 años al diagnóstico. También pueden aparecer pólipos benignos en
localizaciones extraintestinales, incluyendo nariz, bronquios, vejiga, vesícula biliar y conductos
biliares. En el 5- 12% de mujeres, aparecen quistes o tumores ováricos. En varones jóvenes,
pueden aparecer tumores testiculares de células de Sertoli con signos de feminización. Otros
tumores que pueden aparecer son: mama (a menudo bilateral), páncreas, colangiocarcinoma y
cáncer de vesícula biliar.(R1)

16. ¿Qué anomalía genética se da en las células del linfoma de Burkitt?

1. 1. Hibridación in situ.
2. 2. Inversión cromosómica.
3. 3. Translocación entre los cromosomas 8 y 14.
4. 4. Haploidismo.
Gráfico de respuestas
Comentario

El linfoma de Burkitt es una variante de linfoma linfoblástico B. En su histología, es bastante


característica la denominada imagen de cielo estrellado. Desde el punto de vista genético, es típica
la t(8;14), que sobreexpresa el oncogén c- myc.

El linfoma de Burkitt es el linfoma más agresivo que existe. Su tiempo de duplicación tumoral es
inferior a los tres días. Por este motivo, la respuesta a la quimioterapia suele producir gran
destrucción de células, con el consiguiente síndrome de lisis tumoral. Para evitar dicha situación,
se aconseja hidratación importante previa a la quimioterapia y administración de alopurinol, para
evitar la nefropatía por ácido úrico.(R3)

17. Son factores de mal pronóstico en el cáncer de mama todos menos uno, señálelo:

1. 1. El número de ganglios afectados.


2. 2. El tamaño mayor de 2 cm.
El oncogen Her-2 neu negativo, ya que no se puede beneficiar de la terapia con
3. 3.
trastuzumab.
4. 4. Edad menor de 35 años.
Gráfico de respuestas
Comentario

El oncogen Her- 2- neu positivoes de mal pronóstico, no su negatividad, porque dichos tumores
tienen una historia natural más agresiva, un riesgo más alto de recurrencia y una probabilidad
inferior de presencia de receptores hormonales. Paradójicamente, la presencia de este oncogen
permite el tratamiento con trastuzumab.(R3)

18. Cuál de las siguientes complicaciones NO es más frecuente en los prematuros:

1. 1. Enfermedad de membrana hialina.


2. 2. Enterocolitis necrotizante.
3. 3. Apnea recurrente.
4. 4. Hipercalcemia.

 
 
 
 
Gráfico de respuestas
Comentario

La hipercalcemia es anecdótica en los prematuros. Sólo deberíamos tenerla en cuenta si


apareciese por motivos iatrogénicos, por ejemplo, si hemos calculado mal los aportes de calcio en
una nutrición parenteral.

La hipocalcemia sí es más frecuente en el prematuro, al igual que otras carencias, como la


hipoglucemia o la ferropenia, motivadas ambas por la escasez de reservas de los pretérminos.(R4)

19. Paciente de 52 años, con diagnóstico de carcinoma invasor de cérvix, que en estudio
de extensión se observa afectación del 1/3 inferior de vagina. ¿Cuál de los tratamientos
propuestos sería de elección?

1. 1. Radioterapia y quimioterapia.
2. 2. Histerectomía y doble anexectomía.
3. 3. Conización cervical y controles cada 6 meses.
4. 4. Intervención de Wertheim-Meigs.
Gráfico de respuestas
Comentario
La paciente presenta un estadio tumoral III- A por lo que el tratamiento debería ser la Radioterapia
y Quimioterapia concomitante.(R1)

20. Paciente de 34 años gestante de 30 semanas que padece estenosis mitral de origen
reumático. Señale lo FALSO:

1. 1. La actitud debe ser esperar parto vaginal.


2. 2. Hay que evitar periodos de dilatación prolongados.
3. 3. Habría que programar una cesárea en semana 37.
4. 4. Es la cardiopatía más habitual en las gestantes.
Gráfico de respuestas
Comentario
En las pacientes cardiópatas la actitud es esperar un parto vaginal, si bien es necesario evitar
periodos de dilatación y expulsivo prolongados. La cesárea aumenta la mortalidad en pacientes
cardiópatas.(R3)

21. Paciente de 7 años con eritema aclarado en el centro, migratorio, poliartralgias


migratorias en rodillas, tobillo y carpo. Se le detecta a la auscultación un soplo sistólico
de localización mitral irradiado a axila. El cuadro remite con tratamiento a los 10 días.
¿Cuál sería la medida preventiva más adecuada?

1. 1. Penicilina benzatina 6.000.000 UI semanales durante 1 año.


2. 2. Penicilina 250 mg/6 horas antes de intervención.
3. 3. Penicilina benzatina 600.000 UI - 1.200.000/mes mínimo hasta los 18 años.
4. 4. No requiere ninguna medida además del tratamiento realizado.
Gráfico de respuestas
Comentario

El caso nos describe una fiebre reumática. Entre otras medidas terapéuticas, se debe administrar
tratamiento antibiótico de inmediato. En los adultos, se recomienda un ciclo completo (10 días) con

 
 
 
 
500 mg de penicilina V (fenoximetilpenicilina) o bien bencilpenicilina benzatina (una inyección i.m.
aislada de 1,2 millones de UI). Posteriormente, es importante seguir una profilaxis de nuevos
episodios de fiebre reumática, para lo que está indicado tratamiento cada 3-4 semanas con una
inyección intramuscular de 1,2 millones de UI de penicilina G benzatina. La mayor proporción de
las recidivas se producen en los 5 años posteriores al primer episodio. Por ello, la profilaxis está
indicada al menos durante este período en los adultos, o hasta llegar a adultos en los niños y
adolescentes. Algunos autores defienden una profilaxis más prolongada (incluso de por vida), si
hay afectación cardíaca con alteraciones valvulares residuales.(R3)

22. Mujer de 30 años


que se está sometiendo a un tratamiento de inducción de la ovulación con
gonadotropinas recombinantes para conseguir estimulación ovárica dentro de un
programa de FIV y hace 24 horas se realizó la administración de HCG como paso previo
a punción folicular. Acude a Urgencias por malestar general, nauseas y vómitos, aumento
del perímetro abdominal y molestias abdominales de 48 horas de evolución y que en las
últimas 12 horas han empeorado junto con la aparición de disnea leve/ortopnea. Al
realizarle una USG pélvico observa unos ovarios como los que se muestran en la imagen
junto con abundante líquido libre. Respecto a la entidad clínica que usted sospecha,
señale la afirmación VERDADERA:

Este cuadro se debe a un aumento de mediadores estimulados por la HCG (sobre todo a
1. 1.
través del VEGF) que inducen un aumento de la permeabilidad vascular.
Es un tromboembolismo pulmonar y requiere ingreso en UVI, monitorización cruenta de
2. 2. la presión venosa central, intubación orotraqueal con ventilación mecánica asistida y altas
dosis de anticoagulantes.
Se ha de realizar una laparoscopia diagnostico-terapéutica para aspiración del líquido
3. 3. libre y cauterización del punto sangrante ya que lo más probable es que estemos ante un
folículo hemorrágico.
Con toda probabilidad habrá que realizar FIV-ICSI ya que es importante conseguir gran
4. 4.
número de embriones debido a que este cuadro mejorará con la gestación.
Gráfico de respuestas
Comentario

El cuadro clínico corresponde a un Síndrome de Hiperestimulación Ovárica. Es una complicación


asociada a las técnicas de reproducción asistida y a la inducción de la ovulación, que aparece en el
10%. Los primeros síntomas aparecen los últimos días de la fase de estimulación ovárica y se
hacen más intensos y frecuentes después de la administración de HCG. El mecanismo
fisiopatológico implicado es el aumento de mediadores vasculares estimulados por la HCG. En el
tratamiento es importante evitar la gestación ya que esta agravaría el cuadro clínico.(R1)

23. La falta de sangrado por deprivación que ocurre después de la administración de


cantidades de estrógenos y progesterona en una mujer amenorreica puede indicar:

 
 
 
 
1. 1. Un factor de sinequias uterinas.
2. 2. Insuficiencia ovárica.
3. 3. Un factor hipofisario.
4. 4. Todos los anteriores.
Gráfico de respuestas
Comentario

En el algoritmo de amenorreas, en pacientes que no menstruan con la administración de estrógeno


y progesterona se deberá pensar en alteraciones anatómicas uterinas, por lo que la respuesta
correcta es la número 1. Sinequias uterinas o Síndrome de Asherman.(R1)

24. Hombre de 40 años, dedicado a la ganadería, acude al hospital por presentar desde
hace 1 semana una lesión en el cuello de 1 semana de evolución. A la exploración se
observa una escara necrótica con abundante edema perilesional. ¿Cuál es la etiología del
cuadro de este paciente?

1. 1. Bacillus cereus.
2. 2. Bacillus anthracis.
3. 3. Pseudomonas aeruginosa.
4. 4. Candida albicans.
Gráfico de respuestas
Comentario

Lo que nos están describiendo es un carbunco cutáneo. Recuerda sus dos características típicas:

- Lesión costrosa, una escara necrótica.

- Edema DURO alrededor de la misma.

El agente causal es B. anthracis y el tratamiento es la penicilina.(R2)

25. La malnutrición es un problema de salud importante porque en el año 1999 afectaba


a 165 millones de niños menores de 5 años en el mundo. Se producen alteraciones en el
crecimiento, en la función inmunológica y en el desarrollo cerebral. Señale cuál de las
siguientes opciones acerca de este problema NO es cierta:

1. 1. La malnutrición primaria es aquella que se produce por un déficit en la ingesta.


2. 2. La malnutrición secundaria se relaciona con enfermedades subyacentes.
El marasmo, o déficit energético, se produce fundamentalmente tras el segundo año de
3. 3. vida, ya que se evita en parte por la lactancia materna, y en él predomina la pérdida de
peso.
El kwashiorkor es una malnutrición proteica que cursa con edema, atrofia muscular y
4. 4.
pérdida de peso no tan llamativa como en el marasmo.
Gráfico de respuestas
Comentario

Concepto que se pregunta prácticamente año con año en el ENARM.

Debes saber diferenciar a la perfección Marasmo y Kwashiorkor.

 
 
 
 
El marasmo aparece a cualquier edad por una ingesta calórica insuficiente, siendo frecuente en los
lactantes en zonas donde los alimentos son escasos. Kwashiorkor, significa "niño depuesto", es
decir, niño que deja de mamar, y se produce por déficit de proteínas de alto valor biológico, mucho
más alto en las de la leche materna que en otros alimentos. Aunque el déficit de micronutrientes
puede ser más grave en el Kwashiorkor, generalmente acompaña a cualquier tipo de
desnutrición.(R3)

26. En una campimetría, para que el defecto encontrado sea más congruente, la lesión
anatómica deberá encontrarse en:

1. 1. El quiasma óptico.
2. 2. El cuerpo geniculado externo.
3. 3. La cintilla óptica.
4. 4. El córtex occipital.
Gráfico de respuestas
Comentario

Se dice que un defecto es más congruente cuanta mayor similitud existe entre ambos campos
visuales. La máxima incongruencia sería una lesión unilateral de un nervio óptico, ya que
solamente habría afectación en un campo visual. Sin embargo, las lesiones occipitales son muy
parecidas entre ambos campos (respuesta 5 correcta). En general, las lesiones más congruentes
son las que se producen en las regiones más posteriores de la vía óptica.(R4)

27. En un paciente con metástasis hepáticas de primario no conocido, ¿cuál de los


siguientes tumores considera más improbable como origen?

1. 1. Tumores pulmonares.
2. 2. Tumores gastrointestinales.
3. 3. Tumores prostáticos.
4. 4. Melanomas.

 
 
 
 
Gráfico de respuestas
Comentario

Las metástasis son los tumores malignos más frecuentes del hígado, siendo hasta 20 veces más
frecuentes que los tumores malignos primitivos. Cualquier tumor puede producir metástasis
hepáticas, pero son más frecuentes las metástasis de adenocarcinomas y los carcinomas
indiferenciados que los tumores de células escamosas. Los más frecuentes son los tumores de
aparato digestivo, fundamentalmente del cáncer colorrectal, seguidos por pulmón, mama,
melanoma y afectación por linfomas. En cambio, son raras las metástasis de tiroides y próstata
(respuesta 3 correcta).

Cuando producen clínica, lo más frecuente es el dolor en el hipocondrio derecho. En el laboratorio,


lo más frecuente es el patrón denominado de colestasis disociada, sobre todo con el aumento de
fosfatasa alcalina. El diagnóstico se realiza mediante pruebas de imagen y biopsia percutánea. El
tratamiento habitualmente es paliativo, aunque existen circunstancias excepcionales.(R3)

28. Señale cuál de los siguientes fármacos antituberculosos NO es bactericida:

1. 1. Rifampicina.
2. 2. Estreptomicina.
3. 3. Isoniacina.
4. 4. Etambutol.
Gráfico de respuestas
Comentario
Esta pregunta podrías intentar razonarla, por lo menos para intentar luchar por ella. Espero que
sepas que los fármacos más efectivos a la hora de tratar la TBC son los que se emplean en la
terapia de ciclo corto, la de seis meses: isoniacida + rifampicina + pirazinamida. Por eso en esta
terapia el tiempo necesario para lograr la curación es el más corto de las posibles terapias. Luego
de aquí sería lógico pensar que estos tres fármacos son bactericidas y por eso son tan efectivos
(en cuanto no se puede usar alguno de ellos, por la razón que sea, la terapia se tiene que
prolongar más, por lo menos nueve meses). Por eso ya podríamos descartar tres opciones. Si
recuerdas, la estreptomicina es un aminoglucósido y como todos ellos es un fármaco bactericida
que actúa a nivel de la subunidad 30S del ribosoma bacteriano. Por lo tanto nos quedamos con la
respuesta 5.(R4)

29. Todos lass siguientes manifestaciones son esperables en el déficit de ácido fólico
EXCEPTO:

1. 1. Glositis.
2. 2. Demencia.
3. 3. Diarrea.
4. 4. Alteraciones morfológicas en el epitelio del cérvix uterino.
Gráfico de respuestas
Comentario
No podéis fallar esta pregunta, sin dudar debéis de marcar la opción 3 como muy falsa. No olvidéis
que la anemia megaloblastica por déficit de ácido fólico (a diferencia de la por déficit de Vit. B12)
no tiene clínica neurológica (por no estar involucrada en la síntesis de la mielina) y por lo tanto
incluye también a la demencia. Las demás opciones son una series de manifestaciones clínicas
inespecíficas comunes a gran parte de los síndromes anémicos, recordaros que tanto la
cobalamina como el ácido fólico intervienen en el trofismo adecuado de la piel y las mucosas
(incluido el epitelio del cerviz uterino, opción 5 correcta).(R2)

 
 
 
 
30. Niño de 2 meses de edad que acude a Urgencias por presentar rinorrea hialina y fiebre
de 38° C en los tres últimos días. Desde hace 24 horas, tiene tos en accesos y dificultad
respiratoria progresiva. En las últimas 12 horas rechaza todas las tomas. En los
antecedentes personales destaca un embarazo y parto normal, es alimentado con
lactancia materna. Ha recibido la primera dosis de DTP acelular, Hemophilus influenzae
tipo B, Meningococo C, Polio oral y 2 dosis de Hepatitis B. En la exploración física destaca
una frecuencia cardíaca de 135 l.p.m., una frecuencia respiratoria de 55 r.p.m. y una
saturación de O2 con aire ambiental de 90%. Polipnea con tiraje intercostal y subcostal
leve-moderado y en la auscultación destaca la presencia de subcrepitantes y sibilancias
generalizadas. Ante este paciente, ¿qué diagnóstico de presunción realizaría?

1. 1. Bronquiolitis.
2. 2. Neumonía.
3. 3. Tos ferina.
4. 4. Infección respiratoria de vías altas.
Gráfico de respuestas
Comentario

Aunque la definición de bronquiolitis habla de lactantes menores de dos años con su primer
episodio de dificultad respiratoria, aparece más en los menores de seis meses, como sucede en
esta pregunta.

Es importante tener en cuenta los síntomas previos para su diagnóstico, porque en los lactantes de
mayor edad, sobre todo con antecedentes familiares u otros factores de riesgo para
hiperreactividad bronquial (dermatitis atópica, alergias alimentarias, tabaquismo pasivo), la
bronquiolitis puede confundirse con asma.

Las crisis de broncospasmo, a diferencia de las bronquiolitis, suelen tener una aparición brusca; en
la auscultación se escuchan fundamentalmente sibilancias (sin ruidos de secreciones), y lo más
importante, las crisis se repiten con frecuencia. Además, en este caso nos mencionan un episodio
catarral previo, que también respalda la respuesta 1.(R1)

31. En un niño sano de 3 meses, el valor de la hemoglobina es:

1. 1. 13-17 g/dL.
2. 2. 9.5-14 g/dL.
3. 3. < 9 g/dL.
4. 4. Prácticamente igual que al nacer.
Gráfico de respuestas
Comentario

Pregunta memorística, pero que puede responder, recuerde que el RN y en los primeros meses los
niveles de Hb están un poco disminuidos, por lo que la respuesta correcta es la número 2.(R2)

32. Señale lo FALSO con respecto a las complicaciones del sarampión y de la rubéola:

La Neumonía de Hecht o de células gigantes es una neumonía intersticial causada por el


1. 1.
virus del sarampión.
2. 2. La sobreinfección bacteriana es una complicación frecuente de la rubéola.
La artritis que complica la rubéola suele afectar a mujeres adultas y las articulaciones
3. 3.
metacarpofalángicas.

 
 
 
 
4. 4. La miocarditis es una complicación grave, pero poco frecuente, del sarampión.
Gráfico de respuestas
Comentario

La neumonía de células gigantes o de Hecht está producida por el sarampión y se presenta como
una grave afectación intersticial.

La artritis que se produce en la rubéola afecta típicamente a mujeres y a articulaciones de pequeño


tamaño y la miocarditis es muy rara en el contexto del sarampión, pero de mucha gravedad .

Así pues, la opción correcta es la 2, ya que la sobreinfección bacteriana es poco frecuente en los
pacientes afectos de rubéola. Sí que es habitual en la varicela, al romperse las vesículas, que
tienden a la sobreinfección por bacterias grampositivas que forman parte de la flora cutánea.(R2)

33. Vemos a un paciente de 21 años de edad, hijo de paciente con carcinoma medular de
tiroides y feocromocitoma. Destaca su elevada estatura y su delgadez. La exploración
pone de manifiesto neuromas mucosos. Señale el diagnóstico más probable:

1. 1. MEN 1.
2. 2. Síndrome pluriglandular autoinmune tipo 2.
3. 3. MEN 2a.
4. 4. MEN 2b.
Gráfico de respuestas
Comentario

Describen un MEN 2b. Éste síndrome comparte la predisposición al carcinoma medular de tiroides
del MEN 2a pero asocia además neuromas mucosos, ganglioneuromas intestinales y, a veces,
hábito marfanoide. Sin embargo, no estará presente la hipertrofia paratiroidea. El carcinoma
medular es el componente más frecuente del MEN 2b y aparece a edades más tempranas y es
más agresivo que en el MEN 2a.

Recordemos que el MEN 2a presenta carcinoma medular, hipertrofia paratiroidea, feocromocitoma


y, en algunas familias, amiloidosis cutánea.(R4)

34. La presencia de alopecia en un niño con raquitismo grave debe hacerle pensar en:

1. 1. Déficit carencial de vitamina D.


2. 2. Déficit de 25-hidroxilasa hepática.
3. 3. Déficit de 1-alfa-hidroxilasa renal.
4. 4. Déficit funcional del receptor de vitamina D.
Gráfico de respuestas
Comentario

Pregunta muy difícil sobre un tema muy concreto de endocrinología pediátrica. No se preocupe en
absoluto si la ha fallado.

El raquitismo resistente al calcitriol, también llamado tipo II, es de herencia autosómica recesiva.
Se debe a una resistencia periférica a las acciones de la vitamina D, en relación con una anomalía
de su receptor. Clínicamente, se manifiesta como un síndrome hipotónico y con signos y síntomas

 
 
 
 
de raquitismo severo. Característicamente, a diferencia de lo que ocurre en otras carencias de
vitamina D, asocia alopecia total.(R4)

35. After a fight at the door of a nightclub, a 30-year-old man is brought to the emergency
department. He is unconscious, hypotensive, with respiratory distress and open fractures
in both legs. In the initial assessment, you should perform the protocol called ABCDE of
trauma patient care. Point out the wrong step:

1. 1. Airway maintenance and cervical spine protection.


2. 2. Breathing and ventilation.
3. 3. Circulation with hemorrhage control.
4. 4. Genitourinary examination.
Gráfico de respuestas
Comentario

Tema con suma importancia en los últimos años. El protocolo ABCDE de la atención al
politraumatizado es el siguiente:

A: Airway with cervical spine control (vía aérea con control de la columna cervical)

B: Breathing, (respiración)

C: Circulation with hemorrhage control, (circulación y control de la hemorragia)

D: Disability, (deterioro neurológico)

E: Exposure and prevention of hypothermia, (exposición del paciente y prevención de la


hipotermia).(R4)

36. Niño de 2 años de edad, en buen estado general y nutricional asintomático. Tiene
todas sus vacunas en el primer año de vida, incluyendo BCG al nacer. Al realizar
screening familiar se le encuentra PPD 2mm y radiografía de tórax normal. BAAR negativo
Ud. indica para este niño:

1. 1. INH 10 mg/K/día por 3 meses y luego repetir PPD.


2. 2. INH 1 mg/k/día por 6 a 12 meses.
3. 3. INH + Rifampicina por 6 a 9 meses.
4. 4. Observación sin tratamiento alguno.
Gráfico de respuestas
Comentario

Al ser un niño, la presencia de un PPD+ sería indicación por sí sola de iniciar quimioprofilaxis con
isoniacida y posteriormente repetir el PPD, respuesta 1 correcta.(R1)

37. Niño de 28 días de vida, nacido a término. Antecedentes de embarazo y parto


normales. Ha sido alimentado con lactancia materna desde el nacimiento. Consulta
porque hace 8 días comenzó con vómitos, inicialmente esporádicos y desde hace 5 días,
después de todas las tomas. Está permanentemente hambriento. Los vómitos son de
contenido alimentario ”a chorro”. El abdomen, en la palpación, es blando y depresible,
sin visceromegalias. En los análisis destacaban, en la gasometría pH 7.49, bicarbonato

 
 
 
 
38. 30 mEq/l, PCO2 53 mmHg, exceso de base +8 mEq/l. Electrolitos: Na 137 mEq/l, K 3.1
mEq/l, Cl 94 mEq/l. Teniendo en cuenta el diagnóstico más probable en el cuadro clínico
descrito, ¿cuál es la prueba complementaria de elección para confirmar ese diagnóstico
clínico?

1. 1. Radiografía simple de abdomen.


2. 2. pHmetría.
3. 3. Ultrasonido abdominal.
4. 4. Determinación de electrolitos en el sudor.
Gráfico de respuestas
Comentario

Pregunta fácil sobre un tema típico. Este tipo de preguntas no se pueden fallar.

El cuadro clínico corresponde a una estenosis hipertrófica del píloro: lactante masculino que
debuta en torno a los 20 días de vida con vómitos alimentarios (no biliosos), “a chorro” o
proyectivos, que le provocan una alcalosis metabólica hipoclorémica (se pierde HCl), con tendencia
a la hipopotasemia, un estado permanentemente hambriento e irritable, deshidratación y
desnutrición.

En la exploración física, se palpa en algunos casos la “oliva pilórica”. La técnica de elección para
confirmar el diagnóstico es el ultrasonido abdominal (respuesta 3 correcta). Cuidado con la
respuesta 1, la Rx simple de abdomen es útil en el diagnóstico diferencial, pudiendo ver una única
burbuja tanto en la hipertrofia como en la atresia pilórica y una doble burbuja en la atresia
duodenal, pero nos piden la prueba complementaria de elección.

El tratamiento preoperatorio consiste en hidratación i.v. para corregir las alteraciones


hidroelectrolíticas, siendo el tratamiento curativo la piloromiotomía de Ramsted.

Hay que tener presente en la patología digestiva pediátrica los diagnósticos diferenciales, en este
caso con la atresia pilórica (vómitos no biliosos desde las primeras tomas), la atresia duodenal
(vómitos biliosos precoces, regla de la D: duodenal, doble burbuja, síndrome de Down), el reflujo
gastroesofágico (vómito atónico de contenido alimentario desde la primera semana de vida, la
pHmetría, respuesta 2, es la prueba más sensible y específica pero no la primera a realizar) y con
la insuficiencia suprarrenal congénita (vómitos desde el nacimiento con alcalosis metabólica
hiperpotasémica).(R3)

Diagnóstico diferencial de la patología digestiva pediátrica

39. Un niño de 26 meses es llevado al servicio de urgencias por sus padres que refieren
fiebre de 6 días de evolución. Ha alcanzado un máximo de 40ºC, cede temporalmente con

 
 
 
 
paracetamol e ibuprofeno alternos y desde el tercer día asocia además tratamiento con
amoxicilina a 40 mg/kg al ser diagnosticado de faringitis aguda. No refieren síntomas
digestivos, ni respiratorios pero desde hace 4 días su apetito ha disminuido y se
encuentra cada vez más irritable. Durante la exploración física el niño está decaído,
temperatura 39.6ºC, TA 67/45 mmHg, FC 107 lpm, FR 32 rpm. Inspección: adecuada
coloración, buena perfusión periférica, bien hidratado, no exantemas, ni petequias y se
aprecia un leve edema de manos y pies. La auscultación cardiopulmonar es normal.
Abdomen blando, no doloroso, no masas ni vísceromegalias. La puño-percusión renal es
negativa. ORL: otoscopia sin hallazgos, faringe hiperémica, sin exudados. No
adenopatías. Signos meníngeos negativos. Se realiza radiografía de tórax (informada
como normal), BH (leucocitos 16,500 mm3, neutrófilos 42%, linfocitos 37%, Hb 12.3 gr/dL,
plaquetas 725,000 mm, bioquímica (glucosa 96 mg/dL, creatinina 0.4 mg/dL, urea 22
mg/dL, sodio 140 mmol/L, potasio 4.8 mmol/L, cloro 110 mmol/L, calcio 8.7 mmol/L,
proteínas 5.5 gr/dL, albúmina 2.7 gr/dL, PCR 43 mg/L, GOT 36 UI/L, GPT 33 UI/L , GGT 19
UI/L) y en la tira reactiva de orina (leucocitos ++, hematíes -, proteínas -, nitritos -). Se
enviaron muestras para hemo y urocultivo. Señale la opción CORRECTA:

El diagnóstico más probable dada la clínica, la exploración física y los hallazgos


1. 1. radiológicos y de laboratorio es una pielonefritis aguda secundaria a reflujo
vesicoureteral.
Aunque es una forma de presentación atípica de su enfermedad, el tratamiento a iniciar
2. 2.
debe ser el mismo que si presentara la forma clásica.
Dada la situación lo más prudente es el ingreso para observación e iniciar el estudio de
3. 3.
fiebre de origen desconocido antes de iniciar cualquier otro tratamiento.
A pesar de la negatividad de los signos meníngeos, debido al tratamiento antibiótico
4. 4.
iniciado, el siguiente paso sería una punción lumbar.
Gráfico de respuestas
Comentario

Considere en el diagnóstico diferencial la enfermedad de Kawasaki siempre que en una pregunta


le hablen de un niño pequeño (<5 años) y con fiebre prolongada (>5 días). La enfermedad de
Kawasaki es una vasculitis sistémica febril y típica de la infancia. Su diagnóstico se hace por
criterios clínicos, reservándose las pruebas complementarias para el diagnóstico de las
complicaciones y de las formas atípicas o incompletas. Los criterios de Kawasaki incompleto
incluyen: Fiebre > 5 días y PCR > 30 o VSG>40 + al menos 3 de 6 criterios de laboratorio que son:
Albúmina < 3 gr/dl, anemia, elevación de GPT, plaquetas > 450,000 (7 días post-inicio de
síntomas), leucocitos >15,000 y en orina>10 leucocitos x campo con cultivo negativo (piuria estéril).
Su diagnóstico es importante porque la afectación coronaria es igual y el tratamiento el mismo.
Además es típico en lactantes que precisamente son los que más riesgo tienen de enfermedad
coronaria. De las otras opciones:

Pielonefritis: Los nitritos negativos hablan en contra de infección y los leucocitos positivos en orina
sugieren una piuria estéril.

El tratamiento de la faringitis estreptocócica se hace con penicilina o amoxicilina a dosis bajas.

No es una fiebre sin foco si se considera correctamente el diagnóstico de Kawasaki incompleto, por
lo que tampoco habría que buscar foco meníngeo.(R2)

40. En relación a las complicaciones de la cirugía gastrointestinal, señale la respuesta


INCORRECTA:

 
 
 
 
1. 1. La sepsis es la causa más común de muerte en pacientes con fístulas gastrointestinales.
En las fístulas gastrointestinales está indicada la cirugía tras 6-8 semanas de tratamiento
2. 2.
conservador (antibióticos y reposo digestivo con NPT).
Las anastomosis colorrectales están más predispuestas a la disrupción que las del
3. 3.
estómago e intestino delgado.
Suele ser habitual la existencia de un íleo paralítico, de duración de alrededor de una
4. 4.
semana, tras la cirugía.
Gráfico de respuestas
Comentario

Debe elegir la respuesta incorrecta. La respuesta 1, 2 y 3 son correctas, pero concentresé en lo qu


dice la 4... después de una cirugía gastrointetinal es habitual la existencia de un íleo paralítico...
eso no es posible, por obvias razones la respuesta a elegir es esta.(R4)

41. ¿Qué Ig NO activa el complemento por la vía clásica?:

1. 1. IgG1.
2. 2. IgG2.
3. 3. IgG4.
4. 4. IgM.
Gráfico de respuestas
Comentario

Pregunta de dificultada media sobre las características diferenciales de los distintos tipos y
subtipos de Ig.

Recuerde bien las características y funciones principales de cada Ig. La IgG es la más frecuente en
las secreciones internas tanto en el adulto, como en el feto y RN y es la única que pasa la
placenta.

La IgA es la principal en las secreciones externas y tiene actividad antivírica.

La IgM es la mayor, de estructura pentamérica, no atraviesa la placenta, es la que se secreta en el


primer contacto con la infección y es muy eficaz fijando complemento, lo que confiere mucha
actividad antibacteriana.

La IgD podemos encontrarla como receptor en la membrana de linfocitos B.

La IgE se encuentra involucrados en las infecciones por parásitos y procesos alérgicos.

Es importante recordar la característica de fijar complemento, pues sólo reside en la IgM y en las
clases de IgG: 1,2,3. Para recordar el orden en el que fijan IgG mejor, recuerde la regla del cartero:
el cartero cuenta primero impares y luego pares: IgG1 > IgG3 > IgG2 y la IgG4 no fija
complemento.(R3)

42. Uno de los siguientes NO se considera factor de mal pronóstico en el cáncer de


endometrio:

1. 1. Edad joven.
2. 2. Grado 3 de diferenciación o grado histológico indiferenciado.
3. 3. Citología peritoneal positiva.

 
 
 
 
4. 4. Gran tamaño tumoral.
Gráfico de respuestas
Comentario

Pregunta que puede resolver descartando las otras.

Como sabe un grado 3 de diferenciación o indiferenciado es de peor pronóstico que un grado 1 o


diferenciado, por lo que la respuesta 2 no es la correcta.

En un cáncer de endometrio, cuando tienes citología peritoneal positiva quiere decir que el tumor
se ha extendido más allá del endometrio, por lo que sería de mal pronóstico, respuesta 3
incorrecta.

Un tumor de gran tamaño será siempre de peor pronóstico que uno pequeño, respuesta 4
incorrecta.

Por tanto la respuesta que debes elegir como correcta es la 1.(R1)

43. Estando de guardia en un servicio de Urgencias de pediatría atiende a un lactante de


5 meses de edad. La madre refiere que lleva varios días con rinorrea y tos, y que en las
últimas 24 horas le parece que respira con dificultad. Está afebril y toma adecuadamente
el biberón. Es un niño correctamente vacunado y sin ningún antecedente de interés.
Acude a guardería. En la exploración se observa un buen estado general, una frecuencia
respiratoria de 38 respiraciones por minuto y un leve tiraje subcostal. Al auscultarle
descubre una buena entrada de aire en ambos campos pulmonares, con subcrepitantes
y sibilancias espiratorias difusas. La saturación de oxígeno recogida por pulsioximetría
es del 97%. Los padres son responsables y comprenden bien las indicaciones, y tienen
pedida cita con su pediatra para el día siguiente a primera hora. Ante el cuadro descrito,
¿qué prueba complementaria estima más oportuna?

1. 1. Radiografía de tórax.
2. 2. Cultivo de secreciones nasofaríngeas.
Ninguna prueba en el episodio agudo, pero lo remitiría al especialista para realización de
3. 3.
una espirometría basal.
4. 4. Ninguna.
Gráfico de respuestas
Comentario

La pregunta describe un cuadro de bronquiolitis aguda no complicada.

El diagnóstico de la bronquiolitis es únicamente clínico, al hallarse ante un lactante menor de 24


meses con primer episodio de dificultad respiratoria con anomalías en la auscultación en el
contexto de un cuadro catarral. No son necesarias de rutina ninguna prueba complementaria.

La radiografía de tórax se practicará cuando existan dudas razonables en el diagnóstico diferencial


(descompensación cardíaca, cuerpo extraño, neumonía…)

La bronquiolitis es un cuadro viral, pero el diagnóstico etiológico (detección de antígenos virales en


secreciones nasofaríngeas) se practica con fines epidemiológicos, ante niños que requieran
ingreso, sin variar la actitud terapéutica.

 
 
 
 
Por último, la relación entre bronquiolitis y asma es muy controvertida. Los niños que padecen una
bronquiolitis tienen mayor riesgo de padecer cuadros respiratorios con sibilancias durante los
primeros años de vida, sin que parezca incrementado el riesgo de padecer asma bronquial a largo
plazo. El estudio espirométrico tras un primer episodio no está indicado.(R4)

44. En el niño desnutrido tipo Kwashiorkor se encuentra:

1. 1. Espacio extracelular expandido.


2. 2. Puede presentar estatosis hepática.
3. 3. Se asocia a destete tardío.
4. 4. Todas las anteriores.
Gráfico de respuestas
Comentario

Pregunta sencilla. La respuesta correcta es la número 4(R4)

45. La nefropatía de los Balcanes esta producida por:

1. 1. Infección por Adenovirus.


2. 2. Infección por Coronavirus.
3. 3. Infección por Herpesvirus.
4. 4. Infección por Enterovirus.
Gráfico de respuestas
Comentario
Esta enfermedad nunca ha caído en el MIR, así que no te preocupes si has fallado. La nefropatía
de los Balcanes está producida por la infección renal por coronavirus, siendo la fuente de
transmisión el agua de la zona. La padecen los individuos que llevan viviendo en la zona más de

 
 
 
 
10 años. Es importante recordar la mayor incidencia de neoplasia del tracto urinario (uroteliomas)
que padecen estos enfermos.(R2)

46. What should be done if a child presents a white pupillary reflex?

1. 1. Watchful waiting
2. 2. Cover the normal eye
3. 3. Cover the affected eye
4. 4. Referral to an ophthalmologist
Gráfico de respuestas
Comentario
Referral to an ophthalmologist. Leukocoria is a white pupillary reflex. This symptom is associated
with congenital cataracts, corneal scarring, retrolental fibroplasia of the preterm children and
retinoblastoma. Retinoblastoma is a tumor that presents in children between two and four years of
age in most cases. This tumor requires rapid treatment administration (surgery, brachytherapy,
laser, cryotherapy…) in order to avoid visual loss and tumor extension. Early diagnosis is only
posible by referring the child to an ophthalmologist .(R4)

47. ¿Cuál de las siguientes enfermedades puede degenerar en cáncer de colon?

1. 1. Adenoma velloso.
2. 2. Poliposis familiar.
3. 3. Enfermedad de Crohn.
4. 4. Todas las anteriores.
Gráfico de respuestas
Comentario

Las lesiones precancerosas se presentan en adenomas vellosos, en la poliposis familiar y en


enfermedad de Crohn, por lo que la respuesta corecta es la 4.(R4)

48. Which of the following is the most appropriate treatment for hypertrophic
cardiomyopathy?

 
 
 
 
1. 1. Digitalis and diuretics.
2. 2. Beta-blockers, verapamil and antiarrhythmic agents.
3. 3. Digitalis, diuretics and vasodilators.
4. 4. Beta-blockers, angiotensin-converting enzyme inhibitors and digitalis.
Gráfico de respuestas
Comentario

Pregunta sobre una entidad importante en cardiologia como es la MCH. Debe recordar que esta
enfermedad produce fundamentalmente una disfunción diastólica. Por eso, los fármacos inotrópico
positivos, como la digoxina, están contraindicados, ya que aumentan la contractilidad del VI y
producen un deterioro hemodinámico con un incremento de la sintomatología. También deben
evitarse los vasodilatadores, como los antagonistas del calcio dihidropiridínicos (nifedipino). Los
diuréticos deben usarse con mucho cuidado para no disminuir la precarga.

Los fármacos que SI están indicados son los que disminuyen la contractilidad miocárdica, como los
betabloqueantes o los antagonistas del calcio no dihidropiridínicos (verapamil y diltiacem), y
algunos antiarrítmicos, como la amiodarona y la disopiramida.

De hecho, el tratamiento más usado son los betabloqueantes, que mejoran la sintomatología y el
gradiente ventrículo- aórtico, pero no disminuyen el riesgo de muerte súbita.(R2)

49. Una mujer de 45 años acude a su médico por disnea de esfuerzo. En la radiografía de
tórax se demuestra crecimiento del ventrículo izquierdo. A la auscultación destaca un
soplo pansistólico en la punta grado III/VI, un soplo diastólico también en la punta y un
tercer tono. ¿Cuál cree que es la patología predominante en esta paciente?:

1. 1. Estenosis aórtica.
2. 2. Insuficiencia mitral.
3. 3. Insuficiencia aórtica.
4. 4. Miocardiopatía dilatada.
Gráfico de respuestas
Comentario
Pregunta de dificultad considerable, pero útil para entender la fisiopatología de las valvulopatías de
la mitral. Nos presentan un caso con dos soplos, uno sistólico y otro diastólico, y por ello la
pregunta nos puede hacer dudar. Sin embargo, el soplo pansistólico es el predominante, y la
localización nos orienta a la válvula mitral. Además se observa un agrandamiento del VI y un tercer
ruido. Estos datos son característicos de la IM, por la sobrecarga de volumen que supone la
regurgitación constante de sangre del VI a la AI, que de nuevo vuelve al VI. El VI trabaja con
mayores volúmenes y por ello también se dilata. En la EM hay dificultad para el paso de la sangre
de la AI al VI, así que aumentan las presiones en la AI y con el tiempo se dilata. Sin embargo, el VI
mantiene presiones normales y no se dilata. En la EM el daño es retrógrado, de forma que las altas
presiones y la dilatación de la AI, repercuten sobre el VD que debe trabajar con mayores
presiones. Con el tiempo puede provocar una dilatación del VD e incluso Insuficiencia cardíaca
derecha.(R2)

50. A 26-year-old woman is diagnosed with ulcerative colitis, with involvement of the
distal 30 cm of colon. After a 15-day treatment with enemas of mesalazine, the patient
improves significantly, and she decides to discontinue the treatment. Two weeks later,
she presents again with increased number of bowel movements with little blood and mild
abdominal pain. She is afebrile and tolerates solid food. Which of the following is the most
appropriate next step in management?

 
 
 
 
1. 1. Immunosuppressant therapy.
2. 2. Parenteral prednisone therapy.
3. 3. Perform another colonoscopy.
4. 4. Restart treatment with mesalazine enemas and gradually taper it off, if it is effective.
(R4)

51. ¿Qué es el llamado efecto lusotrópico?:

El secuestro de la sangre hacia las partes declives como consecuencia de la fuerza de la


1. 1.
gravedad.
2. 2. La dificultad que existe para el llenado ventricular durante un taponamiento cardíaco.
3. 3. El aumento de la precarga que se observa en los vuelos espaciales.
Un acortamiento de la sístole y un aumento de la velocidad de relajación como
4. 4.
consecuencia de un efecto inotrópico positivo.
Gráfico de respuestas
Comentario

Pregunta bastante difícil sobre un concepto que muy pocas veces aparece en el ENARM. El efecto
lusotrópico va unido al efecto inotrópico positivo provocado por el sistema simpático en respuesta a
un estrés. Las catecolaminas no solo aceleran la sístole para aumentar la frecuencia y la
contractilidad, sino que también aceleran la diástole, es decir el corazón también se relaja más
rápido. Este es el denominado efecto lusotrópico positivo, luego la opción cierta es la 4.(R4)

52. ¿Cuál de los siguientes fármacos resulta útil en el tratamiento de la bronquiolitis


infantil?:

1. 1. Corticoides.
2. 2. Amoxicilina.
3. 3. Salbutamol.
4. 4. Ribavirina.
Gráfico de respuestas
Comentario

De esta pregunta acerca de el tratamiento de la bronquiolitis es importante recordar ciertos


aspectos. Los broncodilatadores tienen dudosa eficacia pero se utilizan por si existe
broncoespasmo reflejo. El antiviral útil para el VRS es la ribavirina, no el aciclovir (útil para herpes).
Su uso puede ser considerado en casos graves en niños con cardiopatía, displasia pulmonar o
inmunodeficiencias. No existen evidencias de la utilidad de los corticoides en el tratamiento de la
bronquiolitis. Los antibióticos sólo están indicados si existe neumonía bacteriana secundaria.(R4)

53. La lepra lepromatosa se caracteriza por:

Baciloscopia (-), Mitsuda (+), inmunidad celular disminuida, inmunidad humoral


1. 1.
conservada.
Baciloscopia (+), Mitsuda (-), inmunidad celular disminuida, inmunidad humoral
2. 2.
conservada.
Baciloscopia (-), Mitsuda (-), inmunidad celular conservada, inmunidad humoral
3. 3.
conservada.
4. 4. Baciloscopia (+), Mitsuda (-), inmunidad celular y humoral disminuida.
Gráfico de respuestas
Comentario

 
 
 
 
La lepra es un tema muy poco preguntado en el MIR, por lo que no debemos dedicarle demasiados
recursos. La lepra lepromatosa es aquella que se produce en los sujetos infectados por el bacilo de
Hansen que tienen una mala inmunidad celular. Esto hará que la baciloscopía tienda a ser positiva
y el Mitsuda negativo (equivalente al PPD). La inmunidad humoral no juega un papel relevante en
las micobacteriosis, y estará en general conservada en estos pacientes. Por lo tanto la opción
correcta será la 2.(R2)

54. La enfermedad celíaca es una intolerancia permanente al gluten. ¿Cuál de las


siguientes afirmaciones es INCORRECTA en relación con esta enfermedad?

1. 1. La lesión intestinal está mediada por mecanismos inmunológicos.


Suele existir un intervalo libre de síntomas entre la introducción del gluten en la dieta y el
2. 2.
comienzo de la clínica.
La determinación de anticuerpos antigliadina es la prueba serológica más específica en
3. 3.
esta enfermedad.
El diagnóstico de la enfermedad se basa en la demostración de una lesión de la mucosa
4. 4.
intestinal.
Gráfico de respuestas
Comentario

La enfermedad celíaca tiene una base inmunopatológica (respuesta 1 verdadera), habitualmente


asintomática durante un tiempo, con un período libre de enfermedad que puede durar años, hasta
que aparecen las primeras manifestaciones clínicas (respuesta 2 cierta). Dentro del diagnóstico
diferencial, entrarían otras enfermedades digestivas, como la giardiasis (respuesta 4 cierta), que
afecta también a duodeno e íleon proximal, compartiendo síntomas como flatulencia, esteatorrea,
molestias epigástricas inespecíficas, etc.

El diagnóstico de enfermedad celíaca requiere una biopsia de la unión duodeno-yeyunal. La lesión


histológica no es específica, ya que puede verse en otras enfermedades, pero es muy sugestiva:
atrofia de vellosidades e hiperplasia de las criptas (respuesta 5 correcta). No obstante, existen
anticuerpos que también nos dan una orientación diagnóstica. Los más específicos son los
antitransglutaminasa tisular (respuesta 3 falsa).

Repasa con los esquemas siguientes la patogenia y diagnóstico de la enfermedad celíaca.(R3)

Patogenia de la enfermedad celíaca

Diagnóstico de la enfermedad celíaca

 
 
 
 

55. What is the most important examination for the diagnosis of a breast tumor in a 45-
year-old woman with a family history significant for breast cancer?

1. 1. Axial Tomography.
2. 2. Ultrasonography.
3. 3. Mammography.
4. 4. Breast self-examination.
Gráfico de respuestas
Comentario

La MASTOGRAFIA es actualmente el método imprescindible pra el diagnóstico precoz del cáncer


de mama. En mujeres con familirares de primer grado con cáncer de mama, se practicará una
mamogradia anual al menos dos años antes de la edad de diagnóstico o a los 40 años.(R3)

56. ¿Cuál de las siguientes valvulopatías aisladas es la más frecuente como secuela de
la fiebre reumática?:

1. 1. Estenosis aórtica.
2. 2. Insuficiencia aórtica.
3. 3. Estenosis mitral.
4. 4. Insuficiencia mitral.
Gráfico de respuestas
Comentario
La válvula que más se afecta por la carditis reumática es la mitral, donde la lesión más común es la
doble lesión (estenosis más insuficiencia) aunque como lesión aislada es más frecuente la
estenosis mitral. El diagnóstico de la fiebre reumática aguda se realiza por la presencia de los
famosos criterios clínicos de Jones más la evidencia de infección estreptocócica (cultivo faríngeo
positivo o ASLO elevado o en aumento). Los criterios mayores que debes conocer son la artritis
(migratoria no deformante, que cura sin secuelas), la carditis (cardiomegalia, soplo, derrame
pericárdico, insuficiencia cardiaca, es el que marca el pronóstico por la gravedad de las lesiones),
nódulos subcutáneos, eritema marginado y el corea minor o corea de Sydenham (que es la
manifestación habitualmente más tardía y puede aparecer meses después del episodio agudo). Se
consideran criterios menores la aparición de fiebre, artralgia, aumento de VSG, proteína C reactiva,
prolongación del intervalo PR en el ECG. Para llegar al diagnóstico se exigen dos criterios mayores
o un criterio mayor y dos menores.(R3)

 
 
 
 
57. Indique la respuesta INCORRECTA sobre la microtoma de sangre fetal:

1. 1. El valor normal del pH fetal es de 7.25 a 7.45.


2. 2. El pH fetal tiene valor pronóstico sobre el estado del recién nacido.
La diferencia de pH materno fetal en caso de feto no hipóxico es de 0.10-0.15 unidades
3. 3.
de pH.
Cuando el pH se encuentra en valor prepatológico (7.15-7.25) se debe tratar a la madre
4. 4.
con bicarbonato y repetir la toma a los 15-20 minutos.
Gráfico de respuestas
Comentario

Pregunta sobre la microtoma de sangre fetal bastante fácil. La microtoma se obtiene realizando
una pequeña incisión en el cuero cabelludo del feto y, acto seguido, nos informan del valor del pH,
siendo normal entre 7.25 y 7.45; prepatológico entre 7.20 y 7.25, y patológico si es menor de 7.20.
Teniendo en cuenta todo esto, la respuesta incorrecta es la 4, ya que incluye como prepatológico el
intervalo de 7.15 a 7.20.

El resto de opciones son correctas: la acidosis materna puede hacernos variar el valor del pH fetal,
y habría que tenerlo en cuenta para los valores prepatológicos. El enunciado de la opción 2
también es cierto, pues el pH fetal es una muestra objetiva del estado del feto; de ahí que se utilice
como criterio para tomar determinadas decisiones (por ejemplo, cesárea cuando es inferior a
7.20).(R4)

58. ¿Cuál de los siguientes NO es un factor de riesgo para la atonía uterina?:

1. 1. Miomas.
2. 2. Multiparidad.
3. 3. CIR.
4. 4. Infección uterina.
Gráfico de respuestas
Comentario
Los miomas dificultan la contracción uterina al ser tumores benignos en el espesor del miometrio.
La multiparidad se ha visto que es un factor de riesgo para atonía. El parto prolongado, así como el
uso de oxitocina durante mucho tiempo actúan agotando al músculo miometrial y repercutiendo en
su contracción postparto. La infección uterina, debido a los mecanismos inflamatorios se ve
alterada la contractilidad del miometrio.(R3)

59. Femenino de 62 años con lesión en dorso nasal de 5 meses de evolución. La lesión
es eritematoviolácea, áspera, con tapones foliculares y telangiectasias. En las pruebas
complementarias se encuentra disminución de los linfocitos T supresores y aumento de
la excreción renal de calcio. En la radiografía de tórax se detectaron adenopatías hiliares
bilaterales. ¿Cuál es el diagnóstico más probable?:

1. 1. Lupus eritematoso sistémico con lesión de lupus eritematoso cutáneo discoide.


2. 2. Lupus pernio más sarcoidosis.
3. 3. Leishmaniasis mucocutánea.
4. 4. Lupus pernio más tuberculosis.
Gráfico de respuestas
Comentario

 
 
 
 
La afectación cutánea de la sarcoidosis no es objeto de muchas preguntas aunque es importante
que recuerdes el lupus pernio como manifestación más característica. La sarcoidosis es una
enfermedad granulomatosa multisistémica que afecta al pulmón en casi todos los casos
produciendo una enfermedad intersticial. Hay adenopatías intratorácicas en el 80- 90% de los
pacientes (las más frecuentes son las hiliares). La hipercalciuria es típica, asociada o no a
hipercalcemia, aunque es infrecuente. En el estudio de sangre periférica pueden encontrarse
linfopenia, eosinofilia, hipergammaglobulinemia y elevación de la VSG. Dentro de las
manifestaciones cutáneas lo más importante es recordar el lupus pernio (es la más característica)
que consiste en una lesión en placa indurada, violácea, crónica con tendencia a aparecer en nariz,
pabellones auriculares y labios y es más frecuente en mujeres con sarcoidosis persistente y
afectación pulmonar, uveitis crónica y lesiones óseas.(R2)

60. A 60-year-old man comes to your office complaining of tremor in both hands while
holding things, especially if he is nervous or tired. He says that his symptoms get better
when he drinks small amounts of wine. On further questioning, it is known that his father,
now deceased, had trembling hands and head. Neurological examination shows
symmetrical postural tremor in both hands. Which of the following options is most likely
responsible for this patient's tremor?

1. 1. Familial hypothyroidism
2. 2. Symptoms of alcohol deprivation.
3. 3. Essential tremor
4. 4. Hereditary anxiety disorder (organic neurosis).
Gráfico de respuestas
Comentario

Pregunta tipo caso clínico sobre trastorno del movimiento.

El cuadro que se nos expone refleja una historia de carácter familiar, con temblor de tipo postural, y
en la que es llamativa la mejoría con consumo en pequeña cantidad de vino. Todo ello orienta
claramente a un temblor esencial (opción 3), que constituye la forma más corriente de temblor
postural hereditario. Típicamente mejora con el alcohol. Ningún otro dato del enunciado es
sugestivo, por otra parte, de cualquiera de las otras etiologías.

Vale la pena recordar que la presencia de otros signos neurológicos excluye el diagnóstico (salvo
el signo de la rueda dentada, característico de la enfermedad de Parkinson, pero posible también
en esta entidad). A su vez, la enfermedad de Parkinson puede asociar frecuentemente un temblor
postural (hasta un 60%).(R3)

61. La valoración del test de Apgar en el recién nacido nos permite:

1. 1. Determinar la edad gestacional.


2. 2. Realizar una evaluación fisiológica general.
3. 3. Evaluar el grado de dificultad respiratoria.
4. 4. Determinar la conducta a seguir en el pretérmino.
Gráfico de respuestas
Comentario

Respuesta que no puede fallar del test de Apgar, el cual NO sirve para determinar la edad
gestacional (Capurro y Ballard modificado si), ni tampoco sirve para evaluar el grado de dficultad

 
 
 
 
respiratoria (Silverman-Andersen) y tampoco determina la conducta a seguir en el pretérmino.
Respuesta correcta 2.(R2)

62. En la reanimación neonatal, la frecuencia de ventilación: masaje cardiaco es de:

1. 1. 5:1.
2. 2. 2:15.
3. 3. 2:30.
4. 4. 1:3.
Gráfico de respuestas
Comentario

El masaje cardíaco se realiza si luego de instaurar adecuada ventilación por 30 segundos la


frecuencia cardíaca permanece inferior a 60 latidos por minuto.

Se realiza comprimiendo el tercio inferior del esternón, por debajo de una la línea imaginaria que
las se traza uniendo los dos pezones, con una profundidad de un tercio de la circunferencia
torácica. Existen dos técnicas para realizar masaje cardíaco que dependen en parte de la cantidad
de personas disponibles. El método más efectivo es la aplicación de los 2 pulgares sobre el
esternón (uno sobre el otro o uno al lado del otro) con el resto de los dedos rodeando el tórax.

La secuencia en el niño es 2:15 (dos ventilaciones 15 compresiones). En el adulto es 30:2,


mientras que en en el neonato es 1:3.(R4)

63. Una paciente de 56 años presenta poliartritis simétrica de IFP, MCF, carpos y rodillas
desde hace tres meses, con rigidez matutina de 90 minutos. En la radiología de manos
muestra aumento de partes blandas. El factor reumatoide es negativo y no tiene nódulos
reumatoides. ¿Cuántos criterios para el diagnóstico de artritis reumatoide presenta esta
paciente?

1. 1. Dos.
2. 2. Tres.
3. 3. Cuatro.
4. 4. Cinco.
Gráfico de respuestas
Comentario

El diagnóstico de AR se establece por la coexistencia de 4 de los 7 criterios diagnósticos


propuestos por la ACR para el diagnóstico de la enfermedad. Estos son:

- Rigidez articular después de la inactividad de al menos 1 hora de duración.

- Afectación poliarticular (al menos tres áreas articulares de forma simultánea)

- Distribución simétrica de la afectación articular.

- Participación de las articulaciones de las manos (MCF, IFP, Carpo).

- Nódulos subcutáneos.

 
 
 
 
- Presencia de Factor Reumatoide.

- Alteración radiológica (erosiones, osteoporosis yuxtaaricular).

Como ves, la paciente presenta los cuatro primeros criterios.(R3)

64. A 73-year-old female comes to your office complaining of hip pain, that started 6
months ago. Her medical history is positive for hypothyroidism and hypertension, but the
rest is unremarkable. Physical examination reveals: BP 148/78 mm Hg, pulse 75/min and
temperature 36,9ºC. Lab tests show elevated alkaline phosphatase. Which of the following
tests would you perform to assess the extension of the disease?

1. 1. Body MRI
2. 2. Bone gammagraphy
3. 3. X-ray
4. 4. PET scan
Gráfico de respuestas
Comentario
Bone gammagraphy. Paget disease is a localized disorder of bone remodeling that typically begins
with excessive bone resorption followed by an increase in bone formation. This osteoclastic
overactivity followed by compensatory osteoblastic activity leads to a structurally disorganized
mosaic of bone (woven bone), which is mechanically weaker, and more susceptible to fracture than
normal adult lamellar bone. Approximately 70-90% of persons with Paget disease are
asymptomatic; however, a minority of affected individuals experience various symptoms, including
pain (the most common symptom), osteoarthritis, bony deformity and neurologic complications.
Plain radiographs are less sensitive than bone scan scintigraphy in the diagnosis of Paget disease.
An entire skeletal survey with plain radiographs to assess the extent of skeletal involvement is not
recommended since bone scanning would be more sensitive and involve less radiation
exposure.(R2)

 
 
 
 

65. Estando usted de guardia le avisan


desde la Unidad de Cuidados Intensivos porque una paciente que acaban de extubar
presenta una saturación de oxígeno en sangre que no supera el 87-88%. Como
antecedente inmediato esta paciente ha sido intervenida hace 2 horas de una
tiroidectomía total por un carcinoma papilar de tiroides. Al realizarle la exploración
laríngea mediante nasofibrolaringoscopio, la imagen muestra lo que usted observa,
durante la inspiración de la paciente. ¿Qué síntomas y signos esperaría usted haber
encontrado en la paciente previos a la realización de la nasofibrolaringosocopia?

1. 1. Disnea de predominio espiratorio sin trabajo muscular respiratorio asociado.


Disnea de predominio inspiratorio, estridor inspiratorio y trabajo muscular respiratorio
2. 2.
asociado.
3. 3. Disfonía sin disnea.
4. 4. Disfonía y aspiración tanto de alimentos como de saliva.
Gráfico de respuestas
Comentario

Dados los antecedentes y la posición paramediana de las dos cuerdas vocales, debemos
sospechar una lesión de ambos nervios recurrentes, en relación con la cirugía recién practicada,
de ahí la disnea y el estridor que asociarían trabajo muscular respiratorio. Recuerda que la parálisis
recurrencial unilateral no produce disnea, sino únicamente disfonía (voz bitonal).(R2)

66. ¿Qué tratamiento realizaría a esta paciente?

1. 1. Amoxicilina-ácido clavulánico, corticoides intravenosos y vigilar vía aérea.


2. 2. Aerosoles con corticoides, adrenalina y vigilar vía aérea.
Traqueotomía de forma urgente, incluso en la misma cama de la UVI, ya que la paciente
3. 3.
presenta ya signos de insuficiencia respiratoria aguda grave.
Esperar y volver a realizar una nasofibrolaringoscopia en una hora, ya que por el
4. 4.
momento la paciente no presenta signos de inestabilidad respiratoria.
Gráfico de respuestas

 
 
 
 
Comentario

Teniendo en cuenta la imagen y la importante desaturación que presenta el paciente, debemos


garantizar una vía respiratoria permeable. Por este motivo, la solución no consiste en administrar
antibióticos ni en repetir la prueba una hora después, sino en realizar una traqueotomía ante la
situación de gravedad que nos describen.(R3)

67. A 7 month-old
boy is brought to the ER with intermittent episodes of crying and irritability. His mother
refers that during these episodes, the baby looks pale, starts sweating and draws his
knees up to his chest. He has been refusing the bottle for the past few hours and presents
with bloody stools. Physical examination is very difficult, due to his lethargy and
irritability. He has been having gluten for over a week now. The image shows his
ultrasound test, which of the following is the most likely diagnosis?

1. 1. Acute gastroenteritis
2. 2. Acute apendicitis
3. 3. Intestinal intussusception
4. 4. Food intolerance
Gráfico de respuestas
Comentario

Es una pregunta sencilla. Lactante con crisis de llanto intermitentes con encogimiento de piernas,
sudoración, palidez… y que además presenta rechazo de las tomas (también podría presentar
vómitos) y evacuaciones hemorrágicas, sólo por la clínica ya hay que diagnosticar una
invaginación intestinal. Además asocian una imagen de una ecografía abdominal (técnica de
elección) donde se observa la característica imagen de la rosquilla o “dona”. Así que sin ninguna
duda la respuesta correcta sería la invaginación. Recordemos que el protagonista de este caso
clínico es siempre un niño muy pequeño, de pocos meses de edad.(R3)

68. Which of the following treatments is not indicated in the previous patient:

1. 1. Barium or water-soluble contrast enema.


2. 2. Air-contrast enema.

 
 
 
 
3. 3. Gluten-free diet.
4. 4. Surgery.
Gráfico de respuestas
Comentario

En el tratamiento de la invaginación intestinal se puede realizar una reducción hidrostática con


bario o también una reducción con aire o suero mediante control ecográfico. En el caso de que ya
existan signos de perforación intestinal, choque, neumatosis intestinal o más de 48 horas de
evolución del cuadro, sería de elección el tratamiento quirúrgico para que en el caso de que se
haya producido necrosis intestinal, realizar una resección de la zona afecta y una anastomosis
término-terminal. En el caso del tratamiento quirúrgico se debería administrar al paciente
tratamiento antibiótico también. La incorrecta sería la opción de dieta exenta de gluten ya que ese
sería el tratamiento de una celiaquía que no se presenta con esta clínica y que necesita un tiempo
de latencia de meses desde que se introduce el gluten para que se manifieste clínicamente (no una
semana). El tratamiento de la invaginación es una urgencia, el retraso en el diagnóstico y en el
tratamiento mediante su reducción se asocia a una mayor morbimortalidad.(R3)

69. Referente al cáncer de endometrio es más frecuente en:

1. 1. Recién nacidas
2. 2. Adultas
3. 3. Posmenopáusicas
4. 4. Premenopáusicas
Gráfico de respuestas
Comentario

El riesgo de cáncer de endometrio aumenta con la edad, y es por tanto más frecuente después de
la menopausia.(R3)

70. A 65-year-old male goes to his primary care physician’s office with the complaint of
progressive shortness of breath for the past two months. He also reports a 11,9kg weight
loss during this period. His previous medical history is significant for hypertension and
myocardial infarction two years ago. He currently takes aspirin, bisoprolol, enalapril and
simvastatin. He has smoked two packs of cigarettes daily for 50 years. On physical
examination, BP is 120/70 mmHg, pulse is 70/min, temperature is 36.9ºC and respiratory
rate is 22/min. low breath sounds and dullness to percussion are noted on the right,
middle and lower lobes of the lung. CXR is shown below. Which of the following is the
most appropriate next step in the management of this patient?

1. 1. Mediastinoscopy
2. 2. Chest CT
3. 3. Thoracentesis
4. 4. Placing of a chest tube
Gráfico de respuestas
Comentario
Thoracentesis. This patient with previous history of heavy smoking and constitutional syndrome
symptoms is a high risk patient for lung cancer. The pleural effussion should be analyzed.(R3)

71. Señale lo INCORRECTO en relación a la fibrosis quística:

1. 1. En niños mayores es característica la tos matutina o después de una actividad.

 
 
 
 
Las complicaciones pulmonares más frecuentes (atelectasias, hemoptisis, neumotórax y
2. 2.
cor pulmonale) aparecen en la primera década de la vida.
3. 3. En un 10% de los recién nacidos con fibrosis quística se presenta íleo meconial.
Además de la insuficiencia pancreática exocrina también puede observarse hiperglucemia
4. 4.
y glucosuria.
Gráfico de respuestas
Comentario

Esta pregunta de la clínica y complicaciones de la fibrosis quística es importante.

La tos productiva es el síntoma mas constante. Empeora con la edad, por las mañanas y con el
esfuerzo. El germen que con mas frecuencia coloniza la vía respiratoria es Pseudomonas
aeruginosa. Las complicaciones son frecuentes a partir del primer decenio de vida, y no antes.
Recuerde que la afectación cardiovascular que se ve con mas frecuencia es el cor pulmonale
crónico.

Aproximadamente el 10% de los pacientes nace con obstrucción intestinal a consecuencia del
espesamiento del meconio. El 10% de los adolescentes pueden experimentar una insuficiencia de
insulina relativa y la hiperglucemia puede hacerse sintomática, aunque la cetoacidosis es
infrecuente.(R2)

72. Un paciente de 60 años acude a Urgencias por presentar desde hace 1 hora dolor
opresivo retroesternal irradiado a la mandíbula, y en el ECG se observa elevación del
segmento ST en las derivaciones V1, V2, V3 y V4. ¿Cuál es el diagnóstico más probable?:

1. 1. Pericarditis aguda.
2. 2. Angina de Prinzmetal.
3. 3. Infarto inferior.
4. 4. Infarto anterior.
Gráfico de respuestas
Comentario

Cuando nos plantean una pregunta de diagnóstico diferencial de dolor torácico, debemos fijarnos
en primer lugar en las características del dolor. En este caso, el dolor es muy típico de cardiopatía
isquémica, con lo que la opción 1 sería improbable. Posteriormente, con el ECG diferenciamos
entre angina inestable e IAM. Si hay elevación del ST, pensaríamos en un IAM o una angina de
Prinzmetal. Como el IAM es más frecuente que el vasoespasmo, debemos decantarnos por el IAM.
Además, la angina de Prinzmetal produce un dolor que aparece y desaparece, como consecuencia
del vasoespasmo, y en este paciente tiene ya una hora de duración. Finalmente, para elegir la
localización del infarto, debemos fijarnos en las derivaciones afectadas, que en este caso son las
de la cara anterior.(R4)

73. Los aneurismas arteriales cerebrales de tipo sacular se localizan con mayor
frecuencia en:

1. 1. Arteria comunicante anterior y cerebral anterior.


2. 2. Arteria cerebral media.
3. 3. Arteria carótida interna y comunicante posterior.
4. 4. Arteria cerebral posterior.
Gráfico de respuestas
Comentario

 
 
 
 
Una pregunta puramente memorística que no se presta en exceso a posibles razonamientos.
Recuerda que los aneurismas saculares aparecen con mayor frecuencia en la porción anterior del
polígono de Willis (cerebral anterior y comunicante anterior). En cambio, los que se forman a partir
de la basilar son fusiformes.(R1)

74. Paciente varón de 47 años, portador de trasplante renal desde hace 2 meses que
acude a urgencias por dolor sobre injerto renal, hematuria y febrícula de 37,3 ºC. Se extrae
analítica de control que evidencia importante deterioro de función renal y un urocultivo
que es negativo. Se realiza eco doppler renal donde sólo destaca aumento de la
ecogenicidad cortical. ¿Cuál debe ser la actitud?

1. 1. Biopsia por sospecha de rechazo del injerto.


2. 2. Urografía por sospecha de estenosis del uréter.
3. 3. TC abdominal.
4. 4. Nefrostomía percutánea.
Gráfico de respuestas
Comentario
Caso clínico sobre deterioro de función renal en paciente trasplantado. Ante hematuria con dolor
sobre el injerto debemos descartar siempre un rechazo.(R1)

75. Lactante de 21 días de vida que presenta, desde hace 4 días, vómitos pospandriales
que cada vez son más intensos y frecuentes. Señale cuál de las siguientes afirmaciones
es cierta en relación a la enfermedad del niño:

1. 1. Esta enfermedad es más frecuente en niños.


2. 2. Suele cursar con acidosis metabólica.
3. 3. El tratamiento es médico y la recidiva frecuente.
4. 4. La enfermedad está producida por una bacteria.
Gráfico de respuestas
Comentario

Pregunta básica aparece año con año en el ENARM, es imposible que la falles.

Es una pregunta fácil sabiendo la clínica típica de la estenosis hipertrófica de píloro EHP.

Nos muestran un neonato de 21 días que presenta vómitos postprandiales cada vez más
frecuentes. En un niño de esta edad y con esta clínica nos debemos plantear el diagnóstico de
EHP. No olvide que el vómito no es bilioso y que el niño queda hambriento e irritable tras el mismo.
De entrada podemos descartar la respuesta 4, puesto que la EHP es de etiología desconocida,
pese a que se ha relacionado con factores que predisponen a ello como la administración de
prostaglandina E para mantener la permeabilidad del ductus. Si nos fijamos en la opción 2, los
vómitos persistentes efectivamente generan alteraciones electrolíticas, pero no acidosis
metabólica, sino alcalosis metabólica hipoclorémica.

Si el problema de la EHP es una hiperplasia e hipertrofia de la musculatura lisa del antro gástrico y
del duodeno, es lógico pensar que el tratamiento será quirúrgico: pilorotomía de Ramsted
(respuesta 3 falsa). Sabiendo que la EHP es más habitual en primogénitos varones de raza blanca
en la tercera semana de vida, podemos afirmar que la 1 es cierta. Recuerde que el polihidramnios
no se relaciona con la EHP.(R1)
Estenosis hipertrófica de píloro. Imagen anatómica

 
 
 
 

76. En un examen rutinario de empresa, el médico descubre que uno de sus trabajadores
tiene hipertransaminemia. La exploración física es totalmente normal. Pide perfiles
serológicos de infección de los virus causantes de hepatitis, siendo los resultados: HBs
Ag (+), Anti-HBs (-), Anti-HBc tipo Ig G (+), HBe Ag (+), Anti-HBe (-), Anti HC (-), Anti HD (-
). ¿Cuál es su diagnóstico de sospecha?:

1. 1. Portador de VHB.
2. 2. Infección crónica por virus B.
3. 3. Periodo de ventana de anti-HBc.
4. 4. Cirrosis biliar primaria.
Gráfico de respuestas
Comentario
La presencia de AgHBs indica que el paciente tiene virus en su cuerpo. Como los anticuerpos
contra la proteína del CORE son positivos de tipo IgG indicaría infección crónica. Además se
encuentra en fase replicativa al tener AgHbe positivo.(R2)

77. La hemorragia uterina disfuncional (HUD) no resulta un diagnóstico apropiado cuando


un curetaje revela:

1. 1. Polipo endometrial.
2. 2. Endometritis TBC.
3. 3. Respuestas 1 y 2.
4. 4. Ninguno de los anteriores.
Gráfico de respuestas
Comentario

Queda claro que si una paciente presenta hemorraga uternia disfuncional y en el curetaje se revela
polipo endometrial, endometritis por TBC, o cualquier otra alteración anatómica, infecciosa, no
puede realizarse el diagnóstico de HUD. Respuesta 3 correcta.(R3)

78. Acude a su consulta una paciente de 83 años con un bocio multinodular. No presenta
disnea, disfagia ni disfonía. Aporta hormonas tiroideas con T3, T4 y TSH normales. Al
elevar los brazos presenta ingurgitación yugular, plétora facial y síncope. ¿Cuál es el
tratamiento de elección?

1. 1. Radioyodo.
2. 2. Cirugía.

 
 
 
 
3. 3. Antitiroideos.
4. 4. Vigilancia periódica.
Gráfico de respuestas
Comentario

Esta pregunta hace referencia al tratamiento del bocio multinodular (BMN). Nos presentan a una
paciente diagnosticada de BMN con hormonas tiroideas dentro de límites normales. En esta
situación, el tratamiento correcto sería realizar seguimiento con determinación de hormonas y
ecografía de forma periódica. Sin embargo, cuando el paciente presenta síntomas compresivos
como la disfagia, disnea o signo de Pemberton, está indicada la intervención quirúrgica,
realizándose una tiroidectomía subtotal (respuesta correcta 3) El radioyodo se encontraría indicado
en caso de hiperfunción, y el tratamiento con dosis supresoras de levotiroxina parece poco
apropiado, ya que en el BMN casi nunca hace regresar el bocio.(R2)

79. Todos los siguientes se utilizan en las crisis hipertensivas MENOS uno:

1. 1. Nitroglicerina.
2. 2. Diazóxido.
3. 3. Urapidilo.
4. 4. Verapamilo.
Gráfico de respuestas
Comentario
En relación a las crisis hipertensivas debes saber reconocer un caso clínico, así como su manejo
terapéutico, que ya ha sido preguntado en el MIR en alguna ocasión. Recuerda que en una
emergencia hipertensiva es necesario el tratamiento inmediato del paciente, generalmente con
fármacos intravenosos. En este contexto son útiles el nitroprusiato sódico, la nitroglicerina, la
furosemida, el diazóxido, el labetalol o el urapidilo. Este último fármaco, que puede que te suene
menos, es un antihipertensivo con acción dual bloqueante alfa- 2 y beta- 1 adrenérgico, que se
emplea vía parenteral en las crisis hipertensivas. El verapamilo es un calcioantagonista y no se
emplea en las crisis hipertensivas; su mayor indicación es el tratamiento de la HTA sistólica aislada
y el tratamiento de pacientes hipertensos que además tienen angina.(R4)

80. Indique cuál de las siguientes patologías se asocia con mayor frecuencia a telorragia
serohemática unilateral:

1. 1. Fibroadenoma.
2. 2. Carcinoma lobulillar.
3. 3. Papiloma intraductal.
4. 4. Enfermedad de Paget.
Gráfico de respuestas
Comentario

Es una pregunta que se debe tener bien clara. La causa principal de telorrea serohemática
unilateral es el papiloma intraductal. El carcinoma también lo puede dar; pero la causa más
frecuente es claramente el papiloma.(R3)

81. A five-year-old boy presents to his pediatrician with a sudden rash located on his face
(especially cheeks) that extended to trunk, with central clearing of the skin lesions. On
further questioning, it is known that these lesions worsen after a hot water bath. What
would you advise his 15-week pregnant mother to do?

 
 
 
 
1. 1. Put her at ease, since the majority of pregnant women have antibodies against rubella.
Human parvovirus B19 is the causative agent of hydrops fetalis. Therefore, she should
2. 2.
avoid contact with the patient.
3. 3. Herpes virus type 6 is associated with a high risk of abortion.
The mother should avoid contact because of a high risk of developing complications
4. 4.
throughout her pregnancy.

(R2)

82. El síndrome de feminización testicular de Morris se caracteriza por todo lo siguiente


EXCEPTO:

1. 1. Cromosoma XO.
2. 2. Amenorrea primaria.
3. 3. Gónadas disgenéticas inguinales.
4. 4. Talla elevada.
Gráfico de respuestas
Comentario

Es una pregunta bastante difícil del síndrome de Morris del que debe conocer algunas
características principales para poder resolver un posible caso clínico.

La feminización testicular o resistencia androgénica completa se debe a un trastorno del receptor


androgénico. Se hereda de forma recesiva ligada a X. El cariotipo es masculino 46 XY.

Los conductos de Wolf están ausentes y no existe ningún órgano genital interno. Los testículos son
normales sin espermatogénesis, pueden estar situados en el abdomen o en cualquier lugar del
trayecto del conducto inguinal.

Seno urogenital femenino y vagina corta que acaba en un fondo de saco ciego. Los genitales
externos son femeninos con ausencia de vello axilar y pubiano. La mama también es normal. La
testosterona se encuentra elevada igual que los estrógenos y la LH (resistencia a la acción
androgénica en el hipófisis).(R1)

83. Femenino de 39 años. G1P1+embarazo de 13 semanas de gestación, acude a su centro


para realización de ultrasonido se reporta: gestación intrauterina, embrión con CRL de
70 mm y movimientos cardiofetales positivos, líquido amniótico en cantidad normal y
placenta en cara posterior. En la exploración usted observa un edema nucal de 5 mm.
Señale lo FALSO:

La gestante tiene un riesgo clínico (edad materna mayor de 35 años) que justificaría la
1. 1.
realización de una amniocentesis precoz, si así lo desean.
2. 2. El edema nucal en esta semana de gestación es normal, USG trimestral.
3. 3. El origen de este edema parece ser una alteración en el desarrollo del sistema linfático.
4. 4. El edema nucal puede aparecer en embriones cromosómicamente normales.
Gráfico de respuestas
Comentario

Se presenta un caso clínico de una gestante de 39 años, a la 13ª semana. Dada la edad materna,
habría que indicar una amniocentesis, cuyo riesgo de pérdidas fetales es de 1%.

 
 
 
 
Un CRL de 70 mm coincide más o menos con las 13 semanas de amenorrea. Ante un edema
nucal de más de 3 mm, debemos informar de la posible existencia de cardiopatías fetales,
cromosomopatías.

Esto hace aún más necesaria la amniocentesis. El edema nucal suele deberse a alteraciones del
drenaje linfático, lo cual no implica que obligatoriamente el feto tenga alguna malformación (pero sí
tiene más probabilidades).(R2)

84. En la amenaza de parto pretérmino ¿cuál es el fármaco de elección?:

1. 1. Oxitocina
2. 2. Beta-miméticos
3. 3. Nifedipino
4. 4. Atosibán
Gráfico de respuestas
Comentario

Pregunta directa y fácil. El fármaco de elección para la tocolisis es el atosibán, quedando los beta-
miméticos como fármaco de segunda línea. El nifedipino y la indometacina tienen gran cantidad de
contraindicaciones y se consideran opciones de 3ª línea de tratamiento.(R4)

85. Femenino de 42 años con antecedente de depresión mayor y en tratamiento desde


hace 3 días con fluoxetina que acude a urgencias por intento autolítico con ingesta de
sosa cáustica hace 3 horas. En la exploración oral se observa lengua depapilada y
negruzca. ¿Cuál será la primera exploración a realizar?:

1. 1. Gastroscopia.
2. 2. Rx tórax y abdomen.
3. 3. TC toraco-abdominal.
4. 4. Tránsito con gastrografin.
Gráfico de respuestas
Comentario
La ingesta de cáusticos, más siendo con intención autolítica y con datos de lesiones marcadas en
boca que traduzcan que a niveles inferiores pudieran ser graves, requieren de Rx tórax y abdomen
para descartar en primer lugar una perforación. .(R2)

86. El mejor signo clínico de una fluidoterapia correcta en el tratamiento de choque


hemorrágico es:

1. 1. Aumento de la presión sanguínea.


2. 2. Aumento de la diuresis.
3. 3. Disminución de la sed.
4. 4. Disminución de la taquicardia.
Gráfico de respuestas
Comentario
Si pensamos en las posibles respuestas, observamos que las cinco respuestas que nos dan son
verdaderas ya que todas marcan una recuperación del estado de choque; pero el MEJOR signo
clínico, basándonos en que el choque hemorrágico conlleva una pérdida de líquido, es el control de
la diuresis, ya que nos marca el correcto funcionamiento renal y por lo tanto una recuperación real
de la perfusión corporal. El resto de respuestas implican una recuperación, pero son signos más
inespecíficos.(R2)

 
 
 
 
87. En el caso de un CIR asimétrico o tipo II, ¿cuál es el primer parámetro ecográfico que
se altera?

1. 1. Longitud del fémur.


2. 2. Diámetro biparietal.
3. 3. Diámetros abdominales.
4. 4. Cantidad de líquido amniótico.
Gráfico de respuestas
Comentario

El CIR tipo I, también llamado simétrico, tiende a reducir el tamaño fetal de forma homogénea. En
cambio, el CIR tipo II se puede entender como un feto excesivamente “delgado”, es decir, lo
primero que se alteran son los diámetros abdominales, mientras que el resto de los parámetros
biométricos permanecen relativamente inalterados. Respuesta correcta 3.(R3)

88. Mario es un paciente varón, que acude al servicio de urgencias por un cuadro de 6
días de evolución de ictericia cutánea y conjuntival. Al realizar la anamnesis nos refiere
una pérdida de peso de 15 Kg en los últimos 6 meses. Entre las pruebas diagnósticas
solicitadas está un TAC, que pone de manifiesto una masa en los conductos biliares
intrahepáticos, motivo por el que se realiza una colangiografía, que nos proporciona el
diagnóstico de colangiocarcinoma. Señale la respuesta INCORRECTA sobre el
colangiocarcinoma:

1. 1. Se relaciona con el virus de la hepatitis B.


2. 2. El tratamiento de elección es la resección hepática, aunque pocas veces es posible.
3. 3. La recurrencia en el trasplante es muy frecuente, por lo que no es aconsejable realizarlo.
4. 4. Es más frecuente en pacientes ancianos que a edades jóvenes.
Gráfico de respuestas
Comentario

El colangiocarcinoma no presenta relación con el virus de la hepatitis B ni con la cirrosis. Ten


cuidado con esta pregunta porque es un engaño muy común. Lo que sí predispone a este tumor es
la infección por Clonorchis sinensis, las infecciones biliares de repetición y las malformaciones de
las vías biliares que determinen cierta lentitud del flujo biliar, como la enfermedad de Caroli.(R1)

89. Los factores de riesgo denominados "mayores" para la enfermedad coronaria


comprenden los siguientes con una EXCEPCION:

1. 1. Diabetes.
2. 2. Niveles plasmáticos bajos de colesterol unido a lipoproteínas de alta densidad (HDL).
3. 3. Niveles plasmáticos altos de colesterol unido a lipoproteínas de alta densidad (HDL).
4. 4. Hipertensión.
Gráfico de respuestas
Comentario
Son factores de riesgo indiscutibles para la cardiopatía isquémica la diabetes, el tabaquismo, la
hipercolesterolemia.. Hay otros factores de riesgo coronario más dudosos o menos importantes
como la elevación de la lipoproteína a, la hipertrigliceridemia, el sedentarismo, la obesidad, el
estrés psíquico, el sexo masculino, la historia familiar, los antecedentes de ACVA y los síndromes
de hipercoagulabilidad. Los niveles altos de HDL son un factor protector.(R3)

 
 
 
 
90. Una paciente de 32 años de edad presenta una auscultación cardíaca con un click
mesosistólico y un soplo mesotelesistólico que se incrementa de intensidad con la
maniobra de Valsalva. El diagnóstico de presunción será:

1. 1. Comunicación interauricular.
2. 2. Estenosis mitral.
3. 3. Estenosis aórtica.
4. 4. Prolapso mitral.
Gráfico de respuestas
Comentario
Pregunta típica e importante para el MIR, donde ya ha aparecido en alguna ocasión. Ante un caso
clínico con los datos: mujer joven + clic mesositólico y soplo mesotelesistólico + el soplo se
incrementa con el Valsalva, debemos sospechar un Prolapso mitral. Recuerda que en esta
enfermedad los datos auscultatorios se hacen más intensos y precoces con la bipedestación, la
maniobra de Valsalva, o el nitrito de amilo. En cambio el ejercicio isométrico o la posición de
cuclillas hacen que sean más tardíos y menos intensos. Esto también ocurre en la Miocardiopatía
hipertrófica, pero en este caso aparece un soplo sistólico rudo localizado en el ápex y el borde
esternal izquierdo, y pulso bisferiens; y además no aparecen los clics mesotelesistólicos.(R4)

91. Alguna de las siguientes parejas que relacionan un trastorno defensivo con los
gérmenes que provocan infecciones por dicho déficit es INCORRECTA, señálela:

1. 1. Inmunodeficiencia celular - parásitos intracelulares.


2. 2. Déficit de IgA - Giardia.
3. 3. Esplenectomía - Bacilo DF-2
4. 4. Neutropenia - M. tuberculosis.
Gráfico de respuestas
Comentario

Pregunta fácil y lógica sobre un tema que debes dominar porque sus conocimientos pueden
ayudarte en muchos temas. Si dominas la tabla que aparece en el Manual sobre enfermedades
asociadas a trastornos inmunitarios, habrás contestar correctamente. La esplenectomía equivale a
un déficit humoral, con alteración en la producción de inmunoglobulinas y las infecciones
características en estos pacientes son las causadas por bacterias encapsuladas: neumococo,
meningococo, Haemophilus, bacilo DF2, paludismo y babesiosis. (respuestas 2 y 4 verdaderas). En
las inmunodeficiencias celulares (recuerda que el prototipo es el SIDA), son características las
infecciones por hongos, micobacterias, parásitos intracelulares y herpes virus (opción 1 verdadera).
El déficit de Ig A es la más frecuente de las inmunodeficiencias. Aunque en la mayoría de los casos
es asintomática, es característica la infección intestinal por Giardia Lamblia. La que no es cierta
claramente, es la opción 5. Los pacientes neutropénicos sufren infecciones, típicamente por
bacterias como S. aureus, BGN como la Pseudomona aeruginosa y hongos, siendo el más
frecuente el Aspergillus.(R4)

92. ¿Cuál de los siguientes es un factor de riesgo del carcinoma invasor de cérvix?:

1. 1. VHS tipo II.


2. 2. Nuliparidad.
3. 3. Síndrome de ovario poliquístico.
4. 4. Tamoxifeno.
Gráfico de respuestas
Comentario

 
 
 
 
El Virus Herpes tipo II es reflejo de la promiscuidad sexual (factor de riesgo), considerándose
además un factor promotor. Si bien la infección por virus del papiloma humano es un factor de
riesgo para cáncer cervix, hay que tener en cuenta que los serotipos 6 y 11 son de bajo riesgo y
que se asocian a los condilomas en genitales y no al cáncer de cervix. Son los serotipos de alto
riesgo como el 16 y 18 los que se asocian al cáncer. Otro factor de riesgo es la multiparidad.
Recuerda que tanto el tamoxifeno como el síndrome del ovario poliquístico son factores de riesgo
para el cáncer de endometrio.(R1)

93. Una mujer de 35 años presenta una historia de un mes de prurito y astenia. En los
últimos días se ha hecho evidente un tinte ictérico. Se constatan xantelasmas
palpebrales. Refiere asimismo sequedad ocular desde hace algunos meses. En el
diagnóstico de su probable patología, NO participa uno de los siguientes criterios:

1. 1. Elevación de la fosfatasa alcalina.


2. 2. Biopsia hepática compatible.
3. 3. Clínica de colestasis.
4. 4. Elevación de la bilirrubina.
Gráfico de respuestas
Comentario

Lo más importante, en esta pregunta, es que sospeches el diagnóstico correcto: la cirrosis biliar
primaria. La clave para acertarla está en saber que, en la mayor parte de los casos, se diagnostica
en fase asintomática, donde casi nunca existirá elevación de la bilirrubina, sino únicamente
enzimas de colestasis y AMAs positivos. La bilirrubina comienza a elevarse más tardíamente, y
tiene valor pronóstico, más que diagnóstico. Recuerda que está demostrado un paralelismo entre
la evolución de la enfermedad y la elevación de la bilirrubina sérica. De hecho, es el mejor
parámetro analítico para el control evolutivo de esta patología.(R4)

94. Gestante de 12 semanas que acude a su consulta para realizar un ultrasonido en la


que usted observa una gestación intrauterina con un embrión único con LCC de 16 mm
(acorde con 8 semanas) sin latido cardíaco. Señale lo CORRECTO:

Gestación mal calculada. La LCC nos indica el tiempo de gestación en el primer


1. 1.
trimestre. Corregiremos la fecha probable del parto.
2. 2. Diagnóstico de aborto diferido: legrado.
3. 3. Diagnóstico de feto acardio.
4. 4. Amenaza de aborto. Recomienda reposo y gestágenos.
Gráfico de respuestas
Comentario

Se habla de aborto diferido cuando, durante varias semanas, se produce la retención de una
gestación no evolutiva. Esta entidad puede producir complicaciones importantes, como una
coagulación intravascular diseminada por liberación de tromboplastinas placentarias, por lo que
debemos medir productos de degradación del fibrinógeno (PDFs, dímero D).

Clínicamente, como sucede en el caso que nos presentan, hablamos de aborto diferido cuando
detectamos muerte embrionaria (o ausencia de embrión, huevo muerto) antes de que comience un
aborto en curso. Sería entonces, como dice la opción 2, indicación de legrado.

Revisemos algunos datos importantes:

 
 
 
 
•   El latido cardíaco debería verse a partir de las 6 semanas (ultrasonido transvaginal) o 7
semanas (ultrasonido abdominal). Sea cual sea la verdadera edad gestacional (8 o 12
semanas), no existe latido. Ante un hallazgo como éste, carece de interés corregir la fecha
probable del parto o medir periódicamente la beta-hCG.
•   El feto acardio es una posible complicación de la transfusión entre dos fetos, en un
embarazo gemelar. Por otra parte, estemos en la 8ª o en la 12ª semana, en este momento
se habla ya de embrión, no de feto.
•   La amenaza de aborto consiste en una metrorragia en la primera mitad de la gestación, a
veces acompañada de dolor hipogástrico leve, clínica que nada tiene que ver con el caso
que aquí nos presentan.

(R2)

95. Señale la afirmación FALSA en relación con la resección laparoscópica del colon:

Casi todas las enfermedades del colon y recto, susceptibles de tratamiento quirúrgico, se
1. 1.
pueden abordar mediante laparoscopía.
En la cirugía del cáncer de colon y recto, la disección ganglionar que se puede realizar
2. 2.
por laparoscopía es más limitada que la efectuada por laparotomía.
El abordaje laparoscópico favorece un alta hospitalaria postoperatoria más precoz que
3. 3.
tras una resección de colon convencional.
En la cirugía del cáncer de colon y recto, las tasas de supervivencia a largo plazo de los
4. 4.
pacientes intervenidos por laparoscopía o laparotomía son similares.
Gráfico de respuestas
Comentario

Una pregunta difícil sobre la cirugía laparoscópica del colon. Una de las ventajas que ofrece la
laparoscopía respecto al abordaje convencional es una visión más amplia del campo quirúrgico,
sobre todo en pacientes obesos. Esto permite que la disección ganglionar que puede obtenerse no
tenga por qué ser más limitada que a través de una laparotomía (respuesta 2 falsa).(R2)

96. Mujer de 55 años, fumadora desde hace 30, a la que se le descubre, en su revisión
anual, un nódulo de 1.5 cm duro, indoloro y fijo en el cuadrante superior externo de la
mama izquierda. En la exploración física no se palpan adenopatías axilares ni
supraclaviculares. La mastografía muestra la existencia de un nódulo con características
de malignidad. Se realiza una biopsia intraoperatoria que se informa como carcinoma
ductal infiltrante con bordes de resección libres de tumor. La actitud terapéutica más
adecuada es:

1. 1. Mastectomía radical, ya que tiene factores de mal pronóstico como es la edad.


2. 2. Tumorectomía + linfadenectomía + radioterapia postoperatoria en el lecho quirúrgico.
3. 3. Mastectomía subcutánea + linfadenectomía + letrozol.
4. 4. Quimioterapia + radioterapia + hormonoterapia postoperatoria (tamoxifeno).
Gráfico de respuestas
Comentario

Esta pregunta es fácil si recordamos el algoritmo de tratamiento del cáncer de mama.

Cuando una mujer presenta un nódulo palpable en la mama, si además la mastografía tiene signos
sospechosos de malignidad, debemos realizar una biopsia de la tumoración, ya sea en consulta
con una PAAF, o en quirófano mediante una biopsia diferida. Como el diagnóstico es de un

 
 
 
 
carcinoma infiltrante, es obligado hacer linfadenectomía axilar ipsilateral. En este caso, al ser una
tumoración pequeña y la biopsia incluye toda la pieza tumoral con márgenes quirúrgicos libres, no
es necesario realizar mastectomía, pero sí será obligado añadir radioterapia en el lecho quirúrgico
de la mama (ya que la tumorectomía es una cirugía conservadora), por lo que la respuesta correcta
es la 2.(R2)

97. A previously healthy 5-year-old girl with normal development is brought to the
doctor's office presenting with irritability and sleeping difficulty during the past two
weeks. Her mother has perceived a mucous and purulent secretion coming from her
genital area. The girl refers itchiness. Visual examination shows an erythematous perineal
area with vaginal discharge and a foreign body protruding from the vagina. Which of the
following is the first step to be carried out in this case?

1. 1. Manual extraction
2. 2. Notify child protective services for sexual abuse
3. 3. Blood extractions for serologic tests
4. 4. Saline irrigation on perineal area
Gráfico de respuestas
Comentario
Saline irrigation on perineal area. Before making misleading judgments about abuse, you must
confirm the finding. Foreign bodies in genitals are common in childhood. In any case, the first step
is to wash the area with saline, and make a visual examination of the area. Foreign bodies in the
vagina have an increased risk of infection. The extraction can be done manually but may require
the use of anesthesia or light sedation. There are various maneuvers useful for this purpose. A
continuous dialogue and communication with parents is necessary to clarify what happened.(R4)

98. Select the correct answer in relation to psoriasis:

The typical psoriatic lesions have diffuse borders and are composed by scaling and
1. 1.
erythematous areas.
No specific laboratory findings are seen in uncomplicated forms, although serum uric
2. 2.
acid levels may be elevated.
3. 3. Nail lesions are typical but not frequent.
The lesions are heterogeneous and may have a variety of size and shape. The most
4. 4.
frequent type are the polycyclic confluent plaques.
Gráfico de respuestas
Comentario

El paciente psoriásico tiende a la hiperuricemia, por la elevada tasa de recambio que se produce
en sus epitelios (respuesta 2 correcta). El resto de las opciones son falsas, por los motivos
siguientes.

R1. La placa psoriásica no es difusa, sino bien definida.

R3. La afectación ungueal es muy frecuente.

R4. Se trata de placas, que incluso pueden ser confluyentes, pero difícilmente encontraremos
formas tan caprichosas como contornos policíclicos.(R2)

99. Todos los siguientes datos son característicos de la ictericia causada por lactancia
materna EXCEPTO:

 
 
 
 
1. 1. Elevación significativa de la bilirrubina indirecta.
2. 2. Descenso rápido de la bilirrubina al suspender la lactancia.
3. 3. La ictericia al reanudar la lactancia después de una suspensión temporal es menor.
4. 4. Elevación significativa de la bilirrubina directa.
Gráfico de respuestas
Comentario

La ictericia por leche de madre o síndrome de Arias se da en 1/200 recién nacidos alimentados por
esta vía, debido a que la leche de madre contiene sustancias que inhiben la conjugación de la
bilirrubina. Provoca, por tanto, hiperbilirrubinemia indirecta. Su curso suele ser generalmente
benigno. Hace años se hacía una prueba diagnóstica consistente en interrumpir la lactancia
durante 1-2 días, con lo cual se conseguía un rápido descenso de la bilirrubina; al reintroducir la
lactancia, la cifra volvía a aumentar pero a menor nivel que el inicial. Hoy por hoy, simplemente se
recomienda a las madres que continúen dando el pecho a sus pequeños.(R4)

100. ¿Cuál de estas circunstancias es irrelevante en los antecedentes de una paciente


que acude al servicio de urgencias de maternidad con sospecha clínica de embarzo
ectópico?

1. 1. Esterilización tubárica.
2. 2. Embarazo ectópico previo.
3. 3. Portadora de DIU.
4. 4. Primigesta.
Gráfico de respuestas
Comentario

Entre los factores favorecedores de gestación ectópica tenemos: los antecedentes de gestación
ectópica, la cirugía tubárica previa, la enfermedad inflamatoria pélvica, el DIU (aunque
controvertido, disminuye los intrauterinos y aumenta relativamente los extrauterinos),
endometriosis, ligadura tubárica, infertilidad (posible obstrucción tubárica), técnicas de
reproducción asistida... No se ha relacionado con la paridad.(R4)

101. Señale la FALSA respecto a los efectos del derrame pleural en la fisiología
respiratoria:

1. 1. El derrame pleural produce una alteración ventilatoria restrictiva.


2. 2. En pacientes con patología pulmonar subyacente, las consecuencias serían más severas.
3. 3. Los derrames masivos pueden afectar a la función cardíaca al disminuir el gasto cardíaco.
El descenso del diafragma ocasiona un empeoramiento de la función de los músculos
4. 4.
inspiratorios.
Gráfico de respuestas
Comentario

Las preguntas sobre derrames pleurales en el examen MIR suelen ser sobre un tipo concreto de
derrame. Sin embargo, esta pregunta es interesante porque recoge un aspecto menos conocido:
sus implicaciones en la fisiología respiratoria. Cuando existe líquido en las pleuras, ocupa un
espacio que el pulmón no puede utilizar para expandirse, produciendo una alteración ventilatoria
restrictiva. Esto causa una disminución de la capacidad residual funcional y de la capacidad vital,
también por razones de ocupación de espacio, limitando la expansión pulmonar. La tercera opción
cae por su propio peso: ante un paciente cuya función pulmonar está inicialmente comprometida,
las consecuencias serán más severas. En caso de derrame masivo, el abundante líquido pleural

 
 
 
 
puede comprimir los vasos pulmonares, cava inferior y cava superior, de forma que disminuye el
retorno venoso a cavidades derechas. La opción claramente falsa es la 5. El descenso del
diafragma no empeora la función de la musculatura inspiratoria, sino que la mejora. Lo que haría
que empeorase sería su ascenso, ya que limitaría todavía más la expansión del pulmón.(R4)

102. Mujer de 27 años que padece una hepatitis crónica por virus de la hepatitis C,
conocida desde hace un año. Entre sus antecedentes destaca ex-ADVP y, desde hace
años, numerosos episodios de cefaleas que ceden con paracetamol. Actualmente está
recibiendo tratamiento con interferón. Entre las medidas y orientaciones que
recomendaría, se encuentran las siguientes, EXCEPTO:

1. 1. Administrar paracetamol en dosis inferiores a 4 g al día como analgésico.


2. 2. Si no hay complicaciones, una dieta normal.
3. 3. Estudiar las serologías virales a sus familiares.
4. 4. Advertir del elevado riesgo de infección fetal en caso de gestación.
Gráfico de respuestas
Comentario

El virus de la hepatitis C es un tema frecuente en el examen MIR; préstale especial atención


cuando estudies las hepatitis víricas. Analicemos las distintas respuestas:

- El paracetamol, cuando se administra en dosis superiores a 10 gramos diarios, es un fármaco


tóxico para el hígado. Sin embargo, en un paciente con una hepatopatía, la toxicidad puede
manifestarse con dosis inferiores, por lo que sería prudente limitar su uso en este caso.

- El interferón no debe administrarse durante el embarazo, puesto que es teratógeno.

- La dieta no tiene por qué diferir de la de una persona sana, salvo que aparezcan complicaciones
(por ejemplo, encefalopatía hepática, en cuyo caso habría que limitar la ingesta proteica).

- Dada la presencia del VHC en esta paciente, no sería descabellado iniciar una búsqueda en las
personas de su entorno. Sin embargo, debes recordar que la transmisión intrafamiliar del VHC es
bastante baja.

- En caso de gestación, la probabilidad de afectación fetal es muy escasa, por lo que la respuesta
falsa es la 5.(R4)

103. La fase 3 de la curva del potencial de acción transmembrana en las células


musculares cardíacas viene determinada por:

1. 1. Entrada de iones calcio a las células y salida de iones potasio al exterior.


2. 2. Entrada de potasio a la célula e impermeabilización para los iones sodio y calcio.
3. 3. Entrada de sodio a la célula y salida de iones cloro y potasio.
4. 4. Intercambio activo sodio/potasio a través de la membrana celular.
Gráfico de respuestas
Comentario
Pregunta relativamente fácil, hay que conocer las distintas fases del potencial de acción de las
células miocárdicas. En la fase 0 se produce entrada de Na por los canales rápidos con lo que se
despolariza la célula. Durante las fases 1 y 2 o meseta hay salida de K y entrada de Ca con lo que
se mantiene el potencial de membrana durante un tiempo ligeramente positivo. La fase 3 o
repolarización está producida por la salida de K y va restableciendo el potencial de membrana en

 
 
 
 
reposo, de unos - 90mV. En la fase 4 la célula recupera el equilibrio iónico a ambos lados de la
membrana gracias a la bomba Na/K. Luego la opción correcta es la 1.(R1)

104. Uno de los siguientes datos NO es propio de un RN normal:

1. 1. Hematocrito 60%.
2. 2. Perlas de Ebstein en mucosa bucal.
3. 3. Reflujo GE postprandial.
4. 4. Ictericia en las primeras 24 horas tras el parto
Gráfico de respuestas
Comentario
La presencia de ictericia en las primeras 24 horas de vida es siempre patológica, y has de
descartar que el niño presente una de las cuatro patologías siguientes: hemólisis, sepsis, infección
connatal o hemorragia interna. El eritema tóxico, a pesar de su alarmante nombre, no tiene
significado patológico. Se admite como hematocrito normal hasta 65 %. Las perlas de Ebstein
también son benignas. El RGE es fisiológico es fisiológico hasta los dos años de vida.(R4)

105. A 10-day-old full-term newborn is brought to the emergency department presenting


with tendency to fall asleep and refusal to breastfeeding. The child had an uncomplicated
delivery and the mother was healthy. Physical examination shows ambiguous genitalia
with partial fusion of the labioscrotal folds but with no palpable testes. Vital signs are 50
respirations per minute, hear rate 130 beats per minute, blood pressure 70/50 mmHg and
the fontanelles seem sunken. Blood Laboratory test results are: Sodium 125 mEq/L,
Potassium 7 mEq/L, BUN 32 mg/dL, Creatinine 0.6 mg/dL. Karyotype is 46 XX. Which of
the following hormones would probably be increased in this clinical picture?

1. 1. Thyroid stimulating Hormone


2. 2. 17 Hydroxyprogesterone
3. 3. Insulin
4. 4. Glucagon
Gráfico de respuestas
Comentario
17-Hydroxy progesterone. The child in this case suffers from congenital adrenal hyperplasia, a
congenital autosomal recessive disease. At birth, the female child has ambiguous genitalia due to
the exposure to high levels of androgens during pregnancy. Male newborns show no abnormal
signs. Laboratory test results are very suggestive of this condition. The lack of glucocorticoid and
mineralocorticoid action produce hyponatremia with hyperkalemia. The most common enzyme
deficit is 21-hydroxylase and very high concentrations of 17-hydroxyprogesterone will be
observed.(R2)

106. La policitemia vera NO se asocia con:

1. 1. Trombocitosis.
2. 2. Niveles elevados de B12.
3. 3. Niveles elevados de eritropoyetina.
4. 4. Esplenomegalia.
Gráfico de respuestas
Comentario

Clásicamente, la policitemia vera se consideró independiente de la secreción de eritropoyetina,


estando aumentada la producción de hematíes de forma primaria. No obstante, hoy día se sabe

 
 
 
 
que, en este proceso, lo que en realidad ocurre es un exceso de sensibilidad de los precursores
eritropoyéticos a mínimas cantidades de eritropoyetina. En cualquier caso, en esta entidad es
fundamental un déficit en la secreción de esta sustancia, al contrario de lo que sucede en las
policitemias secundarias (hipoxia, tumores productores de EPO, etc.). Por lo tanto, es la respuesta
4 la que debe considerarse válida.

El resto de las opciones hacen referencia a datos muy conocidos: alteración de las otras series en
el hemograma (leucocitosis, trombocitosis), aumento de los niveles de vitamina B12 y, típicamente,
esplenomegalia en la exploración física, como también sucede en otros síndromes
mieloproliferativos. A este respecto, merece la pena que recuerdes un detalle: no existe
esplenomegalia en la policitemia secundaria a la hipoxia, dato que puede ayudarte frente a un caso
clínico donde te pidan un diagnóstico diferencial.(R3)

107. Usted ve en consulta por primera vez a un paciente de 55 años, diagnosticado de


enfermedad pulmonar obstructiva crónica (EPOC). Desde los 15 años había fumado un
promedio de 20 cigarrillos al día (40 paquetes-año). El síntoma fundamental es la tos, con
expectoración abundante, especialmente por las mañanas. Esto lleva ocurriéndole varios
años y, según dice, raro es el día que no expectora al despertarse. En la exploración física
destacan abundantes roncus en la auscultación y edemas hasta los tobillos. ¿Cuál de las
siguientes actuaciones considera FALSA en este paciente?:

1. 1. Es probable que exista cor pulmonale crónico.


2. 2. En este paciente no son aconsejables los diuréticos.
Si el paciente tuviera enfisema en vez de bronquitis crónica, la auscultación sería
3. 3.
diferente.
4. 4. En el hemograma no sería sorprendente encontrar un aumento del valor HCTO.
Gráfico de respuestas
Comentario

Aunque el uso de diuréticos no cambiará el pronóstico del paciente a largo plazo, en este paciente
sí que tendrían su utilidad para el tratamiento de los edemas, por lo que la respuesta 3 es falsa.
Como sabes, este grupo de fármacos se utiliza mucho en la insuficiencia cardíaca, aliviando los
edemas cuando es derecha, y la disnea si es izquierda.(R2)

108. Los episodios recidivantes de enrojecimiento paroxístico de cara, cuello y parte


superior de tronco son característicos de:

1. 1. Síndrome carcinoide.
2. 2. Amiloidosis.
3. 3. Urticaria colinérgica.
4. 4. Angioedema hereditario.
Gráfico de respuestas
Comentario

Las dermatosis paraneoplásicas son poco preguntadas en el ENARM, pero esta pregunta es muy
fácil y no debería haberte planteado dificultad alguna.

El síndrome carcinoide es un cuadro que aparece en relación con neoplasias neuroendocrinas,


procedentes de las células del sistema APUD. El tumor carcinoide más frecuente es el de la región
ileocecal (90%). Son tumores secretores de aminas vasoactivas, como serotonina, histamina,

 
 
 
 
calicreína… La clínica del tumor depende de las sustancias que secrete, pero lo más característico
es la aparición de episodios paroxísticos de rubor facial (flushing), diarrea, dolor abdominal,
broncoespasmo, taquicardia e hipotensión. En la piel, es posible observar una dermatosis
pelagroide debido al consumo de triptófano para la síntesis de serotonina, lo que dificulta la
síntesis de niacina. El diagnóstico de estos tumores se realiza por la clínica y mediante la
detección en orina del metabolito de degradación de la serotonina: ácido
5- hidroxi- indol- acético.(R1)

109. Acude a urgencias una paciente de 55 años entre cuyos antecedentes de interés
destaca: IMC=30; intolerancia a los hidratos de carbono en tratamiento con dieta y
apendicetomía. Antecedentes gineco-obstétricos: 2 embarazos, 1 parto y 1 aborto. Última
regla hace 4 meses. Consulta por sangrado genital escaso de 4 días de evolución que
actualmente ha cedido. En la exploración ginecológica no se aprecian hallazgos
patológicos y el USG muestra los hallazgos que se muestran en la imagen. ¿Cuál de las
siguientes opciones es la correcta?:

Comenzar tratamiento con análogos de la GnRH y posteriormente realizar una biopsia


1. 1.
con cánula de Cornier.
Comenzar tratamiento con danazol oral y posteriormente realizar histerectomía con doble
2. 2.
anexectomía.
Comenzar tratamiento con progestágenos y realizar histerectomía total simple
3. 3.
(respetando anejos).
4. 4. Realizar histeroscopía diagnóstica y toma de biopsias.
Gráfico de respuestas
Comentario

Se debe estar alerta ante toda hemorragia genital ya que puede ser el síntoma de de un tumor de
origen ginecológico. No se debe tratar hasta llegar a un diagnóstico. La visualización de la cavidad
endometrial mediante histeroscopía permite obtener muestras de biopsias dirigidas con lo que se
convierte en la prueba diagnóstica de elección en mujeres peri y post menopáusicas con
metrorragia para descartar el cáncer de endometrio y la hiperplasia endometrial.(R4)

110. ¿En cuál de las siguientes enfermedades NO esperaría encontrar hipoalbuminemia?:

1. 1. Enfermedad de Whipple.
2. 2. Enfermedad de Crohn.
3. 3. Déficit de lactasa.
4. 4. Esprue tropical.
Gráfico de respuestas
Comentario
Tanto en la enfermedad de Whipple, como en la enfermedad de Crohn, en la celíaca o el esprúe
tropical, hay o puede haber clínica de malabsorción de cualquier nutriente (incluyendo las
proteínas, para aportar albúmina), porque lo que está dañado es la propia pared intestinal. Sin
embergo, en el déficit de lactasa, únicamente se afecta el metabolismo de un solo nutriente, la
lactosa, impidiendo su degradación a glucosa y galactosa, para absorberse; y provocando, por
tanto, diarrea osmótica, pero no hipoalbuminemia.(R3)

111. Una de las siguientes afirmaciones sobre la función suprarrenal de la mujer


embarazada es FALSA. ¿Cuál?:

1. 1. Existe un aumento importante de las cifras de cortisol en plasma.

 
 
 
 
2. 2. La transcortina se encuentra disminuida durante el embarazo.
3. 3. Está elevada la secreción de mineralcorticoides.
4. 4. Está ligeramente incrementada la producción de androstenodiona.
Gráfico de respuestas
Comentario

Una pregunta complicada sobre los cambios hormonales en la gestante.

Durante el embarazo, se produce un aumento del nivel de cortisol, por una mayor síntesis y una
menor eliminación de esta hormona. Por otra parte, aumentan los niveles de renina y angiotensina,
con lo que también van en aumento los niveles de aldosterona.

En el embarazo, existe un incremento de las globulinas transportadoras de hormonas sexuales


(SHBG), lo que se traduce en una mayor síntesis de testosterona y androstendiona. Al igual que la
SHBG, está aumentado el nivel de transcortina, que es la hormona encargada del transporte del
cortisol, con lo que la respuesta falsa es la 2.(R2)

112. Un paciente de 62 años acude a su consulta porque ha notado últimamente


disminución de la potencia del chorro de la orina con goteo postmiccional y para el inicio
de la micción. Refiere nicturia de dos veces. A este paciente usted le realizará todo lo que
sigue excepto:

1. 1. Ecografía abdominal.
2. 2. Flujometría.
3. 3. IPSS.
4. 4. Ecografía transrectal.
Gráfico de respuestas
Comentario
Cuando evaluamos a un paciente con una posible HBP no está indicada la realización de una
biopsia transrectal, que tiene su utilidad en guiar las BTRE y valorar la estadificación local de ca de
próstata. Para valorar el volumen prostático en un paciente con HBP será suficiente la realización
de una ecografía abdominal, que nos informará además de las características de riñones y vejiga.
Además será recomendable la realización de una flujometría y de un cuestionario IPSS para
gradación de los síntomas, así como un tacto rectal para valoración subjetiva del tamaño prostático
y descartar la presencia de un ca de próstata.(R4)

113. A 14-year-old boy goes to the emergency room complaining of severe sore throat,
fever and swollen cervical lymph nodes for the past two weeks. His past medical history
is unremarkable other than asthma and hay fever. He denies smoking cigarettes and
drinking alcohol. His mother reports that for the last two months he has been running
away from home at night and going out with his friends. Vital signs are within normal
limits. Physical examination reveals cervical lymphadenopathy, splenomegaly and
swollen tonsils with palatal petechiae. CBC shows lymphocyte-predominant
leukocytosis. Which of the following tests will help you confirm the suspected diagnosis?

1. 1. Tzanck stain
2. 2. Ziehl Neelsen
3. 3. Mononuclear spot test (Paul Bunell Test)
4. 4. Detection of Legionella antigen in urine
Gráfico de respuestas
Comentario

 
 
 
 
Mononuclear spot test (Paul Bunell Test). Infectious mononucleosis (colloquially known as the
kissing disease from its transmission by saliva) is an infectious, widespread viral disease most
commonly caused by the Epstein–Barr virus (EBV). In adolescents and young adults, symptoms are
thought to appear around 4–6 weeks after initial infection. Onset is often slow, though it can be
abrupt. The disease usually presents with the characteristic triad: FEVER + SORE THROAT +
SWOLLEN LYMPH NODES. Splenomegaly is common in the second and third weeks.(R3)

114. El marcador morfológico de progresión de una hepatitis crónica es:

1. 1. La hepatitis de interfase.
2. 2. La esteatosis.
3. 3. La densidad del infiltrado inflamatorio portal.
4. 4. La destrucción ductal.
Gráfico de respuestas
Comentario

La clasificación anatomopatológica de las hepatopatías crónicas es un aspecto importante y


sencillo que nos permitirá conocer el manejo y evolución de las hepatitis.

Si bien la hepatitis se considera crónica a partir de los seis meses de inflamación, el diagnóstico de
certeza es anatomopatológico. Suelen describirse tres fases en la progresión de hepatitis aguda a
crónica, siendo la primera la hepatitis crónica persistente o de interfase, en la que observamos
infiltrado mononuclear portal, permaneciendo intactos el lobulillo y la membrana limitante del
espacio porta; datos histopatológicos que caracterizan la hepatitis crónica lobulillar y la activa,
respectivamente, que implican peor pronóstico.(R1)

Histopatología de la hepatitis

115. Se denomina ATELIA a:

1. 1. Ausencia completa de mama


2. 2. Ausencia de pezón y areola
3. 3. Falta de pigmentación del pezón y areola
4. 4. Ninguna de las anteriores

 
 
 
 
Gráfico de respuestas
Comentario

Ausencia de pezón. Es una anomalía rara, que puede asociarse a la amastia (ausencia de mama)
y, excepcionalmente, es una malformación congénita. En los casos de atelia asociados con
ausencia de tejido mamario, el músculo pectoral puede estar ausente. Precisa un tratamiento
quirúrgico similar al realizado en la reconstrucción del pezón en cirugía posmastectomía.(R2)

116. A 45-year-old woman is referred to a gynecologist because of an 8-month history of


frequent vaginal bleeding for a few days every 3 weeks. She says that her menses
occurred regularly before this episode. Her past medical history is unremarkable. She has
one male partner, with whom she uses condom inconsistently. Her family history is
notable for a mother who died of breast cancer and a grandfather who died of colon
cancer. Physical examination shows no abnormalities. Test of the stool for occult blood
is negative. Which of the following is the most advisable next step in management?

1. 1. Endometrial biopsy.
2. 2. Colposcopy.
3. 3. Abdomen and pelvis CT scan.
4. 4. Cystoscopy.

(R1)

117. Un niño de 7 años de edad con atresia de vías biliares que fue intervenido con 3
meses de vida mediante portoenterostomía de Kasai. Presenta hematomas por las
piernas y sequedad de piel con disminución de la agudeza visual. El diagnóstico más
probable será:

1. 1. Coagulación intravascular diseminada.


2. 2. Déficit de vitamina A y K
3. 3. Déficit de vitamina K y D.
4. 4. Hipoprotrombinemia aislada.
Gráfico de respuestas
Comentario

Es muy fecuente que pregunten deficits carenciales de cualquier forma. En la atresia de vías
biliares hasta que lleguemos al tratamiento definitivo que será el transplante hepático debemos
corregir las alteraciones asociadas como el déficit de vitaminas liposolubles debidas a
malabsorción. En este caso la clínica de sequedad de piel y alteraciones visuales nos harían
sospechar un déficit de vitamina A y los hematomas un déficit de vitamina K. El déficit de vitamina
E nos daría clínica de neuropatía periférica y ataxia y el déficit de vitamina D raquitismo.(R2)

118. Acude a consulta niña de 15 años de edad, estatura corta, cuello alado, infantilismo
sexual y coartación aórtica. El diagnóstico probable es:

1. 1. Un grupo no relacionado de problemas.


2. 2. Síndrome de Turner.
3. 3. Síndrome de Marfan.
4. 4. Síndrome de Ellis-Van-Creveld.
Gráfico de respuestas
Comentario
 
 
 
 
Pregunta sencilla sobre el síndrome de Turner, que no debe fallar. En las preguntas tipo caso
clínico debe de tomar los datos más improtantes que relatan: niña, coartación de aorta, talla baja,
pterigium coli. Al darle estos datos inmediatamente debe pensar en Síndrome de Turner y se
buscará la monosomía 45X0 en el cariotipo. Respuesta 2 correcta.(R2)

119. A 3-month-old infant is brought to the pediatrician’s office for a routine check-up. He
presents with increased peripheral pulses and a continuous (systolic and diastolic)
murmur that radiates to the chest. He had a difficult delivery and presented respiratory
distress at birth, but was otherwise healthy. What is the most likely diagnosis?

1. 1. Patent ductus arteriosus


2. 2. Interventricular septal defect
3. 3. Tetralogy of Fallot
4. 4. Congenital lung hypoplasia
Gráfico de respuestas
Comentario
Patent ductus arteriosus. Patent ductus arteriosus is a congenital disorder where the ductus
arteriosus fails to close after birth. Early symptoms are uncommon, but in the first year of life
increased work of breathing and poor weight gain appear. With age, this condition may lead to
congestive heart failure if left uncorrected. Interventricular septal defect produces pulmonary
hypertension in the adult if the shunt is significative. Fallot's tetralogy is a cyanotic congenital
disorder, no cyanosis is described.(R1)

120. Niño de 2 meses, acude a urgencias traído por la madre por presentar llanto intenso
y “dificultad respiratoria” de 6 horas de evolución. Al examen: Afebril, abdomen
distendido, presencia de ruidos hidroaéreos en hemitórax izquierdo. En la radiografía de
tórax se observa la presencia de asas intestinales en campo pulmonar izquierdo. Su
sospecha diagnóstica es:

1. 1. Es necesario realizar un TC de tórax.


2. 2. Hernia umbilical incarcerada.
3. 3. Hernia diafragmática.
4. 4. Ninguna de las anteriores.
Gráfico de respuestas
Comentario

Pregunta que no debe fallar, ya que le haria perder puntos importantes en comparación de otros
aplicantes... El cuadro es muy claro: ruidos hidroaéreos en hemitórax izquierdo y en la radiografía
asa intestinal en pulmón izquierdo!!!, por lo que la respuesta es hernia diafragmática de
Bochdalek!!.(R3)

121. La causa más frecuente de convulsiones en el periodo neonatal es:

1. 1. Encefalopatía hipóxico-isquémica.
2. 2. Infecciones.
3. 3. Traumatismos durante el parto.
4. 4. Enfermedades maternas.
Gráfico de respuestas
Comentario

 
 
 
 
Debe conocer al menos la causa más frecuente para cada grupo de edad, que le señalamos en la
siguiente tabla.(R1)

 
 
 
 

122. Paciente femenino de 56 años


a la que se le detecta una tumoración en el cuadrante infero-externo de la mama derecha.
Se realiza una mamografía que se muestra en la imagen. ¿Qué tratamiento realizaría
usted?

1. 1. Biopsia intraoperatoria de la lesión y, si es positiva, tumorectomía y ganglio centinela.


2. 2. Biopsia intraoperatoria de la lesión y si es positiva tumorectomía y radioterapia.
Biopsia intraoperatoria de la lesión y si es positiva mastectomía con reconstrucción
3. 3.
mamaria inmediata.
4. 4. Biopsia intraoperatoria de la lesión y si es positiva mastectomía y radioterapia.
Gráfico de respuestas
Comentario

En la ecografía, podemos apreciar una tumoración irregular, con bordes mal definidos, en una
paciente de más de 50 años, con lo que el diagnóstico más probable sería el de cáncer de mama.
Aparentemente, se trata de un tumor de pequeño tamaño, por lo que no habrá que hacer (al menos
de inicio) una mastectomía. Lo que sí debemos realizar es una biopsia intraoperatoria, que nos
confirmará la malignidad histológica, y en caso de ser positiva procederíamos al estudio del ganglio
centinela.

Recuerda que, en el pronóstico del cáncer de mama sin metástasis a distancia, el factor pronóstico
más importante es el número de ganglios afectados a nivel axilar, de ahí la importancia del estudio
del ganglio centinela. En cuanto a la radioterapia, se realiza cuando se trata de un tumor de
pequeño tamaño, como parece ser el caso, pero no durante el propio acto quirúrgico, sino de forma

 
 
 
 
diferida. De este modo (tumorectomía + radioterapia) se alcanzan resultados similares a las
mastectomías clásicas, en términos de supervivencia, sin realizar intervenciones tan
mutilantes.(R1)

123. En la paciente del caso anterior el resultado definitivo anatomopatológico informa


de carcinoma ductal infiltrante, de 34 mm de diámetro máximo, indiferenciado, con
importante invasión vascular, receptores hormonales negativos, Ki67 30% y Her-2-neu
negativo. Señale cuál sería el tratamiento complementario más adecuado:

1. 1. Radioterapia.
2. 2. Radioterapia y quimioterapia.
3. 3. Radioterapia, quimioterapia y hormonoterapia.
4. 4. Quimioterapia.
Gráfico de respuestas
Comentario

Confirmado que se trata de un tumor pequeño (34 mm de diámetro máximo), debemos administrar
radioterapia. Aunque no nos dicen el estado de los ganglios, sí que nos mencionan varios factores
de mal pronóstico: importante invasión vascular y carácter indiferenciado, por lo que sería
aconsejable utilizar quimioterapia. Desde el punto de vista de los receptores hormonales, al ser
negativos, no precisamos administrar tamoxifeno, inhibidores de la aromatasa ni otras formas de
hormonoterapia. Por ello, la respuesta correcta es la 2.(R2)

124. Señale cuál de los siguientes factores NO se considera de mal pronóstico en el


cáncer de mama de la paciente del caso anterior:

1. 1. Receptores hormonales negativos.


2. 2. Her-2-neu negativo.
3. 3. Índice de proliferación Ki67 alto.
4. 4. Invasión vascular importante.
Gráfico de respuestas
Comentario

La presencia de Her- 2- neu se considera un factor de mal pronóstico, este concepto hasta ahora
no se ha preguntado en el nacional, pero es importante y debes saberlo. Recuerda que, para las
pacientes en las que es positivo, existe un tratamiento relativamente reciente, el trastuzumab, un
anticuerpo monoclonal anti Her- 2- neu.(R2)

125. ¿Qué es FALSO en el síndrome de Duchenne-Erb?

1. 1. Es causa de síndrome de Claude-Bernard-Horner.


2. 2. Causa parálisis de los músculos abductores y rotadores externos del hombro.
3. 3. Los reflejos bicipital y estilorradial están abolidos.
4. 4. Su origen son los traumatismos y parálisis obstétricas.
Gráfico de respuestas
Comentario

Pregunta de dificultad baja sobre un tema que también se estudia en pediatría. El síndrome de
Duchenne-Erb es una parálisis braquial de las raíces superiores (C5 y C6) que puede comprometer
a C4 y por tanto dar una parálisis frénica. El síndrome de Horner se asocia a una parálisis de las
raíces C7 y C8 cuando se afecta también T1, en cuyo caso se hablaría de la parálisis de Klumpke.

 
 
 
 
Como dice la opción 4, la causa más frecuente son los traumatismos obstétricos. Recuerda las
manifestaciones mencionadas, porque te ayudara a responder varias preguntas en forma de caso
clínico.

En el caso de la parálisis de Duchenne- Erb, el brazo se encuentra en adducción y rotación interna.


El antebrazo está en pronación, adoptando la postura “en propina de maitre”. Recuerda que el
reflejo estilorradial depende de C6 y el reflejo bicipital depende de C5.(R1)

126. A 3-month-old infant is brought to the doctor's office by her worried mother who has
perceived a loss of appetite and fatigability. The child had a preterm delivery at 28 weeks'
gestation. Laboratory blood test results are: HB 7 g/dL, Leukocytes 6000/mm3, Platelets
160000/mm3, Reticulocytes 2%. What is the most likely explanation for this child's
anemia?

1. 1. Anemia of prematurity
2. 2. Blackfan-Diamond anemia
3. 3. Sickle cell anemia
4. 4. Beta thalassemia minor
Gráfico de respuestas
Comentario
Anemia of prematurity. The history of preterm birth is essential to make the diagnosis. Prematurity
anemia is multifactorial: repeated blood sampling and reduced erythropoiesis with extremely low
serum levels of erythropoietin (EPO) are major determining factors.(R1)

127. Respecto al divertículo de Meckel, señale la opción verdadera:

1. 1. Es una anomalía congénita poco frecuente.


2. 2. Puede existir tejido pancreático ectópico a nivel del divertículo.
3. 3. Si se aloja en una hernia inguinal directa se denominada de Littré.
4. 4. Requiere siempre cirugía.
Gráfico de respuestas
Comentario
El divertículo de Meckel es un resto del conducto onfalomesentérico y es la anomalía congénita
más frecuente en el tubo digestivo, puede existir tejido gástrico o pancreático ectópico a nivel del
divertículo. La hernia de Littré se produce cuando se aloja en una hernia inguinal indirecta y sólo
requiere cirugía en los casos en que es sintomático.(R2)

128. ¿Cual de estos fenómenos indica peor pronóstico en un paciente enfisematoso?

1. 1. Disnea grave en reposo.


2. 2. Episodios de insuficiencia aguda hipercárbica.
3. 3. PaO2 de 60 mmHg.
4. 4. Afectación grave anatomopatológica (cuantificada con TAC).
Gráfico de respuestas
Comentario
Pregunta difícil sobre los factores pronósticos en el paciente enfisematoso, pero que debes saber
manejar porque muchos de los mismos se corresponden con factores pronósticos generales de
cualquier enfermedad cardiorrespiratoria. En general todas los afirmaciones anteriores indican mal
pronóstico pero debemos ver cuál es la peor, y para ello podemos seguir la secuencia
fisiopatológica del enfisema. La afectación anatomopatológica vista por un TAC no suele guardar

 
 
 
 
relación estrecha con la clínica, por lo que no sería el criterio de peor pronóstico. Cuando baja la
PaO2 en el enfisema, el paciente experimenta un agravamiento de su disnea habitual para poder
compensarlo. Si la situación no mejora, se reclutan los músculos respiratorios accesorios para
intentar resolver la situación. En un primer momento puede ser suficiente, pero si no se corrige la
situación, al final se produce una claudicación de todos los mecanismos compensadores y el
paciente comienza a hipoventilar, con el consecuente aumento de CO2, siendo éste pues, el
criterio de peor pronóstico de los que se nos dan. Recuerda que en general cualquier paciente con
hipercapnia es un paciente grave y que debe ser tratado urgentemente pues se encuentra en una
situación potencialmente mortal.(R2)

129. La epifisiólisis traumática tiene como localización más frecuente la:

1. 1. Epífisis distal de fémur.


2. 2. Epífisis distal de radio.
3. 3. Epífisis distal de tibia.
4. 4. Epífisis proximal de humero.
Gráfico de respuestas
Comentario
La epifisiolisis es la fratura del cartílago de crecimiento, y por tanto es una lesión que se presenta
exclusivamente en niños durante el periodo de crecimiento; la mas frecuente es la de radio distal,
generalmente tras caída y apoyo del talón de la mano, siendo también lo mas frecuente el tipo II en
esa localización, en la que el plano de clivaje se produce a nivel de la capa de células hipertrófica,
sin llegar a lesionar la capa hipertrófica, y por tanto es infrecuente las alteraciones del crecimiento,
a diferencia de lo que ocurre con las de tipo III y IV.(R2)

130. El tumor abdominal más frecuente, de los abajo citados, en la infancia es:

1. 1. Teratoma retroperitoneal.
2. 2. Neuroblastoma.
3. 3. Estenosis pilórica.
4. 4. Tumor ovárico.
Gráfico de respuestas
Comentario
De todos los tumores abdominales, el más frecuente en los niños es el nefroblastoma o tumor de
Wilms. Sin embargo, no está entre las opciones por lo que hay que saber que, después de aquél,
el siguiente en frecuencia es el neuroblastoma. Recuerda que los neuroblastomas, aunque la
mayoría son abdominales (70%), pueden localizarse en distintos lugares (mediastino, pelvis, etc.) y
que son los tumores sólidos extracraneales más frecuentes.(R2)

131. Un niño de 5 años, previamente sano, debuta bruscamente con palidez, astenia y
anorexia; se le practica una BH y se evidencia anemia, leucopenia y trombocitopenia. En
la exploración se aprecia hepatoesplenomegalia. ¿Qué diagnóstico le parece más
probable?:

1. 1. Anemia ferropénica.
2. 2. Linfoma no Hodgkin.
3. 3. Linfoma de Hodgkin.
4. 4. Leucemia aguda.
Gráfico de respuestas
Comentario

 
 
 
 
En caso de un niño con pancitopenia y visceromegalias, se debe descartar leucemias agudas, por
lo que la respuesta correcta es la 4. La anemia ferropénica no causaria pancitopenia, únicamente
anemia microcítia hipocrómica y en ocasiones trombocitosis reactiva. En cuanto a los linfomas
generalmente se presenta con adenomegalias, sin tanta afección a la BH.(R4)

132. Niño de 10 años, con abdomen péndulo, peso sobre el percentil 97, talla en percentil
75, genitales externos difíciles de ver en posición erguida. El diagnóstico es:

1. 1. Déficit de hormona del crecimiento.


2. 2. Hipogonadismo.
3. 3. Hipopituitarismo.
4. 4. Obesidad.
Gráfico de respuestas
Comentario

Claramente nos están describiendo un niño obeso, con un peso por encima del percentil 97,
abdomen péndulo. Nos intentan despistar diciendo que no se ven los genitales, pero es a causa de
la barriga que se los tapa, no de que haya un hipogonadismo.(R4)

133. Usted encuentra 5 miomas en una mujer nulípara de 27 años de edad, que acude por
dismenorrea, 3 de ellos son subserosos y 2 intramurales, siendo el mayor de 5cm de
diámetro. ¿Qué le indicaría?

1. 1. Uso de agonistas de GnRH.


2. 2. Embolización de arteria uterina.
3. 3. Miomectomía por laparotomía.
4. 4. Resección histeroscópica.
Gráfico de respuestas
Comentario

Ante una miomatosis uterina sintomática en pacientes con paridad insatisfecha, las guías
recomiendan miomectomía por laparotomía como la técnica de elección. No se recomienda el uso
de agonistas de GnRH por el riesgo de hipoestrogenismo. Respuesta 3 correcta.(R3)

134. La oclusión de la arteria cerebral media izquierda en un sujeto diestro produce


generalmente:

1. 1. Hemiparesia y hemihipoestesia derechas de predominio crural.


2. 2. Hemiparesia y hemihipoestesia derechas de predominio crural y disfasia.
3. 3. Hemiparesia y hemihipoestesia derechas de predominio faciobraquial y disfasia.
4. 4. Hemiparesia derecha de predominio crural y disfasia, sin trastornos sensitivos.
Gráfico de respuestas
Comentario
Esta es una pregunta muy importante para el MIR, pues la oclusión de la arteria cerebral media da
lugar al síndrome vascular más frecuente. Éste cursa con 1) hemiparesia y hemihipoestesia
contralaterales de predominio faciobraquial 2) hemianopsia homónima contralateral, afasia de
Brocca, Wernicke o global, dependiendo de la localización y la extensión (en lesiones del
hemisferio dominante) 3) puede haber también asomatognosia, anosognosia y desorientación
espacial en lesiones del hemisferio no dominante.(R3)

 
 
 
 
135. Las fracturas de Looser-Milkman son un dato típico que aparece en ocasiones en
una de las siguientes enfermedades:

1. 1. Osteomalacia.
2. 2. Osteoporosis.
3. 3. Displasia fibrosa.
4. 4. Osteogénesis imperfecta.
Gráfico de respuestas
Comentario

Esta pregunta te servirá para introducirte en el tema de la osteomalacia, enfermedad que se ha


preguntado en el ENARM pocas veces, pero siempre igual. La asociación de pseudofracturas de
Looser- Milkman con la osteopenia es una de ellas. Son bandas radiotransparentes que cruzan
perpendicularmente la cortical. Existen verdaderas fracturas, por supuesto, como las de cuello de
fémur. Recuerda que es un trastorno por déficit de vitamina D casi siempre, y que genera
deformidades óseas con dolor a la palpación, fracturas patológicas a veces y debilidad muscular.
La analítica refleja el déficit de vitamina D y el hiperparatiroidismo compensatorio: calcemia normal
o baja, hipofosfatemia y fosfatasa alcalina elevada.(R1)

136. Mujer de 42 años,


con antecedentes quirúrgicos de 2 cesáreas, que acude a urgencias refiriendo intenso
dolor abdominal de reciente aparición, no irradiado y de tipo continuo. En el
interrogatorio a la paciente, refiere leucorrea aumentada de días de evolución a la que no
había dado importancia, ligeramente maloliente. Presenta una temperatura de 38 grados
centígrados y está taquicárdica al ingreso. Se decide realizar una placa abdomino-pélvica
posteroanterior con el resultado que se muestra en la imagen. Usted sospecha como
primera causa del dolor:

1. 1. Enfermedad inflamatoria pélvica complicada con peritonitis aguda.


2. 2. Tuberculosis genital complicada con peritonitis aguda.
3. 3. Tuberculosis genital no complicada.
4. 4. Peritonitis aguda.
Gráfico de respuestas
Comentario
La radiografía de pelvis nos muestra un dispositivo intrauterino (DIU) migrado de la cavidad uterina,
lo que ha producido una perforación a nivel intestinal y secundariamente una peritonitis aguda, en
el contexto de una enfermedad inflamatoria pélvica. La paciente comenzó con una infección pélvica
localizada en el aparato genital, de ahí la leucorrea y los síntomas genitales, secundaria a ser
portadora de DIU, que se complica con la migración del mismo a la cavidad peritoneal.(R1)

 
 
 
 
137. Ante el diagnóstico que usted sospecha, decide realizar un hemograma completo
con coagulación, con los siguientes resultados: 23.450 leucocitos con el 93% de
neutrófilos. Plaquetas de 450.000 y hemoglobina 9 g/dl. La coagulación comienza a estar
alterada. En el periodo de espera de los resultados la paciente se hipotensa, con pérdida
del conocimiento. Usted decide:

1. 1. Llamar al intensivista para estabilizar a la paciente.


2. 2. Laparoscopía urgente.
3. 3. Laparotomía urgente.
4. 4. Repetir analítica en 2 horas.
Gráfico de respuestas
Comentario

La paciente que presenta una perforación a nivel peritoneal, comienza con signos y síntomas de
shock hipovolémico, secundario a la infección producida por el DIU en la cavidad peritoneal. No
podemos plantear una laparoscopía en una paciente que no se encuentra estable, para eliminar el
dispositivo y limpiar la cavidad. La laparotomía urgente debe de anteponerse a cualquier prueba de
imagen o complementaria, dada la inestabilidad de la mujer del caso clínico que nos proponen.(R3)

138. La metodología para el examen de mamas comprende los siguientes medidas,


excepto:

1. 1. Autoexploración mamaria.
2. 2. Biopsia y aspiración.
3. 3. Radiografía de tórax.
4. 4. Exploración mamaria.
Gráfico de respuestas
Comentario

Tema de suma importancia en el ENAM.

Si bien en la radiografía de tórax se pueden ver algunas alteraciones, no forma parte del screening
del cáncer de mama. Respuesta incorrecta 3.(R3)

139. ¿Cuál es la manifestación digestiva más frecuente de la esclerodermia?:

1. 1. Ulcera péptica.
2. 2. Disfagia.
3. 3. Divertículos del colon.
4. 4. Apendicitis aguda.
Gráfico de respuestas
Comentario
Esta pregunta te servirá de introducción al capítulo de la esclerodermia, que es bastante
preguntado. De las manifestaciones digestivas, la más preguntada y frecuente es la esofágica, por
lo que es la disfagia el síntoma más frecuente. Existe fibrosis de los 2/3 inferiores del esófago con
atrofia de la capa muscular, lo que dificulta una propulsión del alimento adecuada, y entonces
aparece la disfagia para sólidos. También existe hipomotilidad del esfínter esofágico inferior, y ello
puede provocar una

 
 
 
 
esofagitis por reflujo con todas sus consecuencias. Tanto en el intestino delgado como en el
grueso se evidencia también esa "deficiente motilidad", con íleo y sobrecrecimiento bacteriano en
el primero, y estreñimiento y diverticulosis en el segundo.(R2)

140. Gestante de 27 semanas acude a urgencias por fiebre de 38ºC, contracciones


uterinas dolorosas y pérdida de líquido desde hace varias horas. En el monitor fetal: 170
lpm y 2 contracciones/10 min. Por USG podemos constatar que el feto está en podálica y
que la longitud cervical es de 12 mm. Los exámenes de laboratorio muestran: leucocitos
= 17,000/ml con un 87% de neutrófilos y una proteína C reactiva = 85 UI/ml (0.0-5.0). ¿Cuál
sería la actitud obstétrica más adecuada a seguir?

Realizar un test de fibronectina, y si es positivo ingresar a la paciente y administrar


1. 1.
tocolisis con atosibán i.v.
Ingreso hospitalario para maduración pulmonar con corticoides, tocolisis intravenosa con
2. 2.
beta-miméticos y antibióticos i.v.
Ingreso hospitalario para maduración pulmonar con corticoides, tocolisis intravenosa con
3. 3.
atosibán y antibióticos i.v.
4. 4. Administración antibióticos i.v. y realizar una cesárea.
Gráfico de respuestas
Comentario

Se presenta un ejemplo claro de corioamnionitis (fiebre materna, taquicardia fetal, datos analíticos
claros de infección y un foco etiológico como es la rotura prematura de membranas) y su
diagnóstico obliga a finalizar la gestación de forma inmediata. En este caso en concreto además se
haría una cesárea porque el feto tiene una presentación podálica y es prematuro.(R4)

141. Paciente de 43 años que es traída por su familia al hospital por encontrarla confusa
y desorientada. Como antecedentes destaca un hábito enólico importante y mantenido
en el tiempo, y "algo de hígado" que la familia no sabe precisar. En la exploración destaca
la presencia de telangectasias malares y una circulación colateral abdominal prominente.
La auscultación cardiopulmonar es normal. En la palpación abdominal existe
esplenomegalia sin hepatomegalia, y ligera matidez en flancos que se desplaza con el
movimiento. Neurológicamente no se aprecia focalidad, pero está desorientada, agitada
y con tendencia al sueño. ¿Cuál de las siguientes medidas, diagnóstica o terapéutica,
usted NO cree indicada de forma inicial?

1. 1. Tiapride y loracepam i.v.


2. 2. EEG.
3. 3. Punción lumbar.
4. 4. Sodio, potasio, magnesio, calcio, glucemia sérica.
Gráfico de respuestas
Comentario

Se trata de una paciente alcohólica con hepatopatía crónica ya que presenta signos de
hipertensión portal (circulación colateral). Además presenta ascitis y encefalopatía. El diagnóstico
más probable dados los antecedentes es una encefalopatía hepática en una paciente con cirrosis
de etiología alcohólica. Una punción lumbar no estaría indicada de inicio, ya que no tiene fiebre y
es mucho más probable que se trate de encefalopatía hepática. De todas formas, esta pregunta
podría ser impugnada, ya que el loracepam es una benzodiacepina y no son aconsejables en este
tipo de pacientes, porque pueden contribuir a una mayor descompensación (respuesta 1).(R3)

 
 
 
 
142. Un paciente varón de 35 años consulta por cuadro de astenia generalizada y
poliartritis inflamatoria asimétrica en interfalángicas proximales y distales en manos.
Presenta llamativas lesiones ungueales así como cutáneas sugestivas de psoriasis
severa y extensa. Señale cuál de las siguientes determinaciones solicitaría en primer
lugar:

1. 1. HIV.
2. 2. Virus de la Hepatitis C.
3. 3. Ziehl-Nielsen.
4. 4. CMV.
Gráfico de respuestas
Comentario
Los pacientes con infección por VIH tiene una mayor tendencia a presentar artritis reactiva, que de
hecho constituye la causa más frecuente de artritis en estos pacientes). Lo más llamativo de estos
pacientes es que pueden presentar manifestaciones mucocutáneas muy espectaculares. No
obstante en el paciente que nos describen la afectación poliarticular con predominio en las manos
nos debe hacer inclinar el diagnóstico a la artropatía psoriasica. Los pacientes con infección por
VIH muestran también formas de artropatía psoriasica muy agresivas desde el punto de vista
articular y cutáneo por lo que en un paciente que presente, como el del caso estas características
estará indicado confirmar o descartar la infección por VIH.(R1)

143. Paciente de 27 años que refiere disuria, leucorrea mucopurulenta hemorrágica, ¿qué
sospecha usted?

1. 1. Sífilis.
2. 2. Gonorrea.
3. 3. Infección por C. trachomatis.
4. 4. Infección por T. vaginalis.
Gráfico de respuestas
Comentario

Una pregunta sencilla. Como sabes, las causas más frecuentes de cervicitis son Chlamydia
trachomatis y Neisseria gonorrhoeae. En ambos casos, la secreción es purulenta, aunque ésta es
más abundante cuando se trata de un gonococo. Sin embargo, en este caso nos ofrecen un dato
extra que nos ayuda a orientar la etiología: el carácter hemorrágico de la secreción. Este dato
orienta mucho más a Chlamydia que a gonococo, por lo que la respuesta correcta es la 4.(R3)

144. Paciente femenino de 29 años de edad, sexualmente activa, con proceso febril y
dolor pélvico de 3 días de evolución. Última regla normal hace 3 días. Al examen:
movilización del cuello uterino doloroso, anexos engrosados. Gonadotrofina subunidad
beta: 5 mUI. El diagnóstico probable es:

1. 1. Enfermedad pélvica inflamatoria aguda


2. 2. Embarazo ectópico
3. 3. Apendicitis aguda
4. 4. Amenaza de aborto
Gráfico de respuestas
Comentario

 
 
 
 
Fiebre más dolor pélvico en paciente sexualmente activa siempre debe hacer pensar en EIP, y
mucho más ante test del embarazo negativo, debes darte cuenta que la cantidad de gonadotrofina
es muy baja, no porque te anexen un dato de laboratorio tiene porque ser positivo.(R1)

145. En una colecistectomia laparoscopica el cirujano identifica el triangulo de Calot


como referencia de:

1. 1. Conducto cístico.
2. 2. Arteria cística.
3. 3. Colédoco.
4. 4. Conducto hepatico
Gráfico de respuestas
Comentario

Pregunta difícil sobre la anatomía de la vía biliar. El triángulo de Calot es un plano quirúrgico
delimitado por el conducto cístico, la vía biliar principal y la cara inferior del hígado, que sirve para
delimitar la localización anatómica de la arteria cística.(R2)

146. ¿Cuál es la causa más común de pancreatitis en niños?:

1. 1. Infección VIH.
2. 2. Fibrosis quística.
3. 3. Parotiditis.
4. 4. Infección micótica.
Gráfico de respuestas
Comentario

Pregunta sencilla, a la que puede llegar por descarte. Todas podrían ser causa de pancreatitis,
pero le piden que elija la más frecuente. Obviamente de las 4 la más frecuente es la infección por
virus de parotiditis. Respuesta 3 correcta.

No olvide la causa más común en adultos en México es colelitiásica ha sido preguntada desde
hace muchos años en el ENARM.(R3)

147. La mejor actitud ante el diagnóstico por ultrasonido de un endometrioma ovárico es:

1. 1. Tratamiento con análogos de GnRH 6 meses.


2. 2. Laparotomía exploradora.
3. 3. Laparoscopía quirúrgica.
4. 4. Observación y repetir en 3 meses.
Gráfico de respuestas
Comentario

La mejor forma de tratar un endometrioma ovárico es extirpándolo por laparoscopía. Los análogos
de GnRH sólo proporcionan un alivio temporal.(R3)

148. Paciente de 55 años, diagnosticada de diabetes mellitus tipo 2 hace 10 años. Está en
tratamiento con insulina en dos dosis. Presenta retinopatía diabética proliferativa y
polineuropatía diabética, sin otros antecendentes ni otras complicaciones crónicas de su
diabetes. En la exploración física destaca un BMI de 38. Es remitida al hospital por mal

 
 
 
 
control glucémico crónico pese a aumento progresivo de dosis de insulina. ¿Qué actitud
es la más adecuada?:

1. 1. Sustituir la insulina por sulfonilureas.


2. 2. Sustituir la insulina por acarbosa.
Añadir metformina para mejorar la resistencia insulínica e intentar reducir la dosis de
3. 3.
insulina.
4. 4. Aceptar que un buen control metabólico es inalcanzable en este caso.
Gráfico de respuestas
Comentario
La obesidad casi mórbida de esta paciente produce una gran resistencia a la insulina y puede ser
la causa de la dificultad de alcanzar un control metabólico adecuado a pesar de ir subiendo las
dosis de insulina progresivamente. En este caso lo más adecuado es añadir metformina con el fin
de reducir la insulino- resistencia. A pesar de tener complicaciones microvasculares, no nos dice el
enunciado que tenga ningún motivo para que esté contraindicada (insuficiencia renal, hepatopatía,
insuficiencia cardiaca o respiratoria...). Reducir la resistencia a la insulina nos permitirá reducir las
dosis de insulina que, entre otras cosas contribuyen a aumentar el peso de la paciente.(R3)

149. Una mujer con ciclos menstruales de 32 días de duración, con unos niveles de
progesterona el día 22 de ciclo con resultado indetectable, ¿qué afirmación es la
CORRECTA?:

1. 1. Es un ciclo anormal, ya que es excesivamente largo.


2. 2. Es un ciclo ovulatorio porque tiene una duración normal.
3. 3. Es un ciclo anovulatorio.
4. 4. La cantidad de ácido siálico del moco cervical estará disminuida en la segunda fase.
Gráfico de respuestas
Comentario

Si no hay progesterona en la segunda fase del ciclo, es porque no ha habido ovulación. Recuerde
que el cuerpo lúteo es el órgano encargado de su producción.

La duración normal del ciclo oscila entre 21 y 35 días. Si no hay progesterona tampoco se
observarán los efectos derivados de la misma, como la respuesta 3.(R3)

150. Hombre de 22 años que acude por diarrea de 8


semanas de evolución, con moco pero sin sangre. Fiebre vespertina de 37,8º C y

 
 
 
 
disminución de 5 kg en los últimos 4 semanas. En los exámenes de laboratorio
destacaban unos leucocitos de 18,000 con 92% de neutrófilos, VSG 66 mm en la primera
hora y hemoglobina 10 g/dl. Los coprocultivos y parásitos fueron negativos. El recuento
de inmunoglobulinas, proteinograma y serología contra enfermedad celíaca fueron
normales. Se realizó una colonoscopia donde observó un colon izquierdo de aspecto
normal y la imagen adjunta en ciego. Acerca de la enfermedad que usted sospecha,
señale la afirmación CORRECTA:

1. 1. El recto suele encontrase afectado de forma casi constante.


2. 2. La ausencia de anticuerpos ASCA permite descartar el diagnóstico.
La afectación de la mucosa puede ocurrir en cualquier parte del aparato digestivo de
3. 3.
forma discontinua.
4. 4. El tabaco es factor protector de esta patología.
Gráfico de respuestas
Comentario

Una pregunta muy interesante sobre la enfermedad inflamatoria intestinal. Dada la ausencia de
afectación en colon izquierdo, con lesiones principalmente en la zona del ciego, el diagnóstico que
habría que sospechar es una enfermedad de Crohn. La imagen que nos muestra la pregunta
corresponde a sus típicas úlceras serpinginosas. Respecto a las opciones que nos presentan,
debemos decir:

R1: El recto se afecta de forma casi constante en la colitis ulcerosa, no en el Crohn.

R2: Los ASCAs suelen estar presentes, pero también pueden no estarlo.

R4: El tabaco es protector en la colitis ulcerosa. En la enfermedad de Crohn, es factor de


riesgo.(R3)

151. Tras el diagnóstico realizado, ¿qué tratamiento propondría en este paciente, de entre
los expuestos a continuación?:

1. 1. Mesalazina en enemas.
2. 2. Prednisona oral.
3. 3. Infliximab i.v.
4. 4. Ciclosporina i.v.
Gráfico de respuestas
Comentario

Dado que se trata de una enfermedad inflamatoria intestinal en brote agudo, el tratamiento de
elección serían los corticoides. En este caso, teniendo en cuenta la estabilidad del paciente, no
sería necesario pautar tratamiento intravenoso, al menos de inicio, por lo que bastaría comenzar
con prednisona oral, en dosis altas (0.5- 1 mg/kg/día), que habrá que descender de forma muy
paulatina para evitar el efecto rebote.(R2)

152. A 45-year-old male comes to your office due to pitting edema. His previous medical
history is significant for aspirative pneumonia 2 years ago. He has not been seen by a
physician since then. He takes no medications. He admits to drinking 6 glasses of whisky
per night. Physical examination reveals bilateral gynecomastia and spider angiomas on
his upper extremities. Percussion of the abdomen shows ascites. Which of the following

 
 
 
 
physical findings has the same pathogenesis as his gynecomastia?

1. 1. Spider angioma
2. 2. Ascites
3. 3. Asterixis
4. 4. Pedal edema
Gráfico de respuestas
Comentario
Spider angioma. Both spider angioma and gynecomastia are related to high estradiol levels. Spider
angiomas are due to failure of the sphincteric muscle surrounding a cutaneous arteriole. The
dilation, in turn, is caused by increased estrogen levels in the blood (many pregnant women, or
women using hormonal contraception, have spider angiomas, due to high estrogen levels in their
blood).(R1)

153. A 7-year-old girl previously diagnosed with Down's syndrome, with no other medical
history, is brought to consultation for presenting weakness and urinary incontinence de
novo. Physical examination reveals marked hyporeflexia and bilateral positive Babinsky
reflex. What is the most likely diagnosis?

1. 1. Spinal cord tumor


2. 2. Craniopharyngioma
3. 3. Atlantoaxial instability
4. 4. Hypothyroidism
Gráfico de respuestas
Comentario
Atlantoaxial instability. The child in this case presents symptoms of spinal cord injury. On the basis
of the personal medical history of Down’s syndrome, an atlantoaxial instability must be suspected
because this malformation is present in up to 15% of patients with such chromosomopathy. Spinal
cord tumors may induce similar symtoms but are less common than congenital disorders at this
age.(R3)

154. A 7-year-old child is brought to the emergency department presenting with low level
of consciousness after a head trauma during a bike accident. Her neurological status
improved after the accident but her Glasgow score is back down to 11/15 at her arrival.
Asymmetric pupils are the most remarkable finding in neurological examination. Cranial
CT scan shows a hyperdense collection with concave edges that shifts brain structures.
What should be done next?

1. 1. Emergency craniotomy for drainage


2. 2. Antibiotherapy
3. 3. Dexamethasone
4. 4. Cranial X ray examination
Gráfico de respuestas
Comentario
Emergency craniotomy for drainage. Epidural hematoma is a traumatic accumulation of blood
between the inner table of the skull and the stripped-off dural membrane. It often occurs after a high
intensity blow on the skull, such as a traffic accident. The typical presentation includes a lucid
interval between the initial trauma and subsequent neurological deterioration. The consequences
are dramatic if surgical decompression is not performed.(R1)

 
 
 
 

155   Un varón de 80
años acude a su consulta a revisión. Usted le diagnosticó hace 10 días una otitis
externa difusa y le pautó gotas tópicas de antibiótico y corticoides. Al realizarle la
anamnesis, el paciente sigue refiriendo una intensa otalgia y otorrea del oído derecho
que no han cedido a pesar del tratamiento. En la exploración se objetiva otorrea en el
conducto auditivo externo del oído derecho asociada a erosión de la piel subyacente a
dicha otorrea, que además presenta esfacelos y zonas de necrosis; la piel de la región
preauricular está inmdemne. Usted decide realizar un estudio radiológico que se
muestra en la imagen. Indique lo que considere verdadero:

Podemos considerar el TC como normal, ya que ambas mastoides están neumatizadas y


1. 1.
con ausencia de ocupación.
La presencia de ocupación de la mastoides derecha por una masa de partes blandas y la
2. 2. erosión, al menos parcial, de las celdillas mastoideas también derechas, obliga a descartar
un proceso expansivo en dicha región.
Las complicaciones de las otitis externas difusas se localizan exclusivamente en el oído
3. 3.
externo, por lo que el oído medio no tiene por qué estar afecto.
No está indicado en este caso solicitar una prueba de imagen, porque el curso que está
4. 4.
teniendo la otitis externa de este paciente entra dentro de lo normal.
Gráfico de respuestas
Comentario

Aunque nunca hayas visto un corte de TC como éste, no es difícil darte cuenta de que existe una
alteración en las celdillas mastoideas derechas (mitad izquierda de la imagen), por comparación
con el lado izquierdo, cuya estructura está preservada. La respuesta correcta, por tanto, sería la 3.
Por otra parte, existe cierta erosión ósea (observa que no solamente se limita a las celdillas, sino
que también afecta a hueso extracraneal), por lo que la sospecha debería ser una patología
maligna (probablemente un carcinoma epidermoide) o algún tipo de otitis capaz de producir erosión
ósea, como la otitis externa maligna.(R2)

156. Sobre la patología que sospecha en este paciente, señale cuál sería su actitud tras
ver la prueba de la imagen anterior:

Hacer una resección quirúrgica ampliada de la mastoides (petrosectomía) y radioterapia


1. 1. posterior, ya que se trata de un carcinoma epidermoide de conducto auditivo externo
(CAE).

 
 
 
 
Comenzar con quimioterapia paliativa, ya que se trata de un carcinoma epidermoide de
2. 2.
conducto auditivo externo en un estadío avanzado.
Tomar una biopsia de la piel del conducto auditivo externo para filiar histológicamente la
3. 3. lesión: lo más probable es que se trate de una otitis externa maligna o de un carcinoma
epidermoide de CAE.
Iniciar tratamiento antibiótico intravenoso de amplio espectro a altas dosis, ya que es con
4. 4.
toda seguridad una otitis externa maligna.
Gráfico de respuestas
Comentario

Tal como explicábamos en el comentario de la pregunta 1, debemos realizar una biopsia de la


región afecta para descartar malignidad.(R3)

157. ¿Cuál es el signo característico del sarampión?

1. 1. Manchas de Koplik.
2. 2. Manchas de Forschheimer.
3. 3. Máculas eritematosas pruriginosas.
4. 4. Vesículas con contenido purulento.
Gráfico de respuestas
Comentario

Las manchas de Koplik son lesiones pequeñas e irregulares de tipo granular, de color rojizo con el
centro blancoazuláceo que aparecen en la mucosa oral, en la cara interna del carrillo, a nivel del
primer molar inferior, aproximadamente el décimo día posterior a una infección por el virus del
sarampión, 2 a 3 días antes de que aparezca el exantema característico y desaparecen al iniciar el
exantema dejando una lesión eritematosa. Es un signo patognomónico del sarampión.(R1)

158. Which of the following bradyarrhythmias should NOT be treated with permanent
pacemaker placement?

1. 1. Complete atrioventricular block due to acute inferior wall myocardial infarction.


2. 2. An 80-year-old patient with symptomatic sinus node disease.
3. 3. Mobitz II second-degree AV block unrelated to AV nodal blocking agents administration.
Atrial fibrillation and dizziness in a patient with seven-second pauses seen in ambulatory
4. 4.
Holter monitoring.
Gráfico de respuestas
Comentario
El BAVc en el contexto del IAM inferior se suele resolver tras la revascularización, y suele ser
transitorio, por lo que al tratarse de una causa reversible no tendría indicación de marcapasos
definitivo, aunque si podría tenerlo de marcapasos temporal.(R1)

159. Una mujer de 40 años acudió a Urgencias por presentar una lesión dolorosa
enrojecida y caliente en la pierna. Tres días antes, había sido mordida por su perro. Se
drenó un absceso y el examen del pus reveló pequeños cocobacilos Gramnegativos.
¿Cuál de los siguientes es el más probable agente causal de esta infección?

1. 1. Pasteurella multocida.
2. 2. Aeromonas hydrophila.
3. 3. Bacteroides melaninogenicus.
4. 4. Escherichia coli.

 
 
 
 
Gráfico de respuestas
Comentario

Aunque la mordedura de gato es la que más se asocia con la infección por Pasteurella, no debes
descartarla porque nos hablen de un perro. De hecho, nos están describiendo el signo típico de
esta infección: celulitis alrededor de la mordedura, aparte de los cocobacilos gramnegativos, por lo
que la respuesta correcta es la 1.(R1)

160. Para detectar malformaciones del sistema nervioso del feto se suele medir:

1. 1. Glicoproteína-beta o SP1.
2. 2. Alfa-fetoproteína.
3. 3. Diaminooxidasa.
4. 4. Fosfatasa alcalina.
Gráfico de respuestas
Comentario
Pregunta sobre el screening bioquímico del segundo trimestre. La proteína que está elevada
característicamente en aquellas gestantes con fetos portadores de malformaciones del sistema
nervioso (como los defectos del tubo neural, por ejemplo un encefalocele) es la alfafetoproteína
(opción 2 correcta). Recordarte que la alfafetoproteína la podemos encontrar también elevada en
mujeres con tumor del seno endodérmico (tumor de ovario de extirpe germinal altamente maligno)
y en gestantes fumadoras.(R2)

161. Señale la respuesta verdadera respecto a la exploración en el RN:

1. 1. El lanugo es típico de los RN postérmino.


2. 2. El vérmix es típico de los prematuros.
3. 3. La mancha mongólica aparece más frecuentemente en la raza caucásica.
4. 4. El fenómeno arlequín es un proceso transitorio y benigno.
Gráfico de respuestas
Comentario
El fenómeno del arlequín es un fenómeo vasomotor sin trascendencia clínica. El lanugo es típico
de los prematuros. El vérnix caseoso es típico de los recién nacidos a término. El líquido amniótico
teñido de meconio es típico de los posmaduros. La mancha mongólica (sin trascendencia clínica
también ) es más frecuente en los niños de procedencia asiática y latinoamericana.(R4)

162. ¿A qué edad inicia el niño el sostén cefálico?:

1. 1. 4 semanas.
2. 2. 3 meses.
3. 3. 5 meses.
4. 4. 1 mes.
Gráfico de respuestas
Comentario

Quédese con los hitos proncipales: sonrisa social (1.5 mes ), inicio sostén cefálico 2 meses pero al
no encontrar la respuesta exacta busca la que más se aproxime en tes caso respuats número 2, (3
meses), inicio sedestación (6 meses), inicio bipedestación (10-11 meses).(R2)

 
 
 
 
163. Señale la respuesta INCORRECTA acerca del cáncer de mama:

1. 1. El tipo histológico más frecuente es el carcinoma lobulillar.


2. 2. En el sarcoma de la mama no está indicada la quimioterapia.
La presencia del marcador tumoral Ca 15.3 en el cáncer de mama resecado indica
3. 3.
enfermedad metastásica.
El factor pronóstico más importante lo constituye la presencia o ausencia de diseminación
4. 4.
en ganglios axilares.
Gráfico de respuestas
Comentario

Pregunta sobre el cáncer de mama bastante difícil, pero que podemos contestar de forma directa.
El tipo histológico más frecuente de cáncer de mama es el canalicular invasor o ductal infiltrante,
representando el 70-80% de los casos, con lo que debemos elegir la opción 1 por ser la incorrecta.

En el sarcoma de mama, el tratamiento es la extirpación quirúrgica con o sin radioterapia; no está


indicada la quimioterapia (opción 2).

El Ca 15.3 no es específico de cáncer de mama, pero es útil para la detección de metástasis a


distancia y valorar la respuesta terapéutica tras la recidiva (opción 3).

El número de ganglios afectos es el elemento pronóstico más importante (opción 4).(R1)

164. Paciente de 58 años con insuficiencia renal terminal en espera de la realización de


un trasplante, acude a Urgencias porque se ha caído al notar que su pierna se "quebraba".
En la exploración radiológica encontramos una fractura espontánea de diáfisis femoral,
pero indagando un poco, realizamos otras radiografías en las que encontramos
reabsorción perióstica en falanges y huesos largos, formación de quistes óseos pardos
y cráneo en "sal y pimienta". Aunque usted ya sospecha que el paciente padece
osteodistrofia renal, ¿qué tipo específico de lesión sufre?

1. 1. Osteítis fibrosa quística.


2. 2. Osteosclerosis.
3. 3. Osteomalacia.
4. 4. Calcificaciones metastásicas.
Gráfico de respuestas
Comentario

La osteítis fibrosa quística es una enfermedad ósea secundaria a insuficiencia renal avanzada,
caracterizada por:

- Quistes óseos pardos, por exceso de PTH

- Cráneo en sal y pimienta

- Reabsorción subperióstica en falanges y huesos largos.

165. Trasladan a nuestro centro a un paciente al que han encontrado en el suelo e


inconsciente junto a una torre de alta tensión. Al ingreso, el paciente ha recuperado el
conocimiento, presentando un Glasgow de 15 puntos, y se constata la existencia de

 
 
 
 
inestabilidad hemodinámica. A nivel del tórax presenta una quemadura de segundo
grado. En el brazo derecho presenta una quemadura con carbonización de la mano,
necrosis tisular extensa y fractura abierta de cúbito y radio. En el brazo izquierdo, el
paciente presenta dolor e impotencia funcional, con dolor a la movilización pasiva de
dicho miembro superior. En ambos muslos presenta quemaduras de primer grado
profundo. Entre las siguientes afirmaciones sobre las quemaduras eléctricas, señale la
CORRECTA:

El daño aparente (lesión cutánea) suele ser más aparatoso de lo que en realidad son las
1. 1.
lesiones internas.
La resistencia a la corriente eléctrica es muy escasa en el tejido nervioso y músculos, y
2. 2.
elevada en la piel, salvo cuando está mojada.
Si aparece daño renal, es improbable que se justifique por rabdomiolisis, ya que la
3. 3.
afectación muscular es rara.
Si se producen a nivel doméstico, se hace muy improbable la aparición de arritmias
4. 4.
cardíacas.
Gráfico de respuestas
Comentario

La respuesta 1 es falsa: la quemadura cutánea puede ser mínima pero el daño interno ser muy
grande si la corriente atraviesa el corazón o da parada respiratoria por fallo bulbar, por ejemplo. La
3 también es falsa: el daño muscular puede ser importante por varios mecanismos, ya sea
destrucción directa por calor, destrucción por devascularización (daño arterial), síndromes
compartimentales o infección local sobreañadida.

La electrocución doméstica sí puede dar arritmias, sobre todo si el trayecto es mano-mano, mano-
pecho o mano-pie y afecta al corazón. RecuerdE que no hace falta mucha corriente para inducir
una FV.(R2)

166. Una adolescente de 17 años es llevada al médico por sus padres porque en los
últimos meses ha perdido unos 10 Kg de peso y además se le han retirado las reglas.
Usted sospecha una anorexia nerviosa. Es poco probable que esta paciente:

1. 1. Sienta hambre.
2. 2. Tenga un hipogonadismo primario, causa de la amenorrea.
3. 3. Realice bastante ejercicio físico.
4. 4. Presente alteración en las analíticas.
Gráfico de respuestas
Comentario
ES una pregunta muy sencilla que con la que repasamos las consecuencias secundarias de la
anorexia. Es típico y forma parte de los criterios diagnósticos además de la pérdida de peso, la
retirada de las reglas consecuencia de un hipogonadismo hipotálamo- hipofisario y por lo tanto
secundario ( opción 3 falsa) las demás opciones nos mencionan características y consecuencias
propias de la anorexia. Manual CTO 4ª Edición, Psiquiatría, Tema 6, Págs. 40- 41(R2)

167. Señale cuál de las siguientes afirmaciones sobre los genes BRCA1 y BRCA2
implicados en el cáncer de mama hereditario es FALSA:

El riesgo de padecer además cáncer de ovario es mayor en las mujeres portadoras de


1. 1.
mutación en el BRCA1 que en las portadoras de mutación en el BRCA2.

 
 
 
 
Los cánceres de mama asociados a mutaciones en el BRCA1 son generalmente bien
2. 2.
diferenciados en comparación con los cánceres de mama esporádicos.
Mutaciones en los genes BRCA1 y BRCA2 son muy poco frecuentes en cánceres de
3. 3.
mama esporádicos.
4. 4. La prevalencia de mutaciones del BRCA1 es mayor que la del BRCA2.
Gráfico de respuestas
Comentario

El cáncer de mama está muy de moda no sólo en el examen, sino también en los medios de
comunicación.Sobre los genes BRCA1 y 2, has de saber:

•   Incrementan el riesgo de padecer cáncer de mama de forma alarmante, según algunos


trabajos hasta un 80%, y para la mayoría de los autores en más de un 50% de las
pacientes portadoras. Este aumento de riesgo es mayor para el BRCA1 que para el
BRCA2. Recuerda que, igualmente, el BRCA1 es más frecuente que el BRCA2.
•   Los genes BRCA1 y 2 se relacionan con formas familiares de cáncer de mama, siendo
muy raros en el contexto del cáncer de mama esporádico.
•   El grado de diferenciación es menor que en el cáncer de mama no asociado a estas
mutaciones (respuesta 2 falsa).

(R2)

168. Paciente de 41 años que en su revisión ginecológica se descubre SIL de bajo grado
en la citología. Se le realiza una colposcopía objetivándose una lesión acetoblanca a las
12 horarias, por lo que se practica biopsia y tipaje de VPH. El resultado es de CIN I con
VPH 18 +. ¿Qué tratamiento es el más adecuado?:

1. 1. Aplicación de imiquimod al 5% en el cérvix.


2. 2. Observación periódica.
3. 3. Electrocoagulación de cérvix.
4. 4. Legrado endocervical.
Gráfico de respuestas
Comentario

El tratamiento del CIN I puede ser la observación o la destrucción local mediante frio o electricidad.
La observación sin tratamiento puede estar indicada en las pacientes con lesiones de bajo grado,
jóvenes, con ausencia de CIN previa, ausencia de inmunosupresión, test VPH negativo, lesión
totalmente visible y correcto seguimiento. Si en el periodo de seguimiento la lesión persiste o
progresa, se procederá a su tratamiento. En este caso clínico la paciente tiene el VPH +, además
de alto riesgo oncogénico, por lo que las posibilidades de progresión de la lesión a CIN II- III son
mayores, por eso se prefiere la electrocoagulación a la observación. El imiquimod es tratamiento
para los condilomas, y no debe aplicarse en mucosas. La conización se utiliza en los SIL de alto
grado y el legrado endocervical cuando la colposcopía es insatisfactoria.(R3)

 
 
 
 

169.
Secundigesta de 32 años, amenorrea de 12 semanas, análisis de laboratorio de primer
trimestre normal, serologías negativas excepto para rubeola que es inmune; grupo
sanguíneo A, Rh negativo con anticuerpos irregulares negativos. Se realiza USG de
primer trimestre, midiendo el embrión 63 mm de LCC. A tenor de la imagen mostrada
señale la opción correcta:

Está indicada la amniocentesis de forma inmediata por el elevado riesgo de


1. 1.
cromosomopatía.
2. 2. El screening combinado de primer trimestre es normal.
Estará indicado estudio del cariotipo fetal sólo si hay antecedentes de cromosomopatía en
3. 3.
gestaciones anteriores, al ser menor de 35 años.
4. 4. Deberemos recomendar estudio del cariotipo fetal.
Gráfico de respuestas
Comentario

Una pregunta complicada, ya que requiere bastante intuición para darse cuenta de lo que
pretenden mostrarnos en esta imagen. Para alguien versado en ecografía obstétrica, no tiene
mucha dificultad, pero la primera vez que se ve, hay que hacer un esfuerzo para darse cuenta. Es
una traslucencia nucal patológica. Recuerda que este hallazgo es típico del síndrome de Down, y
tendremos que orientar nuestros esfuerzos diagnósticos en este sentido.(R4)

170. En relación con la misma paciente, señale la respuesta FALSA:

El riesgo de cardiopatía fetal es de un 20%, por lo que estará indicada la realización de


1. 1.
una ecocardiografía precoz a las 14 semanas.
2. 2. Estará indicada la amniocentesis a las 15 semanas.
3. 3. Se debe descartar isoinmunización fetal.
El estudio de la onda de velocimetría del ductus venoso y del hueso nasal nos dará el
4. 4.
diagnóstico.
Gráfico de respuestas

 
 
 
 
Comentario

En los fetos con síndrome de Down, no aparece el hueso nasal o se desarrolla de forma muy
tardía, de ahí la importancia de que la investiguemos mediante técnicas de imagen. Por otra parte,
la flujometría del ductus también puede darnos pistas en este sentido.(R4)

171. La pileflebitis es una inflamación, a veces supurada, de la vena:

1. 1. Cava superior.
2. 2. Cava inferior.
3. 3. Pulmonar.
4. 4. Porta.
Gráfico de respuestas
Comentario

De la pileflebitis, destacamos 2 aspectos. Por un lado decir que se trata de una tromboflebitis
séptica de la vena porta de muy mal pronóstico y por otro, que una de sus complicaciones más
frecuentes es la emisión de émbolos al hígado, con la consecuente formación de microabscesos
hepáticos.(R4)

172. Respecto a las patologías originadas después de un trasplante de órganos, uno de


los siguientes enunciados NO es correcto:

El rechazo agudo de los injertos de órganos HLA incompatibles se debe a la respuesta de


1. 1. las CD8 citotóxicas contra las moléculas HLA I de las células del injerto y de la
activación de los CD4 contra los HLA II.
La mayoría de los casos de enfermedad injerto contra huésped (EICH) aparecen tras un
2. 2.
trasplante alogénico de médula ósea.
3. 3. La EICH es una patología que solo se presenta en los trasplantes de órganos.
4. 4. La clínica del rechazo hiperagudo surge al poco tiempo del transplante.
Gráfico de respuestas
Comentario

La enfermedad injerto contra huésped aparece típicamente en relación con el trasplante de médula
ósea… Pero esto no quiere decir que sea una complicación exclusiva. Aunque no es frecuente en
absoluto, también está descrita en relación con transfusiones de sangre o hemoderivados
(respuesta 4 falsa). En general, ante respuestas tan restrictivas “sólo se presenta en…”, debes
sospechar su falsedad, aunque no tengas una seguridad del 100%.(R3)

173. Una enferma de 60 años, diagnosticada de broncopatía crónica, diabetes mellitus,


hipertensión arterial y tratada desde hace 4 meses con Omeprazol, Metformina,
Salbutamol, Bromuro de Ipratropio y Enalapril 20 mg+Hidroclorotiazida 25 mg. Acude a
su médica por cansancio, disminución de apetito, con ligera disnea y tos ocasional,
deposiciones variables, a veces blandas y sin síntomas urinarios. Unos análisis muestran
leucocitos 10.000/mm3, Hto 35%, VCM 80, Glucosa 150 mg/dl, Urea 80 mg/dl, Creatinina
1,6 mg/dl, Sodio 133 meq/l y Potasio 2,9 mEq/l. ¿Cuál es la causa más probable de la
hipopotasemia?:

1. 1. Insuficiencia renal.
2. 2. Hiponatremia.
3. 3. Déficit de aporte de potasio.

 
 
 
 
4. 4. Antihipertensivo.
Gráfico de respuestas
Comentario
Dos de los fármacos que recibe son capaces de producir hipopotasemia: el salbutamol y la tiazida
del enalapril + tiazida. Del resto del enunciado clínico, la astenia es debida a la propia
hipopotasemia, y el ritmo intestinal no sugiere diarrea. Las respuestas no consideran la opción del
salbutamol, que es la causa más frecuente de hipopotasemia entre los pacientes con EPOC. La
combinación de enalapril + tiazida (antihipertensivo) queda como única alternativa.(R4)

174. Un enfermo con poliuria, hipopotasemia, altas concentraciones de renina y


aldosterona presenta una hiperplasia de las células yuxtaglomerulares y medulares
intersticiales. Seguramente nos encontramos ante:

1. 1. Acidosis tubular renal tipo 1.


2. 2. Síndrome de Liddle.
3. 3. Síndrome de Bartter.
4. 4. Cistinuria.
Gráfico de respuestas
Comentario

El síndrome de Bartter es un trastorno tubular hereditario, que consiste en una alteración del
transportador de sodio, cloro y potasio, en la rama ascendente del asa de Henle. Por ello, existe
una deficiencia en la reabsorción de K+ (hipopotasemia) y de Na+. Parte de este sodio se
intercambiará por calcio en el túbulo contorneado distal, lo que produce hipercalciuria. Otra fracción
llegará al túbulo colector, donde se intercambia por H+ y K+, cuyas consecuencias serán una
alcalosis metabólica y una mayor hiperpotasemia. Estas alteraciones producen las siguientes
consecuencias:

- Disminución de la reabsorción de sodio. Esto aumenta la diuresis, con la consiguiente


depleción de volumen.

- La depleción de volumen incrementa la actividad del sistema


renina- angiotensina- aldosterona. Crónicamente, se produce una hipertrofia del aparato
yuxtaglomerular, por este motivo.

- La hipopotasemia es un estímulo directo para la liberación de aldosterona, lo que incrementa


aún más el hiperaldosteronismo.

- A pesar del hiperaldosteronismo, no existe HTA. Otro factor que se opone al efecto presor de
la aldosterona son las prostaglandinas, cuya síntesis está aumentada en este síndrome, como
consecuencia de la hipopotasemia.(R3)

175. Una de las siguientes afirmaciones es FALSA respecto a la gastritis crónica A:

1. 1. Existe hiperplasia de las células G del antro.


2. 2. La anemia perniciosa se produce al poco tiempo de originarse la enfermedad.
3. 3. La etiología es autoinmune.
4. 4. Se han encontrado Ac contra factor intrínseco y células parietales.
Gráfico de respuestas
Comentario

 
 
 
 
La anemia perniciosa se debe a un déficit de vitamina B12. Las reservas hepáticas de vitamina
B12 tardarían varios años en agotarse, desde el comienzo de la gastritis autoinmune, porque son
ciertamente abundantes. Por este motivo, la respuesta incorrecta es la 2. Sobre la anemia
perniciosa, debes tener muy claro que, como consecuencia de la destrucción de las células
parietales, disminuye la secreción de ácido clorhídrico. Esto hace que llegue al duodeno una menor
cantidad de ácido, produciéndose una hiperplasia de células G, que sintetizarán una mayor
cantidad de gastrina. Por tanto, siempre debes relacionar la anemia perniciosa con hipoclorhidria e
hipergastrinemia, igual que ocurre con los pacientes en tratamientos prolongados con
omeprazol.(R2)

176. Señale la opción INCORRECTA en relación a la espondilodiscitis infecciosa y la


osteomielitis vertebral:

La pauta de tratamiento antibiótico se establece generalmente en dos semanas vía


1. 1.
endovenosa, seguida de dos a cuatro semanas más de tratamiento vía oral.
En pacientes ADVP se produce sobre todo por diseminación hematógena, siendo P.
2. 2.
aeruginosa el agente más frecuente en nuestro medio.
3. 3. La resonancia magnética constituye el procedimiento diagnóstico de elección.
4. 4. Ante una espondilitis crónica debemos incluir la tuberculosis como opción diagnóstica.
Gráfico de respuestas
Comentario

Sobre las artritis sépticas a nivel vertebral debe recordar que el agente implicado con mayor
frecuencia en nuestro medio es el S.aureus, tanto en ADVP como en personas mayores (respuesta
falsa la 2) a pesar del incremento relativo de P.aeruginosa y Gram negativos respectivamente.
También debe incluir la TB vertebral en el diagnóstico si se trata de una espondilitis crónica.
Recuerde que la resonancia magnética y si es posible la punción- aspiración serán las
herramientas diagnósticas más útiles. Por último, recuerde que el tratamiento antibiótico se
realizará entre 4 y 6 semanas, inicialmente vía parenteral.(R2)

177. ¿Qué progesterona tiene efecto antiandrogénico?:

1. 1. Progesterona natural micronizada.


2. 2. Levonorgestrel.
3. 3. Desogestrel.
4. 4. Acetato de ciproterona.
Gráfico de respuestas
Comentario

Es importante que conozca que el acetato de ciproterona es un antiandrógeno y que como tal se
utiliza en ciertas patologías en las que este efecto nos es beneficioso, como en el hirsutismo del
SOP en el que se da anticonceptivos orales, que disminuyen la producción de esteroides
suprarrenales y ováricos. Hay otros fármacos también antiandrógenos como la espironolactona,
acetato de ciproterona, flutamida, cimetidina y finasteride.

El acetato de ciproterona también se utiliza en el tratamiento del acné, su mecanismo de acción


parece ser por inhibición de la unión de los andrógenos a los receptores periféricos de la glándula
sebácea, impidiendo el paso de testosterona a 5- alfa- dihidrotestosterona, con lo que disminuye
de forma importante la producción de sebo. Por su acción feminizante sólo tiene indicación en la
mujer, debe asociarse a un estrógeno como el etinilestradiol para conseguir una acción
anticonceptiva, ya que el acetato de ciproterona es teratógeno.(R4)

 
 
 
 
178. La tríada de Hutchinson de la sífilis congénita se caracteriza por:

1. 1. Retinitis, nariz en silla de montar y deformidades dentales.


2. 2. Sordera, queratitis y alteraciones dentales.
3. 3. Frente olímpica, sordera y alteraciones dentales.
4. 4. Trombocitopenia, queratitis y alteraciones dentales.
Gráfico de respuestas
Comentario

La tríada de Hutchinson es una de las tríadas con nombre propio del capítulo de las infecciones
connatales, e incluye los siguientes puntos: queratitis intersticial (que, a su vez, es la manifestación
más frecuente de la tríada), hipoacusia neurosensorial (que es una de las tónicas constantes de
todas las TORCH) y alteraciones dentarias (como los dientes de Hutchinson o los molares en
mora).(R2)

179. La dieta sin gluten es útil en:

1. 1. Tuberculosis verrucosa.
2. 2. Lupus eritematoso.
3. 3. Amiloidosis secundaria.
4. 4. Dermatitis herpetiforme.
Gráfico de respuestas
Comentario

Pregunta fundamental dentro del tema de las enfermedades ampollosas. No debe olvidarse este
dato.

La dermatitis herpetiforme (enfermedad de Duhring- Brocq) es una enfermedad benigna y crónica


caracterizada por una erupción papulovesiculosa muy pruriginosa, localizada en áreas extensoras
de manera simétrica en pacientes con una enteropatía sensible al gluten, habitualmente
asintomática. En su tratamiento se utiliza la sulfona como medicamento de elección además del
mantenimiento de una dieta sin gluten de por vida (normaliza la enteropatía y mejora las lesiones
cutáneas a largo plazo). Las demás opciones no tienen ninguna relación con el gluten de la
dieta.(R4)

180. A 38-year-old woman complains of a abnormal vaginal discharge with bad smell, like
"rotten fish", which becomes more intense after intercourse. She denies genital itching.
On examination, an abundant white-gray discharge, which does not adhere to the vaginal
walls, is observed. When mixing a sample of the discharge with a drop of 10% potassium
hydroxide, the referred bad smell is confirmed. Which of the following is the treatment of
choice?

1. 1. Oral amoxicillin-clavulanate
2. 2. Intravaginal clotrimazole
3. 3. Oral doxycycline
4. 4. Intravaginal clindamycin
Gráfico de respuestas
Comentario
Intravaginal clindamycin. This patient presents with signs and symptoms of bacterial vaginosis, the
most common vaginal infection in women ages 15-44. It is caused by an abnormally high
concentration of G. vaginalis, M. hominis and anaerobic bacteria in the vaginal flora. Most women

 
 
 
 
(50-70%) are asymptomatic. Treatment can be done with topic or oral metronidazole or with
intravaginal clindamycin.(R4)

181. Un paciente de 68 años, con antecedentes de fibrilación auricular crónica en


tratamiento con ácido acetilsalicílico (200 mg/día) y digoxina, presenta de forma súbita
disminución de fuerza en extremidades derechas y dificultad para la expresión,
nominación, comprensión y repetición del lenguaje, mejorando en las siguientes 24 horas
de su déficit motor y del habla, siendo capaz de comprender y repetir las órdenes verbales
pero no de nominar, y manteniendo una escasa fluidez verbal con monosílabos. Lo más
probable es que su trastorno del lenguaje actual corresponda a:

1. 1. Afasia nominal
2. 2. Afasia transcortical motora
3. 3. Afasia de conducción
4. 4. Afasia transcortical mixta.
Gráfico de respuestas
Comentario

Esta es una pregunta importante para el examen porque se deben conocer los diferentes tipos de
afasias y diferenciarlas entre sí. Recordemos que existen 5 tipos de afasias, que se diferencian
atendiendo a los conceptos de fluencia, comprensión, nominación y repetición. La afasia de Brocca
(motora pura) se caracteriza por tener alterada la fluencia, nominación y repetición, conservando
intacta la comprensión. La afasia de Wernicke (sensitiva) se distingue de la motora pura porque
conserva la fluencia pero tiene alterada la comprensión. La afasia de conducción conserva la
fluencia y la comprensión, y las afasias transcorticales son las únicas que preservan la capacidad
de repetición. Y por último existe la afasia global que tendrá alteradas las 4 características del
lenguaje. Finalmente recuerde también que la afasia de Brocca y de Wernicke son debidas a la
oclusión de la arteria cerebral media, que se trata del síndrome vascular más frecuente.(R2)

182. Un RN de 38 semanas, pesa 2,200 g, mide 45 cm, tiene un perímetro cefálico de 32


cm y presenta los siguientes rasgos dismórficos: blefarofimosis, micrognatia, labio
superior fino y anomalías menores de las articulaciones. ¿Qué habrá tomado la madre
probablemente durante el embarazo?:

1. 1. Alcohol.
2. 2. Fenobarbital.
3. 3. Warfarina.
4. 4. Heroína.
Gráfico de respuestas
Comentario

El síndrome alcohólico fetal cursa desde el punto de vista clínico con retraso de crecimiento
intrauterino (fíjese que el niño del enunciado es muy pequeño), rasgos dismórficos faciales (labio
superior fino, micrognatia), alteraciones de los dermatoglifos y cardiopatía (por lo general, suele
haber alteraciones septales) y por retraso psicomotor de entidad variable.(R1)

183. Un varón de 50 años acude a consulta refiriendo dolor abdominal epigástrico


ocasional y diarrea de dos años de evolución. Se le practica un test de cuantificación de
grasa en heces de 24 h, determinándose la presencia de esteatorrea. La siguiente prueba
que le realizamos es la prueba de la D-xilosa, que es normal. ¿Cuál es el diagnóstico más
probable en este paciente?:

 
 
 
 
1. 1. Amiloidosis.
2. 2. Gastritis eosinófila.
3. 3. Pancreatitis crónica.
4. 4. Linfoma intestinal.
Gráfico de respuestas
Comentario

La D- xilosa se absorbe en el yeyuno por difusión pasiva, sin necesidad de que intervengan las
enzimas pancreáticas o las sales biliares. Por ello, está alterada en procesos que afectan a la
pared del tubo digestivo, sin verse influida por alteraciones pancreáticas.

El test de la D- xilosa consiste en administrar este azúcar en ayunas, y luego se mide su presencia
en orina, cuando pasan unas horas. Podría estar más o menos alterada en la enfermedad celíaca,
en la infiltración intestinal por amiloide, en un linfoma intestinal o en el contexto de la enfermedad
de Whipple. En cambio, en la pancreatitis crónica no habría ningún tipo de problema para absorber
D- xilosa, porque su absorción es independiente de las enzimas de esta glándula.(R3)

184. En relación con el diagnóstico de la úlcera duodenal, indique la respuesta adecuada:

1. 1. Un estudio radiológico puede ser suficiente para el diagnóstico.


La presencia de úlceras duodenales en segunda o tercera porción de duodeno no tiene más
2. 2.
importancia clínica que las de primera porción.
Tras realizar el estudio radiológico se realizará estudio endoscópico para la toma de
3. 3.
biopsias.
4. 4. El estudio radiológico es de primera elección.
Gráfico de respuestas
Comentario
En el diagnóstico de las úlceras duodenales (como en las gástricas), la técnica de imagen de
primera elección es la endoscopia. Sin embargo, como en las duodenales, en principio no es
obligatoria la toma de biopsias, si este diagnóstico se ha obtenido por estudio radiológico baritado,
es igualmente válido, y no sería necesaria la realización de endoscopia. En cuanto a las técnicas
radiológicas, las de doble contraste (enteral y parenteral), aportan más y mejor información que las
de un solo contraste.(R1)

185. Un neonato producto de un parto cuya madre 10 días antes tiene cultivo de cérvix
positivo a Chlamydia presentará:

1. 1. Conjuntivitis.
2. 2. Neumonía.
3. 3. Otitis media aguda.
4. 4. Pielonefritis.
Gráfico de respuestas
Comentario

Concepto muy importante y preguntad en reiteradas ocasiones. La Chlamydia, junto con la


Gonorrea son las dos causas más frecuentes de conjuntivitis neonatal.(R1)

186. ¿Cuál es el origen embriológico de la parte proximal de la vagina?:

1. 1. Los conductos de Müller.


2. 2. Los conductos de Wolff.

 
 
 
 
3. 3. El seno urogenital.
4. 4. Los pronefros.
Gráfico de respuestas
Comentario

Si ha fallado esta pregunta, no se preocupe… es muy complicado recordar este tipo de cosas.

El aparato genital femenino se encuentra relacionado estrechamente con el aparato urinario.


Deriva de 4 fuentes: mesodermo intermedio, células germinativas primordiales, epitelio celómico
(mesotelio) y mesénquima subyacente.

Los conductos mesonéfricos o de Wolff desaparecen por falta de testosterona. Desarrollan el


epidídimo, el conducto deferente, la vesícula seminal y el canal eyaculador.

Los conductos paramesonéfricos o Müller forman la mayor parte del aparato genital femenino.
Forman las trompas de tubo-ováricas, útero, cérvix y los dos tercios superiores de la vagina, y
obviamente en el hombre desaparecen. Y por último la vejiga es la que proviene del seno
urogenital.(R1)

187. Un paciente de 50 años ha sido sometido a un recambio valvular mitral. El


postoperatorio se complica con la aparición de un cuadro de mediastinitis aguda
supurada. Señale cuál de las siguientes medidas le parece la MENOS adecuada para el
manejo de esta patología:

1. 1. Desbridamiento mediastínico amplio, cerrando la cavidad con un colgajo muscular.


2. 2. Desbridamiento mediastínico amplio, dejando abierta la herida.
3. 3. Desbridamiento mediastínico amplio e irrigación mediastínica a presión.
4. 4. Desbridamiento mediastínico amplio y antibióticos tópicos.
Gráfico de respuestas
Comentario

Pregunta muy difícil de un tema muy poco importante. Lo principal es que recuerde que el
tratamiento de la mediastinitis aguda consiste en drenaje quirúrgico inmediato, reposo alimentario y
antibióticos. No hay ningún motivo para dejar la herida abierta (respuesta 2 falsa).(R2)

188. De todas las hiperlipemias, ¿cuál es la forma más frecuente de presentación?:

1. 1. Hipercolesterolemia familiar.
2. 2. Hipercolesterolemia poligénica.
3. 3. Hiperlipemia familiar combinada.
4. 4. Hipertrigliceridemia familiar.
Gráfico de respuestas
Comentario
La forma más frecuente de dislipemia es la hipercolesterolemia poligénica que afecta a 1/10- 20
individuos, como su nombre indica, en su patogenia influyen diversos genes, además de factores
ambientales como la edad, el consumo de grasas saturadas y colesterol y la actividad física. Afecta
a menos del 10% de los familiares de primer grado. Los individuos que la padecen cifras de
colesterol entre 200- 300 mg/dl y tienen aumentado el riesgo cardiovascular. No existen xantomas
tendinosos.(R2)

 
 
 
 
189. Durante el tercer trimestre de embarazo, una madre presenta HBsAg positivo. Señale
la pauta correcta de actuación en el recién nacido para prevenir la hepatitis B:

1. 1. Administración de inmunoglobulina e iniciar pauta de vacunación.


2. 2. Sólo administración de inmunoglobulina.
3. 3. Sólo vacunación.
4. 4. Sólo controles serológicos.
Gráfico de respuestas
Comentario

Pregunta sumamente importante en el nacional. Las madres que pueden transmitir la hepatitis B a
sus hijos son aquéllas que muestran positividad para el HbsAg (madres con infección aguda,
portadoras crónicas o con hepatitis B crónica activa).En los hijos de estas madres hay que iniciar
con rapidez acciones preventivas: administrar gammaglobulina específica antihepatitis B y la
primera dosis de la vacuna.(R1)

190. El test de Silverman y Anderson incluye lo siguiente, EXCEPTO:

1. 1. Tono muscular.
2. 2. Retracción xifoidea.
3. 3. Aleteo nasal.
4. 4. Quejido espiratorio.
Gráfico de respuestas
Comentario

Pregunta básica no puedes ni dudarla. El test de Silvermann sirve para cuantificar el grado de
dificultad respiratoria de los neonatos. Sirve como compendio para que repase la expresión del
trabajo respiratorio de este período tan especial de la vida: aleteo nasal, quejido espiratorio,
retracción xifoidea, tiraje subcostal y disociación toraco- abdominal. Cada ítem se valora entre 0 y 2
puntos, según su gravedad (0= inexistente, 1= moderado, 2= grave).(R1)

191. En la estructura histológica de la matriz celular del cartílago hialino, la llamada línea
de marea se sitúa a nivel de:

1. 1. Zona II o de transición.
2. 2. Entre la zona I y la II.
3. 3. Entre la zona III (radial) y la IV (calcificada).
4. 4. Entre la zona IV (calcificada) y el hueso subcondral.
Gráfico de respuestas
Comentario
En el cartílago articular normal se pueden evidenciar cuatro zonas diferentes: la zona I superficial o
tangencial, donde los condorcitos adquieren una forma alargada y aplanada; la II, media o de
transición, donde tienen una forma más redondeada; la III, profunda o radial, donde son más
abundantes los condorcitos, pero de forma similar a la previa; y la IV, el cartílago calcificado, que
se separa de la anterior por una línea divisoria: el frente de calcificación o línea de marea. Por
debajo de la zona calcificada aparece el hueso subcondral.(R3)

192. Para el estudio de un paciente de 65 años, hipertenso, que refiere disfunción eréctil,
NO será necesario realizar:

1. 1. Tacto rectal.

 
 
 
 
2. 2. ECG.
3. 3. Prolactina.
4. 4. Testosterona total y libre.
Gráfico de respuestas
Comentario
Se deben descartar las causas corregibles de la disfunción eréctil y para ello no es necesaria la
realización de un ECG.(R2)

193. A 31-year-old man is brought to the emergency department with fever, chills and
malaise for the past 4 days. He also complains of vomiting and headaches. The patient
denies a history of sexually transmitted diseases but does use IV drugs occasionally.
Physical examination reveals painful, raised and red lesions on his hands and feet. A
holosystolic murmur is noted on auscultation. Which of the following is the most
appropriate next step in management?

1. 1. Echocardiogram.
2. 2. Blood cultures.
3. 3. Chest MRI.
4. 4. Cardiac catheterization.
Gráfico de respuestas

(R1)

194. Entre los antecedentes personales de nuestro paciente destaca enolismo moderado-
severo desde hace aproximadamente 20 años, sin otras patologías concomitantes. En la
Rx de abdomen no se aprecian calcificaciones. ¿Cuál sería la prueba diagnóstica que
tiene mayor sensibilidad y especificidad para el diagnóstico de pancreatitis crónica?:

1. 1. Rx abdomen.
2. 2. ECO abdominal.
3. 3. TAC abdominal.
4. 4. CPRE.
Gráfico de respuestas
Comentario

En el nacional será mucho más frecuente que te pregunten pancreatitis aguda, de la crónica sólo
debes saber los aspectos más importantes. La CPRE ha sido reconocida ampliamente como la
prueba más sensible y fiable para el diagnóstico de pancreatitis crónica. La sensibilidad de la
CPRE está en el 90 % con igual especificidad. En la pancreatitis crónica precoz los cambios de los
ductos están limitados a los ductos secundarios o terciarios que muestran dilataciones irregulares.
En los casos moderados el conducto principal puede estar dilatado con áreas alternantes de
estenosis. En casos avanzados de pancreatitis crónica los cambios del ducto principal tienen la
apariencia de una cadena de lagos. La mayoría de estudios que comparan la sensibilidad de la
CPRE frente a las pruebas de función pancreática (pruebas de secreción pancreática) han
demostrado una buena correlación entre las dos en los casos de enfermedad avanzada. Sin
embargo en las fases precoces la correlación entre los cambios morfológicos y la función
pancreática es baja.(R4)

195. ¿Cuál de las siguientes pruebas se considera como diagnóstico definitivo del
embarazo?

 
 
 
 
1. 1. Prueba de embarazo positiva.
2. 2. Aumento del tamaño uterino.
3. 3. Auscultación de latido cardiaco.
4. 4. Signo de Hegar.
Gráfico de respuestas
Comentario

Una pregunta que puede resolver por sentido común, por las siguientes razones:

- La causa más frecuente de amenorrea secundaria es el embarazo, pero puede deberse a


muchas otras causas (anovulación crónica, síndrome de Asherman, etc…).

- Una prueba de embarazo se basa en la detección de beta- hCG… Por ello, también puede dar
positiva en la enfermedad trofoblástica, en la gestación ectópica, en tumores productores de
beta- hCG…

- El aumento del tamaño uterino y el signo de Hegar (reblandecimiento del segmento inferior)
pueden verse en la enfermedad trofoblástica, en miomas, etc…

Sin embargo, la presencia de latido cardíaco fetal implica la presencia de un feto viable, por lo que
necesariamente debe tratarse de un embarazo. No hay ninguna otra cosa que dé lugar a que
auscultemos un latido al ritmo característico en el hipogastrio de la mujer (no va a tener un Alien; si
oímos un latido ahí, es un feto).(R3)

196. ¿Cuál de las siguientes patologías de mama es la más frecuente?

1. 1. Cáncer de mama.
2. 2. Enfermedad de Paget.
3. 3. Mastopatía fibroquística.
4. 4. Quistes mamarios.
Gráfico de respuestas
Comentario

En esta pregunta no Le especifican si patología benigna o maligna, por lo que inmediatamente


deberásde contestar mastopatía fibroquística, que es una patología benigna.(R3)

197. A 65-year-old veteran comes to his physician with a 3-month history of unstable gait,
memory problems and mood disturbances. His sons claim that his personality has
profoundly changed and he is becoming more and more forgetful. Physical examination
reveals unsteady and high-stepping gait along with a curious finding consisting of
bilateral small pupils that constrict when focusing on a near object, but don't constrict
when exposed to bright light. Assuming the adequate ancillary tests confirm the
suspected diagnosis, which of the following is the most appropriate treatment for this
patient?

1. 1. Oral doxycycline and intramuscular ceftriaxone.


2. 2. Intramuscular benzathine penicillin.
3. 3. Oral azithromycin.
4. 4. IV aqueous crystalline penicillin.
Gráfico de respuestas

 
 
 
 
Comentario
IV aqueous crystalline penicillin. Dementia-like history + Argyll-Robertson pupil =
NEUROSYPHILIS. For patients with this disease, IV aqueous crystalline penicillin is the drug of
choice. Remember that Argyll Robertson pupil is also known as 'Prostitutes pupil'. And like a
prostitute, the pupil accommodates but does not react...(R4)

198. Morbidity and mortality due to accidents are an important issue during childhood.
Therefore, their prevention and management is also very important. Regarding this
subject, which of the following is incorrect?

General measures such as using infant car seats or keeping all medicines out of the reach
1. 1.
of children are very important.
Active protective measures that encougrage behavioral changes in parents or caregivers
2. 2.
are more effective than passive measures applied on the environment.
Children between 1 and 2 years old have the highest risk of accidents such as falls,
3. 3.
drowning or burns.
Boys between 1 and 7 years old have higher risk of suffering accidents than their female
4. 4.
counterparts.
Gráfico de respuestas
Comentario

Esta pregunta porque no tiene mucha relevancia. Simplemente recuerda que los accidentes, en
concreto los de tráfico, son la primera causa de muerte en los niños después del primer año y
constituyen una causa importante de muerte evitable. En cuanto a las medidas de protección o
medidas de control de lesiones las más eficaces son las que implican cambios en el diseño del
producto (ej. Airbag, tapas a prueba de niños...). Son intervenciones pasivas que protegen a todos
los individuos con independencia del grado de colaboración de cada persona. Tienen más éxito
que las medidas activas que requiere una cambio de conducta de los padres o del niño. Es mejor
dar normas especificas como el uso de sillas homologadas para el coche, que consejos generales
del tipo. Los niños que empiezan a andar ( 1- 2 años) tiene un riesgo elevado de quemaduras,
ahogamiento y caídas. Después su curiosidad aumenta el riesgo de intoxicaciones. Las caídas son
la principal causa de asistencia a urgencias. El que entre 1- 7 años los varones tengan mayor
frecuencia de accidentes que las niñas no se explica por diferencias en el desarrollo psicomotor, la
fuerza o coordinación sino porque los niños se exponen más al riesgo (ej montan más tiempo en
bici).(R2)

199. Señale el signo serológico que predice más tempranamente una evolución ulterior
favorable en la hepatitis aguda B, HBsAg seropositiva, de curso común:

1. 1. La aparición de anti-HBc de clase IgM.


2. 2. La negativización del antígeno HBs.
3. 3. La negativización del antígeno HBe.
4. 4. La aparición de anti-HBs.
Gráfico de respuestas
Comentario
La negativización del AgHBe es el primer acontecimiento que ocurre en los pacientes que se están
curando ya que lo primero que ocurre es el cese de replicación, para posteriormente producirse la
negativización del antígeno de superficie (AgHBs), momento en el cual se considera que la
infección está curada.(R3)

200. ¿Cuál de los siguientes fármacos actúa como hormonoterapia competitiva en el


carcinoma de la mama?:

 
 
 
 
1. 1. Aminoglutetimida.
2. 2. Estrógenos.
3. 3. Tamoxifeno.
4. 4. Progestágenos.
Gráfico de respuestas
Comentario

El tamoxifeno es una de las opciones hoy disponibles para el tratamiento hormonal del cáncer de
mama. A nivel mamario, actúa como antiestrógeno, pero sobre otros receptores tiene,
paradójicamente, cierta acción estrogénica (por ello, aumenta la incidencia de hiperplasia y cáncer
de endometrio). Sin embargo, esto no ocurre con el raloxifeno, ya que carece de actividad
estrogénica a todos los niveles, con lo que carece de efectos proliferativos a nivel endometrial. Otro
fármaco que merece la pena recordar, a este respecto, es el fulvestrant, un aitiestrógeno puro.

Actualmente, en el cáncer de mama se recurre con más frecuencia a los inhibidores de la


aromatasa, como el exemestrano, el letrozol o el anastrozol.(R3)

201. Usted recibe a un niño de 7 años de edad. A los 15 meses de edad, ante un cuadro
de diarrea crónica y escasa ganancia ponderal, fue estudiado por su pediatra quién
encontró positividad de los anticuerpos IgA antigliadina y antitransglutaminasa tisular,
por lo que indicó biopsia yeyunal que demostró atrofia vellositaria total. Desde entonces
ha estado con dieta exenta de gluten (y los 3 primeros meses después de la biopsia con
fórmula a base de proteínas hidrolizadas) con lo que la diarrea ha cedido, el peso se ha
normalizado y los anticuerpos antigliadina y antitransglutaminasa tisular se han
negativizado. ¿Qué haría usted ahora?

1. 1. Le daría de alta indicando dieta libre.


2. 2. Indicaría dieta libre y lo volvería a ver en revisión 1 año después.
Le indicaría la realización de una biopsia yeyunal para confirmar la normalización
3. 3.
histológica. Entonces indicaría dieta exenta de gluten de forma indefinida.
Después de comprobar la normalización histológica en una 2.ª biopsia, indicaría prueba
4. 4.
de provocación con gluten.
Gráfico de respuestas
Comentario

Una pregunta difícil sobre el diagnóstico de la enfermedad celíaca.

Ante la sospecha de esta enfermedad, lo habitual es realizar una determinación de anticuerpos


(recuerda que los más sensibles y específicos son los IgA-antitransglutaminasa). Si esta
determinación resulta positiva, estaría indicado realizar una biopsia intestinal, que también podría
plantearse ante una determinación negativa, pero con una firme sospecha clínica. Por tanto, el
papel de los anticuerpos en el diagnóstico de esta enfermedad tiene valor fundamentalmente como
cribado.

La biopsia intestinal es una prueba necesaria para establecer el diagnóstico. Se realizaría por lo
menos en una ocasión, estando el paciente consumiendo gluten. En este caso clínico ya tenemos
una biopsia intestinal compatible con enfermedad celíaca (la duda estaría en la necesidad de
realizarla por segunda vez). La confirmación de la recuperación histológica después de la retirada
del gluten (segunda biopsia) se realizaría en casos dudosos. No obstante, en este caso lo tenemos
bastante claro: buena evolución clínica después de retirar el gluten de la dieta y negativización de

 
 
 
 
los anticuerpos, por lo que en principio bastaría continuar con la dieta. Pero la respuesta correcta
es la número 3.(R3)

202. ¿Cuál de los siguientes datos de laboratorio es inesperado en un niño con


deshidratación hiponatrémica moderada (del 10 al 12%)?:

1. 1. Potasio sérico de 4.5 mEq/L..


2. 2. Nitrógeno ureico en sangre de 30 mg/dl.
3. 3. Hematocrito del 28%.
4. 4. Osmolaridad = 260 mOsm/L.
Gráfico de respuestas
Comentario

Esta pregunta acerca de los datos de laboratorio en una deshidratación hiponatrémica es


importante. Las cifras de sodio definen el tipo de deshidratación: hiponatrémica (<130mEq/l),
isonatrémica (130- 150) e hipernatrémica (>150). La osmolalidad extracelular esta determinada en
gran parte por el sodio por lo que es frecuente que las denominaciones de cada tipo se denominen
indistintamente como hipo, iso, e hipertónica.

Los valores de potasio son casi siempre normales. Los niveles de nitrogeno ureico y creatinina
serica suelen estar elevados en casos intensos por la disminución del filtrado glomerular.

En la deshidratación hiponatrémica el LEC es relativamente hipotónico con respecto al LIC, y el


agua pasa del compartimiento extracelular al intracelular. Así, el déficit de líquidos por perdidas al
exterior se acentúa por la desviación interna al LIC, siendo frecuente el colapso circulatorio.

Lo normal es encontrar hemoconcentración y no un descenso del hematocrito. Cundo la


hemoglobina y el hematocrito aparecen normales a pesar de la deshidratación hay que pensar que
el paciente sufre anemia.(R3)

203. La indicación de realizar una laparoscopía en un paciente con EPI es:

1. 1. Absceso tubo-ovárico.
2. 2. Persistencia del dolor.
3. 3. Persistencia de la fiebre.
4. 4. Diagnóstico dudoso.
Gráfico de respuestas
Comentario

Pregunta de un tema muy importante en el nacional. La utilidad de la laparoscopía es


eminentemente diagnóstica, pues es el gold-estándar para diagnosticar enfermedad inflamatoria
pélvica. Por ello, la respuesta correcta es la 4.

Aun así, no se debe olvidar que también tiene utilidad terapéutica, pues permite drenar las
colecciones y deshacer adherencias.(R4)

204. A 70-year-old man complains of diffuse osseous pain. Laboratory studies reveal:
creatinine 1.9 mg/dl, proteinuria 2.5 gr/day, total serum protein 8.5 gr/dl, serum calcium
11.5 mg/dl. Which of the following is the most likely diagnosis?

 
 
 
 
1. 1. Primary hyperparathyroidism.
2. 2. Paraneoplastic membranous glomerulonephritis.
3. 3. Multiple myeloma related amyloidosis.
4. 4. Prerenal acute kidney injury.
Gráfico de respuestas
Comentario

No se asuste con este pregunta de primera instancia, lea con calma. Esta pregunta es fácil si vas
descartando opciones a partir de la clínica relatada en el enunciado.

El hiperparatiroidismo primario explicaría los dolores óseos y la hipercalcemia pero no la proteinuria


por lo que no es probable.

La insuficiencia renal prerrenal no cursa de esta forma.

La GN membranosa suele cursar en la mayoría de los pacientes como un síndrome nefrótico, que
se define por la presencia de una proteinuria SUPERIOR a 3.5gr/día, nuestro paciente presenta
una proteinuria de 2.5 por lo que descartamos la opcion 2.

Por último, podríamos haber elegido de primera mano la opción 3, ya que el mieloma nos explica
tanto los dolores óseos como la hipercalcemia, y el desarrollo de amiloidosis renal nos explica la
proteinuria.(R3)

205. ¿Cuál de los siguientes NO es un criterio de gravedad en la preeclampsia?:

1. 1. Proteinuria intensa.
2. 2. Trombocitosis.
3. 3. Papiledema en fondo de ojo.
4. 4. TA sistólica superior a 160 mmHg.
Gráfico de respuestas
Comentario

Debe repasar los criterios de gravedad de la preeclampsia. La trombocitosis no es uno de ellos. Al


contrario, lo son la cifra de plaquetas por debajo de 100 000 o la anemia hemolítica
microangiopática.(R2)

206. ¿Cuál de las siguientes sustancias NO atraviesa la placenta?

1. 1. Insulina.
2. 2. Proteínas complejas.
3. 3. TRH.
4. 4. Virus.
Gráfico de respuestas
Comentario

No atraviesan la placenta:

1.- Heparina.

2.- Insulina.

 
 
 
 
3.- Inmunoglobulinas (excepto IgG, que sí la atraviesa).

(R1)

207. Señale la respuesta ERRONEA en relación al tratamiento de la disección aórtica:

1. 1. La frecuencia cardíaca debe mantenerse por debajo de 60 lpm.


2. 2. Los narcóticos son muy útiles.
La reparación quirúrgica urgente de un aneurisma aortico abdominal requiere resección
3. 3.
de toda la porción aórtica disecada y sustitución por injerto de dacrón.
4. 4. Durante la cirugía se mantiene al paciente en hipotermia.
Gráfico de respuestas
Comentario
El tratamiento médico de la disección de aorta se dirige a frenar la progresión del hematoma que
produce la disección de la íntima, lo que se consigue mediante la reducción de la presión arterial
sistólica y de la fuerza de eyección del ventrículo izquierdo. La disminución del dolor con morfina y
derivados contribuye a la reducción de la presión sistólica y a la sedación del paciente. En la
actualidad existe un amplio acuerdo en realizar el tratamiento quirúrgico en las disecciones tipo A.
Esta pregunta sería impugnable ya que efectivamente la cirugía consiste en la resección de la
porción de aorta disecada y su sustitución por un injerto de dacron, pero, si te fijas, en la opción 4
pone que éste es el tratamiento indicado en el aneurisma de aorta abdominal (que también lo es)
pero en el enunciado te están preguntando por el tratamiento de la disección de aorta.(R3)

208. De los siguientes antibióticos activos frente a Staphylococcus aureus meticilín


resistente, indique cuál se puede administrar por vía oral:

1. 1. Ceftobiprole.
2. 2. Linezolid.
3. 3. Tigeciclina.
4. 4. Daptomicina.
Gráfico de respuestas
Comentario
Staphylococcus aureus meticilín resistente (SAMR) es una bacteria que produce infecciones
habitualmente asociadas a los cuidados sanitarios, especialmente infecciones en relación con
catéteres de acceso venoso e infecciones en relación con material protésico. La resistencia a
meticilina implica también resistencia a todos los antibióticos pertenecientes al grupo de penicilinas
resistentes a penicilinazas (meticilina, oxacilina, cloxacilina). Hasta hace muy pocos años la única
alternativa terapéutica para la infección por SAMR era la vancomicina o la teicoplanina (ambas
deben administrarse por vía parenteral). Recientemente han surgido alternativas como la
daptomicina, que presenta la ventaja de administrarse por vía intravenosa pero en una sola dosis
diaria y es un antibiótico con menor riesgo de nefrotoxicidad que la vancomicina. La tigeciclina es
una nueva tetraciclina de administración intravenosa con actividad frente a bacterias Gram
negativas y bacterias Gram positivas (incluido SAMR). Este fármaco presenta una buena
penetración en piel y partes blandas, pero los estudios realizados en bacteriemia no lo sitúan como
una alternativa de primera elección si la bacteria aislada en los hemocultivos es SAMR.
Ceftobiprole es una cefalosporina de “quinta generación”, aún no comercializada, que presenta la
peculiaridad de presentar actividad frente a S aureus, incluido SAMR. Por último, linezolid es un
antibiótico alternativo a la vancomicina para el tratamiento de SAMR que presenta la gran ventaja
de poder ser administrado por vía oral, con una excelente biodisponibilidad por vía digestiva
(superior al 98%), lo cual permite el tratamiento ambulatorio de muchos pacientes. Además
linezolid se caracteriza por su excelente penetración en parénquima pulmonar, de modo que se

 
 
 
 
han publicado algunos estudios que sugieren que podría tratarse del tratamiento de elección de la
neumonía nosocomial (con frecuencia asociada a ventilación mecánica) producida por SAMR.(R2)

209. Una lesión del núcleo del tercer par craneal produce:

1. 1. Lagoftalmos y desviación del ojo hacia fuera y hacia abajo.


2. 2. Lagoftalmos y desviación del ojo hacia dentro y abajo.
3. 3. Ptosis palpebral y desviación del ojo hacia fuera y arriba.
4. 4. Ptosis palpebral y desviación del ojo hacia fuera y abajo.
Gráfico de respuestas
Comentario

Es importante que recuerdes la anatomía y funciones de los pares craneales.

El tercer par inerva toda la musculatura extraocular salvo el recto externo y el oblicuo superior, el
elevador del párpado superior, el músculo ciliar y el constrictor de la pupila. Sin embargo, las fibras
que inervan el músculo ciliar y el constrictor de la pupila provienen del núcleo de Edinger-
Westphal. Por tanto, la lesión aislada del núcleo del tercer par producirá:

- Ptosis palpebral

- Desviación de la mirada hacia fuera y abajo.(R4)

210. Una mujer de 38 años consulta por una secreción vaginal anormal con mal olor como
a “pescado podrido”, que se hace más intenso después del coito. No tiene prurito genital.
En la exploración se observa una abundante secreción blanco-grisácea, que no se
adhiere a las paredes vaginales. Al mezclar una muestra de la secreción con una gota de
hidróxido potásico al 10% se aprecia claramente el mal olor referido. ¿Cuál de los
siguientes es el tratamiento de elección?

1. 1. Amoxicilina con ácido clavulánico por vía oral.


2. 2. Clotrimazol por vía intravaginal.
3. 3. Clindamicina por vía intravaginal.
4. 4. Fluconazol por vía oral.
Gráfico de respuestas
Comentario

Nos encontramos ante un caso evidente de vaginosis bacteriana. Se trata de una enfermedad
causada por un desequilibrio de la flora bacteriana vaginal. Clínicamente se caracteriza por una
leucorrea maloliente, sin prurito ni inflamación de la pared vaginal: no es realmente una vaginitis,
porque no hay inflamación; por eso se le llama "vaginosis".

Para el diagnóstico, tres pruebas nos pueden ayudar:

-Al administrar hidróxido potásico a una muestra de flujo vaginal, aumenta el mal olor.

-El pH vaginal está aumentado, por encima de su pH normal en la mujer fértil de 4,5.

-Al analizar el flujo vaginal con el microscopio, vemos que las células epiteliales descamadas están
cubiertas de bacterias; son las llamadas "células clue".

 
 
 
 
Para el tratamiento de la vaginosis se usan dos antibióticos dirigidos contra bacterias anaerobias:
la clindamicina y el metronidazol. La única respuesta que incluye uno de estos antibióticos es la
3.(R3)

211. Una mujer de 52 años de edad presenta desde hace 2 años disnea de esfuerzo. En la
exploración física, destaca un soplo diastólico. De entre los siguientes, el diagnóstico
más probable será:

1. 1. Estenosis mitral.
2. 2. Estenosis aórtica.
3. 3. Miocardiopatía hipertrófica.
4. 4. Miocardiopatía dilatada.
Gráfico de respuestas
Comentario

Los datos auscultatorios de la estenosis mitral son en ritmo sinusal, primer tono reforzado,
chasquido de apertura de la mitral y soplo diastólico con refuerzo presistólico. Los datos
auscultatorios de severidad de las estenosis mitrales son: que aumenta la duración del soplo y que
el chasquido de apertura valvular mitral está progresivamente más cerca del segundo ruido.
Además, con el tiempo se calcifica la válvula, con lo que desaparece el aumento de intensidad del
primer ruido, ya que disminuye la movilidad de la válvula, y además desaparece el chasquido de
apertura. Cuando el paciente cae en fibrilación auricular, desaparece el refuerzo presistólico, ya
que la aurícula pierde su contracción efectiva.(R1)

212. Sobre la pleuresía tuberculosa, señale lo FALSO:

1. 1. El derrame es un exudado.
2. 2. Hay muchas células mesoteliales.
3. 3. La cutirreacción a la tuberculina es negativa en una tercera parte de enfermos.
4. 4. La respuesta a la quimioterapia es buena.
Gráfico de respuestas
Comentario
Vamos a enumerar algunas características importantes del derrame pleural tuberculoso, que debes
recordar para el examen MIR:

- Predominio de linfocitos entre las células.

- Ausencia o pobreza en células mesoteliales.

- Glucosa disminuida.

- Adenosindeaminasa aumentada (ADA +).

- Características de exudado.(R2)

213. Mujer de 28 años, primípara; parto eutócico hace 10 días. Puerperio afebril. Lactancia
materna. Acude a urgencias refiriendo cuadro de dolor en mama derecha de 48 horas de
evolución, Temperatura de 38º C, enrojecimiento y calor local. Ha presentado dificultad
para vaciar esa mama durante la alimentación del RN. Exploración: mamas asimétricas
(derecha de mayor tamaño); sin alteraciones a nivel de areola-pezón; secreción láctea

 
 
 
 
espontánea. Mama derecha ingurgitada, eritematosa a nivel del cuadrante superior
externo y caliente; no alteraciones tróficas en piel. A la palpación se detecta una zona
indurada, dolorosa, pero sin zonas fluctuantes evidentes en CSE. Señale la respuesta
INCORRECTA:

Los gérmenes más frecuentemente implicados en este proceso son estafilococos,


1. 1.
estreptococos y colibacilos.
2. 2. Es necesario retirar la lactancia.
3. 3. El tratamiento incluye la administración de antibióticos y antiinflamatorios.
4. 4. Es posible la formación de un absceso que requiere drenaje quirúrgico.
Gráfico de respuestas
Comentario

El caso clínico que nos presentan es el de una mastitis. Se trata de una infección muy frecuente en
mujeres con lactancia materna. La producen gérmenes que pasan de la piel a la glándula mamaria
a través de grietas y heridas que se forman secundariamente a la lactancia. Ante una mastitis, se
debe pensar en microorganismos grampositivos, como S. aureus.

El tratamiento de la mastitis se realiza con antibióticos y analgésicos, evacuación periódica del


pecho y aplicación de calor local.

Si se forma un absceso, será necesario completar el tratamiento anterior con el drenaje quirúrgico.

No es necesario inhibir la lactancia, salvo que sea una mastitis bilateral.(R2)

214. Acerca del mecanismo de acción como antiarrítmico de los betabloqueantes, señale
la respuesta FALSA:

Reducen la frecuencia sinusal y suprimen los marcapasos ectópicos y el automatismo


1. 1.
anormal inducido por catecolaminas.
2. 2. No modifican el QRS ni el intervalo HV.
Deprimen la velocidad de conducción y prolongan el periodo refractario efectivo del
3. 3.
nodo auriculoventricular, prolongando los intervalos PR y AH.
Producen un gran enlentecimiento de la velocidad de conducción intraauricular e
4. 4.
intraventricular.
Gráfico de respuestas
Comentario

Pregunta de gran dificultad de un tema que es muy preguntado en el MIR, como son los fármacos
cardiovasculares, aunque si se ha fallado no hay que preocuparse. Los b- bloqueantes se utilizan
en las arritmias causadas por un automatismo anormal inducido por catecolaminas, en las arritmias
asociadas a isquemia miocárdica y además puede utilizarse cuando es necesario bloquear el nodo
AV como en la taquicardia supraventricular por reentrada o en la FA con respuesta ventricular
rápida. Está claro que los betabloqueantes no van a modificar el QRS ni el intervalo HV, ya que no
modifican la conducción intraauricular ni la interventricular.(R4)

215. A 28-year-old female presents to the ED with a six-day history of pelvic pain. She
began with suprapubic pain and burning micturitions with an increase in the number of
voids. 2 days ago, she developed right lumbar pain and fever. Physical examination
shows right costovertebral angle tenderness and suprapubical pain to deep palpation.
Her vital signs are heart rate 120/min, respiratory rate 27/min, blood pressure 85/60 mmHg

 
 
 
 
and temperature 38.5C (101.3F). Laboratory workup: Hb 12.5 g/dL RWC 17.500 /mm3 Renal
ultrasound reveals non-dilated renal pelvic calyceal system. Urinalysis: Esterase Positive
Nitrite Positive Protein Negative WBC >100 /hpf RBC 2-3 /hpf Which of the following is the
most appropriate next step in the management of this patient?

1. 1. Discharge the patient and prescribe Amoxicillin-Clavulanate for 7 days.


2. 2. Admit the patient to the hospital and place a nephrostomy tube.
3. 3. Admit the patient to hospital and initiate intravenous Ceftriaxone and Gentamicin
4. 4. Admit the patient to hospital and initiate oral Cefuroxime axetil.
Gráfico de respuestas
Comentario
Admit the patient to hospital and initiate intravenous Ceftriaxone and Gentamicin A possible
complication of an initially uncomplicated cystitis if it is not treated correctly is an ascending spread
of the infection, causing fever and requiring iv antibiotics.(R3)

216. ¿Cuál de los procedimientos proporciona un diagnóstico más preciso de


Enfermedad Inflamatoria Pélvica?

1. 1. Culdoscopía.
2. 2. Histerosonografía.
3. 3. Laparoscopía.
4. 4. Histeroscopía.
Gráfico de respuestas
Comentario

Recordemos que el patrón de oro para el diagnóstico de enfermedad inflamatoria pélvica es la


laparoscopía.(R3)

217. Mujer 36 años que ingresa por dolor en cuadrante superior derecho, hiperamilasemia
y aumento de fosfatasa alcalina, en el ultrasonido se muestra colelitiasis, por lo que se
inicia hidratación intravenosa y analgésicos, el cuadro se resuelve rápidamente y a las
48 horas los valores de amilasa y fosfatasa alcalina se normalizan. ¿Cuál el manejo mas
apropiado para esta paciente?:

1. 1. Colecistectomía laparoscópica electiva en 8 semanas.


2. 2. Colecistectomía laparoscópica con colangiografía intraoperatoria antes del alta.
Colangio endoscopia retrograda antes del alta y colecistectomía laparoscópica en 8
3. 3.
semanas.
4. 4. Observación.
Gráfico de respuestas
Comentario

Pregunta sencilla. Respuesta correcta 2, ya que en este caso se realizará colecistectomía


laparoscópica con colangiografía intraoperatoria antes del alta.(R2)

218. Femenino de 48 años de edad, con dolor tipo cólico en hipocondrio derecho desde
hace 8 horas, con vómitos y sensación febril. No cede al usar antiespasmódicos. Al
examen físico: signo de Murphy (+) laboratorio: leucocitosis. ¿Cuál es el diagnostico más
probable?

 
 
 
 
1. 1. Coledocolitiasis.
2. 2. Cólico vesicular.
3. 3. Colecistitis aguda
4. 4. Colecistitis crónica calculosa.
Gráfico de respuestas
Comentario

La opción correcta es la 3 (colecistitis aguda) ya que la paciente tiene cólico biliar junto con fiebre y
leucocitosis, lo cual solo puede corresponder a la opción 3.

La coledocolitiasis en principio no se presentaría con leucocitosis o fiebre salvo que se complicara


con colangitis (triada de Charcot: cólico, ictericia, leucocitosis)

El cólico vesicular por sí solo podría explicar el dolor pero tampoco los signos de infección.

La colecistitis crónica acalculosa no se manifiesta como dolor agudo sino más bien como malestar,
intolerancia a los fritos, etc.(R3)

219. Uno de los siguientes microorganismos NO tiene un hábitat natural en la boca ni en


la nasofaringe. Señálelo:

1. 1. Haemophilus influenzae.
2. 2. Moraxella catarrhalis.
3. 3. Bacteroides.
4. 4. Escherichia coli.
Gráfico de respuestas
Comentario

Ésta es una pregunta de dificultad media.

E. coli es una enterobacteria cuyo hábitat natural es el intestino. H. influenzae y Moraxella


catarrhalis son habitantes de la cavidad oral y, por microaspiración, causan infección
broncopulmonar. Los bacteroides forman parte de la flora anaerobia de la cavidad oral; hay que
recordar que los B. fragilis forman parte de la flora intestinal.(R4)

220. Entre las diversas manifestaciones clínicas propias de la artritis reumatoide, hay una
que es muy característica y que tiene un gran peso a la hora de establecer el diagnóstico:

1. 1. Artritis de los hombros.


2. 2. Fiebre.
3. 3. Rigidez matutina.
4. 4. Artritis de tobillos bilateral.
Gráfico de respuestas
Comentario
Ninguna manifestación de forma aislada permite establecer el diagnóstico de artritis reumatoide,
por ello necesitamos reunir 4 de los 7 criterios propuestos por el ACR. De las opciones que se te
ofrecen la más característica de la artritis reumatoide es la rigidez, aunque insisto en que no es
exclusiva de la enfermedad ya que otros procesos inflamatorios o incluso degenerativos pueden
mostrar rigidez, aunque no suele ser tan prolongada como en la AR. El adelgazamiento o la fiebre
no son manifestaciones cardinales y pueden no aparecer en la artritis reumatoide. En cuanto a las

 
 
 
 
localizaciones articulares mencionadas lo característico es la bilateralidad que se menciona más
que en sí las articulaciones afectadas.(R3)

221. ¿Cuáles de las siguiente NO es causa de trombocitopenia en el paciente quirúrgico?

1. 1. Sepsis.
2. 2. Transfusiones masivas.
3. 3. Farmacológica.
4. 4. Aplasia.
Gráfico de respuestas
Comentario

Tanto la sepsis como el uso de transfusiones masivas y algunos fármacos son causa de
trombocitopenia en el paciente quirurgico, la única respuesta que no puede ser es la aplasia, ya
que no es frecuente en pacientes quirúrgicos.(R4)

222. Recién nacido, varón de 15 días de edad, que presenta ictericia y acolia, con escasa
ganancia ponderal. Se detecta hepatomegalia dura a la palpación abdominal, siendo el
resto de la exploración normal. Se le practica una biopsia hepática y se recibe el siguiente
informe anatomopatológico: fibrosis portal, proliferación ductal y trombos biliares
intraportales. ¿Cuál es el diagnóstico más probable?:

1. 1. Síndrome de Alagille.
2. 2. Atresia de vias biliares extrahepáticas.
3. 3. Hepatitis congénita por citomegalovirus.
4. 4. Enfermedad de Gilbert.
Gráfico de respuestas
Comentario

Nos hallamos ante un caso clínico que describe un cuadro de colestasis neonatal. La enfermedad
de Gilbert consiste en una hiperbilirrubinemia indirecta leve y monosintomática, por lo que queda
descartada. Las otras tres entidades son posibles causas de colestasis neonatal.

La atresia de vías biliares extrahepáticas es la causa más frecuente de hepatopatía y de trasplante


hepático en la infancia. El resto de causas posibles producen colestasis intrahepática, por lo que la
anatomía patológica es importante en la diferenciación.

La proliferación ductal y los trombos biliares sugieren una obstrucción distal al paso de la bilis,
mientras que la normalidad en la arquitectura del lobulillo hepático descartan lesión hepatocelular,
típica de las hepatitis idiopática y viral. La hepatitis en el contexto de una infección por
citomegalovirus puede presentar además otros síntomas de infección congénita (púrpura, ictericia,
bajo peso, calcificaciones intracraneales, sordera…). El síndrome de Alagille consiste en lesión de
la vía biliar intrahepática (la biopsia nos lo diferencia) y se acompaña de facies peculiar, cardiopatía
(estenosis pulmonar), malformaciones vertebrales…(R2)

223. ¿Cuál de estas circunstancias NO se ha relacionado con una mayor probabilidad de


embarazo ectópico?

1. 1. Enfermedad inflamatoria pélvica.


2. 2. Endometriosis.
3. 3. Conización cervical.

 
 
 
 
4. 4. Técnicas de reproducción asistida.
Gráfico de respuestas
Comentario

Pregunta sencilla sobre el embarazo ectópico. Se puede deducir fácilmente que de las opciones
que nos dan, la conización cervical es la única que no está relacionada con el desarrollo de
embarazos ectópicos. Como puedes comprobar, es fácil deducir el resto de las opciones.(R3)

224. Entre los pacientes en mayor riesgo de carcinoma gástrico están los siguientes,
EXCEPTO:

1. 1. Los sometidos a resección gástrica por úlcera duodenal.


2. 2. Los que tienen anemia perniciosa.
3. 3. Los que se someten a derivación gástrica por obesidad mórbida.
4. 4. Los del grupo sanguíneo A.
Gráfico de respuestas
Comentario

Existen determinadas intervenciones quirúrgicas sobre el estómago que aumentan el riesgo de


cáncer, como sucede en las Billroth II (adenocarcinoma de la anastomosis). Sin embargo, cuando
se realiza una cirugía resectiva sobre un estómago por lo demás sano (cirugía bariátrica), no se ha
documentado un aumento de riesgo en este sentido.(R3)

225. Una de las siguientes es una paniculitis septal sin vasculitis:

1. 1. Eritema nodoso.
2. 2. Paniculitis inducidas.
3. 3. Eritema indurado de Bazin.
4. 4. Vasculitis nodular.
Gráfico de respuestas
Comentario
Esta pregunta no debe fallarse, ya que las paniculitis esenciales de cara al MIR son el eritema
nodoso y la vasculitis nodular. Es preciso conocer de qué tipo son y sus características
diferenciales. El eritema nodoso es una paniculitis septal sin vasculitis (opción 1 correcta). La
vasculitis nodular/eritema indurado de Bazin es una paniculitis lobulillar con vasculitis. Las
paniculitis inducidas y neonatales son lobulillares sin vasculitis.(R1)

226. Recién nacido de 12 horas de vida, de parto eutócico, peso 3000 gramos,
asintomático, con madre que presenta serología positiva para hepatitis B. ¿Cuál es la
conducta más apropiada en este recién nacido?

1. 1. Administrar solo vacuna anti hepatitis B.


2. 2. Administrar inmunoglobulina más vacuna contra hepatitis B.
3. 3. Serología para hepatitis B.
4. 4. Sólo vacuna contra hepatitis B.
Gráfico de respuestas
Comentario

Pregunta muy importante para el nacional. La profilaxis especial (gammaglobulina específica-IGHB


y vacuna) es muy eficaz en la prevención de transmisión de VHB en el momento del parto. Sin
embargo, algunos niños nacidos de madre HBeAg+ resultan infectados.(R2)

 
 
 
 

227. Señale, de entre las siguientes, cuál es la asociación INCORRECTA:

1. 1. Conjuntivitis - Loa-loa.
2. 2. Serositis - Mansonella perstans.
3. 3. Carcinoma epidermoide de vejiga - Wuchereria bancrofti.
4. 4. Linfedema en miembros inferiores - Brugia malayi.
Gráfico de respuestas
Comentario

La Wuchereria bancrofti produce una filariasis linfática, clínicamente manifiesta en forma de


elefentiasis y linfedema perineal y genital, mientras que el carcinoma epidermoide puede
producirse tras la parasitación vesical por Schistosoma haematobium.(R3)

228. Todo lo siguiente es normal en un RN, EXCEPTO:

1. 1. Cierre del conducto de Arancio y del ductus.


2. 2. Eliminación de meconio en las primeras 48 horas vida.
3. 3. FC de 140 lpm y FR de 30-40 rpm.
4. 4. Hemoglobina más alta y leucocitos más bajos que en un niño de 2 años.
Gráfico de respuestas
Comentario

Conceptos importantes, que es importante saberlos para saber reconocer algo anormal en un
cuadro clínico. Recuerden los signos vitales en el RN:

•   Frecuencia cardiaca: 100 a 180 latidos/min.


•   Frecuencia respiratoria: 30 a 60 respiraciones/min.
•   TA sistólica: 40 a 70 mm Hg.
•   TA diastólica: 20 a 45 mm Hg.

Además, en el recién nacido ocurre el cierre del conducto de Arancio y del ductus arterioso durante
las primeras 48 hrs.

El meconio, deposición viscosa y espesa de color verde oscuro a negro compuesta por células
muertas y secreciones del estómago e hígado, que reviste el intestino del recién nacido, suele ser
eliminado las primeras 48 hrs.

Aunque es verdad que los valores de Hb son más altos en el recién nacido, los leucocitos también
lo son, con lo que la respuesta falsa es la 4.(R4)

229. A 34-year-old female presents to the office with a history of several episodes of
abdominal pain. She reports that the pain is triggered by fatty food intake and it is located
in right upper quadrant, it lasts for few hours and relieves spontaneously . Her medical
history includes Crohn's disease with severe ileal involvement. Vital signs are within
normality. Physical examination and laboratory tests show no significant abnormal

 
 
 
 
findings, with no pain to abdominal palpation at the moment. Which of the following
mechanisms is most likely involved in the patient's pain?

1. 1. Autoimmune cholecystitis
2. 2. Decreased total reabsorption of bile salt
3. 3. Augmented total bile salt excretion
4. 4. Increased cholesterol absorption
Gráfico de respuestas
Comentario
Decreased total reabsorption of bile salt. Ileal implication decreases bile salt reabsorption, which
favours gallstone formation causing the biliary colics our patient is suffering from.(R2)

230. Señale qué actitud NO es correcta en el manejo de un neonato con infección urinaria:

1. 1. Obtención de cultivos por punción suprapúbica, si otros métodos no son posibles.


2. 2. Tratamiento con antibióticos parenterales durante al menos 7 días.
3. 3. Estudio con cistografía, aunque el ultrasonido haya sido normal.
4. 4. Seguimiento con análisis del sedimento urinario exclusivamente.
Gráfico de respuestas
Comentario

El método más fiable para la toma de urocultivos es la punción suprapúbica. Este método es el de
elección en el período neonatal, pues el sondaje puede ser muy dificultoso a esta edad. La ITU ha
de ser tratada por vía parenteral durante al menos 7 días en los neonatos, y además hay que
verificar el estado renal (con US) y de la vía urinaria (a través de cistouretrografía miccional
seriada). El sedimento urinario no tiene ninguna utilidad durante el período neonatal (opción 4
falsa).(R4)

231. El tratamiento del tumor del estroma gastrointestinal (tumor GIST) en principio es
quirúrgico. No obstante, en las ocasiones en que no puede ser resecado, el tratamiento
sería uno de los siguientes:

1. 1. Imatinib.
2. 2. Infliximab.
3. 3. Azatioprina.
4. 4. Etanercept.
Gráfico de respuestas
Comentario

El tumor GIST expresa c-kit. El tratamiento farmacológico de este tumor, en caso de no ser
resecable, sería el imatinib.(R1)

232. Paciente femenino de 37 años que acude a la consulta por dismenorrea que ha ido
en aumento en los últimos meses, con dispareunia asociada. A la exploración, los
genitales externos y el cérvix son normales. El útero está en retroversoflexión, fijo y
doloroso a la movilización, además de dolor a la palpación de los ligamentos uterosacros.
Procedemos a realizar ultrasonido transvaginal, donde vemos un quiste de 23 mm en
ovario derecho y otros dos de 18 y 31 mm en ovario izquierdo, todos ellos de aspecto
ecomixto con fino punteado en su interior. El diagnóstico más probable será:

 
 
 
 
1. 1. Enfermedad inflamatoria pélvica.
2. 2. Endometriosis.
3. 3. Quiste de ovario torsionado.
4. 4. Síndrome de ovario poliquístico.
Gráfico de respuestas
Comentario

Es un cuadro típico de endometriosis

El dolor es el síntoma más característico y frecuente 95%. Su intensidad es muy variable y se


presenta como dismenorrea progresiva que no cede con la toma de ACO. También puede
aparecer dispaurenia. Las alteraciones menstruales se presentan en un 65%, la infertilidad en un
41%.

El diagnóstico de sospecha se hará por clínica y ultrasonido, en la que es habitual la visualización


de los quistes endometriósicos, si bien el diagnóstico definitivo es a través de laparoscopia, que
además será terapéutico.(R3)

233. Indique cuál NO suele ser un síntoma o signo del síndrome de apnea obstructiva:

1. 1. Hipersomnia diurna.
2. 2. Deterioro intelectual.
3. 3. Estridor.
4. 4. Hipertensión arterial.
Gráfico de respuestas
Comentario
Pregunta que se contesta fácilmente teniendo claros unos conceptos básicos acerca del SAOS. El
SAOS es una patología con predominio en varones obesos de edad media y mujeres
postmenopáusicas. Las manifestaciones neuropsiquiátricas y de conducta (las más frecuentes) son
consecuencia de los despertares transitorios y recurrentes que terminan con cada apnea y
fragmentan el sueño, siendo la queja más común la somnolencia diurna (opción 1 y 2 correctas);
los ronquidos suelen estar presentes durante años antes que se desarrollen los otros síntomas
(opción 5 correcta). Asimismo, las alteraciones cardíacas también son frecuentes, y entre ellas
podemos encontrar arritmias e hipertensión arterial (opción 4 correcta). No suele ser signo o
síntoma del SAOS el estridor, por lo que la respuesta falsa es la 3, a la que se puede llegar por
descarte.(R3)

234. El oligohidramnios está asociado a:

1. 1. Trisomía 18.
2. 2. Hipoplasia pulmonar.
3. 3. Atresia duodenal.
4. 4. Isoinmunización Rh.
Gráfico de respuestas
Comentario

Oligohidramnios es un término médico que significa poco líquido amniótico. Afecta a menos del
10% de los embarazos, y aunque es más común que se desarrolle en el primer trimestre, puede
aparecer en cualquier nivel del embarazo. Es un trastorno que se diagnostica durante un
ultrasonido revelando un índice de líquido amniótico menor de 5 cm³.

 
 
 
 
La falta de líquido amniótico permite la compresión del abdomen del feto, lo que limita el
movimiento de su diafragma. Además de la fijación de la pared torácica, la falta de líquido
amniótico que fluye dentro y fuera del pulmón fetal lleva a hipoplasia pulmonar.(R2)

235. ¿Qué valores de pH intraparto se consideran normales?:

1. 1. Por encima de 7.45.


2. 2. Entre 7.25 y 7.45.
3. 3. Por debajo de 7.20.
4. 4. Por encima de 7.50.
Gráfico de respuestas
Comentario

Pregunta muy fácil sobre la microtoma de sangre fetal intraparto.

Esta pregunta requiere que hayas memorizado los valores de la pHmetría fetal.

La opción correcta es la 2, ya que los valores normales oscilan entre 7.25 y 7.45. Por encima de
7.45 indican alcalosis fetal, con lo que serán patológicos. Por debajo de 7.20 es patológico ya que
existe al menos una acidosis moderada, y deberemos finalizar la gestación de forma
inmediata.(R2)

236. Señale lo INCORRECTO respecto a la farmacología del litio:

1. 1. Eliminación por vía biliar en el 95%.


2. 2. En la profilaxis del trastorno bipolar, la litemia óptima está entre 0,6-1,0 mEq/l.
3. 3. Comparte los mecanismos de reabsorción renal del sodio.
4. 4. No está clara su farmacodinamia.

(R1)

237. Una mujer se encuentra en el sexto mes de gestación. La tensión arterial es de 105/60
mmHg y el sedimento de orina es normal. ¿Qué esperaría encontrar en el estudio
sanguíneo de esta gestante?:

1. 1. Hemoconcentración.
2. 2. Aumento del filtrado glomerular.
3. 3. Aumento del ácido úrico.
4. 4. Ausencia de variación del aclaramiento de creatinina.
Gráfico de respuestas
Comentario

Una pregunta muy sencilla sobre la fisiología de la gestación. El caso que nos presentan tiene una
presión arterial normal y no existen alteraciones patológicas en el sedimento, por lo que debemos
pensar que se trata de una gestante sana. Durante el embarazo, el filtrado glomerular aumenta de
forma fisiológica, por lo que aumenta el aclaramiento de creatinina, eliminándose una mayor
cantidad de esta sustancia.

De todos los cambios que se producen en el organismo de la gestante, el más preguntado del
examen es el aumento del filtrado glomerular, así que no olvide esta pregunta. En cuanto a la

 
 
 
 
opción 1, debes saber que aumenta la hematopoyesis, pero en proporción crece más el volumen
plasmático, con lo que se produce una hemodilución.(R2)

238. La forma quística de la dilatación congénita del colédoco se exterioriza clínicamente


antes de los 20 años de edad en la mayoría de casos y se manifiesta con la triada:

1. 1. Pérdida de peso, ictericia, masa.


2. 2. Ictericia, dolor, acolia.
3. 3. Ictericia, dolor, fiebre.
4. 4. Ictericia, dolor, masa.
Gráfico de respuestas
Comentario

Pregunta un tanto compleja de un tema muy poco preguntado en el ENARM.

La triada clásica de este tipo de padecimiento es ictericia, dolor y la palpación de una masa,
respuesta correcta 4.(R4)

239. Paciente de 2 años de edad, desde hace 2 días presenta evacuaciones líquidas
acuosas, 10 veces al día, fiebre, hiporexia, vómitos postprandiales. A la exploración
física: T°: 38.5 °C, FC: 100 por minuto, Peso: 12 kg, piel y mucosas secas, eutrófico,
abdomen no distendido, RC rítmicos y de buena intensidad. El tratamiento inicial es:

1. 1. Hidratación IV con electrolitos.


2. 2. Rehidratación oral.
3. 3. Administración de cloruro de sodio IV.
4. 4. Hidratación con solución Ringer lactato.
Gráfico de respuestas
Comentario

El grado de deshidratación se corresponde con el porcentaje de peso corporal que se ha perdido,


pero se asume que una pérdida de agua de peso corresponde a agua y electrólitos y no a masa
corporal. Una vez detectado y valorado el grado de deshidratación, este debe ser corregido
considerando que al paciente se le han de aportar los líquidos y elecrolitos que ha perdido (deficit),
los que necesita para su metabolismo basal y los que puede continuar perdiendo. La tendencia
actual es a utilizar, en la medida de lo posible, rehidratación oral, por ser más adecuada y
fisiológica, reservando la intravenosa cuando la v.o. esté contraindicada (deshidratación grave,
inestabilidad hemodinámica, íleo paralítico o abdomen potencialmente quirúrgico, trastorno de
conciencia o fracaso de rehidratación oral). Durante la fase de rehidratación se dejará al niño a
dieta y una vez, esta esté resuelta, se reintroduciran los alimentos empezando por los lácteos.
Para la rehidratación oral se suelen emplear sluciones hiposódicos (casos moderados) y en
deshidrataciones hiponatrémicas es posible utilizar también soluciones isotónicas.

- Deshidratación leve: pérdida <5% (niños) o <3% (lactantes)

- Deshidratación moderada: pérdida 5-10% (niños) o 3-7%(lactante)

- Deshidratación grave: pérdida de >10%(niños) o >7% (lactante).(R2)

240. Which is the correct answer regarding acute ischemic colitis:

 
 
 
 
1. 1. It usually presents with painless hematochezia.
2. 2. If suspected, an arteriography must be performed as soon as possible.
3. 3. It is usually a non-occlusive problem.
4. 4. Barium enema test is the first test to obtain.
Gráfico de respuestas
Comentario

La colitis isquémica aparece en pacientes ancianos con factores de riesgo cardiovascular que por
déficit agudo del riego a nivel de mesentérica inferior pero no clusivo, produce isquémica de
mucosa cólica. Cursa en forma de dolor abdominal y rectorragia.

En la radiología simple de abdomen se ven las características huellas digitales que son la
expresión de hematomas submucosos en la colonoscopia. Suele respetar el recto ya que esta
zona recibe aporte del plexo hemorroidal, siendo la zona más afectada el ángulo esplénico donde
la vascularización es más pobre por recibir a porte de ramas distales de mesentérica superior e
inferior.

La arteriografía, al deberse a un trastorno de la microvascularización, no es útil para el diagnóstico.


El tratamiento es conservador ya que suele mejorar espontáneamente y en caso de ser grave o
cursar con perforación, el tratamiento es quirúrgico. Los casos crónicos suelen producir estenosis
cicatriciales segmentarias por lesiones repetidas por isquemia.(R3)

241. RN de 1 semana de vida presenta candidiasis oral, lo CORRECTO es:

1. 1. Requiere manejo sistémico con anfotericina B.


2. 2. Tienen buena respuesta a la terapia tópica con nistatina.
3. 3. Debe hacerse búsqueda de inmunosupresión.
4. 4. Se debe administrar alguna equinocandina.
Gráfico de respuestas
Comentario

En pacientes RN con candidiasis oral, el tratamiento se basa en nistatina tópica, por lo que la
respuesta correcta es la 2.(R2)

242. Mujer de 23 años sin antecedentes de interés consulta por placas


eritematodescamativas en codos y rodillas que mejoran en verano y empeoran en los
meses posteriores. Actualmente utiliza corticosteroides tópicos y ha mejorado mucho. Si
hiciéramos una biopsia a esta paciente, ¿cuál de los siguientes hallazgos NO será
compatible con psoriasis?:

1. 1. Acantosis.
2. 2. Ausencia de granulosa.
3. 3. Microabscesos de Pautrier.
Dilatación de los capilares dérmicos y edema papilar con infiltrado perivascular
4. 4.
mononuclear.
Gráfico de respuestas
Comentario
Recuerda que la histología del psoriasis es muy importante para el MIR. Por tanto, los aspectos
básicos debes recordarlos: - Hiperqueratosis paraqueratósica. - Acantosis y papilomatosis -
Hipogranulosis focal - Microabscesos de Munro- Sabouraud (no Pautrier como afirma la opción 4)

 
 
 
 
que son típicos de la micosis fungoide). - Pústula espongiforme de Kogoj. Página 20. Manual CTO
4ª Edición.(R3)

243. A 7-year-old girl is brought to the pediatrician’s office because her parents have
noticed that she is presenting signs of early puberty. Physical examination shows
protrusion of breast buds with areolar pigmentation and incipient pubic and axillary hair.
Weight and height are normal. Thorough skin examination shows several brown
hyperpigmented lesions on both elbows. The girl has no relevant medical history but she
suffered an ulnar and radial fracture two years ago and has chronic pain in extremities.
She has a 10-year-old brother who is healthy. What is the most likely diagnosis?

1. 1. McCune-Albright disease
2. 2. Cushing's syndrome
3. 3. Ovarian tumor
4. 4. Craniopharyngioma
Gráfico de respuestas
Comentario
McCune-Albright disease. McCune-Albright disease associates bone malformations, skin
pigmentation and hormonal problems along with premature puberty. Cushing's syndrome is
associated with intolerance to carbohydrates, cushingoid appearance and striae.
Craniopharyngioma's main manifestation is often visual loss. Adrenal tumors and lung carcinoids
are not common at this age.(R1)

244.
Paciente de 35 años que acude refiriendo prurito genital de dos días de evolución,
acompañado de leucorrea abundante, espumosa y de mal olor. Se decide realizar una
toma para valoración bajo microscopía, con el resultado que se muestra en la imagen. La
paciente refiere que en los últimos meses ha mantenido relaciones sexuales sin
protección, como antecedente de interés. Ante estos hallazgos, ¿qué germen sospecha
que puede estar involucrado en la infección vulvovaginal que presenta la paciente?:

1. 1. Candida albicans.
2. 2. Candida lusitae.
3. 3. Trichomonas vaginalis.
4. 4. Virus del Papiloma Humano.
Gráfico de respuestas
Comentario

 
 
 
 
La imagen muestra el aspecto microbiológico de este protozoo anaerobio, que se acompaña de
una reacción de polimorfonucleares, representados por las células teñidas en azul. Además, la
información clínica que nos aporta el caso clínico es importante para obtener la respuesta correcta,
ya que el aumento de flujo maloliente y espumoso es característico de este tipo de infección.(R3)

245. Ante la sospecha clínica y microbiológica de infección vulvovaginal, ¿qué


tratamiento propondría a la paciente del caso clínico anterior?:

1. 1. Clotrimazol tópico.
2. 2. Metronidazol oral, recomendando tratamiento de la pareja.
3. 3. Metronidazol oral, sin tratamiento de la pareja.
4. 4. Metronidazol tópico, sin tratamiento de la pareja.
Gráfico de respuestas
Comentario

El tratamiento de elección ante una infección vaginal por Trichomonas es el Metronidazol oral,
recordando que es el único microorganismo que no responde de manera adecuada al tratamiento
tópico. Al ser una ITS, la pareja debe recibir también el mismo tratamiento para erradicar la
infección. Recuerde el efecto Antabus que tiene este tipo de medicación.(R2)

246. Hombre de 50 años que acude al médico porque, después de una infección
respiratoria aguda aparentemente sin importancia, ha empezado a sentir disnea con
medianos esfuerzos y tos sin expectoración; ha perdido 2 kg en 3 meses. A la
auscultación presenta crepitantes inspiratorios en bases. En la placa de tórax aparece un
patrón reticular difuso, más acentuado en campos pulmonares inferiores. La VSG es de
50 mm la 1ª hora; factor reumatoide y ANA son positivos a títulos bajos; en la espirometría
aparece una disminución de todos los volúmenes pulmonares. Resultado del lavado
broncoalveolar: 66% de macrófagos alveolares, 20% de PMN, 10% de linfocitos, 4% de
eosinófilos. En espera de pedir una biopsia pulmonar abierta, ¿cuál sería su diagnóstico
de sospecha?:

1. 1. Fibrosis pulmonar idiopática.


2. 2. Neumonía eosinófila crónica.
3. 3. Afectación pleuropulmonar en un LES.
4. 4. Sarcoidosis.
Gráfico de respuestas
Comentario

La fibrosis pulmonar idiopática (alveolitis fibrosante criptogenética) se caracteriza por una fibrosis
progresiva de las paredes alveolares, que suele aparecer hacia los 50- 70 años de vida.
Clínicamente, al principio produce disnea de esfuerzo y tos seca, de intensidad creciente según
evoluciona la enfermedad. Con el paso del tiempo, acaban apareciendo acropaquias, que
constituyen un signo tardío. En la exploración, es típica la presencia de crepitantes
teleinspiratorios.

La radiografía de tórax suele mostrar un patrón reticular o reticulonodular, que acaba


evolucionando hacia el “pulmón en panal” o “en queso suizo” en el estadio terminal de la
enfermedad. Recuerda que se afectan más las bases que los vértices, aspecto que alguna vez se
ha preguntado.

 
 
 
 
Los estudios de función pulmonar revelarán un patrón restrictivo: disminución de la capacidad
pulmonar total, del volumen residual, de la capacidad vital… Es muy frecuente la disminución de la
DLCO y puede preceder al descenso de los volúmenes.

Ante la sospecha de esta enfermedad, se realizarían varias biopsias transbronquiales o, en su


caso, una biopsia pulmonar abierta, en caso de que no aclaren el diagnóstico. El lavado
broncoalveolar revela un aumento de los polimorfonucleares (>20%) y, aunque de forma discreta,
también de los eosinófilos.

El pilar fundamental del tratamiento son los corticoides, pero la supervivencia global es del 50% a
los cinco años, ya que no modifican la historia natural de la enfermedad. En situación terminal,
puede realizarse un trasplante pulmonar.

En cuanto al resto de las opciones, la neumonía eosinófila crónica produce infiltrados pulmonares
periféricos y en la espirometría es habitual la obstrucción al flujo aéreo. La afectación
pleuropulmonar por lupus se caracteriza por derrame pleural, habitualmente bilateral. Para la
sarcoidosis, esperaríamos una edad menor y en el LBA es característica la linfocitosis con aumento
del cociente CD4/CD8. El síndrome de fatiga crónica no presenta afectación parenquimatosa
pulmonar.(R1)

247. ¿Cuál de las circunstancias que se exponen NO es una contraindicación absoluta de


trasplante renal?:

1. 1. Enfermedad coronaria o cerebrovascular grave.


2. 2. Anomalías uretrales o vesicales.
3. 3. Posibilidad de mantener un estilo de vida normal con medidas conservadoras.
4. 4. Tuberculosis activa.
Gráfico de respuestas
Comentario
El trasplante renal es un tema que se disputan nefrólogos y urólogos, en los manuales de CTO lo
encontrarás en la sección de urología. En las últimas convocatorias MIR cayeron preguntas de este
tema. Las contraindicaciones absolutas para el trasplante renal son: 1. Infección activa (ej,
tuberculosis ?respuesta 5), 2. Neoplasia 3. Expectativa de vida reducida por la enfermedad de
base del paciente (ej, enfermedad coronaria o cerebrovascular grave ?respuesta 1) 4. Anticuerpos
preformados frente a antígenos del donante (tal y como dice la respuesta 2) 5. Glomerulonefritis
activa 6. Sospecha de no cumplimiento terapéutico 7. Si las medidas conservadoras (dieta,
restricción de agua y sal?) sería contraproducente realizar un trasplante. Dentro de las
contraindicaciones relativas encontramos: 1. Edad avanzada 2. Oxalosis 3. Amiloidosis 4.
Enfermedad iliofemoral oclusiva 5. Anomalías del tracto urinario (por lo tanto esta es la respuesta
buscada).(R2)

248. En relación a la enfermedad divertícular colónica, señale lo FALSO:

1. 1. Una de las complicaciones es el sangrado.


2. 2. Otra complicación es la obstrucción.
3. 3. Es un divertículo verdadero.
4. 4. La perforación es otro tipo de complicación.
Gráfico de respuestas
Comentario

 
 
 
 
La enfermedad diverticular o diverticulosis está determinada por la presencia de
PSEUDODIVERTÍCULOS (herniación de mucosa y submucosa) en el borde antimesentérico, por lo
que la respuesta 3 es la falsa.

Un divertículo verdadero, con todas las capas, sería por ejemplo el de Meckel.(R3)

249. Femenino de 25 años con historia de cefalea y claudicación de las extremidades


inferiores, presenta en la exploración física: hipertensión de las extremidades superiores
y pulsos femorales retrasados con disminución de la presión en las extremidades
inferiores. El ECG muestra datos de hipertrofia ventricular izquierda. ¿Qué diagnóstico
consideraría más correcto?:

1. 1. Insuficiencia aórtica.
2. 2. Miocardiopatía hipertrófica idiopática.
3. 3. Aorta ateromatosa.
4. 4. Coartación aórtica.
Gráfico de respuestas
Comentario

Tema y pregunta clásica no puedes fallarla. Puesto que el tema de las cardiopatías congénitas es
muy extenso, debes estudiar sólo aquellos datos característicos de cada una de ellas que te
ayuden a identificar los casos clínicos que, por otro lado, es la forma en que habitualmente se
preguntan las cardiopatías congénitas. Es fundamental que ante todo paciente joven hipertenso
sospeches siempre una coartación de aorta. En este caso además te dan otros datos que apoyan
tu sospecha, pues la clave diagnóstica consiste en la disminución y el retraso del pulso femoral, si
se compara con el pulso de las extremidades superiores. Recuerda que en el ECG se observa una
desviación del eje eléctrico a la izquierda indicativo de la existencia de una hipertrofia ventricular
izquierda.(R4)

250. La EPI es consecuencia de ETS, señale que microorganismo que tenga menos
probabilidad de causarla.

1. 1. Tricomonas.
2. 2. Bacteroides.
3. 3. Estreptococo del grupo B.
4. 4. Escherichia coli.
Gráfico de respuestas
Comentario

Caso definitivo de EPI: toda mujer con dolor abdominal con o sin síntomas agregados y cultivo de
secreción vaginal positiva para N. gonorrhoeae, C. trachomatis, M. hominis, U. urealyticum, G.
negativos, anaerobios y estreptococos. La tricomona tiene menos probabilidad de causarla.(R1)

251. Si, al hacer un ultrasonido cerebral a un prematuro, nos encontramos con imágenes
anómalas periventriculares e hiperecogenicidad en más del 50% del espacio ventricular
con dilatación ventricular, el diagnóstico es:

1. 1. Hemorragia intraventricular tipo I y áreas de leucomalacia.


2. 2. Hemorragia intraventricular tipo II y áreas de leucomalacia.
3. 3. Hemorragia intraventricular tipo III y áreas de leucomalacia.
4. 4. Hemorragia intraventricular tipo IV y áreas de leucomalacia.

 
 
 
 
Gráfico de respuestas
Comentario

La hemorragia se produce en la matriz germinal preferentemente y no en otras zonas del cerebro


debido a una debilidad intrínseca de la vasculatura en esta localización. Desde la matriz germinal,
la hemorragia se puede extender al interior del ventrículo e incluso al parénquima periventricular.
Clásicamente, la hemorragia se clasifica en 4 grados según la clasificación de Papile:

Grado I: hemorragia localizada en la matriz germinal subependimaria.

Grado II: hemorragia intraventricular sin dilatación de los ventrículos.

Grado III: hemorragia intraventricular con dilatación del ventrículo.

Grado IV: hemorragia intraparenquimatosa asociada.

Esta clasificación podría ser errónea teniendo en cuenta que el grado IV es una lesión de sustancia
blanca cuyo sustrato es un infarto hemorrágico periventricular y no una simple extensión de la
hemorragia intraventricular. Por eso, algunos autores describen únicamente los 3 primeros
grados.(R3)

252. Si una persona adulta consulta con la siguiente exploración funcional pulmonar:
Capacidad pulmonar total 110%, VEMS/CV 30% y difusión pulmonar 25%, el diagnóstico
más probable es:

1. 1. Embolismo pulmonar.
2. 2. Enfermedad intersticial idiopática.
3. 3. Enfisema pulmonar.
4. 4. Sarcoidosis.
Gráfico de respuestas
Comentario

El aumento de la capacidad pulmonar total, acompañado de una reducción severa del índice de
Tiffenau (VEMS/CV) son muy sugestivas de una enfermedad obstructiva, como el enfisema
pulmonar. Si además nos hablan de una mala difusión pulmonar, encaja todavía mejor. Recuerda
que el enfisema, debido a la destrucción de los vasos de las paredes alveolares, suele presentar
una importante disminución de la DLCO. Este dato permite diferenciarlo del bronquítico crónico
donde la DLCO no se modifica de forma tan llamativa.(R3)

253. Cual de las siguientes es una contraindicación absoluto para la lactancia materna.

1. 1. Recién nacido con fenilcetonuria.


2. 2. El niño presenta síndrome de Down.
3. 3. La madre sufre mastitis unilateral y necesita tratamiento con antibióticos.
4. 4. La madre y el niño son VIH positivos.
Gráfico de respuestas
Comentario

Debe repasar las contraindicaciones para dar lactancia ya están bien establecidas. La respuesta
correcta es la 1, ya que los niños con este tipo de padecimiento no deben consumir leche materna.

 
 
 
 
El estar infectado por VIH es una contraindicación relativa, ya que en algunos paises como en
África está aprobado, además que en la respuesta le dicen que el niño ya está infectado por VIH.
El que presente síndrome de Down no es una contraindicación en absoluto y en caso de mastitis
unilatera, la otra mama podría ser de utilidad.(R1)

254. Respecto al hipotiroidismo congénito, señale la opción FALSA:

1. 1. La causa más frecuente es la disgenesia de la glándula tiroidea.


Antes de iniciar el tratamiento es muy importante conocer la etiología, ya que las dosis de
2. 2.
tiroxina serán diferentes.
El diagnóstico precoz es importante por las consecuencias neurológicas que podría
3. 3.
desarrollar.
El tratamiento con levotiroxina se mantendrá hasta los 3 años de edad, y en este momento
4. 4.
se volverá a valorar el caso.
Gráfico de respuestas
Comentario

La principal causa es la disgenesia de la glándula tiroidea en forma de ectopia, hipoplasia o bien


agenesia (opción 1 correcta).

Es muy importante el diagnóstico temprano debido a la repercusión sobre el desarrollo psicomotor.

Aunque no se disponga de la etiología concreta, es necesario el tratamiento precoz ajustado al


peso y niveles de TSH (opción 2 falsa), que se mantendrá hasta la edad de los 3 años, momento
en que se puede retirar y se completará el estudio etiológico.(R2)

255. ¿Cuál de los siguientes factores desempeña una función importante en el desarrollo
de neoplasia cervical intraepitelial (NIC)?

1. 1. VPH.
2. 2. Coitos frecuentes.
3. 3. Espermatozoide.
4. 4. Vaginosis bacteriana.
Gráfico de respuestas
Comentario

Los factores de riesgo son los mismos que para el cáncer de cérvix y son: promiscuidad sexual,
infección por VPH, inmunosupresión, tabaquismo, déficit de a-1-antitripsina. De todos ellos, el más
importante es la infección por VPH.(R1)

256. Señale la tríada clínica más sugestiva de toxoplasmosis congénita:

1. 1. Hidrocefalia, calcificaciones intracerebrales y coriorretinitis.


2. 2. Microcefalia, calcificaciones intracraneales y púrpura trombocitopénica.
3. 3. Lesiones mucocutáneas, osteocondritis y periostitis.
4. 4. Lesiones cutáneas vesiculosas, queratoconjuntivitis y meningoencefalitis.
Gráfico de respuestas
Comentario

Pregunta básica no falla en el ENARM.

 
 
 
 
Recuerda la triada de Sabin de la toxoplasmosis congénita:

- Hidrocefalia

- Calcificaciones cerebrales difusas

- Coriorretinitis

(R1)

257. ¿Cuál es un signo clínico poco frecuente de la enfermedad de membrana hialina?

1. 1. Polipnea.
2. 2. Desviación traqueal.
3. 3. Aleteo nasal.
4. 4. Cianosis.
Gráfico de respuestas
Comentario

En los niños que padecen una EMH, aparece un distrés respiratorio de inicio inmediato tras el parto
(en menos de seis horas), que se manifiesta en forma de taquipnea, quejido intenso, aleteo nasal,
retracciónes inter y subcostales y cianosis parcialmente refractaria a la administración indirecta de
oxigeno.

En la auscultación de estos niños, suelen aparecer crepitantes en ambas bases junto a


hipoventilación bilateral. Si se produce un deterioro rápido, hay que sospechar neumotorax por
ruptura alveolar.(R2)

258. ¿Cuál de los siguientes signos corresponden habitualmente a infección del tracto
respiratorio alto?

1. 1. Rinorrea.
2. 2. Tiraje.
3. 3. Estridor.
4. 4. Disfonía.
Gráfico de respuestas
Comentario

El estridor es típico de la afectación laríngea, mientras que las sibilancias o crepitantes serían
derivados de una afectación del tracto respiratorio inferior.(R3)

259. En cuanto a las instilaciones con BCG, señale la afirmación INCORRECTA:

1. 1. La BCG produce únicamente efectos secundarios locales y son muy bien tolerados.
2. 2. El modo de acción del BCG en el tumor vesical es como estimulador de la inmunidad.
3. 3. El BCG fue utilizado por primera vez en el tumor de vejiga por Morales en 1976.
Se debe utilizar BCG en los pacientes con tumores de riesgo intermedio y elevado de
4. 4.
recidiva o progresión.
Gráfico de respuestas
Comentario

 
 
 
 
La instilación endovesical de BCG produce mas efectos secundarios que la instilación de
sustancias quimioterápicas como la Mitomicina C, Tiotepa, Adriamicina o Epirubicina. Los efectos
secundarios se clasifican en locales o sistémicos y además en cuanto a la severidad se consideran
como leves, moderados o severos. Se considera que más del 90% de los pacientes tratados con
BCG presentarán síntomas leves o moderados, mientras que se observan efectos severos entre 2
y 5% de los casos y se observan complicaciones que pueden poner en peligro la vida del paciente
en menos del 1% de los casos.(R1)

260. La menopausia tardía aumenta el riesgo de desarrollar un cáncer de mama:

1. 1. En mujeres sometidas a terapia hormonal sustitutiva (THS).


2. 2. En todas las mujeres.
3. 3. En casos de menarquia precoz.
4. 4. En multíparas.
Gráfico de respuestas
Comentario

Tema que debe dominar para fines de ENARM. Como ya debe de saber, toda mujer que se
encuentra expuesta por largos periodos de tiempo a estrógenos tienen un riesgo incrementado de
cáncer de mama y de endometrio. Entre otros FR que debes de conocer son antecedentes
familiares BRCA1 y 2, patología mamaria previa, irradiación, menarquía precoz, nuliparidad, primer
embarazo tardío, lactancia corta o artificial, adiposidad, antecedentes personales de otras
neoplasias.(R2)

261. ¿Cuál constituye la localización más frecuente de la TB genital en la mujer?

1. 1. Epitelio vesical
2. 2. Endotelio tubárico
3. 3. Tejido ovárico
4. 4. Miometrio
Gráfico de respuestas
Comentario

La localización más frecuente de la TB genital en mujeres son las trompas, y en los hombres el
epididimo. El tratamiento es el mismo que el de la TB pulmonar.(R2)

262. ¿Qué sustento tiene el test de Schiller en el diagnóstico del cáncer cervical?

1. 1. La prueba del yodo negativa detecta lesiones cancerosas.


2. 2. En que el epitelio cervical normal es acetoblanco.
3. 3. En que el epitelio canceroso no contiene glucógeno.
4. 4. Detecta lesiones en mosaico.
Gráfico de respuestas
Comentario

La prueba de Schiller se utiliza para diagnosticar tumores o neoplasias en el cuello uterino. Las
células sanas del cuello uterino se tiñen con color marrón oscuro al entrar en contacto con Lugol
debido a su alto contenido de glucógeno, mientras que las células tumorales no se tiñen, viéndose
una región pálida que indica un posible cáncer de cuello uterino. Esta prueba sencilla y poco
invasiva permite visualizar áreas patológicas para toma de muestras y permitiendo un diagnóstico
precoz de un posible cáncer.(R3)

 
 
 
 
263. Un lactante de 6 semanas de vida, nacido por parto domiciliario, sin control posterior
y alimentado a pecho exclusivo, presenta diarrea de 5 días de evolución. Llega a a la
guardia porque tiene múltiples equimosis y las evacuaciones se han hecho
sanguinolentas. Indique cuál de los siguientes es el mejor tratamiento INICIAL a
administrar por vía parenteral, mientras aguarda los resultados de los estudios
complementarios:

1. 1. Vitamina K.
2. 2. Plasma fresco congelado.
3. 3. Concentrado de factor VIII.
4. 4. Ninguna de las anteriores.
Gráfico de respuestas
Comentario

Entre los cuidados estandarizados a realizar a la hora de vida de un RN se incluyen: la profilaxis de


la conjuntivitis neonatal, la profilaxis de la enfermedad hemorrágica del RN la administración de la
primera dosis de la vacuna del VHB, la detección de enfermedades metabólicas y el cribado de
hipoacusia.

El caso que se nos presenta en la pregunta es un lactante de 6 semanas con sangrados. Date
cuenta que insisten en que fué un parto a domicilio sin controles, de ahí que seguramente no se
haya practicado lo nombrado antes, entre ellas la profilaxis de enfermedad hemorrágica que se
realiza con la vitamina K im. De ahí que el mejor tratamiento incial sea la vitamina K.(R1)

264. A 28-year-old woman who has been hospitalized in a psychiatric ward several times
(due to overdose of medications) comes to the emergency department with various cuts
in her forearms. Her family explains that today, after a little argument she suddenly
engaged in a quarrelsome and aggressive behavior, breaking her computer and locking
herself in the bathroom where she cut herself with scissors. She has chronic feelings of
emptiness and marked uncertainty about her body image, relationships and sexual
orientation. The most likely diagnosis is:

1. 1. Borderline Personality Disorder.


2. 2. Pseudoneurotic schizophrenia
3. 3. Schizoaffective disorder.
4. 4. Bipolar disorder (rapid cycler).
Gráfico de respuestas
Comentario

Es una pregunta muy sencilla en forma de caso clínico donde no tendrás ningún problemas en
contestar pues en el enunciado nos dicen que tiene de todo un poco ( antisocial, obsesivo,
neurótico...). En los trastornos de personalidad de todo un poco significa límite o borderline.
Además como clínica principal destaca problemas relacionados con la
separación- individualización ( no toleran la soledad) control afectivo pésimo, vínculos personales
intensos y problemas de autoimagen. Con múltiples gestos autolíticos, manipuladoras. Además
más frecuente en mujeres La esquizofrenia pseudoneurótica y el trastorno esquizoafectivo, se
descartan porque en estas hay un deterioro del juicio de realidad y la opcion 4, trastorno bipolar
(ciclador rápido), aunque es difícil es diagnóstico hay signos de atipicidad.(R1)

265. Paciente de 23 años de edad, polínico, que en los últimos 2 meses presenta disnea,
sibilancias, tos y opresión precordial, sobre todo al levantarse por la mañana. La

 
 
 
 
exploración física es normal, al igual que la radiografía de tórax. Se le solicita una
espirometría forzada cuyos resultados son: FEV1 4140 ml (99%), FVC 5250 ml (104%),
FEV1/FVC 79%. ¿Qué prueba debe solicitar a continuación?

1. 1. Prueba broncodilatadora.
2. 2. Test de metacolina.
3. 3. Pruebas cutáneas frente a neumoalérgenos habituales, hemograma e IgE total.
4. 4. Test del sudor.
Gráfico de respuestas
Comentario
Se trata de un paciente con sospecha de asma bronquial, según la clínica que presenta (tos,
disnea, sibilancias) con espirometría normal. Según lo explicado en la pregunta anterior, lo
indicado si la espirometría es normal es demostrar la presencia de hiperreactividad bronquial con
una prueba de provocación bronquial inespecífica con histamina o metacolina (respuesta correcta
3).(R2)

266. Un anciano de 87 años es traído al servicio de Urgencias de su hospital por presentar


un cuadro compatible con una obstrucción intestinal. Señale cuál de las siguientes es la
entidad con mayores probabilidades de ser la responsable de esta situación:

1. 1. Apendicitis aguda.
2. 2. Cáncer de colon.
3. 3. Íleo biliar.
4. 4. Invaginación intestinal.
Gráfico de respuestas
Comentario

Recuerda las causas más frecuentes de obstrucción intestinal:

- En población general: síndrome adherencial post-cirugía

- En pacientes sin antecedentes de cirugía abdominal: hernias

- En intestino grueso: cáncer de colon

Como las dos primeras causas no se dan como opciones, nos decantaremos por el cáncer de
colon.(R2)

267. Un paciente de 55 años refiere plenitud post-prandial progresiva desde hace 3


semanas. En la actualidad intolerancia a la ingesta con vómitos de carácter alimentario.
La exploración endoscópica muestra restos alimenticios en cavidad gástrica y una gran
ulceración (unos 3 cm de diámetro) en la porción distal próxima a la segunda rodilla
duodenal. El estudio anatomopatológico demostró adenocarcinoma. Refiera, de las
opciones terapéuticas siguientes, cuál es la más CORRECTA:

1. 1. El tratamiento consiste en duodenopancreatectomía cefálica.


El tratamiento del carcinoma duodenal es paliativo, por lo que debe realizarse
2. 2.
gastroyeyunostomía posterior, retrocólica, inframesocólica.
3. 3. Se debe realizar resección segmentaria con anastomosis duodenal término-terminal.

 
 
 
 
Se practicará instalación de prótesis autoexpandible con radioterapia y quimioterapia
4. 4.
sistémica.
Gráfico de respuestas
Comentario

Este caso nos orienta hacia un adenocarcinoma de la segunda porción duodenal. La intolerancia a
la ingesta con vómitos alimentarios sugiere cierto grado de obstrucción a este nivel. Al estar
localizado en la segunda porción duodenal, se trataría de un tumor periampular. El tratamiento
curativo de todos estos tumores es la intervención de Whipple, es decir, la
duodenopancreatectomía cefálica.(R1)

Duodenopancreatectomía cefálica (operaciónde Whipple) con preservación pilórica

268. En las infecciones urinarias nosocomiales el germen más frecuentemente implicado


es:

1. 1. Proteus mirabilis.
2. 2. Klebsiella pneumonie.
3. 3. Escherichia coli.
4. 4. Staphylococcus aureus.
Gráfico de respuestas
Comentario

Si bien es cierto que la frecuencia de aparición de E. coli en infecciones nosocomiales es menor


que en la población general, sigue siendo el germen más frecuenteme.(R3)

269. El síndrome de Asherman se produce por:

1. 1. Hipoplasia uterina.
2. 2. Retroflexión uterina.
3. 3. Legrados uterinos repetidos.
4. 4. Incompetencia cervical.
Gráfico de respuestas
Comentario

 
 
 
 
Para el ENARM es muy importante que conozcas el diagnóstico diferencial de las amenorreas,
porque si que suelen aparecer casos clínicos sobre esto.

Debes recordar que el síndrome de Asherman es una causa de amenorrea secundaria (falta de
menstruación de más de 3 meses en una mujer que previamente ha tenido regla), que está
producida por sinequias uterinas tras legrado. En el diagnóstico diferencial se presentaría como
una mujer con amenorrea con prueba de embarazo negativa, TSH y PRL normales que no regla ni
tras el test con progesterona ni tras el test con anovulatorios orales.(R3)

270. El neumococo es una rara causa de peritonitis. Sin embargo, es posible observar
esta entidad en pacientes con:

1. 1. Cirrosis hepática.
2. 2. Estenosis aórtica.
3. 3. Defectos en la inmunidad retardada.
4. 4. SIDA.
Gráfico de respuestas
Comentario
Aprovechamos esta pregunta para repasar los aspectos etiológicos más importantes de la
peritonitis primaria y secundaria. La peritonitis primaria es una infección sin evidencia de rotura de
víscera ni de inoculación desde el exterior. Característicamente se asocia a dos enfermedades: a)
cirrosis hepática con ascitis considerable; recibe el nombre de peritonitis bacteriana espontánea,
producida mayoritariamente por E. Coli, y por neumococo dentro de los grampositivos b) síndrome
nefrótico, producido por neumococo. Por otro lado, la peritonitis secundaria, producida tras rotura
de una víscera hueca abdominal, es ocasionada por las bacterias que constituyen la flora saprofita
del tracto digestivo, principalmente enterobacterias, anaerobios, y con menos frecuencia,
enterococos. Opción 1 correcta.(R1)

271. Mujer de 70 años de edad, que al examen físico presenta una tumoración anexial
izquierda, de 6 cm de diámetro, de superficie irregular, no móvil, no dolorosa. ¿Cuál es el
marcador tumoral que solicita para el seguimiento de este caso?

1. 1. CA-125.
2. 2. Alfa fetoproteína.
3. 3. HCG.
4. 4. Antígeno carcinoembrionario.
Gráfico de respuestas
Comentario

No se deje confundir, en un paciente adulto mayor, con un tumor de ovario, el marcador tumoral
que le servirá como seguimiento es el CA-125, pensando que es epitelial, ya que en otros tipos de
tumores se elevaran otros marcadores. Repase la siguiente tabla.(R1)

 
 
 
 

272. Paciente que, de forma súbita, comienza con una poliuria de 4 litros/día, sed intensa
y polidipsia, que le obliga a beber grandes cantidades de líquidos fríos. Se le practica una
prueba de deshidratación y la osmolaridad urinaria aumenta en un 12% tras la
administración de vasopresina. Ud. NO consideraría la causa de la enfermedad como una
de las siguientes:

1. 1. Sarcoidosis.
2. 2. Traumatismo craneoencefálico grave.
3. 3. Craneofaringioma.
4. 4. Hipercalcemia crónica.
Gráfico de respuestas
Comentario

Si se tratase de una diabetes insípida central, la osmolaridad urinaria aumenta más de un 9% al


realizarse el test, como es el caso. Por lo tanto, debemos excluir como solución correcta las causas
de DI central que aparecen entre las respuestas 1, 2, 3 y 4. Lo que deberíamos marcar es la
hipercalcemia crónica, ya que produciría una diabetes insípida nefrogénica (respuesta 5
correcta).(R4)

273. Cuando se diagnostica de fimosis a un lactante, al comentarles sobre el proceso a


sus padres tenemos que tener en cuenta que:

1. 1. Hasta los dos años se puede tratar de un proceso fisiológico.


2. 2. Es un hallazgo poco frecuente.
3. 3. Las adherencias peneanas y la fimosis se trata de una misma patología.
En la actualidad el uso de pomadas de corticoides evita la circuncisión por motivos
4. 4.
médicos.
Gráfico de respuestas
Comentario

 
 
 
 
Hasta la edad de los dos o tres años, los lactantes presentan una fimosis fisiológica en un altísimo
porcentaje de casos (respuesta 1 correcta). Por ello, es evidente que no todos deberán ser
intervenidos quirúrgicamente. Las indicaciones para la circuncisión en menores de un año son:

•   Que la fimosis produzca infecciones urinarias.


•   Balanopostitis.
•   Parafimosis de repetición.
•   Fimosis puntiforme (la que deja un orificio prepucial mínimo, de forma que obstaculiza la
libre emisión de orina).

Es cierto que los corticoides pueden ser útiles en algunos casos y evitar la circuncisión, pero esto
no puede entenderse como la regla general, de manera que la respuesta 4 es falsa.(R1)

274. Aspergillus fumigatus puede estar implicado en una amplia variedad de cuadros
clínicos. De las siguientes, ¿cuál NO es una manifestación clínica habitualmente causada
por este microorganismo?

1. 1. Neumonía en granulopénicos.
2. 2. Salpingitis.
3. 3. Otitis aspergilar de colonización.
4. 4. Colonización de cavernas tuberculosas.
Gráfico de respuestas
Comentario

Pregunta de dificultad media en la que lo más fácil es llegar a la respuesta correcta descartando
las otras opciones.

El Aspergillus fumigatus es un hongo ambiental que, aunque podrían crecer en cualquier tejido o
fluido corporal, coloniza o invade con más frecuencia el tejido subcutáneo o las membranas
mucosas. Produce infecciones invasivas, en pulmones (respuesta 1 incorrecta) o senos
paranasales, en sujetos debilitados o inmunodeprimidos.

También produce formas no invasivas en pacientes debilitados y en individuos no


inmunocomprometidos; en esos casos el hongo coloniza una cavidad preexistente en el pulmón tal
como cavernas tuberculosas (respuesta 4 incorrecta), bronquiectasias o quistes pulmonares.

Además de las infecciones pulmonares también puede generar infecciones en otros órganos o
tejidos, como otomicosis externa (respuesta 3 incorrecta), queratitis, onicomicosis, sinusitis, etc.,
pero no existe afectación de áreas ginecológicas (respuesta 2 correcta). Se estima que la
aspergilosis broncopulmonar alérgica (APBA) complica el 1-2% de los casos de asma crónica,
sobre todo corticodependiente; suele aparecer en la 3-4ª década de la vida. Los pacientes suelen
tener historia de atopia con alergia a fármacos, asma, rinitis y conjuntivitis (respuesta 5
incorrecta).(R2)

275. ¿Cuál de las siguientes es la causa más frecuente de escoliosis?:

1. 1. Osteogénesis imperfecta.
2. 2. Parálisis cerebral.
3. 3. Postraumática.
4. 4. Idiopática.
Gráfico de respuestas

 
 
 
 
Comentario
Pregunta perteneciente al bloque de Ortopedia Infantil referente a las deformidades del raquis.
Esta pregunta trata sobre una deformidad en particular como es la escoliosis. No es un tema
demasiado importante porque no lo preguntan muy a menudo, pero no está de más que le eches
un vistazo. La escoliosis se define como la desviación de la columna vertebral en el plano frontal.
Hay que distinguir la escoliosis estructurada, en la que además de la desviación de la columna ésta
rota sobre su propio eje a la vez que se incurva. De esta forma la más frecuente es la idiopática
(respuesta 4 correcta). Las demás también son causas, pero más raras: entre las congénitas
recuerda las fusiones vertebrales (hemivértebras). Recuerda también la asociación poco frecuente
de escoliosis y neurofibromatosis.(R4)

276. Nombre una paniculitis que típicamente sea de predominio lobulillar y sin vasculitis
asociada:

1. 1. Eritema indurado de Bazin.


2. 2. Esclerema neonatorum.
3. 3. PAN.
4. 4. Paniculitis de la esclerodermia.
Gráfico de respuestas
Comentario
Pregunta de dificultad intermedia, no pasa nada si se falla en primera vuelta ya que hace referencia
a conceptos olvidadizos. La opción correcta es la 3. El eritema nodoso es septal sin vasculitis, el
eritema indurado de Bazin es lobulillar con vasculitis, la PAN es septal con vasculitis, la paniculitis
de la esclerodermia es septal sin vasculitis.(R2)

277. A 73-year-old female comes to your office complaining of hip pain, that started 6
months ago. Her medical history is positive for hypothyroidism and hypertension, but the
rest is unremarkable. Physical examination reveals: BP 148/78 mm Hg, pulse 75/min and
temperature 36,9ºC. Lab tests show elevated alkaline phosphatase. Which of the following
tests would you perform to assess the extension of the disease?

1. 1. Body MRI
2. 2. Bone gammagraphy
3. 3. X-ray
4. 4. PET scan
(R2)

278. Señale cuál de las siguientes afirmaciones es FALSA respecto al cáncer de mama:

1. 1. El nivel socioeconómico puede ser un factor de riesgo.


2. 2. El carcinoma inflamatorio cursa con fiebre y leucocitosis.
La retracción del pezón puede ocurrir en cualquier estadio, y no afecta a su clasificación
3. 3.
en estadios.
Entre las técnicas de imagen empleadas en el diagnóstico, en mujeres menores de 40 años
4. 4.
se prefiere la ecografía a la mamografía.
Gráfico de respuestas
Comentario

El cáncer de mama es un tumor cada vez más preguntado en el ENARM. De hecho, incluso está
de moda a nivel social, pues con frecuencia es mencionado en televisión y en artículos de prensa.
Analicemos opción por opción:

 
 
 
 
1.- El nivel socioeconómico es, por término medio, mayor en las pacientes con cáncer de mama.
Ocurre justo lo contrario que en el cáncer de cérvix, asociado a bajo nivel socioeconómico (de
hecho, es muy frecuente en los países pobres).

2.- El carcinoma inflamatorio puede cursar en forma de placa eritematosa, pudiendo confundirse
con una mastitis. A nivel histológico, es característico el infiltrado leucocitario, como su nombre
indica. Pero no produce fiebre y leucocitosis, lo que hace falsa la opción 2.

3.- La retracción del pezón no define el estadiaje del cáncer de mama.

4.- En mujeres de menos de 40 años, la mamografía ofrece imágenes más difíciles de interpretar
que posteriormente, puesto que se trata de mamas con un mayor componente glandular. Por ello,
es preferible el USG antes de esta edad.(R2)

279. ¿Cuál es la prueba más frecuentemente alterada en el síndrome neuroléptico


maligno?

1. 1. Punción lumbar (células y glucosa).


2. 2. Fosfatasa alcalina.
3. 3. Electroencefalograma.
4. 4. CPK.
Gráfico de respuestas
Comentario

El síndrome neuroléptico maligno es una reacción adversa extrapiramidal, de carácter


idiosincrásico, que aparece en personas que toman medicación que reduce la actividad
dopaminérgica. Los fármacos más implicados son los antipsicóticos, pero también se ha descrito
con algunos antidepresivos y con el litio. Puede aparecer en cualquier momento del tratamiento,
aunque es más frecuente al principio del mismo, o cuando se aumenta la dosis. Cuanto más
potente sea el fármaco y más dosis se utilice, aumenta la probabilidad de esta complicación.
Combina cuatro tipos de síntomas:

1) Hipertermia.

2) Síntomas extrapiramidales: rigidez extrema, sobre todo.

3) Síntomas vegetativos: cambios de la presión arterial, taquicardia, sudoración.

4) Síntomas confusionales.

El peligro radica en la lesión muscular (rabdomiolisis) secundaria a la hipertermia y a la rigidez; de


ahí que el marcador analítico más usado sea la CPK. La lesión muscular puede conducir a un
fracaso renal agudo por mioglobinuria, potencialmente letal.

El tratamiento consiste en (1) retirar el antipsicótico, (2) controlar las constantes (sobre todo la
temperatura), (3) administrar un agonista dopaminérgico (bromocriptina) para revertir el efecto del
fármaco y (4) administrar dantroleno en los casos más graves (relajante muscular de acción directa
sobre el músculo).

 
 
 
 
Aparte del síndrome neuroléptico maligno, debes recordar los efectos secundarios más frecuentes
de los antipsicóticos.(R4)

280. ¿Cuál es la localización más frecuente de la angiodisplasia intestinal?:

1. 1. Estómago.
2. 2. Ciego.
3. 3. Colon transverso.
4. 4. Sigma.
Gráfico de respuestas
Comentario

La angiodisplasia de colon es una lesión degenerativa que consiste en dilatación y distorsión de


vasos de pared fina en colon, siendo la localización más frecuente en ciego, contituyendo la causa
más frecuente de este tipo de hemorragia a partir de los 60 años.

Debe sospecharse con antecedentes de estenosis aórtica. El diagnóstico se realiza mediante una
colonoscopía completa, pudiendo realizarse tratamiento con laser- argón para "quemar" las
lesiones y evitar los episodios de sangrado. Otra opción de tratamiento es la administración de
estrógenos- progestágenos que interviene en la reparación vascular. En caso de mala respuesta a
tratamiento, se indica hemicolectomía derecha.(R2)

281. A 12-year-old patient with cystic fibrosis presents to the emergency department with
hemoptysis. Differential diagnosis would include the next options EXCEPT:

1. 1. Hypersplenism with thrombocytopenia.


2. 2. Allergic bronchopulmonary aspergillosis.
3. 3. Goodpasture syndrome.
4. 4. Platelet dysfunction induced by ticarcillin.
Gráfico de respuestas
Comentario

Esta pregunta sobre hemoptisis no tiene mucha relevancia. La hemoptisis es poco frecuente en
pediatría Algunas de las causas son traqueotomía, cuerpo extraño, cardiopatía, MAV...Entre las
causas mas frecuentes están las complicaciones de fibrosis quística. Puede recordar que las
hemoptisis de pequeña cuantía (<20ml) son muy frecuentes y suelen ser síntoma de una
exacerbación respiratoria con erosión de la vía por la infección que requiere intensificar el
tratamiento con antibióticos . En caso de hemoptisis masiva (250ml) pensaremos en rotura
importante de una vaso y además de administrar vitamina K, puede ser necesario la embolización
de la arteria bronquial. En caso se aspergillosis broncopulmonar alérgica es frecuente el esputo
herrumbroso. El déficit de factores de coagulación por falta de vitamina K , la alteración plaquetaria
por el uso de antiagregantes o la trombocitopenia por hiperesplenismo pueden ser causa de
hemoptisis en la FQ. El síndrome de Goodpature (asocia hemorragia pulmonar y nefritis) no me lo
plantearía en este caso.(R3)

282. A 45-year-old woman comes to the ER complaining of severe fatigue and jaundice.
Her previous medical history is significant for untreated HCV infection which she
acquired with a blood transfusion 20 years ago. Over the past months, she has developed
ascites and has been admitted into the hospital three times for upper gastrointestinal
hemorraghe. Physical examination shows jaundice and stigmas of cirrhosis. She has
hepatosplenomegaly.There is tense ascites and severe edema in both legs which extends

 
 
 
 
to the groin. Laboratory results reveal albumin 2.2 g./dl., total bilirubin 12.1 mg/dl and
prothrombin time 20 seconds. Which of the following is the most appropiate treatment for
this patient?

1. 1. Interferon plus rivabirin


2. 2. Sofosbuvir and rivabirin
3. 3. Ribavirin
4. 4. Liver trnasplantation
Gráfico de respuestas
Comentario
Liver trnasplantation. In a patient with advanced signs of cirrhosis and portal hypertension, antiviral
treatment for HCV infection is without value and the patient should undergo evaluation for a liver
transplant.(R4)

283. Señale el método clínico utilizado para calcular la edad gestacional de un recién
nacido prematuro:

1. 1. Test de Silverman.
2. 2. Test de Dubowitz.
3. 3. Test de la hiperoxia.
4. 4. Test de Apt.
Gráfico de respuestas
Comentario

Esta pregunta de neonatología no tiene mucha relevancia, pero si puede ser útil recordar algunos
de los test mas empleados en pediatría, de los que, el que debe dominar, es el de Apgar.

El test de Silverman puntúa del 0 al 2 los signos clínicos de dificultad respiratoria: tiraje costal
retracción esternal, aleteo nasal, quejido espiratorio y disociación toracolumbar. A mas puntuación
mayor severidad.

El test de Apt se utiliza en caso de evacuaciones sanguinolentas durante los primeros días de vida
para diferenciar el síndrome de sangre deglutida de la hemorragia digestiva basándose en que la
sangre fetal es resistente a los álcalis mientras que la materna no lo es.

El test de la hiperoxia se utiliza para diferenciar la cianosis de causa cardiaca y pulmonar Consiste
en la administración de oxigeno a concentraciones elevadas cercanas al 100%. Se determina la
PO2 preductal antes y después de su administración.

El calculo de la edad gestacional es importante porque la morbimortalidad del recién nacido y sus
necesidades difieren según la edad gestacional (EG) alcanzada. Se puede calcular a partir de la
fecha de última regla ( FUM ) según la regla Naegele: FUM +10 días - 3 meses y desde el punto de
vista clínico valorando la maduración morfológica y neurológica del recien nacido. Se valoran
postura, motilidad, tono y aspecto externo (piel, uñas, pliegues, mamilas, pelo, orejas y genitales) y
con estos datos, se han hecho diferentes tests que los combinan, como los de Dubowitz, Ballard,
Capurro, Kempe, etc., que con un sistema de puntaje proporcionarán la EG, con un margen d error
de ± 2 semanas.(R2)

284. ¿Cuál de los siguientes supuestos cumple los criterios de Amsterdam-II para el
diagnóstico del síndrome de Lynch?:

 
 
 
 
Hombre diagnosticado de cáncer colorrectal (CCR) a la edad de 52 años. Padre
1. 1.
diagnosticado de CCR a los 75 años. Hermano diagnosticado de CCR a los 61 años.
Mujer de 48 años diagnosticada de CCR. Madre diagnosticada de CCR a los 78 años.
2. 2.
Prima materna diagnosticada de cáncer de mama a los 39 años.
Mujer de 71 años diagnosticada de CCR. Hermana diagnosticada de cáncer de
3. 3.
endometrio a los 48 años. Abuela materna diagnosticada de CCR a los 80 años.
Hombre diagnosticado de CCR a los 51 años. Padre diagnosticado de CCR a los 70 años.
4. 4.
Hermano diagnosticado de CCR a los 45 años.
Gráfico de respuestas
Comentario

Criterios de Ámsterdam II: Tres o más familiares con cáncer asociados a síndrome de Lych (cáncer
colorrectal, cáncer de endometrio, intestino delgado, uréter, o pelvis renal) más todos de los
siguientes:

•   Un paciente afectado debería ser familiar de primer grado de los otros dos.
•   Dos o más generaciones sucesivas deberían ser afectadas.
•   Cáncer en uno o más familiares afectados debería ser diagnosticado antes de los 50 años
de edad.
•   La poliposis familiar adenomatosa debería ser excluida de los casos de cáncer colorectal.
•   Los tumores deben ser verificados por un examen histopatológico. La única de todas las
opciones que cumple estos criterios es la número 4.

(R4)

285. A 72-year-old man with angina pectoris and recurrent syncopal episodes has a
systolic murmur radiated to both carotid arteries. A paradoxical split of the second heart
sound is heard during his last follow-up visit. Which of the following is the most likely
underlying mechanism of this auscultatory finding?

1. 1. An increasingly severe valvular stenosis.


2. 2. Left ventricle hypertrophy.
3. 3. Atrial fibrillation onset.
4. 4. A contraindication for aortic valve replacement.
Gráfico de respuestas
Comentario

En condiciones normales, la válvula aórtica se cierra antes que la pulmonar. Se habla de


desdoblamiento invertido cuanto la situación es contraria, cerrándose primero la pulmonar y más
tarde la aórtica.

El caso descrito es típico, tanto por clínica y exploración física, de una estenosis aórtica. En un
paciente con esta enfermedad, la aparición de un desdoblamiento invertido podría significar
principalmente dos cosas:

- Un bloqueo de rama izquierda, que retrasaría la contracción del ventrículo izquierdo y, con ello, la
contracción del mismo.

- Un aumento de la severidad de la estenosis (el ventrículo tarda más tiempo en vaciarse y, por
ello, se demora el componente aórtico del segundo ruido).

 
 
 
 
De estas dos posibilidades, el aumento de la severidad de la estenosis aparece en la opción
número 1.(R1)

286. Paciente femenino de 68 años, diabética, que consulta por malestar general. En la
gasometría venosa destaca pH 7.25, bicarbonato 15 mmol/l (normal 24-28 mmol/l). anión
gap: 18 mmol/l (normal 10-12 mmol/l). ¿Cúal de las siguientes entidades descartaría como
diagnóstico?

1. 1. Cetoacidosis.
2. 2. Acidosis tubular renal.
3. 3. Ingesta de salicilatos.
4. 4. Acidosis láctica.
Gráfico de respuestas
Comentario

Las preguntas sobre el equilibrio ácido-base son fundamentales. Debe dominar el Capítulo 1 de
Nefrología, que es donde aparecen explicadas.

En este caso clínico, lo que permite enfocar la pregunta es el anión GAP.

Recuerde que el anión GAP está aumentado en las acidosis debidas a la adición de un ácido
endógeno o exógeno.

Esto sucede en las opciones 1 (cetoácidos), 3 (el propio salicilato es ácido, aparte del lactato por
su toxicidad mitocondrial cuando hay sobredosis) y 4 (lactato).

Sin embargo, el anión GAP es característicamente normal en las acidosis tubulares renales.(R2)

287. Varón de 65 años, hipertenso de larga evolución con cardiopatía hipertensiva en


tratamiento con hidroclorotiacida, que acude al servicio de Urgencias por la aparición de
disnea progresiva y edemas en ambos miembros inferiores. En la analítica destaca una
creatinina de 1 mg/dl, un sodio de 131 mEq/L, potasio de 3 mEq/L y una TA de 180/100
mmHg. La aldosterona plasmática basal está elevada (al doble de su valor normal). ¿Cuál
de los siguientes enunciados es correcto?:

1. 1. Se debe pedir test de captopril para descartar HTA vasculorrenal.


2. 2. Se trata de un hiperaldosteronismo secundario y no hay que proseguir estudio.
El tratamiento de elección sería furosemida junto con tiacidas a dosis altas, para disminuir
3. 3.
los edemas y así mejorar la sintomatología.
Habría que realizar un test de sobrecarga salina para confirmar la supresión de la
4. 4.
aldosterona.
Gráfico de respuestas
Comentario
Esta pregunta hace referencia al hiperaldosteronismo. Nos presentan a un paciente que recibe
tratamiento con tiazidas, y que refiere un cuadro clínico compatible con insuficiencia cardiaca. En la
analítica destaca la existencia de hipopotasemia y una aldosterona basal elevada. Debeis recordar
que la causa más frecuente de hipopotasemia en un paciente hipertenso es el tratamiento con
diuréticos tiazídicos o de asa, luego en estos pacientes de entrada no se debe descartar un
hiperaldosteronismo primario, sino suspender el tratamiento diurético, administrar suplementos de
potasio y reevaluar en 2 semanas. Otro dato en contra de un hiperaldosteronismo primario en este
caso es la existencia de edemas periféricos, signo que no aparece en esta patología debido al

 
 
 
 
fenómeno de escape. Por útlimo recordad que la insuficiencia cardiaca es causa de un
hiperaldosteronismo secundario. En esta patología suele existir una disminución de la presión de
perfusión renal y un aumento del volumen extravascular, que conlleva el aumento de renina y
secundariamente el de aldosterona.(R2)

288. Paciente de 28 años, que presenta en vértice pulmonar derecho una lesión cavitaria
tuberculosa de 4 cm. de diámetro y paredes fibrosadas, que no se ha modificado en los
últimos seis meses con tratamiento tuberculostático; la baciloscopia es negativa. El
tratamiento más aconsejable de los que se enumeran es:

1. 1. Asociar corticosteroides.
2. 2. Resección pulmonar segmentaria.
3. 3. Toracoplastia.
4. 4. Frenicectomía.
Gráfico de respuestas
Comentario

Al ser una lesión bien localizada y que suponemos que no ha respondido al tratamiento correcto
(en este tipo de lesiones suele llegar mal el tratamiento antibiótico), entendemos que es un fracaso
del tratamiento médico. Por lo tanto, es una de las indicaciones del tratamiento quirúrgico, siendo
el más adecuado para este caso la resección de la zona afectada. La corticoterapia se utiliza en
otras lesiones en las que la reacción inflamatoria es potencialmente muy perjudicial: meningitis,
serositis...(R2)

289. Mujer de 60 años de edad, obesa, hipertensa, con sangrado transvaginal no


relacionada con el coito. El diagnóstico más probable es neoplasia de………

1. 1. Cérvix
2. 2. Ovario
3. 3. Endometrio
4. 4. Vagina.
Gráfico de respuestas
Comentario

El cáncer de endometrio se ha asociado con numerosos factores de riesgo que incluyen


hiperplasia endometrial, exposición exógena a estrógenos, obesidad, hipertensión, diabetes
mellitus, menarca temprana, nuliparidad, menopausia tardía, anovulación crónica, y radiación a la
pelvis. En contraste, el tabaquismo y los anticonceptivos orales se asocian a un riesgo menor de
presentar cáncer de endometrio.

La mayoría de las mujeres tiene uno o más episodios de sangrado uterino anormal. La gran
mayoría son mayores de 50 años, por lo que sería un sangrado posmenopáusico, y no importa si
es solo un manchado o un sangrado profuso, pero en ambos casos si dura más de 7 días, se
asocia a cáncer de endometrio. Además con la edad aumenta la prevalencia.

Debemos recordar que en México todavía la principal causa de sangrado uterino anormal es el
cáncer cérvico uterino, por lo que habrá de descartarlo en primera instancia.(R3)

 
 
 
 
290. Un paciente varón de 60 años acude a Urgencias por dolor en la rodilla, dificultad de
flexión y ligero derrame. Comenta que hace unos días al ponerse de cuclillas notó un
chasquido articular y un leve dolor. ¿Cuál es el diagnóstico más probable?

1. 1. Necrosis del cóndilo femoral interno.


2. 2. Rotura del menisco externo.
3. 3. Rotura del menisco interno.
4. 4. Cuerpo libre intrarticular.
Gráfico de respuestas
Comentario

Las lesiones de meniscos y ligamentos de las rodillas son una pregunta recurrente en el MIR, en
las que te has de fijar en varios puntos que te darán el diagnóstico. Mira el mecanismo lesional del
menisco: giro con rodilla en flexión y carga. Después fíjate en el derrame, el del menisco es ligero y
seroso. En la exploración se observará dolor en la interlínea posterior y dolor y chasquidos con
flexión de la rodilla y rotación de la pierna. La localización más usual de la lesión de menisco es el
cuerno posterior del menisco medial.(R3)

291. Paciente que presenta lesión vulvar de meses de evolución, cuya biopsia es
informada de un carcinoma infiltrante de vulva. Tras la cirugía radical se confirma la
afectación de 3 mm del estroma en una lesión de 1.8 cm y afectación del tercio inferior
uretral. El resultado de los ganglios inguinales es informado como positivo, siendo
metastásicos 4 de los 12 ganglios obtenidos. Por tanto usted estadifica la enfermedad
como:

1. 1. Estadio II.
2. 2. Estadio IB.
3. 3. Estadio IIIB.
4. 4. Estadio IIIC.
Gráfico de respuestas
Comentario

Esta pregunta teórica acerca de la clasificación TNM del cáncer de Vulva. La afectación del
estroma >1mm nos llevaría al estadio IB, sumado a la afectación de la uretra (Estadio II) y la
afectación de > de 3 ganglios, no importa el tamaño, nos lleva a clasificar la enfermedad en el
estadio IIIB. Al no existir afectación extracapsular, no se podría corresponder con el siguiente
estadio IIIC.(R3)

292. ¿Cuál de las siguientes afirmaciones sobre el linfoma gástrico NO es cierta?:

1. 1. Es el segundo tumor maligno más frecuente del estómago.


2. 2. Por lo general se trata de un linfoma de Hodgkin.
A menudo son necesarias macrobiopsias incluyendo mucosa y submucosa gástrica para
3. 3.
su diagnóstico histológico.
4. 4. Es sensible a la quimioterapia y a la radioterapia.
Gráfico de respuestas
Comentario
El linfoma gástrico prácticamente siempre se trata de un linfoma no Hodgkin de estirpe B. Asienta
sobre áreas con gastritis crónica previa o con metaplasma intestinal. Y a veces, su histopatología
es tan difusa, que requiere biopsias muy profundas para asegurar el diagnóstico. Algunos de estos
linfomas se asocian a infección por Heliobacter pylori. En los de bajo grado localizados

 
 
 
 
(normalmente MALT), asociados a H.pylori, está indicada la terapia erradicadora previo a cualquier
otro tratamiento, ya que puede ser curativa. Esto, y su quimiosensibilidad alta, les confiere un mejor
pronóstico a los linfomas gástricos que a los adenocarcinomas.(R2)

293. Paciente que tras vomitar de forma intensa y repetida los vómitos se hacen
sanguinolentos dando paso a una importante hematemesis. ¿Cómo se llama este
cuadro?:

1. 1. Síndrome de Plummer-Vinson.
2. 2. Síndrome de Paterson-Kelly.
3. 3. Síndrome de Mallory-Weiss.
4. 4. Síndrome de Potter.
Gráfico de respuestas
Comentario

El síndrome de Mallory-Weiss consiste en el desgarro y erosión de la mucosa esofágica situada


anatómicamente dentro de la cavidad gástrica e inducida por los vómitos repetidos. Generalmente
cursa con una hemorragia digestiva benigna y autolimitada y, por ello, raramente se precisa
cirugía.(R3)

294. La nefropatía diabética:

Suele ser una complicación tardía, apareciendo entre 15 y 20 años después del
1. 1.
diagnóstico.
Su aparición depende del control de la diabetes. En los enfermos mal controlados puede
2. 2.
aparecer en los primeros cinco años después del diagnóstico.
El momento de aparición de la nefropatía diabética es impredecible, pudiendo aparecer
3. 3.
precozmente o tras muchos años de evolución.
4. 4. Suele ocurrir dentro de los primeros 10 años del inicio de la enfermedad.
Gráfico de respuestas
Comentario

La nefropatía diabética es algo preguntado en el ENARM con frecuencia, sobre todo su


tratamiento, por lo que esta pregunta es básico que no la falle. Tras una primera lectura de las
opciones vemos que las opciones 1 y 4 son incompatibles y por cultura médica general debemos
saber que la opción 1 es la correcta y que generalmente, cuando hay nefropatía, ya existen
neuropatía y retinopatía asociadas. El resto de las opciones son similares en cuanto al
razonamiento que debemos utilizar en ellas. Recuerda que el control de la TA es el tratamiento
más efectivo para reducir la progresión de la enfermedad, ya que reduce tanto la lesión vascular
como la HTA intraglomerular y que los antihipertensivos más eficaces son los IECA y los ARA- II.
El control metabólico adecuado también es útil para prevenir la progresión de la afectación
reno- retiniana pero no detiene la progresión de las lesiones y mucho menos las hace
regresar.(R1)

295. Una ampolla intraepidérmica con células acantolíticas, sugiere el diagnóstico de:

1. 1. Porfiria cutánea tarda.


2. 2. Epidermolisis ampollosa distrófica.
3. 3. Penfigoide gestacional.
4. 4. Pénfigo vulgar.
Gráfico de respuestas

 
 
 
 
Comentario
Pregunta básica dentro de las enfermedades ampollosas autoinmunes, no se debe fallar. La
acantólisis es la ruptura de los puentes intercelulares (desmosomas), y es típica del pénfigo,
aunque no exclusiva (puede verse también en la enfermedad de Darier). En la porfiria cutánea
tarda, en la dermatitis herpetiforme y en el penfigoide gestacional, las ampollas son
subepidérmicas (respuestas incorrectas). En la epidermolisis ampollosa distrófica, no hay
acantólisis (respuesta incorrecta), y tampoco en los casos precedentes.(R4)

296. En el síndrome de ovarios poliquísticos se dan trastornos menstruales asociados a:

1. 1. Hipotiroidismo.
2. 2. Obesidad y resistencia a la insulina.
3. 3. Tiroiditis autoinmune.
4. 4. Anemia ferropénica.
Gráfico de respuestas
Comentario

La causa primaria del SOP está en discusión. Hay una elevación de LH, que estimula en exceso la
teca, produciendo hiperplasia tecal, dando lugar a una sobreproducción de andrógenos ováricos.
También hay una sobreproducción de andrógenos suprarrenales.

Este aumento de andrógenos provoca obesidad, hirsutismo y anovulación. La insulina estimula la


actividad aromatasa en las células de la granulosa, convirtiendo los andrógenos de la teca en
estrógenos. En las mujeres con SOP hay insulinresistencia, lo cual contribuye al aumento de los
andrógenos. La obesidad agrava la resistencia a la insulina que pueda existir, o que constituye un
factor importante de mantenimiento de la anovulación crónica.(R2)

Clínica del síndrome de ovarios poliquísticos

297. A 17-year-old female goes to the emergency department presenting with fever and
weakness. She has a history of spherocytosis and stable chronic anemia that has not
required any transfusions. She refers increasing asthenia for the past week. Blood

 
 
 
 
laboratory test results shows: HB 5 g/dL, Reticulocytes 0.2%, Platelets 200000/mm3,
Leukocyte 10000/mm3, Sodium 140 mEq/L, Potassium 3.4 mEq/L, Chloride 101 mEq/L,
Bicarbonate 20 mEq/L, Creatinine 0.7 mg/dL, Glucose 110 mg/dL. Which of the following
is the most likely diagnosis?

1. 1. Acute lymphoblastic leukemia


2. 2. Aplastic crisis
3. 3. Microangiopathic hemolytic anemia
4. 4. Acute mononucleosis
Gráfico de respuestas
Comentario
Aplastic crisis. Spherocytosis usually produces stable chronic anemia with low mean corpuscular
volume. However, marrow aplasia crises can occur related to infectious or immunosuppressive
episodes.(R2)

298. La complicación más frecuente del tratamiento con LEOC es:

1. 1. Hematuria.
2. 2. Hipertensión arterial.
3. 3. Infección.
4. 4. Hematoma renal.
Gráfico de respuestas
Comentario
La complicación más frecuente de la LEOC es la hematuria. No está demostrada que ésta pueda
producir hipertensión arterial.(R1)

299. Con respecto al diagnóstico diferencial entre la histeria y la epilepsia señale la


respuesta INCORRECTA:

1. 1. En la histeria el inicio es progresivo, mientras que en la epilepsia el inicio es brusco.


2. 2. En la histeria la recuperación es rápida, mientras que en la epilepsia es progresiva.
En la epilepsia los gritos aparecen sólo al inicio de la crisis, mientras que en la histeria
3. 3.
pueden aparecer durante la crisis.
La histeria suele finalizar en llanto mientras que la epilepsia suele seguirse de un estado
4. 4.
confusional post-crítico.
Gráfico de respuestas
Comentario
Es un diagnóstico diferencial clásico el de las crisis epilépticas respecto a las pseudocrisis
histéricas.(R2)

300. En la convulsión neonatal, la dosis de carga de fenobarbital es de …….. mg/kg de


peso.

1. 1. 10.
2. 2. 20.
3. 3. 5.
4. 4. 3.
Gráfico de respuestas
Comentario

Pregunta complicada por ser memorística.

 
 
 
 
Convulsiones neonatales: Fenobarbital (IV) 15-25 mg/kg/dosis única de carga y 5 mg/kg/día en 2
dosis de mantenimiento (para administrarlo IV efectuar dilución 1 ml luminal + 9 ml agua
bidestilada, pasar IV lento < 60 mg/min).(R2)

301. Paciente femenino de 19 años, nuligesta, acude a la consulta por irritabilidad,


inestabilidad emocional, cefaleas, sensibilidad y distensión mamaria. La sintomatología
se inicia una semana antes de la mestruación y desaparece tras presentarse la regla. Una
vez comprobada la relación con el ciclo menstrual mediante un diario de síntomas,
decidimos pautar tratamiento pues la paciente refiere que la sintomatología interfiere con
su trabajo y sus relaciones sociales. ¿Qué aconsejamos como primera elección
terapeútica?

1. 1. Aumento de ingesta de cafeína y azúcares refinados en segunda fase del ciclo menstrual.
Evitar ejercicio físico y tomar suplemento de vitamina B12 durante sintomatología a una
2. 2.
dosis de 150 mg/día.
Alprazolam 0.25 mg/8 horas, desde el día 20 del ciclo hasta el 2.º día de la regla,
3. 3.
disminuyendo después a una toma al día.
Anticonceptivos hormonales orales más un diurético suave durante los días que presente
4. 4.
distensión mamaria.
Gráfico de respuestas
Comentario

Pregunta bastante discutible. En el síndrome premenstrual se pueden emplear varias alternativas


terapéuticas (por ejemplo, podríamos administrar AINES como el ibuprofeno). Entre las opciones
que nos presentan, figuran los anticonceptivos orales, que también serían una alternativa
terapéutica. Por consiguiente, si ha marcado la opción 4 no ha de preocuparse, está casi bien. Sin
embargo, debe fijarse en el caso clínico que nos presentan.

Los síntomas que nos mencionan revelan la intención de la pregunta. Nos hablan de irritabilidad,
inestabilidad emocional, interferencia con el trabajo y relaciones sociales... Observe que apenas
nos hablan de molestias de tipo doloroso, predominando las quejas que hemos citado. Por ello,
dada la naturaleza de los síntomas, en este caso sería preferible el alprazolam.(R3)

302. Vemos en consulta a una paciente de 50 años que presenta desde hace unos meses
un temblor llamativo que empeora “cuando se pone nerviosa”. En la inspección general,
usted nota que parpadea menos de lo habitual y que su cara es bastante inexpresiva. En
la exploración física detecta una leve resistencia a los movimientos pasivos y un temblor
de reposo (4-6 Hz) que no incapacita a la paciente para sus tareas habituales. Sin
embargo, la paciente insiste en que está molesta con su situación actual. ¿Cuál sería el
tratamiento más apropiado?:

1. 1. Levodopa-carbidopa.
2. 2. Trihexifenidil.
3. 3. Amantadina.
4. 4. Pergolide.
Gráfico de respuestas
Comentario

El caso que nos presentan corresponde a una enfermedad de Parkinson, que debemos sospechar
ante el temblor de reposo a 4- 6 Hz, la inexpresividad de la cara, la resistencia a movimientos
pasivos (rigidez) y el escaso parpadeo. Al parecer, se trata de un estadio incipiente, ya que no

 
 
 
 
interfiere con sus actividades diarias, por lo que inicialmente no haría falta ningún tratamiento. Sin
embargo, la paciente insiste en que el temblor le hace estar molesta, por lo que debemos emplear
un fármaco que sea eficaz a este respecto. Recuerda que la levodopa es útil en esta enfermedad,
pero sobre todo para la rigidez y la bradicinesia, no para el temblor. Lo más eficaz para el temblor
parkinsoniano son los anticolinérgicos (respuesta 2 correcta). No obstante, debes recordar un
detalle. Estos fármacos sólo se utilizarían en pacientes relativamente jóvenes, como es el caso, ya
que en pacientes mayores tienden a producirles pérdida de memoria, lo que limita su empleo.(R2)

303. El estudio histopatológico de la pieza quirúrgica de histerectomía muestra, tras la


estadificación completa y como única focalidad neoplásica, un adenocarcinoma
endometrioide, limitado al endometrio, con aproximadamente un 30% de patrón de
crecimiento sólido. Indíquese el estadio y grado:

1. 1. Ia G2.
2. 2. Ib G3.
3. 3. Ic G1.
4. 4. Ic G2.
Gráfico de respuestas
Comentario

El adenocarcinoma endometrioide es el tipo más usual de cáncer de endometrio (80% del total).
Como está limitado al endometrio sin invasión del miometrio supone el estadio de Ia de la FIGO.
Nos hablan de aproximadamente un 30% de patrón de crecimiento sólido, lo cual supone
aproximadamente un estadio G2 moderadamente diferenciado.(R1)

304. Todas las afirmaciones son ciertas respecto a la laringitis supraglótica, EXCEPTO:

1. 1. El agente etiológico más frecuente es el H. influenzae tipo B.


2. 2. Evoluciona dando lugar a un cuadro de disnea progresiva grave.
En caso de que el niño no sea hospitalizado, debe realizarse en el domicilio del enfermo
3. 3.
la visualización de faringe y epiglotis.
4. 4. La radiografía lateral de faringe muestra una epiglotis engrosada.
Gráfico de respuestas
Comentario

En el niño, la exploración de la faringe y epiglotis es peligrosa ya que, con frecuencia se produce


un espasmo con cierre completo de la via aeria, lo que prolongado en el tiempo produciría asfixia.
Por esto, ante sospecha firme de epiglotits debemos explorarla en un quirófano con material
disponible para la intubación si fuese necesario.(R3)

 
 
 
 

305. Paciente de 30 años de


edad que acude a la consulta de reproducción asistida tras 3 años de esterilidad. Sin
antecedentes de interés. USG transvaginal normal. Reglas irregulares. FSH y estradiol el
día 4 de ciclo en los parámetros normales para su edad. Progesterona el día 21 de ciclo
con niveles muy bajos. Se realiza una histerosalpingografía cuyo resultado se muestra
en la imagen. Seminograma con REM (recuperación de espermatozoides móviles) de 2
millones. ¿Cuál es el tratamiento más correcto para esta pareja a la vista de los resultados
del estudio de esterilidad?:

1. 1. Coito dirigido.
2. 2. Fecundación in vitro.
3. 3. Inseminación artifical con semen del marido.
4. 4. Biopsia testicular.
Gráfico de respuestas
Comentario

La fecundación in vitro consiste en recoger los ovocitos mediante punción folicular (normalmente
por vía vaginal, con control ecográfico), induciendo previamente la evolución. Se ponen en
contacto los ovocitos extraídos con los espermatozoides y, una vez producida la fecundación, se
transfieren hasta tres embriones a la cavidad uterina. Las indicaciones fundamentales de esta
técnica son:

- Patología tubárica bilateral, como la obstrucción de las trompas (no es este caso, donde la
histerosalpingografía demuestra que están permeables).

- Insuficiente número de espermatozoides para realizar inseminación intrauterina.

- Fracasos repetidos con la inseminación intrauterina, en caso de hacerse.

Dado que el seminograma con REM nos muestra un número de espermatozoides inferior a 3
millones, nos decidimos por la fecundación in vitro.(R2)

 
 
 
 
306. ¿Qué actitud le parece más correcta ante una adolescente remitida desde Atención
Primaria con una curva escoliótica que no se corrige con la inclinación ventral del tronco
y un ángulo de Coob de 35º?:

Actitud expectante con controles anuales, ya que existen muchas posibilidades de


1. 1.
regresión espontánea.
2. 2. No precisa actitud específica, ya que parece una actitud escoliótica.
3. 3. Debe procederse a una instrumentación quirúrgica de la curva escoliótica.
4. 4. Tratamiento rehabilitador y corsé corrector.
Gráfico de respuestas
Comentario
Pregunta perteneciente al bloque de Ortopedia Infantil referente a las deformidades del raquis.
Esta pregunta trata sobre una deformidad en particular como es la escoliosis. No es un tema
demasiado importante porque no lo preguntan muy a menudo, pero no está de más que le eches
un vistazo. La escoliosis se define como la desviación de la columna vertebral en el plano frontal.
Hay que distinguir la escoliosis estructurada, en la que además de la desviación de la columna ésta
rota sobre su propio eje a la vez que se incurva. El ángulo de Cobb valora la magnitud de la curva
y lo forman las perpendiculares a las líneas que pasan por el platillo superior de la vértebra más
alta y el platillo inferior de la vértebra más baja de la curva (normal si es menor de 10º). El test de
Risser valora la madurez esquelética en función del desarrollo del núcleo de crecimiento de la
cresta ilíaca: tiene 5 grados. En cuanto al tratamiento en la escoliosis del adolescente entre 30 y
50º se realiza tratamiento con corsé, aunque en pacientes con Risser 0- 2 y curvas de 40- 50º se
puede considerar tratamiento quirúrgico (respuesta correcta 5). Cuidado con la 3, pues al decir que
no se corrige con la inclinación eliminamos el diagnóstico de actitud escoliótica.(R4)

307. ¿Cuál de los siguientes enunciados NO es cierto sobre la enfermedad de Alzheimer?:

1. 1. Es la causa más frecuente de demencia en el anciano.


Histopatológicamente se caracteriza áreas hematoxinófilas localizadas en lóbulo frontal y
2. 2.
temporal principalmente.
3. 3. La placa difusa es un tipo especial que está formado sólo por amiloide.
4. 4. Existe pérdida de neuronas del núcleo basal de Meynert.
Gráfico de respuestas
Comentario
Sobre la enfermedad de Alzheimer suelen preguntar más en relación con la clínica, no se suelen
centrar acerca de anatomía patológica. Probablemente es la causa más frecuente de demencia en
el anciano (R1), con degeneración progresiva y selectiva de neuronas corticales de asociación
temporales y frontales entre otras, aunque histológicamente se caracteriza por ovillos o madejas
neurofibrilares, y placas amiloides (R2, 3), aunque ninguna de las dos es patognomónicas. La
afectación del núcleo de Meynert (R4) explica el déficit en la síntesis de acetilcolina. La
degeneración neuronal produce atrofia cortical, lo cual se traduce en la neuroradiología en
aumento del sistema ventricular y espacio subaracnoideo (R5). Bibliografía: Manual Psiquiatría
CTO- Medicina 4ªEd, pág 33 Bibliografía: Manual Neurología CTO- Medicina 4ªEd, pág12.(R2)

308. Con respecto a la neumonía por estafilococo, señale lo INCORRECTO:

1. 1. Es más frecuente en niños que en lactantes.


2. 2. Es menos frecuente que la neumonía por neumococo.
3. 3. Es rápidamente progresiva y puede tener alta mortalidad.
4. 4. Puede tener complicaciones como empiema y neumatoceles.
Gráfico de respuestas
Comentario

 
 
 
 
Dadas las características del estafilococo, aunque menos frecuente que el neumococo, provoca
cuadros que pueden progresar rápidamente, con alta mortalidad y tienden a la formación de
empiemas e incluso neumatocele, siendo más frecuente su presentación en lactantes. Respuesta 1
incorrecta.(R1)

309. Un joven de 28 años presenta desde hace dos días un intenso dolor en la región
retroesternal, que se incrementa con la inspiración profunda. No tiene antecedentes
cardiológicos ni factores de riesgo coronario conocidos. En cuanto a la exploración
física, la TA es de 135/85 mm Hg y la temperatura de 37,8º C, siendo normal la
auscultación cardiopulmonar. En el electrocardiograma, aparece el ST elevado con
concavidad superior en prácticamente todas las derivaciones. ¿Cuál es, con más
probabilidad, la causa del dolor de este paciente?

1. 1. Disección de aorta.
2. 2. Pericarditis aguda.
3. 3. Angina de Prinzmetal.
4. 4. Rotura espontánea del esófago o síndrome de Boerhaave.
Gráfico de respuestas
Comentario

La edad del paciente, las características del dolor (cambia con la inspiración) y la elevación del ST
en todas las derivaciones debe hacernos pensar en una pericarditis aguda. Recuerda que, en este
caso, el tratamiento de elección serían los AINEs en dosis elevadas (antiinflamatorias) y que
debemos evitar los anticoagulantes, por el riesgo de sangrado, formándose un hemopericardio,
incluso con taponamiento cardíaco, que podría comprometer la vida del paciente.(R2)

310. Paciente de 68 años que acude al dermatólogo porque, desde hace algunas
semanas, presenta unas pequeñas ampollas en la cara de flexión de ambos brazos
bastante pruriginosos, y también en la mucosa oral. Por lo demás, su estado general es
bueno. La anatomía patológica revela una pequeña zona de despegamiento
subepidérmico con eosinófilos en su interior. La IFD muestra positividad para IgG y C3
la lámina lúcida. Todos estos datos son sugestivos de:

1. 1. Penfigoide cicatricial.
2. 2. Dermatitis herpetiforme.
3. 3. Penfigoide ampolloso.
4. 4. Pénfigo foliáceo.
Gráfico de respuestas
Comentario
Pregunta en forma de caso clínico bastante sencillo, que no debería fallarse. La edad del paciente
(mayor de 60 años), la distribución de las lesiones ampollosas en las zonas flexoras y en la
mucosa oral, el prurito, y el buen estado general del paciente, orientan clínicamente hacia un
penfigoide ampolloso de Lever. Además, la anatomía patológica y la IFD son concordantes con
este diagnóstico clínico. La opción correcta, por tanto, es la 3. En los pénfigos, la distribución de las
ampollas es diferente, el estado general del paciente malo, y no hay prurito; en la anatomía
patológica hay despegamientos intraepidérmicos por acantólisis. El penfigoide cicatricial afecta
fundamentalmente a las mucosas, con importante tendencia a la cicatrización, y con una anatomía
patológica similar al penfigoide ampolloso. La dermatitis herpetiforme produce prurito, pero es
propia de personas jóvenes, se localiza en zonas extensoras, no suele afectar a mucosas, y en la
IFD los depósitos son de IgA granular en la dermis papilar.(R3)

 
 
 
 
311. Uno de los siguientes cuadros NO es causa de insuficiencia pancreática exocrina,
señálelo:

1. 1. Malnutrición proteico - calórica.


2. 2. Alcoholismo.
3. 3. Fibrosis quística.
4. 4. Litiasis biliar.
Gráfico de respuestas
Comentario
La litiasis biliar no es causante de insuficiencia pancreática exocrina. Sí se han asociado a
insuficiencia pancreática exocrina el alcoholismo, la fibrosis quística y el gastrinoma. La
malnutrición proteico- calórica debe ser grave, con hipoalbuminemia, para producir insuficiencia
pancreática.(R4)

312. Entre las alteraciones producidas por la púrpura trombopénica trombótica NO


aparece:

1. 1. Esquistocitos.
2. 2. Leve activación de la coagulación.
3. 3. Descenso de haptoglobina plasmática.
4. 4. Inflamación perivascular en la biopsia.
Gráfico de respuestas
Comentario
El tema de la PTT no es muy preguntado directamente en el MIR, pero es importante conocer sus
características principales para hacer el diagnóstico diferencial con otras alteraciones plaquetarias
que pregunten. En este caso nos hacen una pregunta directa sobre la expresión en pruebas
complementarias de la PTT, las cuales podemos deducir con sabernos la clínica que es lo más
característico: 1) Los esquistocitos es la alteración morfológica de las células sanguíneas que
encontramos cuando existe una anemia hemolítica por traumatismo que en el caso de la PTT es
microangiopática y además en las anemias hemolíticas suele haber un descenso de la
haptoglobina por unión de la Hb que queda libre al romperse los hematíes (respuesta 3); 2)Leve
activación de la coagulación lo que se relaciona con la respuesta 5 dando lugar a trombos hialinos
intravasculares. Por lo tanto las respuestas se van descartando porque van asociadas en parejas y
la única que nos queda es la inflamación perivascular en la biopsia, la cual es falsa porque lo que
encontramos son los ya mencionados trombos hialinos SIN REACCION INFLAMATORIA (lo que la
diferencia de las vasculitis), sobre todo en pequeñas arterias.(R4)

313. Varón de 30 años, con una colitis ulcerosa conocida, que viene con signos de colitis
grave, fiebre elevada, taquicardia, depleción de volumen con desequilibrio electrolítico y
dolor abdominal. La placa simple de abdomen muestra una dilatación del colon de 7 cm
de diámetro. Señalar la FALSA:

1. 1. Puede haberlo desencadenado el empleo de loperamida.


2. 2. La localización más frecuente de la dilatación es el colon descendente-sigma.
3. 3. Si se perfora, la mortalidad es superior al 30%.
4. 4. Hay que administrar corticoides intravenosas y antibióticos de amplio espectro.
Gráfico de respuestas
Comentario

El caso clínico es típico de una complicación muy grave de la colitis ulcerosa: el MEGACOLON
TÓXICO. Por definición, se diagnostica cuando, ante una clínica compatible, se observa en la RX
simple de abdomen una

 
 
 
 
dilatación en colon transverso mayor de 6 centímetros. Inicialmente, debe tratarse con medidas
conservadoras (sueroterapia, corticoides sistémicos, antibióticos de amplio espectro). Y si en 48
horas no hay mejoría, colectomía urgente, dado el riesgo de perforación.

Es muy importante que recuerdes que la zona dilatada es el colon transverso. Ha sido preguntado
en varias convocatorias.(R2)

314. La neoplasia más frecuente de la pleura es:

1. 1. Fibroma pleural.
2. 2. Metástasis.
3. 3. Mesotelioma pleural benigno.
4. 4. Mesotelioma pleural maligno.
Gráfico de respuestas
Comentario
Las neoplasias de la pleura no son muy preguntadas en el MIR por lo que es un tema en el que no
hay que dedicar mucho tiempo. Además está pregunta se puede resolver con conocimientos
generales sobre los tumores, puesto que en general, el tumor más frecuente en cualquier
localización suele ser las metástasis, y en la pleura ocurre lo mismo. Sin embargo recuerda que
una de las excepciones aparece en neumología, ya que el pulmón es uno de los pocos órganos en
que son más frecuentes los tumores primarios que los metastásicos; además más del 90% de las
neoplasias pulmonares primarias son tumores malignos, siendo el tipo histológico más frecuente
entre ellos el carcinoma epidermoide. En cuanto a las neoplasias primarias de la pleura, tan solo
saber las características diferenciales entre el mesotelioma maligno y el benigno; la forma maligna
se relaciona con la exposición al asbesto, su manifestación más frecuente es el derrame pleural y
éste aparece casi siempre, mientras que la forma benigna no se relaciona con el asbesto, suele ser
asintomático y el derrame sólo aparece en el 10% de los casos, aunque presenta dos síndromes
paraneoplásicos típicos, como son la hipoglucemia y la osteoartropatía hipertrófica.(R2)

315. Si usted recibe un paciente con quemaduras por fuego, en una sala de urgencias,
deberá seguir como regla primaria:

Olvidarse de las quemaduras hasta establecer un diagnóstico completo del estado del
1. 1.
paciente.
Canalizar una vía venosa para perfusión de líquidos al tiempo que toma una muestra para
2. 2.
exámenes de laboraotrio e iniciar la cura mientras llega el cirujano de guardia.
3. 3. Administrar pronto la profilaxis antitetánica para evitar que se olvide.
Desbridar las ampollas provocadas por la lesión térmica mientras un asistente busca una
4. 4.
vía de perfusión de líquidos para iniciar la reanimación hidroelectrolítica.
Gráfico de respuestas
Comentario

Una pregunta que puede resolverse por sentido común.

Por supuesto, en un paciente quemado es importante valorar la extensión y profundidad de las


quemaduras. Pero existen aspectos más prioritarios, como el mantenimiento de la vía aérea
permeable y, si precisa, la administración de oxígeno. No debes olvidar que un paciente quemado
también puede haber inhalado humos, o puede proceder de un accidente y padecer lesiones de
mayor complejidad y urgencia que la quemadura en sí (respuesta 1 correcta).

 
 
 
 
El resto de las opciones no tiene sentido. Teniendo en cuenta lo dicho, no podemos dar prioridad a
la aplicación de compresas húmedas, a las curas, a la profilaxis antitetánica ni al desbridamiento
de las ampollas.(R1)

316. La causa más frecuente de quemaduras en niños esta relacionada con:

1. 1. Corriente eléctrica domestica.


2. 2. Líquidos inflamables.
3. 3. Salpicaduras con líquidos calientes.
4. 4. Ácidos fuertes.
Gráfico de respuestas
Comentario

Desde los niños que se lavan bajo el agua de un grifo demasiado caliente hasta el volcado
accidental de una taza de café, las quemaduras son un peligro potencial en todos los hogares. De
hecho, las quemaduras, especialmente aquellas producidas por agua y líquidos calientes, son
algunos de los accidentes más frecuentes que ocurren en la infancia.

Los bebés y los niños pequeños son particularmente susceptibles: son curiosos, pequeños y tienen
una piel sensible que exige protección adicional.(R3)

317. Un niño de 2 años acude con historia de 24 horas de evacuaciones líquidas


abundantes, 10 al día y vómitos. Al examen físico se observa ojos hundidos, peso de 12.5
Kg. 36.8 °C FC: 144 latidos/min. FR: 26 respiraciones/min. PA: 78/40 mmHg, extremidades
frías, llenado capilar de 3 segundos. ¿Cuál seria la mejor actitud terapéutica?

1. 1. 125 mL lactato Ringer.


2. 2. 250 mL 0.9% salino.
3. 3. 250 mL 5% Dextrosa.
4. 4. 125 mL 5% Dextrosa con lactato Ringer.
Gráfico de respuestas
Comentario

Se deben buscar los siguientes síntomas y signos en el niño con diarrea aguda, deshidratación y
datos clínicos de choque:

•   Disminución del nivel de conciencia: estuporoso o comatoso.


•   Piel pálida o marmórea.
•   Extremidades frías.
•   Taquicardia.
•   Taquipnea.
•   Pulsos periféricos débiles.
•   Tiempo de llenado capilar prolongado.
•   Hipotensión (choque descompensado).

Tratar el choque sospechado o confirmado con una infusión IV rápida de SS al 0.9% a 20ml/Kg:

•   Cuando los síntomas y/o signos de choque resuelven después de infusión IV rápida iniciar
la rehidratación con líquidos intravenosos.
•   Si el niño permanece chocado después de la primera infusión rápida:

 
 
 
 
- Inmediatamente dar otra infusión IV rápida de SS al 0.9% a 20ml/kg y

- Considerar causas posibles de choque diferentes a la deshidratación.

•   Considerar consultar al Pediatra Intensivista si el niño permanece chocado después de la


segunda infusión rápida.

(R2)

318. A 4-year-old child is brought to the pediatrician's office presenting with arthralgia
and fever. She suffered a viral cough with mild fever ten days ago that was treated with
antipyretics. Vital signs are temperature 39ºC, blood pressure 100/70 mmHg, heart rate
110 bpm and respiratory rate 22 rpm. Examination of the left knee shows a flexion
contracture that bends the joint in a fixed position with inability for extension. The joint
is swollen and erythematous. General status is affected, presenting a vomiting episode
during the visit Blood laboratory test results are: Leukocytes 15000/mm3, Hemoglobin 12
g/dL, Platelets 250000/mm3, RCP 70 mg/L, ESR 50 mm/h. Blood cultures are extracted and
X ray is pending. What is the most appropriate next step at this point?

1. 1. Arthrocentesis
2. 2. Arthroscopy
3. 3. Arthrotomy
4. 4. Empiric intravenous antibiotics administration
Gráfico de respuestas
Comentario
Arthrocentesis. The symptoms reported are suggestive of a severe systemic infection with
septicemia. We can easily locate the source of infection in the left knee. The diagnostic test
indicated is arthrocentesis. A purulent appearance of the liquid will be indicative of bacterial
infection. An initial approximation to etiology might be done by Gram Stain. Immediately after
arthrocentesis, broad spectrum antibiotics must be initiated.(R1)

319. En los últimos años se está poniendo de moda la determinación de alfa-feto-proteína


en suero materno y líquido amniótico como método de screening. Señale lo FALSO:

1. 1. Si es baja, se asocia con cromosomopatías.


2. 2. Está disminuida en el riñón poliquístico.
El triple test es la medición de los niveles de HCG, estriol no conjugado y alfa-
3. 3.
fetoproteína.
4. 4. El embarazo gemelar puede dar un falso positivo.
Gráfico de respuestas
Comentario

Pregunta fácil sobre los marcadores bioquímicos de diagnóstico prenatal, ya que es un tema en el
que hacemos hincapié en las clases. Los niveles elevados de a-fetoproteína (AFP) en suero o
líquido amniótico implican lesión de los tejidos fetales, y está elevado especialmente en los
defectos del tubo neural, además de otras patologías como atresia duodenal, onfalocele, higroma
quístico y riñón poliquístico; así, la opción 2 es la falsa.

Un descenso de la AFP es típico del síndrome de Down (opción 1). El triple test es la asociación de
HCG + AFP + estriol no conjugado (opción 3), usado en el segundo trimestre. El embarazo gemelar
(opción 4) puede dar falsos positivos en los métodos bioquímicos, ya que las tablas de normalidad

 
 
 
 
están realizadas sobre gestaciones únicas (y además varía según la edad de la paciente, edad
gestacional, tabaquismo, ...).(R2)

320. Son signos de hipoxia perinatal todos, EXCEPTO:

1. 1. Frecuencia cardiaca baja.


2. 2. Disminución de la variabilidad.
3. 3. ph <7,20.
4. 4. Dips tipo I.
Gráfico de respuestas
Comentario

Pregunta fácil sobre la monitorización fetal intraparto. En esta pregunta se nos muestran distintos
signos de sufrimiento fetal intraparto.

La bradicardia fetal es uno de los signos más importantes de sufrimiento fetal, al igual que la
taquicardia.

Una disminución de la variabilidad nos hace pensar en una hipoxia, en una hipoglucemia, fármacos
depresores del SNC.

Una microtoma de sangre fetal menor de 7.20 indica acidosis fetal moderada- grave, por lo que
existe sufrimiento fetal.

Lo que no indica sufrimiento fetal son los dips de tipo I, que son debidos a estímulos del
parasimpático fetal, sobretodo por compresión de la cabeza fetal en las últimas fases del parto.(R4)

321. De las siguientes alteraciones del cordón umbilical, ¿cuál se asocia con
malformaciones fetales?:

1. 1. Vaso accesorio de cordón.


2. 2. Inserción velamentosa de cordón.
3. 3. Arteria umbilical única.
4. 4. Laterocidencia de cordón umbilical.
Gráfico de respuestas
Comentario
El vaso accesorio de cordón ocupa sólo un extremo del mismo y desaparece en la gelatina de
Wharton. La inserción velamentosa de cordón se asocia con Vasa Previa y alteraciones en la
inserción de la placenta. El nudo verdadero de cordón y la laterocidencia son alteraciones en la
posición del mismo. La arteria umbilical única se asocia en 15- 20% de los casos con
malformaciones fetales y cromosomopatias.(R3)

322. Paciente de 28 años en programa de adelgazamiento acude a Urgencias por


palpitaciones, nerviosismo, sudoración, piel caliente; en el estudio hormonal destaca
TSH inferior a 0,03 (0,5-5) mUI/ml, T4L 3,2 (1,2-2,2) ng/dl, gammagrafía abolida y
tiroglobulina disminuida. ¿Cuál será su posibilidad diagnóstica?:

1. 1. Estruma ovarii.
2. 2. Tiroiditis crónica en tratamiento de muy larga evolución.
3. 3. Adenoma tóxico.

 
 
 
 
4. 4. Tirotoxicosis facticia.
Gráfico de respuestas
Comentario
En el estudio de un hipertiroidismo, la gammagrafía tiroidea es fundamental. En los casos de
hipocaptación gammagráfica, debemos descartar como primeras posibilidades la existencia de un
hipertiroidismo en el seno de una tiroiditis (subaguda, indolora o por una Hashitoxicosis) o una
tirotoxicosis facticia. Otras causas menos probables sería el estruma ovarii (teratoma ovárico con
tejido tiroideo diferenciado que produce hipertiroidismo), etc. Los niveles de tiroglobulina pueden
ser útiles en casos de dudoso diagnóstico diferencial, ya que estarán aumentados en las tiroiditis,
normales o levemente altos en el estruma ovario y bajos o suprimidos en los casos de tirotoxicosis
facticia.(R4)

323. Secundípara de 40 semanas de amenorrea, con monitorización interna intraparto. Se


detectan deceleraciones tipo II en el 50% de las contracciones, por lo que se decide
realizar microtoma de pH fetal, con un valor de 7,12. La dilatación es de 5 cm. ¿Qué actitud
tomaría frente a esta situación?

1. 1. Cesárea urgente.
Aumento del goteo de oxitócicos para acelerar la dilatación e intentar abreviar el
2. 2.
expulsivo.
Medir el pH materno para descartar acidosis materna, y si se confirma, administrar
3. 3.
bicarbonato.
4. 4. Usar un tocolítico hasta que se normalice el pH y después hacer cesárea.
Gráfico de respuestas
Comentario

Nos presenta una gestante a término. En una monitorización interna intraparto, se observan dips
tipo II y se realiza una microtoma de sangre fetal (que es la actitud correcta). Recordemos que los
valore normales son de 7,25- 7,45; prepatológicos, 7,20- 7,25, y patológicos por debajo de 7,20.
Ante un valor de 7,12, tenemos una acidosis grave, y la actitud es realizar una cesárea de forma
inmediata (opción 1 correcta).

Este grado de acidosis es demasiado intenso como para intentar revertirla. Por ello, la respuesta 4
es falsa, pues no se puede esperar para hacer la cesárea. Si aumentamos la dosis de oxitocina, lo
único que vamos a conseguir es que aumente el sufrimiento fetal, por aumento de las
contraciones, agravando la acidosis todavía más (opción 2 falsa). Dercartar acidosis materna como
nos dice la opción 4, podría ser válida si el valor del pH estuviera en el rango prepatológico, ya que
la acidosis materna influye poco en el pH fetal (0,01- 0,02, a lo sumo).(R1)

324. Hombre de 55 años acude al médico por astenia de 3 meses de evolución. En las
últimas 2 semanas presenta además prurito, náuseas y vómitos matutinos. Refiere
nicturia de 3-4 episodios/noche, sin síndrome prostático, desde hace años. No presenta
otra sintomatología. En los laboratorios destaca hemoglobina de 8 g/dl, creatinina 9 g/dl,
urea 290 mg/dl, sodio 137 mmol/l, potasio 6.5 mmol/l, cloro 96 mmol/l, bicarbonato 15
mmol/l, calcio 7 mg/dl y fósforo 8 mg/dl. ¿Cuál es el diagnóstico más probable?

1. 1. Insuficiencia renal aguda obstructiva por hipertrofia prostática.


2. 2. Hemorragia digestiva alta.
3. 3. Insuficiencia renal crónica.
4. 4. Insuficiencia renal aguda prerrenal.
Gráfico de respuestas

 
 
 
 
Comentario

En este caso clínico, la insuficiencia renal sería crónica con mayor probabilidad que aguda
(respuesta 3 correcta), por las siguientes razones:

- Aunque el prurito, las náuseas y los vómitos llevan solamente dos semanas, el paciente arrastra
sintomatología (astenia) desde hace varios meses.

- La intensidad de la anemia es excesiva para una insuficiencia renal aguda. Ésta también puede
producir anemia, pero suele ser mucho más leve.

Es cierto que el paciente padece clínica de prostatismo crónico (nicturia de 3- 4 episodios), pero
también nos insisten en lo mismo: “desde hace años”. Si su hiperplasia benigna estuviese
implicada de algún modo en su insuficiencia renal, también se trataría de un proceso crónico.(R3)

325. Un niño de 4 años diagnosticado de diabetes mellitus tipo 1 hace 2 meses tras un
debut con cetoacidosis diabética acude a tu consulta refiriendo reducción importante de
las necesidades de insulina, hasta la dosis actual de 4 unidades de insulina glargina y
entre 1 y 2 unidades de insulina lispro. ¿Cuál es la correcta?

1. 1. Habría que replantearse el diagnóstico de diabetes y realizar una SOG.


Suspendería la insulina porque a esas dosis tan bajas no le está haciendo prácticamente
2. 2.
efecto.
3. 3. Se trata de la fase de "luna de miel", en la que las necesidades insulínicas son bajas.
4. 4. Mantener únicamente la insulina basal.
Gráfico de respuestas
Comentario
Es frecuente que tras el diagnóstico de una diabetes 1 haya un período llamado luna de miel, que
puede durar hasta dos años, en el que las necesidades insulínicas son mínimas. Esto se debe a
una reserva pancreática aun mantenida. Las dosis de insulina serán bajas pero no se recomienda
suspender los bolos de insulina ya que con el tratamiento insulínico intensivo podemos prolongar
esta fase.(R3)

326. Indique cuál de las siguientes afirmaciones NO le parece correcta:

La hepatitis por el virus D se evita con una vacuna recombinante que induce la formación
1. 1.
de anti-HBs.
2. 2. La hepatitis fulminante de origen vírico suele ser causada por el virus tipo C.
La coinfección aguda por virus B y D no induce mayor tendencia a la cronicidad que la
3. 3.
infección aislada por virus B.
4. 4. La infección por el virus C tiende al desarrollo de la hepatitis crónica y de la cirrosis.
Gráfico de respuestas
Comentario

La hepatitis D es producida por un virus RNA defectivo que requiere la presencia de virus B para
poder infectar, de forma que esta infección se prevendrá del mismo modo que el virus B. La
coinfección por ambos virus aumenta la probabilidad de hepatitis fulminante, sin embargo la tasa
de cronicidad es la misma que para la hepatitis B aislada. La infección por virus C cronifica en el
80% de los casos y en torno al 50- 60% desarrollará cirrosis, sin embargo no se trata de una
hepatitis fulminante (opción 3 falsa), salvo en casos muy concretos (pacientes con disfunción
hepática severa previa, por otra causa).

 
 
 
 
Hasta un 40% de los pacientes por virus C desconocen el mecanismo por que contrajeron dicha
infección. Sin embargo, tras analizar con estudios genéticos virales y ahondar en las posibles
causas, esta tasa se reduce y los mecanismos descritos son, fundamentalmente, sanguíneo y
perinatal. La transmisión vía sexual es posible, pero improbable.(R2)

327. La artritis reumatoide suele respetar una de las siguientes articulaciones:

1. 1. Rodillas.
2. 2. Columna cervical.
3. 3. Codos.
4. 4. Interfalángicas distales.
Gráfico de respuestas
Comentario
La AR afecta a articulaciones periféricas y como idea global de forma más frecuente aquellas que
sean más distales. Sin embargo debes recordar la excepción que constituyen las articualciones
más distales del organismo. Ni las IFD de las manos ni las interfalangicas de los dedos de los pies
se suelen afectar. Otra precisión respecto a la distribución articular es lo que ocurre a nivel del
esqueleto axial. El único tramo de la columna que se afecta es la columna cervical y ni la columna
dorsal ni la lumbar son asiento de la sinovitis característica de la enfermedad.(R4)

328. Un paciente con diagnóstico de depresión mayor es traído al servicio de Urgencias.


Se sospecha un intento autolítico con dosis altas de paracetamol. Indique la afirmación
con la que NO está de acuerdo:

1. 1. Si han pasado más de 72 horas de la ingesta, el peligro ya ha pasado.


El tratamiento inmediato consiste en lavado gástrico, medidas de sostén y carbón activado
2. 2.
oral, si han pasado menos de 30 minutos de la ingesta.
3. 3. Si el paciente se recupera, probablemente no presente secuelas hepáticas.
Si previamente ha ingerido alcohol, esto puede haber potenciado el efecto tóxico del
4. 4.
paracetamol.
Gráfico de respuestas
Comentario

La intoxicación por paracetamol produce inicialmente una clínica poco específica, con síntomas
digestivos (náuseas, vómitos, dispepsia…). Más adelante, cuando han pasado 48- 72 horas, se
produce lo verdaderamente típico: una hepatitis fulminante de origen tóxico. Por ello, que hayan
pasado 72 horas desde la ingesta no significa que esté exento de riesgo.

La hepatitis por paracetamol puede ser mortal en el momento agudo, pero si se recupera es difícil
que produzca secuelas. El antídoto de esta intoxicación es la N- Acetil- Cisteína, cuya molécula
contiene un grupo sulfhidrilo, y esto contribuye a reponer el glutatión deficitario.(R1)

329. Todas las afirmaciones siguientes acerca del crecimiento y desarrollo fetal son
ciertas, EXCEPTO:

1. 1. Los movimientos respiratorios no aparecen hasta el nacimiento.


2. 2. Los movimientos deglutorios comienzan tan pronto como a las 14 semanas del embarazo.
3. 3. La hemoglobina es principalmente fetal.
4. 4. El reflejo de prensión aparece a las 17 semanas.
Gráfico de respuestas
Comentario

 
 
 
 
Esta pregunta acerca del desarrollo fetal no tiene mucha relevancia y se contesta por lógica. La
contracción muscular del feto comienza a las 8 semanas seguidas de movimientos de flexo
extensión lateral. Los movimientos respiratorios y deglutorios comienzan a las 13-14 semanas. El
reflejo de prensión aparece a las 17 semanas y a las 27 esta desarrollado del todo. Recuerde que
la hemoglobina fetal predomina hasta los 6 meses de vida postnatal.(R1)

330. Paciente primigesta de


32 semanas de gestación. Acude a urgencias por sensación de dinámica. Exploración:
cérvix cerrado y formado. RCTG: 2 contracciones en 10 minutos, FCF basal 140 latidos,
RAF positivo. Se realiza USG transvaginal para la medición de la longitud cervical,
obteniéndose la imagen que se muestra. ¿Cuál de las siguientes afirmaciones es
INCORRECTA?:

Una longitud cervical como la de la imagen tiene un alto valor predictivo negativo para
1. 1.
parto pretermino.
2. 2. Se aconseja reposo en esta paciente.
3. 3. Si el test de la fibronectina es positivo, es un factor de riesgo para parto pretermino.
4. 4. Ante 2 contracciones en 10 minutos, es necesario instaurar tocólisis.
Gráfico de respuestas
Comentario

La imagen ecográfica adjunta nos muestra una longitud cervical considerable (línea punteada). Tal
como expresa la respuesta 1, es muy poco probable que esta situación termine en un parto
pretérmino. Más bien parece una amenaza de parto pretérmino, pero de bajo riesgo (cérvix no
modificado, ausencia de antecedentes de parto pretérmino, no se trata de una gestación múltiple).
En estos casos, no es obligado instaurar tocolisis, aunque tengamos dinámica uterina (respuesta 4
falsa). Puede ser suficiente con recomendar reposo y alta domiciliaria. En el caso de que no ceda
la dinámica uterina, puede valorarse un período de observación de 12- 24 horas inicialmente.(R4)

 
 
 
 
331. Niña de 10 años de edad, que al examen físico presenta mama y pezón elevados, formando una
pequeña prominencia; vello pubiano escaso, poco pigmento, liso, en el borde interno de los labios.
La madurez sexual según Tanner corresponde al estadio:

1. 1. 4.
2. 2. 1.
3. 3. 3.
4. 4. 2.
Gráfico de respuestas
Comentario

Concepto importante para el nacional el Tanner.

Corresponde a un Tanner 2:

Esbozo de las mamas palpables, aumento de la aréola, con vello mínimamente recio, pigmentado
especialmente sobre los labios Edad promedio 10-11 años.(R4)

332. Ante una paciente de 40 años de edad que presenta un cuadro caracterizado por
Raynaud, poliartritis no deformante, lesiones cutáneas eritematosas en mejillas y
debilidad muscular proximal y cuyo dato de laboratorio más importante es la presencia
de títulos altos de anticuerpos anti-RNP, la primera sospecha diagnóstica debe ser:

1. 1. AR (artritis reumatoide).
2. 2. LES (lupus eritematoso sistémico).

 
 
 
 
3. 3. Polimiositis.
4. 4. EMTC (enfermedad mixta del tejido conectivo).
Gráfico de respuestas
Comentario
La enfermedad mixta del tejido conectivo (EMTC) no es de lo más importante. Es una entidad bien
definida que reúne determinados síntomas de varias enfermedades reumatológicas, y su
característica primordial e imprescindible es la positividad de los anti- RNPs (si tuviera que recordar
un dato de la enfermedad, sería éste). Tiene cualidades de: 1) Esclerodermia, como fenómeno de
Raynaud, edema de manos y esclerodactilia (limitada a dedos); 2) Miopatía inflamatoria, con
debilidad muscular; 3) Lupus, con eritema malar y fotosensibilidad; 4) Artritis reumatoide, con
sinovitis. Sin embargo no afecta a vísceras ni es deformante como las entidades aisladas. Además
asocia neuropatías y neuralgias, Sjögren,?.(R4)

333. Masculino de 25 años, DM1 en tratamiento con insulina de 10 años de evolución bien
controlado, con cifras de HbA1c: 5.5% que progresivamente ha tenido que disminuir la
dosis de insulina necesaria para su control sin causa justificable. Como antecedentes el
paciente presenta tiroiditis crónica autoinmune y áreas de vitíligo dispersas. Con estos
datos, ¿qué prueba diagnóstica solicitaría?:

1. 1. Anticuerpos antitiroideos.
2. 2. Test de estímulo con ACTH para cortisol.
3. 3. Ecografía abdominal.
4. 4. Test de sobrecarga oral de glucosa para medir glucosa e insulina.
Gráfico de respuestas
Comentario

Los síndromes poliglandulares autoinmunes (PGA) son patologías raras, que difícilmente se
preguntarán en el ENARM. Nos encontramos con un paciente que tiene diagnostico de diabetes
mellitus tipo I y que ahora tiene que disminuir sus dosis de insulina por hipoglucemias de causa no
conocida. Entre sus antecedentes cuenta con tiroiditis crónica autoinmune y vitíligo (otra
enfermedad autoinmune). Sospechamos pues un síndrome PGA tipo II o de Schmidt que asocia
característicamente diabetes tipo I, insuficiencia suprarrenal y tiroiditis. Es por eso que por los
síntomas de hipoglucemia, debemos descartar la existencia de adrenalitis autoinmune mediante un
test de estímulo con ACTH.(R2)

334. Niño de 8 años que acude al servicio de urgencias por fiebre de 3 días de evolución
y torticolis de 6 horas de evolución. Suele tener amigdalitis de repetición. El niño no se
deja explorar. ¿Qué prueba le pueda dar el diagnostico?

1. 1. Una BH con recuento leucocitario.


2. 2. TAC cervical.
3. 3. Rx de columna cervical ósea.
4. 4. Hemocultivos.
Gráfico de respuestas
Comentario
Lo más probable es que este niño tenga una infección de los espacios profundos del cuello, por lo
que el TAC es la prueba indicada.(R2)

335. Señale, de entre los siguientes, cuál se considera tratamiento de elección de la


brucelosis:

 
 
 
 
1. 1. Rifampicina más amikacina.
2. 2. Doxiciclina más rifampicina.
3. 3. Doxiciclina más penicilina.
4. 4. Esparfloxacino.
(R2)

336. La gasometría arterial se realiza habitualmente por punción de:

1. 1. Arteria radial.
2. 2. Arteria humoral.
3. 3. Arteria femoral.
4. 4. Arteria subclavia.
Gráfico de respuestas
Comentario
La gasometría arterial se realiza habitualmente por punción en arteria radial.(R1)

337. Señale lo FALSO con respecto a la encefalopatía hipóxico-isquémica:

1. 1. Ocurre fundamentalmente en RN a término.


2. 2. Las lesiones se localizan en sustancia gris cortical y subcortical.
3. 3. Las convulsiones son una manifestación frecuente en la encefalopatía estadio 1.
4. 4. Las convulsiones suelen aparecer en las primeras 24 horas.
Gráfico de respuestas
Comentario

La hipoxia daña de forma difusa el SNC de los recién nacidos a término, dando lugar a la llamada
encefalopatía hipóxico- isquémica. Afecta a muchas zonas cerebrales, tanto a la sustancia blanca
como a la gris. Sus dos secuelas principales son: el desarrollo de parálisis cerebral infantil y el
desarrollo de convulsiones. Éstas aparecen en los estadios más avanzados, y de forma
precoz.(R3)

338. In the primary diagnostic workup of a patient with secondary amenorrhea, which of
the following hormonal blood level should be measured?

1. 1. TSH.
2. 2. ACTH.
3. 3. Dopamine.
4. 4. Estrone.
Gráfico de respuestas
Comentario

Debe manejar con soltura el algoritmo diagnóstico de las amenorreas secundarias., es un tema
importante.

Ante una amenorrea secundaria, lo primero que debemos descartar es una gestación, y para ello
realizamos, en primer lugar un test de embarazo.

 
 
 
 
Si el test es negativo, a continuación es necesario determinar los niveles de TSH (RC-1) y PRL,
que también pueden dar lugar a amenorrea. Si están alterados, haremos el tratamiento
etiológico.(R1)

339. Una paciente de 28 años acude por amenorrea secundaria. Tras el tratamiento con
progesterona no se obtiene regla, y tampoco tras el tratamiento con anovulatorios. La
etiología debe localizarse en:

1. 1. Hipotálamo.
2. 2. Ovario.
3. 3. Útero, cérvix o vagina.
4. 4. Eje hipotálamo-hipófisis-gonadal.
Gráfico de respuestas
Comentario

Si en el estudio y tratamiento de una amenorrea secundaria no se obtiene respuesta con el


tratamiento hormonal, es posible se puede tratar de una alteración a nivel de útero, cervix o
vagina.(R3)

340. Respecto a la varicela, señale lo CORRECTO:

1. 1. La aspirina está indicada como tratamiento sintomático de la varicela.


2. 2. En una neumonía por varicela no está indicado el tratamiento con aciclovir.
Niños con síndrome nefrótico que reciben corticoides deberían recibir la gammaglobulina
3. 3.
antivaricela en las 72 horas postexposición.
4. 4. En inmunosuprimidos no está indicado el tratamiento con aciclovir.
Gráfico de respuestas
Comentario

Pregunta de dificultad media-alta sobre la varicela.

La opción correcta es la 3, ya que un niño que recibe altas dosis de esteroides debemos
considerarlo un inmunodeprimido y por lo tanto debería recibir la profilaxis postexposición con la
gammaglobulina anti- VVZ.

En cuanto al resto de las opciones, no debemos dar aspirina para prevenir la aparición de
síndrome de Reye, tanto en la neumonía por varicela como en los inmunodeprimidos debemos
tratar con aciclovir y también está indicada la profilaxis en RN cuya madre desarrolla varicela para
prevenir la varicela perinatal.(R3)

341. Mujer de 32 años que consulta por esterilidad primaria de 2 años. Antecedentes
personales: meningitis en la infancia que curó sin secuelas; amigdalectomía y
apendicectomía; no refiere alergias a medicamentos. Sin hábitos tóxicos. Casada.
Cónyuge de 24 años sin antecedentes de interés, salvo fumador de 20 cigarrillos al día.
Dentro del estudio básico de esterilidad, señale qué prueba NO está incluida:

1. 1. Serologías (VHB, VHC, VIH, Toxoplasma, Rubéola, Sífilis).


2. 2. Histerosalpingografía.
Examenes de laboratorio hormonal que incluya: estradiol, FSH, LH, progesterona y
3. 3.
prolactina entre los días 7 y 9 del ciclo menstrual.
4. 4. Seminograma.

 
 
 
 
Gráfico de respuestas
Comentario

Debe tener ciertos conocimientos sobre el estudio de la pareja infértil. Lo primero es realizar un
buena anamnesis, exploración (citología triple toma) y laboratorio (BH, bioquimica y serologías
frente a rubeola, toxoplasmosis, sifilis, hepatitis B, C y VIH). También se realiza un ultrasonido
transvaginal y se valora la ovulación mediante la determinación de FSH, LH y estradiol en la fase
proliferativa entre los días 3 y 5 del ciclo, entre los días 20-22 (fase lútea) determinamos
progesterona y prolactina. Se añade también seminograma donde se estudia el número, movilidad
y morfología de los espermatozoides y si es normal se realiza el test de capacitación espermática.
También se realiza una histerosalpingografía que permite valorar obstrucción tubárica o uterina y
ocasionalmente puede resultar terapéutica al repermeabilizar.(R3)

342. Las alteraciones de la variabilidad del registro cardiotocográfico se caracterizan por


todo lo siguiente EXCEPTO:

1. 1. El aumento de variabilidad constituye el signo más precoz de hipoxia fetal leve.


La hipoxia leve produce aumento de la variabilidad que a corto plazo se debe a un
2. 2.
aumento del tono simpático fetal.
3. 3. La variabilidad baja se debe frecuentemente a sueño fetal.
4. 4. El ritmo sinusoidal suele deberse a isoinmunización Rh grave.
Gráfico de respuestas
Comentario

Una pregunta bastante difícil sobre la interpretación del registro cardiotocográfico. No se preocupe
si falló la respuesta.

La hipoxia leve produce un aumento de la variabilidad, pero no es debida (a corto plazo) a un


aumento del tono simpático, sino del parasimpático, lo que permite economizar glucógeno a nivel
miocárdico. Si la hipoxia persiste, entonces aparecería el aumento del tono simpático, pero siempre
en segundo lugar, contribuyendo a redistribuir el flujo sanguíneo hacia cerebro, corazón y
suprarrenales.(R2)

343. ¿A partir de que mes de edad se considera retraso en la erupción dental?

1. 1. 10 meses.
2. 2. 13 meses.
3. 3. 8 meses.
4. 4. 9 meses.
Gráfico de respuestas
Comentario

Debe dominar todo sobre crecimiento y desarrollo cada año hay preguntas sobre esto en el
ENARM. La erupción dentaria suele comenzar con los incisivos centrales inferiores a los 6 o 8
meses. A los dos años, generalmente, están todos los dientes. La caída empieza alrededor de los
6 años, seguida de la erupción de los primeros molares como dientes definitivos. Se considera
retraso de la erupción, la ausencia de piezas dentarias a partir de los 13 meses, siendo la causa
idiopática la más frecuente.(R2)

344. Hombre de 63 años acude a su consulta por un dolor sordo en epigastrio que se
irradia a la espalda. Ha perdido peso y no se encuentra bien. Su familia está preocupada

 
 
 
 
porque cada día le encuentran más deprimido. Está ligeramente ictérico y puede palparse
la vesícula ligeramente agrandada. ¿Qué marcador tumoral solicitaría usted, entre otros
estudios?

1. 1. Alfa-fetoproteína.
2. 2. CA 19.9.
3. 3. Gonadotropina coriónica humana.
4. 4. CA 125.
Gráfico de respuestas
Comentario

Existen múltiples factores que nos orientan a cáncer de páncreas:

- Dolor en epigastrio con irradiación a espalda.

- Ictericia con vesícula palpable (signo de Courvoisier- Terrier).

- Malestar general y pérdida de peso.

Por ello, el marcador tumoral que probablemente estaría elevado es el CA 19.9 (respuesta 2
correcta). El resto de los marcadores se elevan en otros tumores:

- Alfa- fetoproteína: Tumores testiculares (excepto seminoma), hepatocarcinoma.

- Antígeno carcinoembrionario (CEA): Tumores digestivos, sobre todo cáncer de colon.

- Beta- hCG: Tumores gonadales, sobre todo coriocarcinoma (recuerde: gonadotropina


CORIÓnica…).

- CA 125: Ovario. Leve aumento en la endometriosis.

(R2)

345. Una mujer de 22 años cuenta dolores torácicos atípicos, demostrándose en el ECG
T bifásica o invertida en las derivaciones II, III y aVF. En el ecocardiograma se demuestra
un desplazamiento en la proyección paraesternal de las valvas de la válvula mitral que
coaptan en un plano más alto que el anillo mitral. La paciente presenta dolores torácicos
frecuentes e insuficiencia mitral ligera-moderada. No presenta disnea ni dilatación o
disfunción del ventrículo izquierdo. ¿Cuál es el tratamiento idóneo?:

1. 1. Betabloqueantes empíricamente.
2. 2. Cirugía de sustitución valvular mitral.
3. 3. Cirugía de reparación de la válvula mitral.
4. 4. Anticoagulación oral.
Gráfico de respuestas
Comentario
Esta pregunta es complicada, pero el caso es típico e importante para el MIR. Nos presentan un
caso clínico típico de Prolapso de la válvula mitral (mujer joven + dolores torácicos atípicos + IM +
ecocardiograma diagnóstico). Como la IM es ligera- moderada y no presenta disnea, ni dilatación o
disfunción del VI, no está indicado el tratamiento quirúrgico de la IM. El tratamiento de elección del

 
 
 
 
prolapso mitral sintomático son los betabloqueantes. Recuerda que en los casos asintomáticos no
se precisa tratamiento. Esta pregunta es difícil, ya que debes combinar los conocimientos sobre el
tratamiento de la IM y el tratamiento del prolapso mitral.(R1)

346. Un varón de 35 años fue diagnosticado a los 18 años de esquizofrenia paranoide,


habiendo sufrido varios brotes psicóticos hasta la fecha. Desde hace un tiempo la clínica
del paciente se caracteriza por una gran pobreza del lenguaje, aislamiento social
marcado, anhedonia y aplanamiento afectivo. Ya no presenta delirios ni alucinaciones
auditivas claras. Usted escogería para el tratamiento de este paciente:

1. 1. Haloperidol.
2. 2. Risperidona.
3. 3. TEC.
No existe ningún tratamiento que al menos mitigue parcialmente para los síntomas
4. 4.
negativos de la esquizofrenia.
Gráfico de respuestas
Comentario

Dentro de los trastornos psicóticos, hay que tener claros los síntomas de la esquizofrenia, las
diversas formas clínicas y las diferencias entre esquizofrenia y paranoia.

Nos encontramos ante un caso de esquizofrenia residual, que se diagnostica cuando después de
un episodio esquizofrénico desaparecen los síntomas positivos, pero persisten los negativos. Suele
ser la fase final común de muchos pacientes.

Nos piden cual es el tratamiento más adecuado para este paciente, por lo que debemos elegir uno
que actúe fundamentalmente sobre los síntomas negativos, y el más adecuado de todos ellos es la
risperidona, un neuroléptico atípico que mejora tanto los síntomas positivos como negativos con
pocos efectos extrapiramidales.

El haloperidol y la clorpromacina son neurolépticos clásicos con escasa o muy poca acción sobre
los síntomas negativos; y la TEC se usa en cuadros esquizofrénicos resistentes a tratamiento
médico.(R2)

347. Which of the following clinical manifestations is not usually seen in primary
antiphospholipid syndrome?:

1. 1. Stroke.
2. 2. Hemolytic anemia.
3. 3. Hemorrhagic purpura.
4. 4. Livedo reticularis.
Gráfico de respuestas
Comentario

El síndrome antifosfolípido puede aparecer tanto en hematología como en reumatología, por lo que
basta con estudiarlo en un sitio y siempre que aparezca en otro recurrir a lo ya estudiado. El
síndrome fosfolipídico primario sucede en individuos sin enfermedad de base relevante,
generalmente jóvenes, con asociación de trombocitopenia moderada, abortos de repetición y
trombosis tanto en territorios venosos como arteriales, pero no se asocia a hemorragia (por eso la
opción 3 es la incorrecta). En el 70% de los casos coexiste en el suero de estos enfermos un
anticoagulante lúpico y anticuerpos anticardiolipina. Desde el punto de vista de laboratorio, la

 
 
 
 
alteración típica es una prolongación del tiempo de tromboplastina activada y/o tiempo de
protombina con incapacidad para corregirlo con la administración de plasma normal. El tratamiento
de los episodios trombóticos es el convencional.(R3)

348. La enfermedad de Perthes de la cadera infantil es un proceso de etiología:

1. 1. Congénita.
2. 2. Traumática.
3. 3. Inflamatoria.
4. 4. Isquémica.
Gráfico de respuestas
Comentario
La enfermedad de Perthes (también llamada coxa plana) es una enfermedad importante en el MIR
dentro del bloque de trauma y conviene estudiarla a fondo. Es una isquemia de la extremidad
proximal del fémur en crecimiento que condiciona su osteonecrosis y posterior revascularización y
reosificación (respuesta 5 correcta). La mayor parte de los casos ocurren en varones (4- 5 veces
más frecuente que en mujeres), unilateral en el 90% y su edad típica está entre los 4 y los 8 años.
Está en entredicho su relación con la sinovitis transitoria de cadera. La evolución suele ser benigna
y sólo es necesario el tratamiento quirúrgico en un pequeño porcentaje de casos de mala
evolución.(R4)

349. En la edad pediátrica, ¿la infección oportunista más frecuente en el SIDA es?

1. 1. Infección diseminada por Mycobacterium avium intracellulare.


2. 2. Neumonía por Pneumocystis jirovecii.
3. 3. Infección crónica por virus Herpes simplex.
4. 4. La toxoplasmosis.
Gráfico de respuestas
Comentario

Pregunta de dificultad media- alta.

El SIDA pediátrico es un tema poco preguntado. La respuesta correcta es la 2. Debemos de saber,


en comparación con el SIDA del adulto, que las infecciones oportunistas infantiles son menos
frecuentes pero de mayor gravedad.

Entre ellas, la más frecuente es la neumonía por P. jirovecii, que además es la causa más
frecuente de muerte en estos niños.

El tratamiento de elección de esta patología, como en el adulto, es el trimetoprim-


sulfametoxazol.(R2)

350. Paciente con carcinoma de pulmón que, en un análisis de orina, presenta proteinuria
no selectiva de 5 gramos/día, hematíes abundantes, sin hematuria macroscópica. En la
sangre encontramos una albúmina de 2,5 g/dl. El FG es normal y no existe hipertensión
arterial. En este contexto, ¿cuál de las siguientes nefropatías es la causa más probable?:

1. 1. Glomerulonefritis membranosa.
2. 2. Glomerulonefritis rápidamente progresiva.
3. 3. Glomerulonefritis mesangial de IgA.
4. 4. Invasión del riñón por el tumor.

 
 
 
 
Gráfico de respuestas
Comentario

La causa más frecuente de síndrome nefrótico en el adulto es la GN membranosa. Ante un adulto


con síndrome nefrótico, se debe realizar una biopsia renal. Esto es así porque este síndrome
puede deberse a otras varias glomerulonefritis (GN mesangiocapilar, GN focal y segmentaria,
algunas GNs lúpicas, amiloidosis, etc…). Si se trata de una GN membranosa, como en este caso,
encontraríamos una histología como la que describen en la pregunta: engrosamiento difuso,
“spikes” o espigas con la tinción de plata, celularidad glomerular normal o casi normal… En esta
pregunta, no debes pasar por alto el cáncer microcítico de pulmón, ya que los tumores sólidos se
han relacionado con la GN membranosa.

Sobre la nefropatía membranosa, también es importante que recuerdes que el nivel sérico de
complemento se mantiene normal, a diferencia de otras glomerulonefritis, como la
mesangiocapilar.(R1)

351. Los epiteliomas basocelulares:

1. 1. Su aparición no está relacionada con la exposición al sol.


2. 2. Son los epiteliomas cutáneos más frecuentes.
3. 3. Afectan sobre todo a mujeres.
4. 4. Su edad media de aparición son los 70 años.
Gráfico de respuestas
Comentario
Pregunta básica sobre el tumor maligno más frecuente de la especie humana, el carcinoma
basocelular (respuesta 3 correcta). Debes conocer algunas peculiaridades del basalioma, que
aprovechamos para resumir en este comentario. Es absolutamente excepcional que metastatice, a
diferencia del carcinoma epidermoide, que sí lo hace con cierta frecuencia. Su aparición se
relaciona con la exposición solar, por lo que es más frecuente en zonas fotoexpuestas,
normalmente en la cara. Recuerda que el carcinoma epidermoide suele surgir sobre piel dañada,
mientras que el basocelular lo hace en áreas sanas. Son más frecuentes en varones,
probablemente porque las labores como la agricultura, la albañilería y otras profesiones que
implican muchas horas de fotoexposición son más frecuentes en el varón que en la mujer. Existen
muchas variedades de basalioma, pero la presentación más frecuente es como una pápula de
aspecto blanquecino, brillo perlado y con alguna telangiectasia. Crece progresivamente y puede
llegar a ulcerarse en su centro, formándose costras.(R2)

352. El agente más frecuente de la epiglotitis aguda es:

1. 1. Haemophilus influenzae b.
2. 2. Estreptococo beta hemolítico.
3. 3. Estafilococo.
4. 4. Proteus mirabilis.
Gráfico de respuestas
Comentario

Pregunta de dificultad baja que ya debe quedar muy clara en la primera vuelta.

Debemos saber que la respuesta correcta es la 1, Haemophilus influenza tipo b. Este cuadro es
una emergencia en pediatría y no debemos hacer ninguna maniobra exploratoria ni pasar ningún

 
 
 
 
tratamiento hasta que tengamos controlada la vía aérea, puesto que la inflamación y el edema en
la epiglotis puede colapsar totalmente la vía aérea. El tratamiento de elección es la ceftriaxona.(R1)

353. Neonato de 3 días que ha nacido por parto de nalgas y que presenta desde hace
unas horas escasa reactividad y desconexión con el medio. En la exploración lo
encuentra decaído, pálido, con tinte ictérico, taquicárdico y taquipneico. En el abdomen
se palpa una masa redondeada y lisa, en hipocondrio derecho, que parece depender del
hígado. Señale la opción INCORRECTA acerca de la patología que sospecha:

1. 1. La prueba diagnóstica inicial debe ser un TC.


2. 2. Si se rompe hacia la cavidad peritoneal puede producirse shock y la muerte del niño.
3. 3. Los primeros datos suelen aparecer a partir del 3er-4º días de vida.
Suele ser consecuencia de la presión que se ejerce sobre el hígado durante la extracción
4. 4.
de la cabeza en los partos de nalgas.
Gráfico de respuestas
Comentario

La víscera que con mayor frecuencia se ve afectada en el contexto de un parto traumático es el


hígado. Puede sintetizar la lesión hepática del siguiente modo: neonato + parto traumático +
palidez (secundaria a hemorragia) + masa en hipocondrio derecho. Como en cualquier hemorragia,
el niño puede estar en shock, con lo cual sobreviene un aumento de la frecuencia cardíaca. Se
diagnostica inicialmente a través de realizar un ultrasonido abdominal, que mostrará el hematoma
hepático.(R1)

354. La principal causa de parálisis cerebral es:

1. 1. Ictericia nuclear ("kernicterus").


2. 2. Infección cerebral al nacer.
3. 3. Anoxia cerebral al nacer.
4. 4. Epilepsia.
Gráfico de respuestas
Comentario

No se trata de una pregunta difícil, aunque parezca una peculiaridad epidemiológica, siendo la
hipoxia cerebral la causa que encabeza la lista (respuesta 3 correcta).(R3)

355. ¿En cuál de las siguientes situaciones clínicas sería más probable el diagnóstico de
espondilitis anquilosante?:

Durante los últimos 10 años, un hombre de 28 años ha presentado dolor en la porción


1. 1.
inferior de la espalda y rigidez, que empeora por la noche y mejora con la actividad.
Durante los últimos 5 años, un hombre de 32 años ha presentado dolor en la parte inferior
2. 2.
de la espalda, que empeora con la actividad pero mejora con el reposo en cama.
Durante los últimos 10 años, un hombre de 34 años ha tenido brotes intermitentes de
3. 3. dolor leve en la parte inferior de la espalda; sin embargo, ahora se siente incapaz de hacer
la dorsiflexión con el primer dedo del pie derecho.
En los últimos 15 años, un hombre de 72 años ha presentado dolor progresivo en la parte
4. 4.
inferior de la espalda, que empeora al caminar y mejora con el reposo y al inclinarse.
Gráfico de respuestas
Comentario

 
 
 
 
En todos los casos nos mencionan un proceso de larga evolución que es compatible con el
diagnóstico de espondilitis anquilosante, pero la enfermedad suele comenzar por debajo de los 40
años lo que hace posible descartar la opción 4. El dolor propio de la enfermedad es de
características inflamatorias, es decir que no mejora con el reposo y de hecho no solo no mejora
sino que empeora y se acompaña de rigidez tras la inactividad, lo que descarta la opción 2. Entre
las opciones restantes la más compatible es la primera ya que en la opción tres nos hablan de un
cuadro intermitente y leve que no es lo propio de la enfermedad donde se acaba apreciando
afectación neurológica todo lo cual nos haría pensar en una ciática con afectación radicular.(R1)

356. El diagnóstico de una fractura de esternón se obtiene mediante:

1. 1. Radiografía posteroanterior de tórax.


2. 2. Radiografía oblicua anterior-izquierda de tórax.
3. 3. TAC torácico de alta resolución.
4. 4. Radiografía lateral de tórax.
Gráfico de respuestas
Comentario
Esta pregunta no es muy importante, ya que trata de un tema nunca preguntado en el mir, como es
el caso de las fracturas de esternón. Ante un traumatismo con aparición de dolor esternal a la
palpación se ha de realizar una radiografía lateral de tórax, donde se podrá apreciar todo el
contorno del esternón. El tratamiento es igual al de las fracturas costales, y consiste en tres
aspectos fundamentales: instaurar una analgesia adecuada, realización de fisioterapia respiratoria
y el reposo.(R4)

357. ¿Cuál es la complicación más frecuente de la parotiditis en la infancia?:

1. 1. Orquitis.
2. 2. Ooforitis.
3. 3. Pancreatitis.
4. 4. Meningoencefalitis.
Gráfico de respuestas
Comentario

Pregunta un poco complicada. La orquitis es la principal complicación en pacientes adultos, pero


en niños es la meningoencefalitis pregunta 4 correcta.(R4)

358. Le llaman a usted a la sala de partos para atender al alumbramiento del hijo de una
primípara de 39 años. El pequeño (niño A. es normal, pero presenta una sindactilia
bilateral en los dedos segundo y tercero de los pies. Antes de hablar con la madre, le
llaman por el pasillo para atender a otro parto, siendo éste un bebé pequeño para la edad
del embarazo (1.980 gramos, 38 semanas) (niño B), que tiene microcefalia, defectos
cardíacos, ictericia y cataratas. ¿Qué orden daría para el niño B?

1. 1. Cuidados ordinarios de enfermería.


2. 2. Aislamiento precautorio e investigación de infecciones congénitas.
3. 3. Cuidados ordinarios e investigación de diabetes mellitus en la madre.
4. 4. Consejo genético.
Gráfico de respuestas
Comentario

 
 
 
 
Después de las cromosomopatías, la causa más frecuente de crecimiento intrauterino retardado
son las infecciones connatales.

Piensa en las infecciones englobadas en el TORCH, en pacientes con antecedentes de CIR


asociado a ictericia precoz, con hepatoesplenomegalia, adenopatías o alteraciones en la biometría
hemática.

La presencia de alteraciones cardíacas y ocualres te indican la posibilidad de rubéola congénita


(recuerda que son parte de la tríada de Gregg: sordera neurosensorial, cataratas y malformaciones
cardíacas).

Ante dicha sospecha, está indicado el aislamiento del niño y realizar las pruebas oportunas para
confirmar el diagnóstico. Respuesta 2 correcta.(R2)

359. Montar un triciclo, hacer torres de 10 cubos, decir su edad y sexo, y comunicarse
con lenguaje gramatical, son logros que normalmente deben adquirirse a los:

1. 1. 2 años.
2. 2. 3 años.
3. 3. 4 años.
4. 4. 6 años.
Gráfico de respuestas
Comentario

1,5 mes: inicia la sonrisa social.

3 meses: inicia el sostén cefálico.

4 meses: toma objetos grandes con la mano .

5 meses: prensión alternante de objetos.

6 meses: inicia la sedestación, que se completa a los ocho meses.

8- 9 meses: oposición del pulgar.

9- 10 meses: inicia la reptación.

10- 11 meses: comienza la bipedestación.

12-15 meses: da los primeros pasos y emite su primera palabra real.

18-22 meses: realiza combinaciones de dos palabras.

2 4 meses: sube y baja escaleras, corre, apila cuatro o seis cubos para formar una torre.

5-10 años: el niño comprende que la muerte es un fenómeno permanente.

(R2)

 
 
 
 
360. La más alta incidencia de hemorragia intraventricular en los prematuros ocurre:

1. 1. Entre el primer y tercer día.


2. 2. Entre los 10 y 30 días de vida.
3. 3. Después de los 30 días de vida.
4. 4. Ninguna de las anteriores.
Gráfico de respuestas
Comentario

La immadurez del epitelio es lo que causa el sangrado sobretodo en prematuros, a medida que
avanza el tiempo es menos probable. La respuesta correcta es la 1 entre el primer y tercer día.(R1)

361. A 27-year-old black female comes to the emergency room complaining of fever, chills
and abdominal pain throughout the last week. She vomited twice in the last 24 hours. On
physical examination, her blood pressure is 115/66 mmHg and her pulse is 118/min.
Breath sounds are diminished at the left lung base, abdomen is soft but there is left upper
quadrant tenderness. Laboratory results show WBC 25.000/mm3 with left shift, rest
normal. CT scan of the abdomen shows a collection within the spleen. Which of the
following is the most likely diagnosis?

1. 1. Infectious endocarditis
2. 2. Crohn's disease
3. 3. Cirrhosis and portal hypertension
4. 4. CMV infection
Gráfico de respuestas
Comentario
Infectious endocarditis. Spleen abscesses are not frequent, one of the main causes is embolization
from infectious endocarditis.(R1)

362. A newborn is brought to the physician because his mother noticed that "the apple of
her newborn´s eye” is key hole shaped instead of round. Which of the following is the
most likely diagnosis?

1. 1. Iris coloboma.
2. 2. Aniridia.
3. 3. Synechiae as a sequel of a prenatal uveitis.
4. 4. This kind of pupil is normal in some infants. Pupil will regularize its form with growth.
Gráfico de respuestas
Comentario
El ojo se origina a partir de una evaginación del prosencéfalo que luego se invagina para formar la
copa óptica. Esta estructura “abraza” la arteria hialoidea hasta rodearla por completo. Si el cierre
de la fisura no es completo se producen unas malformaciones que reciben el nombre de
colobomas. Puesto que el cierre se produce de arriba abajo, el coloboma se sitúa habitualmente en
la zona inferior, y por ello cuando afecta al iris le da un aspecto muy característico en cerradura.
Los colobomas pueden estar limitados al iris o extenderse hacia atrás afectando al cristalino, la
retina y nervio óptico.(R1)

363. Paciente de 21 años nuligesta que acude por primera vez para realizar una citología
cervico-vaginal, el cual se reporta como ASCUS (células escamosas atípicas de
significado no determinado). Señale el siguiente paso más apropiado en el manejo:

 
 
 
 
1. 1. Colposcopía.
2. 2. Repetir el la citología cervico-vaginal.
3. 3. Antibióticos vía tópica.
4. 4. Biopsia dirigida.
Gráfico de respuestas
Comentario

Tema muy imortante en el bloque de Ginecología. La citología cervicovaginal puede dar los
siguientes resultados:

-Células escamosas normales.

-Células LSIL.

-Células HSIL.

-ASCUS.

El hallazgo de células ASCUS, de significado no determinado, puede deberse a la presencia de


una lesión premaligna en el cuello o no, como su propio nombre indica. Se recomienda en estos
casos repetir la citología a los 6 meses.(R2)

364. ¿En donde se produce la progesterona?

1. 1. Glándulas suprarrenales.
2. 2. En la granulosa.
3. 3. A través de la secreción pulsátil de GnRH.
4. 4. Cuerpo luteo.
Gráfico de respuestas
Comentario

La progesterona se produce únicamente en el cuerpo lúteo, sus funciones son: maduración


endometrial, disminución de a excitabilidad del músculo liso miometrial, elevación del metabolismo
basal y disminución de la cantidad de moco cervical y contenido de ácido siálico.(R4)

365. Señale cuál de las siguientes opciones referentes al recién nacido, su cuidado y
valoración NO es correcta:

La respiración habitual del niño es con el diafragma por lo que el abdomen sobresale y la
1. 1.
parte blanda del tórax se hunde durante la inspiración.
La presencia del reflejo rojo pupilar es indicación de estudio oftalmológico para descartar
2. 2.
patología intraocular.
Ante un hipospadias o epispadias graves debemos descartar alteraciones de los
3. 3.
cromosomas sexuales o de la diferenciación sexual.
A las 24 horas de vida debemos encontrar aire en el recto al practicar una radiografía de
4. 4.
abdomen.
Gráfico de respuestas
Comentario

 
 
 
 
La presencia de un reflejo pupilar rojo es normal; no así la presencia de un reflejo pupilar
blanquecino (leucocoria), que le obliga a descartar muy diversas patologías, como catarata
congénita, retinoblastoma congénito, coriorretinitis,etc.

El resto de las opciones son correctas: recuerde que la ausencia de gas distal en una placa de
abdomen simple le obliga a descartar que haya una obstrucción.(R2)

366. Acude a su consulta un niño de 7 años acompañado de sus padres preocupados por
su lento crecimiento y talla baja. Efectivamente su talla se encuentra por debajo del
percentil 3 para su edad. Su cara tiene algunos rasgos peculiares: presenta
hipertelorismo, ptosis y las orejas parecen de implantación baja. El cuello es corto. Tiene
pectus excavatum. A la auscultación revela un soplo sistólico en foco pulmonar. Al ser
preguntado refiere que tiene cierta facilidad para el sangrado. Antecedentes personales:
Polihidramnios en USG prenatales. RN a término (38+6), peso RN 3 600 gr, talla RN 54 cm,
PC 35 cm. Lactancia materna exclusiva durante 6 meses. Correctamente vacunado. Ha
sido intervenido por una criptorquidia. El diagnóstico es:

1. 1. Síndrome de Down.
2. 2. Síndrome de Turner.
3. 3. Síndrome de Noonan.
4. 4. Síndrome de Williams.
Gráfico de respuestas
Comentario

Dentro de las causa de talla baja hay que conocer algunos síndromes.El síndrome de Turner
(45X0) asocia talla baja, pterigium coli (pliegue del borde externo del cuello desde el pabellón
auricular al hombro), epicantus (pliegue vertical en el canto interno del ojo), paladar estrecho,
mandíbula pequeña, mamilas hipoplásicas o invertidas. Ovarios disgenéticos con retraso puberal.
Linfedema de manos y pies al nacimiento. Anomalías renales (riñón en herradura) o cardíacas
(coartación de aorta).

El síndrome de Noonan (respuesta correcta) cursa con talla baja postnatal, epicantus,
hipertelorismo (aumento de la distancia entre canto interno y externo del ojo), raíz nasal deprimida,
hendiduras palpebrales descendentes, cuello corto + pterigium, hipertelorismo mamilar (aumento
de distancia entre ambas mamilas), pectus carinatum/excavatum y criptorquidia. Alteraciones de la
coagulación. Puede haber retraso mental. La cardiopatía congénita más frecuente es la estenosis
pulmonar y después la miocardiopatía hipertrófica. Herencia autosómica dominante. Mutaciones en
gen PTPN11 (50%). Afecta a 1/1000- 2500 RN vivos.

El síndrome de Williams-Beuren: de herencia autosómica dominante, por delecciones en 7q11.23


(alteraciones de la elastina) conlleva talla baja prenatal, retraso mental moderado, carácter alegre e
hipersociabilidad. Rasgos faciales característicos: frente estrecha, nariz ancha y corta, con
hipotelorismo ocular, estrabismo, patrón de iris estrellado, mejillas prominentes y caídas, mentón
pequeño, labios gruesos, boca grande y maloclusión dental. Se llama al conjunto de estos rasgos
cara de “elfo o duende”. Puede asociar cardiopatía. La más frecuente estenosis aórtica
supravalvular, hipercalcemia neonatal e hipertensión (esta última aparece en la adolescencia o en
la edad adulta).(R3)

 
 
 
 

367. Niña de 2 años que


acude a su pediatra por llanto intenso con importante afectación del estado general.
Comenzó hace 1 hora, habiéndose encontrado previamente bien. No presenta fiebre. Los
padres refieren que, coincidiendo con los episodios de dolor, retrae las piernas hacia el
abdomen. En la exploración presenta aceptable estado general, abdomen difícilmente
depresible y doloroso, y durante la consulta emite una deposición en “jalea de grosellas”.
Usted solicita una ecografía abdominal. La imagen muestra lo que se observa en dicha
ecografía. Respecto al cuadro clínico citado, ¿cuál de las siguientes aseveraciones es
cierta?:

1. 1. La imagen muestra la característica imagen del "donut".


En esta niña no se debe intentar la desinvaginación con enemas de suero o de aire ya que
2. 2.
es muy pequeña.
3. 3. No es una situación de urgencia y se puede optar por actitud expectante.
4. 4. El primer diagnóstico de sospecha es el de megacolon congénito.
Gráfico de respuestas
Comentario

Tema prioritario en pediatría, casi nunca falla su presenca en el ENARM. Con imagen ecográfica o
no, se trata de una pregunta muy fácil. Un niño de esta edad, con dolor abdominal, encogimiento
de piernas coincidente con el mismo y, sobre todo, heces "en jalea de grosella", tendrá
seguramente una invaginación intestinal. La ecografía correspondiente a esta entidad mostraría
una imagen llamada "en donut" o "en rosquilla" (respuesta 1 correcta). Aunque no la hubieras visto
antes, deberías haberte inclinado por ella sólo por el contexto (aparte de que la ecografía
realmente revela una imagen redondeada que recuerda a una rosquilla).(R1)

368. ¿Cuál sería el manejo más adecuado para la paciente de la pregunta anterior?:

Con la sospecha de megacolon agangliónico realizaremos maniobras descompresivas con


1. 1.
enemas para posteriormente iniciar estudio con manometría ano-rectal.
El tratamiento inicial es médico intentando la desinvaginación con enemas de suero o
2. 2.
aire, pero si éstos no fueran efectivos sería necesaria la cirugía.

 
 
 
 
3. 3. Solicitar una TAC abdominal para confirmar el diagnóstico.
4. 4. Actitud expectante y revisión por su pediatra.
Gráfico de respuestas
Comentario
Si orientaste bien la pregunta anterior, ésta no resultará complicada. La invaginación intestinal no
precisa tratamiento quirúrgico de entrada. Suele resolverse únicamente con enemas de aire o
suero, ya que incrementan la presión intraluminal lo suficiente como para reducir la invaginación. Si
esto no es suficiente, entonces es planteable el tratamiento quirúrgico.(R2)

369. Señale la tríada clínica más sugestiva de toxoplasmosis congénita:

1. 1. Hidrocefalia, calcificaciones intracerebrales y coriorretinitis.


2. 2. Microcefalia, calcificaciones intracraneales y púrpura trombocitopénica.
3. 3. Cataratas, lesiones óseas estriadas y sordera.
4. 4. Lesiones mucocutáneas, osteocondritis y periostitis.
Gráfico de respuestas
Comentario

Esta pregunta se contesta como esrímulo-respuesta, no se puede dudar para fines del ENARM.
Recuerda la triada de Sabin de la toxoplasmosis congénita:

- Hidrocefalia.

- Calcificaciones cerebrales difusas.

- Coriorretinitis.
(R1)

370. A 30-day-old female infant who was been previously healthy develops a high-grade
fever, which is followed 2 days later by a maculopapular rash on her trunk and extremities.
She also has reddish discoloration of her lips and conjunctival infection, with
desquamation of the skin at her fingers starting from the fingertips and the anal region.
She is then given intravenous antibiotic for 12 days but do not recover from any of the
symptoms or her fever. Levels of serum acute phase reactants including erythrocyte
sedimentation rate (80 mm/h) and C-reactive protein (5 mg/dL) are remarkably high; no
bacterial growth is detected in any of her cultures. Which of the following is the most
likely diagnosis?

1. 1. Kawasaki's Disease
2. 2. Scarlet Fever
3. 3. Measles
4. 4. Mumps
Gráfico de respuestas
Comentario
Kawasaki's disease. Kawasaki’s disease (also known as lymph node syndrome and
mucocutaneous lymph node syndrome) is an autoimmune disease triggered by a pre-existing viral
infection in most cases. The disorder is caused by a systemic vasculitis. The most feared
complication is the formation of coronary aneurysms. The classic diagnostic criteria are: fever
lasting five or more days, accompanied by four out of five findings: bilateral conjunctival injection,
oral changes such as cracked and erythematous lips and strawberry tongue, cervical

 
 
 
 
lymphadenopathy, extremity changes such as erythema or palm and sole desquamation, and
polymorphous rash.(R1)

371. Una meningitis purulenta de evolución fatal, con Gram y cultivo negativos, después
de haber nadado en un lago es causada por:

1. 1. Criptococcus.
2. 2. Naegleria fowleri.
3. 3. Coccidioides immitis.
4. 4. Streptococcus pneumoniae.
Gráfico de respuestas
Comentario
Pregunta difícil y poco rentable. Las Naeglerias son amebas de vida libre que se han aislado en
aguas dulces y salobres. La infección es más frecuente en adultos jóvenes y niños, con el
antecedente de haber nadado recientemente en lagos o piscinas climatizadas. El contagio se
produce por aspiración de agua contaminada con trofozoítos o quistes, y tras un periodo de
incubación de 2 a 15 días aparece un síndrome meníngeo, con afectación de pares craneales y
evolución rápidamente mortal, sin que haya demostrado, hasta ahora, un tratamiento eficaz.(R2)

372. De acuerdo al Consejo Nacional para el manejo de Asma en Pediatría, para clasificar
la crisis de asma, según puntaje clínico de Bierman Pierson corregido por tal, se valoran
los siguientes parámetros:

1. 1. Sibilancias, frecuencia respiratoria, estridor y retracciones.


2. 2. Frecuencia respiratoria, sibilancias, cianosis y retracciones.
3. 3. Espiración prolongada, frecuencia cardiaca, sibilancias y estridor.
4. 4. Tos ametizante, frecuencia cardiaca, sibilancias y retracciones.
Gráfico de respuestas
Comentario

Pregunta complicada. La respuesta correcta es la 2.

Repasemos la siguiente tabla.(R2)

 
 
 
 
373. Paciente de 50 años sometido hace 6 meses a trasplante cardíaco y en tratamiento
inmunosupresor, que ingresa con fiebre, tos productiva y disnea. En la radiografía de
tórax se observa, en lóbulo superior izquierdo, un infiltrado cavitado. En una muestra de
esputo se observan bacilos grampositivos ramificados. ¿Cuál es el diagnóstico más
probable?:

1. 1. Aspergilosis pulmonar.
2. 2. Neumonitis por CMV.
3. 3. Neumonía por Nocardia asteroides.
4. 4. Neumonía neumocócica.
Gráfico de respuestas
Comentario

Pregunta fácil acerca de Nocardiosis, un tema fácil de dominar pues sabiendo unos pocos datos
clave sabremos reconocerla. La Nocardia es un grampositivo filamentoso aerobio (a diferencia del
Actinomyces israelii que es el otro actinomiceto importante de cara al examen y que es anaerobio).
Este es uno de los gérmenes capaces de dar neumonías cavitadas (recuerda otros como:
micobacterium tuberculosis, Staphilococo aureus, Aspergillus, Actinomyces, Pseudomona,
Klebsiella y Rodococcus). Recordemos que la Nocardia la veremos asociada a contextos de
inmunodepresión celular, como en este caso clínico. En este caso el diagnóstico es fácil gracias al
examen de esputo tan sugerente. Es muy típico (casi patognomónico) la combinación de
inmunodepresión + neumonía cavitada + abscesos cerebrales. Recuerda que el tratamiento es con
cotrimoxazol.(R3)

374. Paciente de 67 años de edad.


Dice que desde hace dos meses no ve por el centro ("es como si tuviera una sombra justo
en el centro"), para ver con detalle la mácula, qué prueba solicitaría:

1. 1. OCT (tomografía de coherencia óptica).


2. 2. Topografía corneal.
3. 3. Ecografía ocular.
4. 4. RMN de alta resolución.
Gráfico de respuestas
Comentario

Las alteraciones en el centro del campo visual deberían hacernos pensar en un posible síndrome
macular. En este tipo de

 
 
 
 
cuadros, se pierde agudeza visual, a diferencia de los síndromes retinianos periféricos, donde lo
que se pierde es campo.

Lo que nos preguntan en este enunciado es algo muy concreto. Queremos ver la mácula CON
DETALLE. Evidentemente, la respuesta 1 no es válida, ya que el campo visual estaría más
indicado para evaluar patologías que afectan más a la periferia de la retina (glaucoma, retinosis
pigmentaria, etc). Tampoco es lógico solicitar una prueba como la 3, que sirve para evaluar otra
estructura (la córnea). La duda, por tanto, estaría entre las opciones 2, 4 y 5.

Aunque la ecografía ocular puede dar información sobre el polo posterior del ojo, incluyendo la
mácula, la resolución es pobre en comparación con la tomografía de coherencia óptica (OCT), lo
mismo que sucede con la resonancia magnética. La respuesta correcta es, por ello, la 2.(R1)

375. En la prueba solicitada se obtiene la siguiente imagen. ¿Cuál es el diagnóstico?

1. 1. Degeneración macular asociada a la edad, forma seca.


2. 2. No hay patología, se trata de una mácula normal.
3. 3. Agujero macular.
4. 4. Drusas maculares.
Gráfico de respuestas
Comentario

El agujero macular se debe a la tracción que ejerce la contracción del vítreo sobre la porción
central de la retina, que es la mácula. Esto produce una distorsión de las imágenes y una
disminución de la visión. Hasta hace poco, el diagnóstico y el seguimiento de sus diferentes
estadios se basaba sobre todo en el aspecto clínico y en la angiografía fluoresceínica.
Actualmente, la tomografía óptica de coherencia desempeña un papel muy importante en su
diagnóstico y seguimiento, así como para estimar el pronóstico del resultado del tratamiento, que
es quirúrgico.

En la imagen que nos presentan, podemos apreciar en el centro una solución de continuidad que
produce una disrupción de todas las capas de la retina. Ésta es la imagen típica de un agujero
macular. Si se tratase de una degeneración macular senil, esta técnica mostraría una imagen
distinta, pudiendo mostrar drusas, neovascularización (DMAE húmeda), etc.

No te preocupes si has fallado esta pregunta. Tanto esta pregunta como la anterior tienen como
objetivo que conozcas la existencia de esta técnica y que sepas que puede darnos información
mucho más precisa que algunas otras, como la RMN o la ecografía ocular.(R3)

376. ¿Cuál de las siguientes moléculas puede servir como marcador bioquímico de la
hipertrofia ventricular izquierda de la hipertensión?:

1. 1. La cardiotrofina.
2. 2. La beta-miosina de cadena pesada.
3. 3. El factor de crecimiento transformante beta.
4. 4. La interleuquina 1.
Gráfico de respuestas
Comentario

No te preocupes si has fallado esta pregunta porque es difícil y su estudio es poco rentable.
Podrías acertar esta pregunta si conoces la fisiopatología de la miocardiopatía hipertrófica, que es

 
 
 
 
una enfermedad miocárdica primaria con una diversa expresión clínica y genética, y una evolución
variable caracterizada por una hipertrofia ventricular simétrica o asimétrica con o sin obstrucción al
tracto de salida del ventrículo izquierdo (VI), en ausencia de enfermedades cardíacas o sistémicas
capaces de producirla. En el 70- 80% de los casos se deben a mutaciones en uno de estos 3
genes: gen de la cadena pesada de la betamiosina (cr 14), gen de la proteina C ligada a la miosina
(cr 11) y gen de la troponina T (cr 1). Sabiendo esto, podrías haber deducido que la cedana pesada
de la betamiosina es un marcador de hipertrofia del VI.(R2)

377. Masculino de 18 años acude por presentar rectorragia ocasional. Presenta


pigmentación melanótica en la mucosa bucal, en manos y pies. La inspección anal
evidenció pigmentación perianal, sin hemorroides externas. Se realizó una colonoscopia
que reveló pólipos en sigma y colon descendente. ¿Cuál de las siguientes afirmaciones
es FALSA respecto a la entidad que presenta su paciente?

1. 1. Los pólipos son hamartomas.


2. 2. Se localizan con mayor frecuencia a nivel de colon distal.
3. 3. Puede haber pólipos en localizaciones extraintestinales.
4. 4. Existe riesgo de presentar cáncer intestinal.
Gráfico de respuestas
Comentario

Lo más importante de esta pregunta es que sea capaz de dar con el dato clave: melanosis
periorificial, que le haría pensar en un síndrome de Peutz-Jeghers. La pregunta, por lo demás, es
difícil, ya que exige conocer con cierto detalle esta enfermedad tan infrecuente.

El síndrome de Peutz-Jeghers se trasmite con herencia autosómica recesiva. El signo cutáneo


típico es la pigmentación mucocutánea. Durante la infancia, aparece una pigmentación melanótica
alrededor de la nariz, labios, mucosa bucal, manos y pies. Ocasionalmente, se acompaña de
hiperpigmentación genital y perianal. En la pubertad, pueden desaparecer las pigmentaciones,
excepto las de la mucosa oral.

Los pólipos pueden aparecer en estómago, intestino delgado y colon, siendo sobre todo frecuentes
en intestino delgado, y rara vez malignizan, pues son de tipo hamartomatoso. Por ello, no está
justificada la colectomía profiláctica. Se han descrito cánceres en duodeno, yeyuno, íleon y colon.
En un estudio, casi el 50% de los pacientes tuvieron cánceres intestinales o extraintestinales, con
una media de edad de 50 años al diagnóstico. También pueden aparecer pólipos benignos en
localizaciones extraintestinales, incluyendo nariz, bronquios, vejiga, vesícula biliar y conductos
biliares. En el 5-12% de mujeres, aparecen quistes o tumores ováricos. En varones jóvenes,
pueden aparecer tumores testiculares de células de Sertoli con signos de feminización. Otros
tumores que pueden aparecer son: mama (a menudo bilateral), páncreas, colangiocarcinoma y
cáncer de vesícula biliar.(R2)

378. Respecto al tratamiento de los pacientes con leucemia linfática crónica, señale la
afirmación CORRECTA:

La quimioterapia tipo CHOP ha demostrado un aumento de la supervivencia respecto a la


1. 1.
monoterapia con clorambucil en los estudios controlados.
La leucocitosis y la hipogammaglobulinemia, sin otros datos de actividad de la
2. 2.
enfermedad, no son criterios de tratamiento quimioterápico.
El desarrollo de una anemia hemolítica autoinmune en un paciente con LLC es una
3. 3.
complicación que justifica per se el tratamiento con fludarabina.

 
 
 
 
La administración de IgG iv en altas dosis en los pacientes con LLC e
4. 4.
hipogammaglobulinemia ha conseguido aumentar la supervivencia.
Gráfico de respuestas
Comentario

La leucocitosis sanguínea y la hipogammaglobulinemia son connaturales con la enfermedad, y por


sí mismos no son indicación de tratar a la LLC, que sí precisaría tratamiento en caso de progresión
o sintomatología. No existe en la actualidad tratamiento curativo, por lo que estos pacientes
pueden ser simplemente observados mientras estén asintomáticos.(R2)

379. ¿En el embarazo que vacunas se pueden aplicar?

1. 1. Vacuna contra difteria.


2. 2. Vacuna contra fiebre amarilla.
3. 3. Vacuna contra sarampión.
4. 4. Todas las vacunas se pueden aplicar.
Gráfico de respuestas
Comentario

Pregunta imortante para el nacional. Las únicas vacunas que se permiten durante la gestación son
los toxoides (tétanos, rabia, difteria), mientras que aquellas de virus vivos atenuados están
totalmente contraindicadas (parotiditis, rubeóla, sarampión y fiebre amarilla).(R1)

380. A 54-year-old woman comes to her physician for a preoperative work-up. She is
scheduled to undergo a hysterectomy. Her past medical history is significant for diabetes
mellitus and hypertension. Current medication includes ACEIs. Laboratory studies: pH
7.32, K 6.1 mEq/L; Cr 1.3 mg/dL. Which of the following is the most likely underlying cause
of these findings?

1. 1. Adrenal insufficiency as part of type 2 autoimmune polyglandular syndrome.


2. 2. Hyperreninemic hypoaldosteronism caused by high doses of ACEIs.
3. 3. Hyperosmolar diabetic decompensation is the main cause of her prerenal failure.
4. 4. Hyperreninemic hypoaldosteronism caused by a deficit of renin release.
Gráfico de respuestas
Comentario

Pregunta compleja en relación a una patología infrecuente. Los datos más importantes son los
antecedentes de diabetes mellitus, y que en los laboratorios presenta una insuficiencia renal leve
con una hiperpotasemia importante y una acidosis metabólica. Estos hechos definen al cuadro
denominado hipoaldosteronismo hiporreninémico, en donde existe un déficit de producción de
renina por el riñón y aparece la clínica asociada. El tratamiento de elección es la furosemida si el
paciente es hipertenso o la fludrocortisona si es normo o hipertenso.

Esta pregunta es de internista no se estrese.(R4)

381. La mayoría de casos de isoinmunizacion ABO en enfermedad hemolítica del recién


nacido, la madre es:

1. 1. Tipo A y el recién nacido tipo AB.

 
 
 
 
2. 2. Tipo O y el recién nacido tipo A.
3. 3. Tipo O y el recién nacido tipo AB.
4. 4. Tipo O y el recién nacido tipo B.
Gráfico de respuestas
Comentario

La enfermedad hemolítica del recién nacido por incompatibilidad ABO es la más frecuente de todas
las incompatibilidades de grupo sanguíneo entre la madre y el recién nacido

La EHRN por incompatibilidad ABO (EHRN-ABO) entre la madre y el recién nacido es la más
frecuente de las EHRN y se produce en gestantes de grupo O con hijo A (lo más frecuente) B o
AB. Esto es así, porque los individuos de grupo O además de la inmunoglobulina (Ig) M natural
contra el antígeno ABO del cual carecen, presentan cierta cantidad de IgG. Así pues, la IgG anti-A
o anti-B presente en el suero de la gestante de grupo O podrá atravesar la placenta y unirse a los
hematíes fetales o del recién nacido. Salvo raras excepciones, se produce en gestantes de grupo A
o B.

La respuesta correcta es la 2, ya que el grupo sanguíneo A es el más frecuente en comparación


con el B y el AB.(R2)

382. Manejo inicial de de amenorrea primaria:

1. 1. Anticonceptivos orales.
2. 2. AINE.
3. 3. Bajar la cafeína.
4. 4. Aumentar la actividad física.
Gráfico de respuestas
Comentario

De todas las opciones, la 1 es la única que nos puede ayudar a resolver la causa de la amenorrea;
por ello, por descarte, se responde.

Si al dar anticonceptivos orales, la paciente menstrúa, significa que el tracto genital femenino se
encuentra bien, y la causa es hormonal: hipogonadismo, anovulación crónica, etc.(R1)

383. La causa más frecuente de amenorrea es:

1. 1. Estrés.
2. 2. Deficiencia de la hormona luteinizante.
3. 3. Embarazo.
4. 4. Enfermedad crónica debilitante.
Gráfico de respuestas
Comentario

Esta pregunta es muy fácil. El embarazo sigue siendo la causa más frecuente de amenorrea.(R3)

 
 
 
 

384. Niña de 10 años que


acude a su pediatra por fiebre de 48 horas de evolución (máximo 39.3ºC) y odinofagia,
con tos escasa. La madre refiere que en las últimas 24 horas le han aparecido unos
“granitos”. A la exploración la niña se encuentra febril, con buena coloración de piel y
mucosas. La auscultación cardiopulmonar es completamente normal, el abdomen no es
doloroso ni se palpan masas ni megalias, los signos meníngeos son negativos y se
palpan múltiples adenopatías látero-cervicales y submandibulares bilaterales,
ligeramente dolorosas a la palpación. En la exploración ORL los tímpanos están
mínimamente opacos e hiperémicos, pero no están abombados, y la faringe es
intensamente eritematosa, con algunos exudados en ambas amígdalas. En la evaluación
de la piel observamos el exantema de la imagen. Respecto a la enfermedad que sospecha,
señale la falsa:

Se trata de una infección producida por un estreptococo beta hemolítico del grupo B
1. 1.
productor de toxina eritrógena.
La facies de Filatov consiste en una zona de eritema facial que respeta el triángulo
2. 2.
nasogeniano.
La presencia de lengua en fresa y lengua saburral es frecuente en la exploración de estos
3. 3.
niños.
4. 4. Es un cuadro muy raro en niños menores de 2 años de edad.
Gráfico de respuestas
Comentario

Resumamos lo que presenta este paciente: fiebre, odinofagia, faringoamigdalitis exudativa,


adenopatías cervicales y un exantema puntiforme, como en piel de lija o piel de gallina, rasposo,
que se palpa mejor que se ve. Es decir, tiene una escarlatina.

La escarlatina es una enfermedad exantemática que se produce por la infección por el S. pyogenes
(un estreptococo beta hemolítico del grupo A (no del grupo B) productor de toxina eritrógena.

La facies de Filatov, las líneas de Pastia y la lengua en fresa roja y blanca son típicas de este
cuadro.

La escarlatina y, por tanto, la faringoamigdalitis estreptocócica, es frecuente en niños en edad


escolar, siendo muy rara en niños menores de 2- 3 años, siendo en estos la causa más frecuente
de infección faríngea los virus.(R1)

 
 
 
 
385. En el paciente de la pregunta anterior, ¿cuál es la actitud más correcta entre las
siguientes?

Realizar un test de detección rápida de antígenos de estreptococo en exudado faríngeo y


1. 1.
si es positivo tratar con penicilina oral.
2. 2. Pautar tratamiento analgésico y antitérmico y reevaluar en 48 horas si persiste la fiebre.
Derivar a un centro hospitalario para realizar exámenes de laboratorio solicitando: BH,
3. 3.
QS, PCR y determinación de anticuerpos antiestreptolisina O (ASLO).
Realizar cultivo de exudado faríngeo, esperar a los resultados y si es positivo tratar con
4. 4.
macrólido oral.
Gráfico de respuestas
Comentario

La actitud más correcta es tomar un exudado faríngeo para una detección rápida de antígenos de
estreptoco. Aunque el gold estándar de toda infección es tener un cultivo positivo, este test tiene
una sensibilidad y una especificidad muy altas, con la ventaja de que obtenemos el resultado en
unos pocos minutos.

Ante un resultado positivo el tratamiento de elección es penicilina oral.

La opción 2 no es correcta si estamos sospechando una escarlatina, ya que la actitud es tratar con
antibioticoterapia, principalmente con el objetivo de evitar las complicaciones a largo plazo (como
la fiebre reumática y la glomerulonefritis).

La realización de un análisis de sangre no es necesario, sólo se llevará a cabo en los casos en que
se sospeche una complicación local (como un absceso faríngeo).

El ASLO consiste en la determinación de anticuerpos contra una enzima del estreptococo y no se


utiliza de manera rutinaria porque no nos sirve para el diagnóstico en el momento agudo y una
elevación aislada del título de estos anticuerpos indica que el niño ha padecido una infección por S.
pyogenes en los últimos meses.

El cultivo faríngeo se realiza cuando no se dispone del test rápido y en los casos en los que este
ha resultado negativo y la sospecha es alta.

El tratamiento con macrólido oral es la elección en pacientes alérgicos a beta- lactámicos.(R1)

386. Seleccione lo INCORRECTO con respecto a la circulación fetal:

1. 1. La circulación pulmonar recibe el 10% del gasto del ventrículo derecho.


2. 2. La presión de la aurícula izquierda es superior a la de la derecha.
3. 3. La resistencia vascular pulmonar es determinada por la baja PaO2 fetal.
4. 4. Algunas infecciones perinatales producen vasoconstricción pulmonar.
Gráfico de respuestas
Comentario

En la circulación fetal la sangre oxigenada procedente de la placenta llega al feto mediante la vena
umbilical, el 50% de esta sangre penetra en el hígado y el resto lo sortea y alcanza la vena cava
inferior a través del conducto venoso de Arancio. En ambos casos, llega a la aurícula derecha, por
lo tanto, la presión en las cavidades derechas será mayor que en las izquierdas algo que variará en
la circulación del RN.(R2)

 
 
 
 
387. En un masculino de 12 años, con retraso mental, macrogenitosomía, orejas grandes
y cara alargada, el diagnóstico más probable es:

1. 1. Síndrome de Klinefelter.
2. 2. Varón XYY.
3. 3. Trisomía 22.
4. 4. Síndrome del X frágil.
Gráfico de respuestas
Comentario

Tema poco importnte para el ENARM. Pregunta dificil sobre genética. El síndrome X frágil puede
afectar a hombres o a mujeres y se debe a una repetición de tripletes CGG.

Se caracterizan por retraso mental, genitales y orejas enormes y una cara alargada. Tienen riesgo
de neoplasias.(R4)

388. El síndrome de la rubéola congénita se asocia ¿a cuál de estas anomalías?:

1. 1. Conducto arterioso permeable (CAP) y estenosis de la bifurcación de la arteria pulmonar.


2. 2. Comunicación interventricular (CIV) y CAP.
3. 3. CIV y CIA.
4. 4. CIV y estenosis de la arteria pulmonar.
Gráfico de respuestas
Comentario

Se pregunta muy frecuente en el ENARM el síndrome de rubeola congénita, en general los


TORCH.

La rubéola congénita cursa clínicamente como una combinación de la tríada de Gregg (hipoacusia
neurosensorial, alteraciones oculares y cardiopatía) y lesiones óseas sin periostitis (esta es de la
sífilis). Dentro de las alteraciones cardíacas, la más frecuente es el ductus arterioso persistente
(DAP). Otra posibilidad es que genere estenosis de ramas pulmonares periféricas.(R1)

389. Ante un hombre de 55 años, que presenta un nódulo pétreo prostático en el tacto
rectal, el siguiente paso a dar es:

1. 1. Ecografía externa.
2. 2. Ecografía transrectal.
3. 3. Gammagrafía ósea.
4. 4. Solicitar una PSA y una ecografía transrectal con biopsia del nódulo.
Gráfico de respuestas
Comentario

Pregunta sencilla si se conocen las indicaciones de biopsia prostática. Ante un tacto rectal
sospechoso en un paciente joven debemos descartar SIEMPRE un cáncer de próstata. Por
supuesto necesitamos el PSA, la opción 4 es mas correcta que la 1, porque SOLO con el tacto
rectal podríamos indicar la biopsia.(R4)

390. ¿Cuál de las siguientes afirmaciones sobre la enfermedad de la membrana hialina


NO es cierta?:

 
 
 
 
1. 1. Su incidencia es inversamente proporcional a la edad de la gestación.
2. 2. Su frecuencia es mayor en los hijos de madres diabéticas.
La falta de surfactante provoca atelectasia y alteraciones graves de la ventilación-
3. 3.
perfusión.
En la radiografía es característico encontrar líneas vasculares pulmonares prominentes y
4. 4.
líquido en las cisuras.
Gráfico de respuestas
Comentario
La enfermedad de membrana hialina es la modalidad de distrés típica de los prematuros. Es tanto
más probable cuanto menor sea la edad gestacional. En los hijos de madre diabética también
puede aparecer, sean o no recién nacidos pretérmino. Esta enfermedad se debe a un déficit de
surfactante, que condiciona colapso alveolar y alteración de la relación V/Q. Su expresión
radiológica típica consiste en la aparición de infiltrados retículo- granulares bibasales con
broncograma aéreo y atelectasias.(R4)

391. Entre las causas más frecuentes de nefritis tubulointersticial aguda no se encuentra
el tratamiento con:

1. 1. Ciprofloxacino
2. 2. Amoxicilina
3. 3. Ibuprofeno
4. 4. Losartán
Gráfico de respuestas
Comentario
Los ARA II no se encuentran entre las asociaciones frecuentes de nefritis intersticial
inmunoalérgica. Al menos debes recordar la asociación con quinolonas, betalactámicos y
AINES.(R4)

392. Una persona de 75 años, asintomática y sin hallazgos en la exploración física,


presenta un hemograma con leucocitosis de 30.000/ul con un 70% de linfocitos pequeños
maduros. ¿Qué actitud le parece la más correcta?

1. 1. Realizar aspirado de médula ósea.


2. 2. Seguimiento del paciente sin tratamiento.
3. 3. Realizar TAC abdominal.
4. 4. Estudio de inmunofenotipo linfocitario.
Gráfico de respuestas
Comentario

El cuadro que nos describen corresponde a una leucemia linfática crónica. Vamos a repasar
algunas características fundamentales de esta entidad:

- Son características las alteraciones de la inmunidad humoral. Dado que la célula afectada es el
linfocito B, su disfunción implica hipogammaglobulinemia. Lógicamente, al tratarse de una leucemia
crónica, habrá leucocitosis, a expensas de células de aspecto maduro.

- La causa más frecuente de muerte son las infecciones, normalmente bacterianas (por ejemplo, el
neumococo).

 
 
 
 
- La incidencia de tumores está aumentada respecto a la población sana, debido a la
inmunosupresión que padecen, y a las transformaciones hacia leucemias más agresivas
(prolinfocítica) o linfomas (síndrome de Richter).

- Suele presentarse en ancianos. Produce linfocitosis, adenopatías y sombras de Gumprecht en el


frotis (sombras nucleares).

Dado que esta enfermedad no tiene tratamiento curativo, lo que se hace son revisiones periódicas
mientras el paciente está asintomático, como en el caso que nos plantean.(R4)

393. De acuerdo con los indicadores de asfixia neonatal se puede afirmar que.

1. 1. Un Apgar bajo es un muy buen indicador.


2. 2. Cuando hay asfixia, sólo se compromete el éncefalo.
3. 3. Una consecuencia muy importante es la depleción de fosfatos de alto nivel energético.
4. 4. El tono muscular del recién nacido nunca se ve afectado.
Gráfico de respuestas
Comentario

Si analizamos detenidamente la pregnta, esta se puede responder sin conocimientos médicos


excesivos. El Apgar como se ha preguntado no es un idicador de asfixia. Cuando hay asfixia el
encéfalo no es lo único que se compromete, sino también todos los demás órganos y decir que el
tono muscular del RN NUNCA se ve afectado es arriesgarse. Por lo que la respuesta correcta es la
3.(R3)

394. En relación con las metástasis hepáticas procedentes de carcinoma colorrectal,


señale, de las siguientes afirmaciones, cuál es la más CORRECTA:

1. 1. La cirugía está contraindicada cuando se aprecia más de una lesión metastásica.


2. 2. La supervivencia a los 5 años después de un tratamiento quirúrgico radical es del 30-50%.
3. 3. Se ha demostrado que la crioterapia consigue resultados mejores a los de la resección.
La presencia de determinadas mutaciones genéticas en las células tumorales permite, en
4. 4.
la actualidad, seleccionar a los pacientes que más se pueden beneficiar de la cirugía.
Gráfico de respuestas
Comentario

Dificultad media, ya que se pueden descartar tres de las opciones fácilmente.

Se descartan como respuestas falsas:

R1: la existencia de más de una lesión no es contraindicación quirúrgica, se pueden resecar varias
metástasis, siempre que la resección completa de todas ellas sea viable. El número máximo de
lesiones que se pueden resecar está en continua revisión.

R3: la crioterapia se considera un tratamiento alternativo en pacientes no candidatos a la resección


quirúrgica. En la actualidad, no hay ningún estudio que compare ambos tratamientos, de modo que
no se puede decir que consiga resultados mejores.

R4: La determinación de mutaciones genéticas no modifica la actitud frente al tratamiento


quirúrgico, pese a que sí puede predecir la respuesta al tratamiento quimioterápico.

 
 
 
 
La respuesta correcta es la 2, obteniéndose una supervivencia tras la resección de alrededor del
30 al 50% en la mayoría de las series.(R2)

395. La infección por estreptococo del grupo B en el recién nacido puede prevenirse
administrando antibioticoterapia profiláctica a las embarazadas portadoras o a las que
presentan factores de riesgo. Respecto a este problema, señale la afirmación
INCORRECTA:

1. 1. En las gestantes alérgicas a penicilina puede utilizarse eritromicina.


2. 2. La vía más frecuente de infección fetal es transplacentaria.
3. 3. El cribado en las embarazadas se realiza entre las semanas 35-37 de gestación.
La incidencia de infección neonatal por estreptococo B proporcionalmente es mayor entre
4. 4.
recién nacidos prematuros que entre recién nacidos a término.
Gráfico de respuestas
Comentario

Pregunta fácil acerca de la infección del recién nacido por el estreptococo del grupo B.

El SGB forma parte de la flora digestiva y coloniza la vagina hasta en un30 % de las mujeres, y
constituye la causa más frecuente de infección bacteriana neonatal.

Los factores de riesgo para la infección del recién nacido son:

•   CIR.
•   Prematuridad (respuesta 4 correcta).
•   Sufrimiento fetal perinatal.
•   Rotura de membranas de más de 18 h.
•   Fiebre materna intraparto.
•   Historia de hijo anterior infectado.
•   Bacteriuria por SGB.

El recién nacido adquiere la infección a su paso por el canal del parto y no por vía transplacentaria
(repuesta 2 incorrecta). La afectación del recién nacido precoz suele dar sepsis y la tardía,
meningitis. El screening se realiza a todas las embarazadas entre las semanas 35-37 mediante el
cultivo del exudado anorrectal y vaginal (repuesta 3 correcta), y la profilaxis se realiza durante el
parto en pacientes con cultivo positivo o aquéllas con cultivo negativo cuando hay factores de
riesgo.

Acostumbra a realizarse con la administración de ampicilina o penicilina cada cuatro horas hasta la
finalización del parto. En alérgicas se puede usar eritromicina (respuestas 1 correcta).(R2)

 
 
 
 

396.
A 60-year-old man with a 40-pack-year history of smoking presents to his physician with
a cervical mass. A fine-needle aspiration biopsy is performed, and the pathology
describes the sample as "papillary thyroid carcinoma". The patient undergoes a total
thyroidectomy. 3 months later, he presents to his physician office again for a regular
follow-up visit. He claims he has been having dysphonia and frequent choking. Findings
of this patient's direct laryngoscopy are shown in the image. Which of the following
options is CORRECT?

1. 1. Idiopathic paralysis of the left vocal cord.


A thoracic CT scan should be performed in order to rule out a pulmonary or mediastinal
2. 2.
tumor that may be causing the observed paralysis.
The presence of paralysis in middle position and arching of the right vocal cord suggests
3. 3. a iatrogenic paralysis or paresis caused by an injury of the vagus nerve sustained during
the thyroidectomy.
The occurence of frequent choking should prompt us to suspect CNS pathology such as
4. 4. Vernet, Collet-Sicard or Villaret syndromes that affect vagus nerves along with other
lower cranial nerves (IX, XI, XII).
Gráfico de respuestas
Comentario

La parálisis de cuerda vocal unilateral de origen troncular (afectación del tronco del nervio vago,
tanto del laríngeo superior como recurrente) conlleva una parálisis en posición intermedia y
acortamiento de la cuerda vocal. El paciente presenta aspiraciones frecuentes y disfonía. En el
caso de tratarse de una lesión del nervio recurrente presentará una parálisis en posición
paramediana sin afectación de la sensibilidad laríngea. Es obligado el estudio mediastínico
pulmonar en casos izquierdos, dado el recorrido intratorácico del nervio laríngeo recurrente
izquierdo. Si la lesión del tronco del vago se da proximalmente a las ramas faríngeas provocará
disfagia, pero las aspiraciones son consecuencia de la lesión del nervio laríngeo superior que
recoge la sensibilidad de la laringe.(R3)

397. Para el diagnóstico de salpingitis, el método más certero es:

1. 1. Laparoscopía
2. 2. Ecografía transvaginal
3. 3. Ecografía pélvica
4. 4. Resonancia magnética pélvica
Gráfico de respuestas
Comentario
 
 
 
 
Sería el método más certero pero raramente es necesario. Se realiza en pocos casos.(R1)

398. En el examen de un recién nacido prematuro se encuentra generalmente uno de los


siguientes signos o eventos:

1. 1. Uñas largas.
2. 2. Lanugo.
3. 3. Taquipnea transitoria.
4. 4. Policitemia.
Gráfico de respuestas
Comentario

En el neonato pretérmino la piel es delgada y suave, y es posible que esté recubierta de un vello
escaso y fino denominado lánugo. que le confiere protección térmica y que desaparece en unas
semanas.(R2)

399. A una paciente embarazada se le quiere recetar un determinado fármaco, pero ante
las dudas de su empleo durante la gestación, se ve que dicho medicamento es
considerado “categoría A” en la clasificación de seguridad de los fármacos durante el
embarazo de la FDA (Food and Drug Administration). ¿Cómo debe considerarse el
empleo de dicho medicamento durante la gestación?

1. 1. Contraindicado de forma absoluta por claros riesgos para el feto.


2. 2. Puede utilizarse pero limitando su utilización a periodos inferiores a 10 días.
3. 3. Solo se aconseja su utilización reduciendo la dosis a la mitad de lo habitual.
4. 4. Puede emplearse con seguridad durante toda la gestación.
Gráfico de respuestas
Comentario

Una pregunta extraordinariamente sencilla.

Tal como menciona la pregunta, existe una clasificación, propuesta por la FDA, sobre los riesgos
potenciales de los diferentes fármacos en la gestación. Lo más importante es que conozcamos los
dos extremos: categorías A y X.

•   Categoría A. En ella están incluidos los fármacos que pueden utilizarse con total seguridad
durante el embarazo como, por ejemplo, el paracetamol (respuesta 4 correcta).
•   Caterogía X. Son los que están absolutamente contraindicados, ya que en ningún caso el
beneficio superaría los riesgos fetales (isotretinoína).

Entre ambas categorías existen otras intermedias (B, C, D), en las que va aumentando el balance
riesgo-beneficio a medida que nos aproximamos hacia la X.(R4)

400. Los pacientes con polimiositis asociada a neoplasia presentan las siguientes
características EXCEPTO:

1. 1. Es un grupo poco frecuente de polimiositis.


2. 2. Aparece generalmente por encima de los 50 años.
3. 3. Se asocian a tumores de mama y pulmón.
4. 4. La afectación cutánea (dermatomiosistis) es inhabitual.

 
 
 
 
Gráfico de respuestas
Comentario
Esta pregunta es muy buena para que repases lo más importante del capítulo de miositis: que a
veces son indicio de neoplasia oculta. Desde luego es más frecuente la dermatopolimiositis
idiopática, que se da en mujeres de edad media. Si el cuadro va ligado a neoplasia oculta, el
paciente "tipo" cambia, suele ser mayor de 60 años, por lo que una miopatía inflamatoria que
debuta en un anciano debe alertarnos a buscar una neoplasia oculta. Lo más frecuente es que esta
sea de pulmón, pudiendo también tener su origen en mama, ovario, estómago,? La neoplasia
puede aparecer antes o después de la miositis, en todo momento son necesarios estudios
complementarios. La entidad que más se relaciona con la neoplasia oculta es la Dermatomiositis, y
no la polimiositis, por lo que la respuesta falsa es la 5. Ten en cuenta que la DM lo tiene casi todo:
manifestaciones cutáneas típicas, predisposición a neoplasia,? sólo la PM "le gana" en una cosa: la
relación de los anticuerpos Jo- 1 con la neumopatía intersticial.(R4)

401. El hallazgo más frecuente en la radiografía de tórax en un paciente con asma es:

1. 1. Condensaciones alveolares bilaterales y difusas.


2. 2. Radiografía de tórax normal.
3. 3. Engrosamiento de paredes bronquiales.
4. 4. Neumomediastino.
Gráfico de respuestas
Comentario

La radiografía de tórax más usual en una crisis asmática es normal (respuesta 3 correcta). La
presencia de signos de hiperinsuflación (diafragmas aplanados, horizontalización costal,
hiperclaridad) sólo se da en las crisis graves. Recuerda que, en el tromboembolismo pulmonar,
también es posible encontrar una Rx de tórax normal, si bien sería más frecuente encontrar
alteraciones radiológicas, aunque leves e inespecíficas (atelectasias laminares, pequeños
derrames, elevación de un hemidiafragma…).(R2)

402. Respecto al carcinoma "in situ", ¿cuál de las siguientes afirmaciones es la correcta?:

1. 1. El carcinoma intraductal suele ser multicéntrico.


2. 2. El carcinoma lobulillar "in situ" tiene carácter bilateral y multicéntrico.
3. 3. El carcinoma intraductal no evoluciona a lesión palpable.
4. 4. El tratamiento quirúrgico no debe ir acompañado de linfadenectomía axilar.
Gráfico de respuestas
Comentario

Pregunta sobre los tipos histológicos del cáncer de mama que puede resultar relativamente fácil.
Hay que tener presente cara al examen que el carcinoma lobulillar in situ es bilateral en un
50- 75% de las ocasiones, mientras que el ductal in situ lo es en un 10- 20% de las veces; esto nos
lleva a elegir como cierta la opción 2 y descartar la 1. Nos podrían surgir dudas con la opción 4 ya
que el tratamiento de un carcinoma in situ suele ser mastectomía o tumorectomía con radioterapia;
generalmente no se realiza linfadenectomía ya que no suele haber afectación linfática, salvo en
casos con factores de mal pronóstico en los que podría estar indicada y nos podemos encontrar un
muy bajo porcentaje de ganglios afectos.(R2)

403. Una mujer de edad media con xerostomía, sequedad ocular e hipertrofia parotídea
bilateral persistente consulta por cuadro de linfadenopatía periférica que se acompaña

 
 
 
 
de linfadenopatía hiliar y mediastínica en la RX de tórax. Tiene fiebre y en la analítica se
objetiva descenso en los títulos del factor reumatoide respecto a los previos. Entre las
siguientes entidades, señale su diagnóstico más probable:

1. 1. Linfoma no Hodgkin de células B asociado a síndrome de Sjögren primario.


2. 2. Sarcoidosis.
3. 3. Mononucleosis infecciosa.
4. 4. Linfadenopatía por citomegalovirus.
Gráfico de respuestas
Comentario
Memoriza bien este detalle, es de lo más importante que debes saber de este tema, que en el MIR
sólo ha sido preguntado una vez en los últimos 10 años. Se reconoce el Sjögren por xerostomía y
xeroftalmía, que no aparecen en el resto de entidades. El linfoma es la complicación tardía más
típica que puede tener un paciente con síndrome de Sjögren. Se debe sospechar cuando lleva ya
unos años de evolución de su enfermedad autoinmune, si persiste la tumefacción parotídea largo
tiempo, especialmente si ésta es unilateral (aunque aquí no es el caso), si existen adenopatías
laterocervicales, si aparecen infiltrados pulmonares o adenopatías hiliares o mediastínicas, y
además, si descienden los títulos de factor reumatoide. Especialmente aparece en las formas
primarias del Sjögren, pero no es muy frecuente de todos modos. Ten en cuenta que aunque los
infiltrados de linfocitos en las glándulas son de linfocitos CD4+, los linfomas son de tipo no
Hodgkiniano de células B. Recuerda que el VIH puede producir un Sjögren secundario
indistinguible del primario, aunque no en esta paciente.(R1)

404. A la urgencia pediátrica acude un niño de 4 años de edad tras la ingesta accidental
de lejía. De las siguientes pruebas diagnósticas, ¿cuál NO aporta beneficio alguno en el
momento agudo?

1. 1. Radiografía simple de tórax.


2. 2. Historia clínica detallada.
3. 3. Endoscopia alta.
4. 4. Tránsito digestivo baritado.
Gráfico de respuestas
Comentario

Ante una causticación esofágica, es obvio que conocer el tóxico ingerido es fundamental, para
distinguir si es de naturaleza ácida o alcalina. La exploración de la cavidad orofaríngea permite ver
si hay úlceras por el cáustico o sospechar lesión de la vía aérea. La Rx tórax es precisa para
evaluar la posible perforación, que se manifestaría como neumomediastino. En cuanto a la
endoscopia, debe realizarse de forma urgente, durante las primeras 24 horas, para valorar la
extensión, localización y severidad de las lesiones (habiendo descartado previamente perforación
esofágica mediante la Rx tórax).(R4)

405. Todas las siguientes afirmaciones respecto al síndrome de apnea del sueño son
verdaderas EXCEPTO:

1. 1. Los varones se afectan más que las mujeres.


2. 2. La hipertensión sistémica es un hallazgo común.
3. 3. El alcohol puede ser un factor contribuidor.
4. 4. Los estrógenos suelen ser útiles.
Gráfico de respuestas

 
 
 
 
Comentario
Se trata de una pregunta de dificultad baja y para resolverla nos deberían bastar los conocimientos
adquiridos en primera vuelta y para responderla correctamente es conveniente conocer la
fisiopatología de la apnea obstructiva del sueño. Se trata de una patología con predominio en
varones obesos de edad media (opción 1 correcta) en la que el suceso definitivo es el movimiento
posterior de la lengua y el paladar en aposición con la pared posterior de la faringe (a lo cual
contribuye el efecto relajante del alcohol (opción 3 correcta), con oclusión de la naso y orofaringe.
Por la apnea hay asfixia progresiva hasta que se produce un breve despertar, transitorio a un
estado de sueño más ligero ("arousal"), que reanuda el flujo aéreo. Estos despertares son la causa
de las manifestaciones neuropsiquiátricas y de conducta (opción 5 correcta), siendo dentro de ellas
la somnolencia diurna la queja más común. El tratamiento son medidas higienicodietéticas y la
ventilación con CPAP (presión positiva continua nasal) fundamentalmente. En algunos casos se
puede recurrir a la uvulopalatofaringoplastia o traqueostomía. Los estrógenos no se utilizan porque
no aportan ningún beneficio (opción 4 falsa).(R4)

406. Paciente con cuadro de deshidratación severa, que presenta volumen urinario de 10
ml/hora, creatinina sérica 2 mg/dl, osmolaridad urinaria 1.025, sodio urinario 10 mEq/L. el
diagnóstico más probable es de insuficiencia:

1. 1. Pre – renal.
2. 2. Renal con necrosis tubular.
3. 3. Post – renal.
4. 4. Renal por papilitis necrótica.
Gráfico de respuestas
Comentario

Sólo con el dato de "deshidratación severa" se puede responder a la pregunta, ya que es una
causa de insuficiencia renal pre-renal.

También el sodio urinario bajo (<20mEq/l) o FENa <1% lo indican, pues la hipoperfusión renal
activa al sistema renina angiotesina, que recuperará el sodio de la orina.(R1)

407. Todos los siguientes son criterios empleados para definir el control de la enfermedad
asmática excepto uno. Señálelo.

1. 1. Frecuencia de síntomas diurnos.


2. 2. Frecuencia de síntomas nocturnos.
3. 3. Dosis de corticoide inhalado empleado.
4. 4. Espirometría.
Gráfico de respuestas
Comentario

El control del asma se define si se cumplen todos los criterios siguientes: 1. síntomas diurnos no
más de 2 veces por semana, 2. uso de fármacos de rescate (beta agnonistas de acción corta) no
más de 2 veces por semana, 3. ausencia de síntomas nocturnos, 4. ninguna limitación de
actividad, 5. ausencia de crisis, y 6. función pulmonar normal. El control de la enfermedad asmática
es el objetivo que se debe alcanzar al tratar a un paciente asmático, y para ello, tras el tratamiento
inicial indicado en función de la clasificación de gravedad del asma, se empleará el escalón más
bajo posible que consiga este objetivo. En estos criterios no aparece la dosis necesaria de
corticoide inhalado necesaria para conseguir el control (respuesta correcta 3) .(R3)

 
 
 
 
408. ¿En cuál de las siguientes circunstancias es excepcional la existencia de
trombocitosis reactiva?

1. 1. Hemorragias.
2. 2. Neoplasias epiteliales.
3. 3. Anemia refractaria con exceso de blastos en transformación.
4. 4. Anemia ferropénica.
Gráfico de respuestas
Comentario

Causas de trombocitosis reactiva.

Las trombocitosis pueden ser primarias, por alteración clonal de las células, como en los síndromes
mieloproliferativos crónicos, o bien secundarias. Causas de trombocitosis secundarias son
neoplasias, infecciones, inflamación crónica, esplenectomía, ferropenia o una hemorragia aguda.
En cambio, en las anemias refractarias lo que se produciría sería una trombopenia.(R3)

409. ¿Cuál de los siguientes fármacos que se citan a continuación cree que NO estaría
indicado en un paciente varón de 82 años, diagnosticado de insuficiencia cardiaca, en
clase funcional III/IV de la NYHA con FEVI del 34%, y cuyos antecedentes personales son:
fumador importante, HTA, IRC con creatinina basal de 3,2mg/dl, fibrilación auricular
crónica y diabetes mellitus controlada con antidiabéticos orales?:

1. 1. Ramipril.
2. 2. Metoprolol.
3. 3. Candesartan.
4. 4. Espironolactona.
Gráfico de respuestas
Comentario
Esta es una pregunta donde los detalles son importantes. Fijaos que el paciente tiene una
creatinina de 3,2 mg/dl. Los IECA y los ARA II (si intolerancia a IECA aunque a veces se asocian)
son tratamiento de primera línea en pacientes con ICC y FEVI deprimida. Hay que ir con cuidado
con la función renal y los iones pero esto no es contraindicación (de hecho están indicados en la
insuficiencia renal crónica y más si son pacientes diabéticos como este). Pero la Espironolactona sí
que está contraindicada en pacientes con creatinina superior a 2,5mg/dl o potasio mayor de 5
mEq/l por riesgo a hiperpotasemia tóxica. Yo me quedaría con la idea de que la función de los
IECA/ARA II en la insuficiencia renal es análoga a los beta- bloqueantes en la insuficiencia
cardiaca. Ambos hacen “trabajar menos” al órgano en cuestión previniendo su deterioro y
mejorando su función siempre y cuando haya una situación estable, de manera que son de
elección en la fase crónica y están contraindicados en la fase aguda.El Metoprolol está dentro de
los beta- bloqueantes indicados en la ICC con FEVI deprimida y que han demostrado mejora de la
supervivencia y además al ser liposoluble es de elección en pacientes con IRC. La digoxina
sabemos que no mejora la SPV pero el paciente tienen FA y puede ayudar a controlar la FC y
también se ha visto que en estos pacientes disminuye los ingresos hospitalarios.(R4)

410. Un bebe de 1 mes de edad presenta súbitamente fiebre elevada y convulsiones


generalizadas, cual es el diagnóstico de presunción mas adecuado?

1. 1. Epilepsia.
2. 2. Convulsión febril.
3. 3. Meningitis bacteriana.

 
 
 
 
4. 4. Tétanos.
Gráfico de respuestas
Comentario

Nos presentan un caso de convulsión febril. Las crisis febriles son un proceso típico de la edad
infantil (entre los 3 meses y los 5 años) que se relaciona más frecuentemente con el aumento de la
temperatura, lo que da lugar a una crisis el primer día de un proceso febril, independientemente del
origen del mismo. Duran menos de 15 minutos, presentan buena recuperacion posterior y los
hallazgos en el periodo intercrítico son normales o negativos. Lo más adecuado en su tratamiento
es el manejo conservador, manteniendo la temperatura en límites normales mediante la utilización
de paracetamol. Si los episodios son recurrentes utilizar diazepam vía recta u oral. No esta
indicado la utilización de anticomiciales como profilaxis de las crisis febriles.

Pero detecte algo muy importante, le mencionan que tiene 1 mes de edad, con lo cual habrá que
descartar una sepsis tardía y muy probablemente una meningitis bacteriana Respuesta correcta
3.(R3)

411. En un paciente de 48 años con adenocarcinoma pulmonar estadio IIIa tratado hace
8 meses se diagnostica una lesión intraparenquimatosa cerebral de 3 cm de diámetro que
produce hemiparesia. En resonancia muestra captación de contraste y produce edema.
Un estudio de extensión no muestra indicios de recidiva del tumor primario. La actitud
más adecuada será:

Radioterapia holocraneal paliativa puesto que el estadio actual del tumor no tiene
1. 1.
supervivencia a largo plazo.
Biopsia estereotáxica, dado que el diagnóstico es incierto y las opciones terapéuticas muy
2. 2.
diferentes.
3. 3. Extirpación quirúrgica mediante craneotomía abierta seguida de radioterapia.
4. 4. Iniciar quimioterapia para tratar la enfermedad sistémica aunque sea invisible.
Gráfico de respuestas
Comentario

Una pregunta de dificultad media sobre el cáncer de pulmón metastásico.

Aunque el diagnóstico más probable es el de una metástasis cerebral, date cuenta de que se trata
de una lesión única, ya que el estudio de extensión no muestra indicios de recidiva a otros niveles.

En estos casos, el tratamiento más adecuado sería la resección quirúrgica, siempre que la lesión
sea accesible quirúrgicamente.

Tal como dice la respuesta 4, debería extirparse y, después, se emplearía radioterapia a nivel
local, para disminuir la probabilidad de recidivas.(R3)

412. En relación a un paciente masculino de 35 años, con colelitiasis asintomático,


marque lo VERDADERO:

1. 1. Debe tratarse con medicación que disuelva los cálculos.


2. 2. El paciente tiene probabilidad menor del 10% de ser sintomático en los siguientes 5 años.
El riesgo de cáncer de vesícula por detección de litiasis a tan temprana edad justifica la
3. 3.
colecistectomía electiva.
4. 4. Debe realizarse colecistectomía solo si tiene comorbilidad como Diabetes Mellitus.

 
 
 
 
Gráfico de respuestas
Comentario

Pregunta de cierta dificultad sobre indicaciones quirúrgicas de colelitiasis en pacientes


asintomáticos

Cálculos mayores de 2.5 cm

Anomalías congénitas con cálculo

Anemia falciforme

Calcificación vesicular

Concomitantemente con cirugía bariátrica.(R2)

413. ¿Cuál es el tratamiento de elección de los trastornos respiratorios durante el sueño


en los pacientes con enfermedades neuromusculares?:

1. 1. Ventilación mecánica no invasiva mediante mascarilla nasal.


2. 2. Oxigenoterapia nocturna.
3. 3. Protriptilina.
4. 4. Relajantes musculares.
Gráfico de respuestas
Comentario
Esta pregunta tiene una dificultad intermedia, ya que para poder responderla hay que tener claro
qué tipo de trastorno respiratorio producen las enfermedades neuromusculares. Los procesos que
presentan defectos del sistema neuromuscular producen aumento de la PaCO2 con unos
pulmones normales, debido a que hay disminución del volumen de ventilación por minuto, y por
tanto, hipoventilación alveolar. Debido a que lo que falla es la ventilación alveolar el tratamiento de
elección es la ventilación mecánica que en la mayoría de los casos se realizará de forma no
invasiva (mediante mascarilla nasal), al ser menos cruenta que la ventilación mediante
traqueostomía (opción 1 correcta).(R1)

414. ¿Cuál de los siguientes cuadros psiquiátricos se relaciona clásicamente con el uso
de fenciclidina?:

1. 1. Psicosis aguda.
2. 2. Manía.
3. 3. Síndrome amotivacional.
4. 4. Ataque de pánico.
Gráfico de respuestas
Comentario
Es una pregunta muy sencilla y directa de esas cosas sobre las drogas que 2nunca se deben de
olvidar por lo típico que son y siempre quiso saber" Clásicamente la psicosis aguda ( opción 1) la
produce la fenciclidina o polvo de ángel. Un cuadro maniforme ( opción 2) la provocaría la cocaína.
El síndrome amotivacional ( opción 3) es típico del cannabis. Y el grado máximo sería la catatonía (
opción 5) Los taques de pánico ( opción 4) son típicos del LSD o alucinógenos, recuerda que los
malos viajes son crisis de pánico. Manual CTO 4ª Edición, Psiquiatría, Tema 5, pags. 35- 39.(R1)

 
 
 
 
415. La denominada reacción de Paul-Bunnell, en pacientes con mononucleosis
infecciosa, determina en realidad la presencia de:

1. 1. Anticuerpos completos.
2. 2. Anticuerpos heterófilos.
3. 3. Antígenos de superficie vírica.
4. 4. Anticuerpos específicos.
Gráfico de respuestas
Comentario

De cara al ENARM es importante dominar bien el síndrome mononucleósico. Recuerda que los
anticuerpos heterófilos son relativamente específicos de la Mononucleosis infecciosa (tienen un
buen valor predictivo positivo) y apoyan la etiología por VEB de un sd mononucleósico, sobre todo
cuando el cuadro clínico es típico. En el ENARM nos pueden decir que estos anticuerpos son
positivos de distintas maneras: "reacción de Paul- Bunnel"; "test Monospot". Recuerda que estos
anticuerpos no son positivos siempre: 90 % adultos, 50% niños. Cuando sean negativos o ante un
cuadro atípico, podemos solicitar IgM ACV (contra el antígeno de la cápside) o constatar la
seroconversión de ANEB (Ac contra antígeno nuclear), que permiten hacer diagnóstico de
primoinfección por VEB.(R2)

416. Cual seria la pauta más correcta a llevar a cabo en el RN de madre con la siguiente
serología: Ags + frente al virus de la hepatitis B.

1. 1. Gammaglobulina.
2. 2. Interferón alfa.
3. 3. Vacuna + Gammaglobulina.
4. 4. Vacuna + Gammaglobulina + inteferón.
Gráfico de respuestas
Comentario

Profilaxis postexposición: vacuna + IGHB (0,5 ml) administrándose en las primeras 8-12 horas en
recien nacidos de madres con AgHBs positivo.(R3)

417. Respecto a la hipertensión arterial en el anciano, es cierto que:

1. 1. Rara vez se produce en ellos el fenómeno de “bata-blanca”.


2. 2. En esta edad no es necesario reducir la sal en la dieta.
3. 3. No produce beneficio tratar la hipertensión arterial en mayores de 80 años de edad.
La cifra de presión sistólica es mejor predictor de cardiopatía isquémica que la de
4. 4.
diastólica.
Gráfico de respuestas
Comentario

Una pregunta de dificultad media, que puede responderse directamente con el Manual CTO.

Antiguamente, se llegó a considerar este tipo de hipertensión como una manifestación normal del
envejecimiento, considerándose más importante la presión arterial diastólica. Con el paso del
tiempo, se ha demostrado la gran importancia de la hipertensión sistólica, que para algunos es
incluso más relevante que la diastólica.

 
 
 
 
La HTA sistólica es especialmente importante en pacientes mayores de 65 años. Se ha
demostrado que su tratamiento disminuye la mortalidad cardiovascular y el riesgo de sufrir
accidentes cerebrovasculares.(R4)

418. Usted atiende en Urgencias a un niño de 8 años que ha sido traído por sus padres
ya que, desde hace 5 días, presenta fiebre, sudoración, intensa astenia, anorexia y tos
seca. La exploración es normal excepto por una frecuencia respiratoria de 22/min y en la
auscultación pulmonar, una disminución del murmullo vesicular en el tercio medio
derecho. Solicita estudios de laboratorio, donde sólo destaca una discreta leucocitosis,
y una placa de tórax que muestra un infiltrado intersticial en el lóbulo medio derecho y
adenopatías hiliares homolaterales. Su diagnóstico de sospecha es:

1. 1. Bronquiolitis por VRS.


2. 2. Primoinfección tuberculosa.
3. 3. Enfermedad de Hodgkin.
4. 4. Neumonía por Haemophilus influenzae.
Gráfico de respuestas
Comentario

En este caso de los que nos están hablando es del complejo primario o de Ghon de la enfermedad
tuberculosa. El resto de síntomas son los de un síndrome constitucional que son también
compatibles con esta enfermedad. De las opciones más fácilmente descartables son aquéllas que
son incompatibles con la edad: la bronquiolitis - opción 1- se da en niños más pequeños, por
debajo de los dos años y sobre todo menores de un año. La infección por H. influenzae - opción
4- produce patología en niños menores de 5 años. Por otro lado la aspergilosis se caracterizaría
por ser una infección oportunista, que por tanto afectaría a inmunodeprimidos. Por lo tanto la
enfermedad de Hodgkin puede confundirnos más: recordemos que esta enfermedad tiene dos
picos de edad, unos a los 60 años y el otro en 2ª- 3ª décadad de la vida y puede acompañarse de
una clínica parecida, pero como vemos por probabilidad sería más frecuente la TBC,
especialmente por la imagen radiológica.(R2)

419. Una paciente de 23 años de


edad consulta por dolor articular en rodillas, tobillos, codos, muñecas y articulaciones
pequeñas de las manos y los pies, de 4 meses de evolución, con rigidez matutina de 6
horas de duración. En la exploración se observa una poliartritis con derrame prominente
en ambas rodillas y nódulos de las características que presenta la imagen. En el estudio
realizado presenta un aumento importante de la velocidad de sedimentación y la proteína

 
 
 
 
C reactiva, factor reumatoide positivo, y varias erosiones radiológicas en carpos. ¿Cuál
de las siguientes afirmaciones es cierta?

El diagnóstico más probable es artritis reumatoide. Se trata de un proceso autolimitado


1. 1.
que debe manejarse de forma conservadora, con antiinflamatorios no esteroideos.
El diagnóstico más probable, dada la edad de la paciente y lo persistente de la afectación
articular, sería lupus cutáneo subagudo. La determinación de anticuerpos anti-Ro será
2. 2.
muy probablemente positiva y habría que iniciar, cuanto antes, tratamiento con
antipalúdicos (hidroxicloquina).
El diagnóstico más probable es artritis reumatoide. Habría que administrar cuanto antes
3. 3. tratamiento con metotrexato, ya que en estas fases iniciales es más probable obtener
respuesta.
Se trata muy probablemente de una enfermedad de Still del adulto y sería útil el
4. 4.
tratamiento con corticoides a dosis altas.
Gráfico de respuestas
Comentario

En esta pregunta, la imagen era absolutamente prescindible. Las características clínicas (poliartritis
simétrica, con rigidez y FR +) y radiológicas (erosiones) eran más que suficientes para diagnosticar
una artritis reumatoide. Lo que nos muestran en la fotografía es un nódulo, hallazgo típico de esta
entidad, que suele asociarse a títulos altos de factor reumatoide y que típicamente aparece en
regiones de apoyo, como los codos, occipital, etc.

La respuesta correcta, por tanto, será la 4. La artritis reumatoide muy pocas veces será
autolimitada, al contrario de lo que dice la opción 1. De hecho, durante los últimos años se ha
implantado la tendencia a añadir metotrexato de forma precoz, porque mejora la respuesta al
tratamiento y la evolución de la enfermedad.(R3)

420. A 55-year-old male undergoes a surgical procedure for repairing an abdominal aortic
aneurysm. His postoperative course is complicated by acute renal failure and pulmonary
edema. The patient needs to stay intubated and on the sixth postoperative day, he
develops a fever to 38,9 ºC. His respirations are 36/min, blood pressure is 105/60 mm Hg.
Chest X-ray reveals right middle and lower lobe infiltrates. WBC count is elevated. Gram
stain of his bronchial aspirates shows gram-negative rods. He is given intravenous
ceftriaxone. His clinical status worsens over the next day. Which of the following is the
most appropriate next step in management?

1. 1. Continue ceftriaxone and add vancomycin.


2. 2. Stop ceftriaxone and start cefepime.
3. 3. Stop ceftriaxone and start linezolid.
4. 4. Continue ceftriaxone and add azithromycin.
Gráfico de respuestas
Comentario
Stop ceftriaxone and start cefepime. Intubated patients in intensive care units are prone to develop
pneumonia. Very typical causative agents of pneumonia in these patients are: St. Aureus (Gram-
positive) and Pseudomonas Aeruginosa (Gram-negative). The Gram-stain of this particular case
should make you suspect infection by P. Aeruginosa. Furthermore, this is a bacteria that can be
particulary resistant to antibiotic therapy. You should rememeber the antibiotics that are especially
effective against Pseudomonas such as: Piperacillin-tazobactam, Cefepime, Ceftazidime, Colistine,
Imipenem-cilastatin, Tobramycin, Amikacin...(R2)

 
 
 
 
421. Señale en cuál de las siguientes enfermedades puede aparecer como complicación
un linfoma intestinal:

1. 1. Enfermedad celíaca.
2. 2. Enfermedad de Crohn.
3. 3. Colitis microscópica.
4. 4. Enteritis eosinofílica.
Gráfico de respuestas
Comentario

En la enfermedad celíaca, de base autoinmune, se pueden asociar otras enfermedades


inmunológicas (como diabetes mellitus, vitíligo). Pero también está aumentada la incidencia de
tumores, como el linfoma intestinal; es muy rentable que recuerdes esto, sin embargo, este
aumento de riesgo de tener una neoplasia desaparece si los pacientes realizan una dieta exenta
de gluten. En la enfermedad de Crohn está aumentada la incidencia de adenocarcinoma de
intestino delgado.(R1)

422. What is the most important prognostic factor in breast cancer?

1. 1. Histological grading.
2. 2. Histological type.
3. 3. Presence of hormone receptors.
4. 4. Nodal infiltration.
Gráfico de respuestas
Comentario

El factor pronóstico más importante en el cáncer de mama es el número de ganglios afectados.(R4)

423. Un amigo de 26 años le comenta que, en su familia, ha habido varios casos de cáncer
de colon. Su abuelo, que actualmente tiene 80 años, fue colectomizado por este motivo a
los 35 años. Su padre fue también diagnosticado de este problema, cuando él era niño,
así como su tío paterno, dos años después que su padre. Usted piensa en una poliposis
colónica familiar, pero su amigo le dice que, en las frecuentes colonoscopias que se
realiza, el endoscopista nunca le ha comentado que haya encontrado pólipos, ni recuerda
haber oído hablar de ella en su casa. Sobre la enfermedad que probablemente padece,
señale la respuesta CORRECTA:

1. 1. Se trata de un síndrome de Turcot y, por ello, tiene elevado riesgo de tumor cerebral.
Es probable que presente dientes supernumerarios, ya que se trata de un síndrome de
2. 2.
Gardner.
Debería realizarse una colectomía total, de forma profiláctica, puesto que la probabilidad
3. 3.
de desarrollar cáncer de colon es prácticamente del 100%.
El problema es muy probablemente genético y probablemente esté en relación con la
4. 4.
reparación del ADN (inestabilidad de microsatélites).
Gráfico de respuestas
Comentario

Nos presentan un caso clínico en el que varios miembros de una familia han tenido cáncer de
colon. Inicialmente, por frecuencia y ya que no nos dan datos de otra clínica acompañante
debemos pensar en PAF o cáncer colorrectal hereditario no asociado a poliposis. No hay pólipos,
por lo que descartamos la PAF. Sobre el cáncer colorrectal hereditario no asociado a poliposis

 
 
 
 
(síndrome de Lynch), recuerda que se hereda de forma AD y característicamente existe una
inestabilidad de microsatélites. No se debe hacer colectomía profiláctica sino sólo cuando se
detecte malignidad durante el seguimiento.(R4)

424. ¿Cuál de los siguientes tipos histológicos en el carcinoma de la mama presenta una
mayor frecuencia de fracasos terapéuticos?

1. 1. Carcinoma ductal infiltrante.


2. 2. Carcinoma medular.
3. 3. Carcinoma mucinoso.
4. 4. Carcinoma lobulillar no infiltrante.
Gráfico de respuestas
Comentario

Pregunta sencilla sobre aspectos muy básicos sobre el cáncer de mama.

El carcinoma ductal infiltrante es el tipo histológico con peor pronóstico, y también el más
frecuente, representando un 80% del total de cánceres de mama.

Recuerde que, en el cáncer de mama, también es muy importante el número de ganglios afectados
como elemento pronóstico.(R1)

425. ¿Cuál de estas características clínicas NO esperaría encontrar en una placenta


previa sintomática?

1. 1. Dolor.
2. 2. Útero relajado en la exploración.
3. 3. Con frecuencia hay rotura prematura de membranas.
4. 4. Feto con presentación alta.
Gráfico de respuestas
Comentario

Tema muy importante para el ENARM, que debe dominar. La placenta previa produce un sangrado
indoloro (respuesta 1 falsa). El resto de las opciones que nos presentan son características de la
placenta previa. La otra hemorragia del tercer trimestre que tampoco presenta dolor abdominal es
la rotura de vasos previos. El diagnóstico diferencial entre ambos cuadros puede basarlo en la
afectación fetal. Mientras en la placenta previa no aparece sufrimiento fetal, en la rotura de vasos
previos se presenta inmediatamente, ya que la sangre procede del feto, y además suele ser
grave.(R1)

 
 
 
 

426. A 3-year-old girl is brought to the ER due to sudden malaise, within the context of
fever with bloody diarrhea. On examination she appears pale and sleepy,
cardiopulmonary auscultation is normal, her abdomen is painful and the rest of the
physical examination only reveals the presence of scattered punctate hemorrhage.
Hypertension and hematuria are observed. The CBC shows HB 7 g/dl, WBC 17,000 with
neutrophilia, 37,000 platelets with normal coagulation tests. What is the most likely
diagnosis?

1. 1. Salmonella sepsis
2. 2. Schönlein-Henoch purpura
3. 3. Postinfectious glomerulonephritis
4. 4. Hemolytic-uremic syndrome
Gráfico de respuestas
Comentario
Hemolytic-uremic syndrome. Hemolytic-uremic syndrome (HUS) is a clinical syndrome defined by
progressive renal failure that is associated with microangiopathic (nonimmune, Coombs-negative)
hemolytic anemia and thrombocytopenia. HUS is the most common cause of acute kidney injury in
children. History findings may include the following: - Prodromal gastroenteritis (83%), fever (56%),
bloody diarrhea (50%) for 2-7 days before the onset of renal failure - Irritability, lethargy - Seizures
(20%) - Acute renal failure (97%) or anuria (55%).(R4)

427. Mujer de 59 años de edad, con antecedentes de tabaquismo de 10 cigarrillos al día,


cólicos biliares de repetición e hipercolesterolemia, que acude por dolor epigástrico
brusco, intenso, de unas tres horas de evolución, que ha comenzado ha irradiarse hacia
la región interescapular. La paciente refiere que es un cuadro clínico muy similar a los
cólicos biliares que ha sufrido. En la exploración física presenta dolor a la palpación en
hipocondrio derecho, con contractura muscular a dicho nivel. No tiene fiebre. En la BH
no hay leucocitosis. Tiene una radiografía simple de abdomen de unos 6 meses de
antigüedad donde ya se apreciaban cálculos biliares. Se le administra metamizol por vía
intravenosa, así como antieméticos, y ante la clínica recurrente, se decide realizar una
colecistectomía programada. La realización de una colecistectomía de forma programada
sería, desde el punto de vista del grado de contaminación, una cirugía:

1. 1. Limpia.
2. 2. Sucia.
3. 3. Limpia-contaminada.

 
 
 
 
4. 4. Contaminada-sucia.
Gráfico de respuestas
Comentario

Una cirugía limpia- contaminada es aquella en la que se abre el tubo digestivo, respiratorio o
genitourinario de forma controlada, sin salida de material. Una colecistectomía programada, por
ejemplo. Si se tratase de una colecistectomía en el contexto de una colecistitis aguda, estaríamos
ante una cirugía contaminada.(R3)

428. Mujer de 45 años, diagnosticada de artritis reumatoide de 15 años de evolución,


acude por presentar hiperpigmentación en áreas expuestas y úlceras en las piernas. En
la exploración se evidencian esplenomegalia y adenopatías axilares e inguinales. En la
analítica encontramos 3.500.000 hematíes con 9,8 g/dl de Hb, 1.200 polimorfonucleares
por microlitro y 90.000 plaquetas/mm3. ¿Cuál de las siguientes afirmaciones NO es cierta
acerca de la entidad que usted sospecha?:

1. 1. Se presenta principalmente en pacientes FR+.


2. 2. Son más frecuentes las infecciones.
3. 3. La neutropenia se produce a expensas tanto de linfocitos como de neutrófilos.
4. 4. Suele haber nódulos subcutáneos.
Gráfico de respuestas
Comentario
La triada de AR, esplenomegalia y neutropenia permiten afirmar que la paciente presenta un
síndrome de Felty. La hiperpigmentación, la trombopenia, la anemia y las úlceras en las piernas
son tambien características de la enfermedad. Esta complicación aparece en las formas
seropositivas y a menudo con nódulos reumatoides. La leucopenia es una neutropenia selectiva y
efectivamente es rara en la raza negra.(R3)

429. Neonato de 8 días de vida que es traído a urgencias por presentar hemorragia
umbilical tras la caída del cordón. No presenta otra sintomatología. En la exploración le
llama la atención la presencia de petequias puntiformes distribuidas por toda la superficie
corporal. Tras interrogar a la madre, nos cuenta que ella también tiene una patología de
la coagulación, aunque no recuerda exactamente el nombre, y que durante el embarazo
ha tenido que seguir tratamiento. En la analítica encontramos un nivel de plaquetas
descendido. Indiquen la opción FALSA acerca de este cuadro:

Sucede aproximadamente en un 30% de los niños cuyas madres tienen la enfermedad


1. 1.
activa durante el embarazo, en cambio es excepcional si la enfermedad está inactiva.
2. 2. Es frecuente la asociación con hepatoesplenomegalia.
La administración de corticoides o gammaglobulina a la madre al final de la gestación
3. 3.
puede disminuir la afectación del recién nacido.
4. 4. No se debe hacer esplenectomía dado que el trastorno suele ser transitorio.
Gráfico de respuestas
Comentario
Entre las causas de trombopenia neonatal, no ha de pasar desapercibida la posibilidad de que
estés ante un hijo de madre con púrpura trombocitopénica de base inmune. En el niño no suele
haber hepatoesplenomegalia. La trombopenia en el niño puede durar bastante, incluso puede que
dure meses. Una manera de minimizar la enfermedad consiste en administrar tratamiento
específico a la madre.(R2)

 
 
 
 
430. A 14-year-old teenage girl accompanied by her parents comes to her pediatrician
reporting a history of three episodes of sudden loss of consciousness that occurred while
she was crying a lot, without specific prodromes. Full recovery occurs spontaneously
with no neurological deficits. Her previous medical history is unremarkable. What is the
best option regarding this case?

1. 1. Automatic defibrillator implantation


2. 2. Beta blockers
3. 3. Schedule a psychiatric consultation
4. 4. To reassure parents about their daughter behavior
(R4)

431. A su consulta acude un niño de 6 años con fiebre y exantema muy pruriginoso, con
elementos vesiculosos y pustulosos de predominio en tronco, que le ha aparecido el día
anterior. Cuando usted aconseja a la madre aislar al niño del resto de los niños y de
embarazadas, ella asegura estar embarazada de 2 meses y no sabe si ha pasado la
varicela. ¿Qué actitud es la más apropiada?

No hacer nada, dado que lo más probable es que la madre haya pasado la varicela en la
1. 1.
infancia.
Ante la duda, aislar al niño de su madre hasta que todas las lesiones estén en fase de
2. 2.
costra.
3. 3. Administrar Ig anti-varicela-zoster al niño y a la madre.
4. 4. Realizar serología inmediata a la madre, y administrar Ig anti-varicela-zoster.
Gráfico de respuestas
Comentario

Pregunta muy importante, que debe dominar y que repasamos previamente. Ante un caso clínico
como éste, debemos hacer las siguientes consideraciones.

- La gestante está en elprimer trimestrey no tenemos evidencia de que haya padecido la


varicela. En caso de que, efectivamente, nunca la haya pasado, podría contagiarse en este
momento.

- El máximo riesgo de afectación fetal es el primer trimestre de la gestación, por ser el


período durante el que se produce la organogénesis fetal. Realmente, no se trata de un riesgo muy
alto, un 1-2%, pero es el período donde es más probable.

Aunque la eficacia de la gammaglobulina para la prevención de la varicela congénita es dudosa


hasta cierto punto, la respuesta más adecuada es la opción 4. El balance riesgo- beneficio
aconseja su uso y, por otra parte, el haber tomado esta medida, también servirá para protegernos
ante posibles reclamaciones legales en caso de que la madre se contagie y el recién nacido
tuviese algún tipo de malformación atribuible a la varicela.(R4)

432. Mujer con historia personal familiar de sangrado por mucosas. El estudio de
coagulación presenta un tiempo de hemorragia y tiempo de tromboplastina parcial
activado alargados. Actividad de protrombina del 100%. ¿Qué patología de la coagulación
le sugieren estos datos?

1. 1. Enfermedad de Von Willebrand.


2. 2. Déficit de factor XI.

 
 
 
 
3. 3. Hemofilia B.
4. 4. Déficit de factor VII.
Gráfico de respuestas
Comentario

En la pregunta se hace referencia a un sangrado de mucosas. Este dato, de entrada, estaría en


contra de las respuestas 2 y 4, ya que en las hemofilias el sangrado suele ser profundo, tardío y
frecuentemente en forma de hemartros o en músculos como hematomas musculares. Ambas
hemofilias tienen herencia recesiva ligada al X.

Los factores VIII, IX y XI hacen parte de la vía intrínseca y el factor VII de la extrínseca de la
cascada de coagulación, siendo parte de la hemostasia secundaria, por lo que el tiempo de
hemorragia en caso de déficit de cualquiera de estos factores sería normal, ya que no interfieren
con la unión de las plaquetas al endotelio lesionado ni en la adhesión interplaquetaria (hemostasia
primaria).

La opción correcta es la respuesta 1, ya que la enfermedad por déficit (cualitativo o cuantitativo) de


factor Von Willebrand afecta principalmente a la hemostasia primaria produciendo alteración en la
adhesión plaquetaria y dificultad para la formación del coagulo inicial, por esta razón es que se
prolonga el tiempo de hemorragia y se producen sangrados superficiales a nivel de piel y mucosas,
sangrado nasal y sangrado de encías.

Hay que tener cuidado porque en la enfermedad de Von Willebrand los niveles de factor VIII de
coagulación pueden alterarse y afectar el tiempo parcial de tromboplastina activada (vía
intrínseca), ya que en condiciones normales el factor VIII se une al FvW, que le protege de la
rápida destrucción dentro de la sangre. La deficiencia de FvW, por tanto, puede dar lugar a una
reducción en los niveles de factor VIII.

Pregunta típica modelo “oveja negra”, ya que hay cuatro opciones que en caso de alterarse
afectarían la hemostasia secundaria, versus una única opción (respuesta 1) en la que la afectación
es típicamente de la hemostasia primaria.(R1)

433. Un hombre de 48 años, con ingesta de alcohol importante desde hace años, acude a
Urgencias por ictericia y malestar general. En los exámenes de laboratorio se observa lo
siguiente: GPT 3,358 mU/ml; GOT 2,855 mU/ml, bilirrubina total 8 mg/dl. ¿Cuál de las
siguientes afirmaciones es CORRECTA ?

1. 1. El pronóstico de la hepatopatía es grave.


2. 2. La realización de un ultrasonido abdominal carece de utilidad.
3. 3. Probablemente se trata de una ictericia obstructiva.
4. 4. Los resultados son sugestivos de necrosis hepática extensa.
Gráfico de respuestas
Comentario

Se trata de una paciente con elevación importante de las transaminasas, lo que traduce una
intensa citolisis hepática. Además, presenta ictericia, que es un hallazgo que puede asociarse en
las hepatitis agudas. El ultrasonido abdominal es necesario para descartar patología biliar, si bien
con una elevación tan intensa de las transaminasas es poco probable que se trate de ictericia
obstructiva. La hepatitis alcohólica suele cursar con elevación moderada de las transaminasas
(menos de 300 mU/ml, generalmente).(R4)

 
 
 
 
434. Un paciente inmunodeprimido por tratamiento corticoideo crónico comienza con
cefalea, náuseas, marcha tambaleante y confusión. Se le realiza una punción lumbar y en
la tinción con tinta china se observan levaduras encapsuladas. ¿Cuál es el
microorganismo más probable?

1. 1. Candida albicans.
2. 2. Cryptococcus neoformans.
3. 3. Candida glabrata.
4. 4. Aspergillus fumigatus.
Gráfico de respuestas
Comentario

La meningitis criptocócica es la más frecuente en pacientes con SIDA, afectando a sujetos con
menos de 100 linfocitos T- CD4/microlitro. También afecta a pacientes sometidos a tratamientos
prolongados con esteroides, ya que éstos justifican una inmunodepresión celular. Produce un
cuadro de meningitis subaguda, siendo las características del LCR las de la infección fúngica. El
diagnóstico de presunción se hace viendo estructuras típicas que se tiñen con tinta china,
confirmándose mediante la detección del antígeno de criptococo en LCR. El tratamiento de
elección es la anfotericina B con 5-fluocitosina, debiendo realizarse profilaxis secundaria con
fluconazol.(R2)

435. Paciente de 78 años, diabético, hospitalizado por una infección del tracto urinario.
Por las noches se agita y dice que otros enfermos ingresados le quieren maltratar y
humillar, y protesta a gritos porque los médicos y las enfermeras no hacen nada para
evitarlo. Durante el día duerme más y está relativamente tranquilo. Ante un cuadro así, lo
primero que habría que explorar es:

1. 1. La memoria a corto plazo.


2. 2. Comprobar si se trata de un delirio y, si es así, valorar su contenido.
3. 3. La orientación y la atención.
4. 4. Si tiene conciencia de enfermedad.
Gráfico de respuestas
Comentario

Lo que se nos plantea en la pregunta es el diagnóstico diferencial entre delirium y demencia. Estos
cuadros tienen en común el deterioro cognitivo del paciente, pero en el caso del delirium o
síndrome confusional agudo, lo característico es que es de forma aguda y tiene una duración corta.
Presenta un ciclo sueño- vigilia alterado, estando agitado y nervioso por las noches, mientras que
por el día duerme. La temática de su lenguaje es incoherente y lo que es característico es que la
atención, la atención y el nivel de consciencia están alterados a diferencia de lo que ocurre en la
demencia en que sólo está alterada la orientación. Es por eso por lo que la opción correcta es la
3.(R3)

436. Las urgencias por enfermedades oculares suelen clasificarse según el grado de
premura con la que deben ser atendidas. ¿Cuál de las siguientes presenta el menor grado
de urgencia oftalmológica?

1. 1. Causticación ocular por sosa cáustica.


2. 2. Fractura orbitaria por estallido.
3. 3. Ataque de glaucoma agudo.
4. 4. Herida ocular penetrante.

 
 
 
 
Gráfico de respuestas
Comentario

Esta pregunta aparenta una gran dificultad, pero en realidad puede ser respondida sin tener
excesivos conocimientos. Sólo hace falta un poco de sentido común. ¿Qué es más importante, de
cara al pronóstico visual, el ojo o la órbita…?

•   La opción 1 es, evidentemente, una urgencia oftalmológica. La primera medida sería el


lavado ocular con agua abundante, tan rápido como sea posible, como en cualquier
traumatismo químico.
•   En un glaucoma agudo (respuesta 3), debemos hacer todo lo posible para disminuir la
presión intraocular. De lo contrario, se producirá isquemia a nivel retiniano y el paciente
tendrá un deterioro visual irreversible.

•   En la arteritis de la temporal (enfermedad de Horton), es imperativo tratar con esteroides


en altas dosis de forma urgente. De lo contrario, la repercusión visual se hará bilateral. No
olvides que es de carácter irreversible, de ahí la premura en el tratamiento.

•   La respuesta 5 es, obviamente, una emergencia oftalmológica.

Observa que, en cambio, la respuesta 2 puede esperar. La palabra “estallido” suena muy
amenazadora, pero ten en cuenta que no es el ojo lo que ha estallado, sino la órbita, que es muy
diferente.

Este tipo de fracturas no pueden ser reducidas de urgencia, porque la órbita se compone de
láminas óseas muy finas.

El tratamiento, por tanto, se realiza de forma diferida; por ejemplo, reconstruyendo la órbita a partir
de implantes de cartílago.(R2)

437. Ante un paciente que ingresa en la unidad de quemados tras haber sufrido una
descarga eléctrica con cables de alta tensión, ¿cuál de las siguientes afirmaciones es
FALSA?:

1. 1. El a
2. 2. Gene
3. 3. Debe
4. 4. Pued
Gráfico de respuestas
Comentario

No es habitual que aparezcan en el ENARM preguntas sobre quemaduras eléctricas; de todo


modos es bueno que conozcas algunas generalidades sobre este tipo de quemadura. Se
caracterizan por una frecuente afectación muscular y ósea con piel apenas lesionada. La
afectación muscular puede generar rabdomiolisis que puede dar lugar a su vez a un necrosis
tubular aguda. Por esta razón es importante realizar escarotomía y fasciotomía si hay tejido
muscular en riesgo.(R2)

438. Masculino de 72 años, con historia de cardiopatía isquémica, insuficiencia cardiaca


congestiva y fibrilación auricular, está en tratamiento con procainamida desde hace

 
 
 
 
varios meses. Comienza con artromialgias generalizadas, dolor torácico de perfil
pleurítico, fiebre y presenta una imagen de derrame pleural derecho en la radiografía de
tórax. Ha seguido tratamiento con cefuroxima durante 48 horas sin respuesta. La
gammagrafía pulmonar de ventilación-perfusión es normal. Se realiza una determinación
de laboratorio que permite el diagnóstico. Por favor, señálela:

1. 1. Anticuerpos anticitoplasma de neutrófilo.


2. 2. Anticuerpos anticentrómero.
3. 3. Anticuerpos antifosfolípido.
4. 4. Anticuerpos anti-histonas.
Gráfico de respuestas
Comentario

En este caso clínico debemos centrarnos para su resolución en los antecedentes del paciente, en
particular en la toma de medicación y en los síntomas que se presentan en la actualidad. El
fármaco que recibe es la procainamida, un antiarrítmico que se ha relacionado con frecuencia con
la aparición del cuadro de lupus inducido por fármacos. La procainamida induce la aparición de
ANA en el 50-75% de los pacientes y el 10-20% de los sujetos ANA-positivos padecen síntomas
lúpicos. Los más frecuentes son los síntomas generales y las artralgias; la poliartritis y la
pleuropericarditis aparecen en el 25-50%. Los anticuerpos anti-histona están íntimamente
relacionados con el lupus inducido por fármacos (respuesta correcta 4).(R4)

439. En relación con la fístula traqueoesofágica congénita, refiera cuál de las


complicaciones siguientes al tratamiento quirúrgico es la más frecuente:

1. 1. Estenosis esofágica.
2. 2. Fístula traqueoesofágica recidivante.
3. 3. Reflujo gastroesofágico.
4. 4. Traqueomalacia.
Gráfico de respuestas
Comentario

Se trata de una pregunta de elevada dificultad sobre un tema poco preguntado en el ENARM.

La atresia y fístula traqueoesofágica ocurren con una incidencia 1/3.000-4.500 RN vivos. Existen
diversas clasificaciones, aunque la más aceptada es la clasificación de Ladd, que distingue cinco
tipos:

•   Tipo 1: atresia sin fístula.


•   Tipo 2: fístula proximal y atresia distal.
•   Tipo 3: atresia proximal y fístula distal.
•   Tipo 4: doble fístula.
•   Tipo 5: fístula sin atresia.

La forma más frecuente es la III, con casi el 85% de los casos.

•   Debemos sospechar esta patología ante:


•   Existencia de polihidramnios.
•   Imposibilidad para pasar sonda nasogástrica en el paritorio.
•   Salivación excesiva.
•   Cianosis y atragantamiento con las tomas.

 
 
 
 
Si existe una fístula traqueoesofágica distal, aparecerá una distensión abdominal importante,
mientras que si no existe fístula distal, el abdomen estará excavado. Las formas con fístula
proximal cursan con aspiraciones masivas con la alimentación. La fístula sin atresia (en H) puede
cursar de forma más larvada y manifestarse como neumonías recurrentes.

El tratamiento ha de ser quirúrgico. Durante el preoperatorio: posición en decúbito prono,


aspiración continua del bolsón esofágico y medidas generales. La corrección quirúrgica suele
realizarse en dos tiempos.

Tras la cirugía la complicación más frecuente será los trastornos de la motilidad esofágica y, en
particular, el reflujo gastroesofágico. Con menos frecuencia aparecen traqueomalacia, estenosis,
refistulización.(R3)

Atresia esofágica

440. Un paciente previamente sano es llevado a Urgencias con cefalea, fiebre y signos
meníngeos. El estudio citobioquímico del LCR muestra pleocitosis monocítica, discreta
elevación de proteínas, con glucosa normal. El examen del líquido con tinción de Gram y
Ziehl, así como los cultivos bacterianos y para hongos, resultaron negativos. ¿Qué
pruebas microbiológicas adicionales le practicaría?

1. 1. Detección de antígeno neumocócico en LCR.


2. 2. Toma de muestras de faringe, heces y LCR para cultivo viral.
3. 3. Cultivo en medios especiales para Haemophilus influenzae.
4. 4. Detección de antígeno de Haemophilus influenzae.
Gráfico de respuestas

 
 
 
 
Comentario

Una pregunta muy sencilla sobre un tema, por otra parte, fundamental para el Examen MIR. Si la
has fallado, revisa urgentemente el capítulo de las meningitis.

El LCR que nos presentan reúne las características típicas de una meningitis vírica:

- Predominio linfocítico.

- Glucosa normal.

- Elevación moderada de las proteínas.

La respuesta correcta es, por tanto, la 2.(R2)

441. En la enfermedad de membrana hialina o síndrome de distrés respiratorio neonatal


existe un déficit de surfactante que se ha relacionado con todas, EXCEPTO una de las
siguientes situaciones:

1. 1. Prematuridad.
2. 2. Hijo de madre diabética.
3. 3. Trastorno metabólico de la síntesis de lecitina.
4. 4. Hijos de madres adictas a heroína.
Gráfico de respuestas
Comentario

Es muy importante que domine el diagnóstico diferencial entre las distintas entidades responsables
de distress respiratorio en el recién nacido.

La enfermedad de membrana hialina se debe a un déficit de surfactante, que no alcanza


plenamente la superficie pulmonar hasta la semana 34-35. Este déficit produce un aumento de la
tensión superficial y una tendencia de los pulmones al colapso.

Hay distintas situaciones que pueden variar la síntesis de surfactante:

- Aumenta con las situaciones de estrés: desprendimiento placentario, la rotura precoz de


membrana, consumo de opiáceos (resp. falsa-4), HTA y vasculopatía renal materna.

- Disminuye en el hydrops fetal, trastorno metabólico de la síntesis de lecitina y diabetes materna.

Afecta sobre todo a prematuros. Su frecuencia también es mayor en los hijos de madre diabética y
en los embarazos múltiples.(R4)

442. ¿Cuál de los siguientes signos es indicativo de mayor gravedad en un recién nacido
a término con encefalopatía hipóxico-isquémica?:

1. 1. Mioclonos.
2. 2. Pupilas desiguales, con mala reacción a la luz.
3. 3. Reflejos tendinosos hiperactivos.

 
 
 
 
4. 4. Postura en flexión.
Gráfico de respuestas
Comentario
La presencia de anisocoria en un niño con encefalopatía hipóxico- isquémica muestra que hay
edema intracraneal importante o han aparecido quistes, y secundariamente hay herniación
transtentorial, con compromiso del tercer par craneal.(R2)

443. Señale el enunciado FALSO:

1. 1. Las crisis parciales simples no cursan con deterioro del nivel de conciencia.
2. 2. Las ausencias típicas se siguen de confusión postcrítica.
3. 3. La crisis jacksoniana es una forma de crisis parcial simple.
Las crisis parciales complejas cursan con una pérdida del contacto consciente con el
4. 4.
entorno.
Gráfico de respuestas
Comentario

Pregunta asequible acerca de la epilepsia. Dentro de la clasificación de las crisis epilépticas hay
que tener claras dos divisiones: Crisis parciales - generalizadas (según afecten a una parte o a
toda la corteza) y crisis simples - complejas (según se afecte o no el nivel de conciencia).

Las crisis simples mantienen el nivel de conciencia y las complejas no. La crisis Jacksoniana es
una crisis parcial simple motora. El enunciado falso es la respuesta 2 ya que las ausencias típicas
no se siguen de confusión posterior y esto precisamente es una de las características que las
diferencia de las crisis parciales complejas.(R2)

444. Paciente con antecedentes de artritis reumatoide, en tratamiento con metotrexato.


En la última revisión estaba bien controlada, con hemograma y bioquímica normales. Sin
embargo, hoy acude al Servicio de Urgencias por fiebre de 38,7º C, dolor y tumefacción
de 12 horas de evolución en la rodilla izquierda. En la exploración, usted objetiva dolor a
la palpación, eritema y aumento de calor en dicha zona. ¿Cuál sería la actitud más
adecuada?

Artrocentesis, examen del líquido sinovial con microscopio de luz polarizada y test de
1. 1.
Gram urgente.
2. 2. Instaurar tratamiento esteroideo en altas dosis.
3. 3. Infiltración intraarticular de esteroides.
4. 4. Antiinflamatorios no esteroideos, protector gástrico y derivar a Reumatología.
Gráfico de respuestas
Comentario

La clínica que nos presentan, monoartritis aguda con dolor local y fiebre alta, debe hacernos
pensar en una causa infecciosa, por lo que lo indicado es puncionar la articulación, drenar su
contenido, y estudiarlo al microscopio con una tinción para bacterias (Gram).(R1)

445. El test de screening de O´Sullivan para diagnóstico de diabetes gestacional se realiza


entre las semanas 24-28 de gestación. Señale cuál de las siguientes afirmaciones es
INCORRECTA:

En mujeres con antecedentes de diabetes gestacional en otros embarazos, sería


1. 1.
conveniente la realización del screening en el primer trimestre de gestación.

 
 
 
 
La detección de una glucemia basal al azar > 126 mg/dL es diagnóstico de diabetes
2. 2.
gestacional.
Dos valores al azar de glucemia > 200 mg/dl también son diagnósticos de diabetes
3. 3.
gestacional directamente.
Sería conveniente también realizar el test de O´Sullivan en el primer trimestre a mujeres
4. 4.
con intolerancia glucídica o con IMC> 30.
Gráfico de respuestas
Comentario

Tema importante en el bloqu de ginecología, lo debes dominar. La respuesta correcta es la 2, ya


que es posible diagnosticar una diabetes gestacional de este modo, pero son precisas dos
determinaciones de glucemia basal en dos días diferentes >126 mg/dl y no sólo una. El resto de
respuestas son ciertas: en caso de factores de riesgo de diabetes gestacional se realiza screening
en primer trimestre de gestación. Con valores de O´Sullivan superiores o iguales a 140 mg/dl es
preciso realizar SOG 100 g. Es diagnóstico de diabetes gestacional dos valores al azar de
glucemia superiores a 200 mg/dl.(R2)

446. Paciente de 70 años sin antecedentes de interés y asintomática con calcemias entre
10,6 y 11 mg/dL (seguimiento de 18 meses). Diagnosticada de hiperparatiroidismo
primario por adenoma paratiroideo superior. ¿Cuál es el tratamiento más recomendable?:

1. 1. Tratamiento con tiacidas.


2. 2. Tratamiento con glucocorticoides.
3. 3. Paratiroidectomía subtotal.
4. 4. Seguimiento.
Gráfico de respuestas
Comentario
Los trastornos del metabolismo del calcio constituyen un tema bastante preguntado en el MIR. El
hiperparatirodismo primario es la causa más frecuente de hipercalcemia. Normalmente es
asintomática y se debe a un adenoma de paratiroides. Dado su carácter benigno y que el tto
quirúrgico no siempre tiene éxito, se han establecido unas indicaciones de cirugía. Se extirpará el
adenoma en todos los menores de 50 años y en los mayores de 50 que tengan además: calcemia
> 11,5 mg/dl, algún episodio de hipercalcemia grave, osteoporosis, fracturas, calciuria > 400
mg/día, litiasis renal o disminución del aclaramto de Creat >30%. Si el pacte no cumple estas
indicaciones, se le recomienda hidratación abundante y seguimiento periódico de la calcemia, la
función renal y la masa ósea.(R4)

447. Uno de los siguientes datos clínicos NO corresponde al síndrome de ovario


poliquístico:

1. 1. Obesidad.
2. 2. Hirsutismo.
3. 3. LH > FSH.
4. 4. Cifras muy elevadas de 17-hidroxiprogesterona en el día 23 del ciclo.
Gráfico de respuestas
Comentario

Esta pregunta es muy importante porque debe conocer perfectamente el síndrome de ovario
poliquístico y sus alteraciones hormonales. Es una afección muy frecuente en que está aumentada
la LH con niveles de FSH bajos o inferiores a los normal, por lo que aumenta la relación LH/FSH.
Hay un aumento leve de andrógenos, aumento de la estrona (los andrógenos circulantes son
convertidos a estrona en la grasa periférica) y descenso del estradiol. La LH aumentada estimula la

 
 
 
 
teca, produciendo más andrógenos (también hay una sobreproducción suprarrenal) provoca
obesidad, hirsutismo y anovulación. Existe insulinresistencia.(R4)

448. El Cushing por ACTH ectópica de tumores agresivos se diferencia de los otros tipos
de síndrome de Cushing en todas las características siguientes, EXCEPTO una:

1. 1. Alcalosis hipopotasémica.
2. 2. Fenotipo cushingoide más intenso.
3. 3. Hiperpigmentación cutánea.
4. 4. Inicio rápido.
Gráfico de respuestas
Comentario

Dentro de las distintas etiologías del síndrome de Cushing, está el producido por secreción
ectópica de ACTH. A su vez, éste se divide en los originados a partir de tumores agresivos y no
agresivos (tumores carcinoides). El producido por tumores agresivos se asemeja al síndrome de
Cushing hipofisario por la existencia de alteraciones iónicas (alcalosis hipopotasémica,
hiperglucemia) y por la existencia de hiperpigmentación mucocutánea. Es más frecuente en
varones y su evolución es muy rápida, lo que impide que desarrollen el fenotipo cushingoide
característico.

Por el contrario, en el Cushing ectópico secundario a tumores no agresivos, la distinción con el de


origen central es compleja, ya que en estos pacientes coexisten las alteraciones metabólicas y el
fenotipo característico.(R2)

449. Un hombre de 65 años, oficinista jubilado y fumador de 1 paquete diario de


cigarrillos, acude a consulta por un cuadro de tos persistente, generalmente seca y
disnea progresiva de 2 años de evolución que en la actualidad es de grado 2. El paciente
niega otros síntomas. La exploración física no muestra datos relevantes. La actitud a
seguir sería:

1. 1. Realizar radiografía de tórax y espirometría con prueba broncodilatadora.


2. 2. Pautar corticoides orales.
3. 3. Realizar TC torácica.
4. 4. Realizar gasometría arterial basal.
Gráfico de respuestas
Comentario

Esta es una pregunta de dificultad media/alta, pues se refiere a la actitud diagnóstica de un


paciente con síntomas respiratorios frecuentes y poco específicos (tos y disnea) con una
exploración física normal. Las opciones de diagnóstico diferencial son amplias, y van desde la
patología intersticial, transtornos obstructivos e incluso patología tumoral, por mencionar sólo las
más frecuentes. Dado que estamos en la etapa de diagnóstico, no es lógico iniciar un tratamiento
(respuestas 1 y 3 incorrectas). De las opciones restantes, se debe escoger las más simples y que
abarquen el mayor espectro diagnóstico inicial posible. Las enfermedades anteriormente
mencionadas en el diagnóstico diferencial se sospechan con una radiografía simple de tórax y una
espirometría (con broncodilatadores si la exploración basal fuera obstructiva) y, por tanto, son la
opción más lógica (respuesta correcta la 2). No es lógico hacer una TAC de tórax antes de la
radiografía simple (respuesta 4 incorrecta), ni empezar por una gasometría (respuesta 5
incorrecta), aunque es posible que en el proceso diagnóstico posterior estén indicadas.(R1)

 
 
 
 
450. Un paciente de 85 años acude a su consulta porque presenta una tumoración
inguinal de dos años de evolución. A la inspección, el paciente de pie y usted delante, se
observa un abultamiento en la región inguinal derecha, que llega hasta el escroto. ¿Cuál
es el diagnóstico?

1. 1. Hernia femoral.
2. 2. Hernia inguinal directa.
3. 3. Hernia inguinal indirecta.
4. 4. Hernia de Spiegel.
Gráfico de respuestas
Comentario

Simplemente por frecuencia debemos saber que la hernia inguinal más frecuente tanto en hombres
como en mujeres es la hernia inguinal indirecta. Pero además hay un dato clave en la historia. Se
trata del hecho de que la llega hasta el escroto; ésta es una característica típica de este tipo de
hernias. Si aparece en el ENARM debemos pensar directamente en una hernia indirecta o
externa.(R3)

451. A 35-year-old man comes to the ER after a horse-riding fall. He only refers pain on
his left arm. Based on these findings, Which of the following statements is true?

1. 1. The most common complication is traumatic brachial artery injury


2. 2. Surgical treatment is always required
3. 3. The most common complication is radial nerve injury
The management of this kind of fracture is the same as that used for proximal humerus
4. 4.
fractures
Gráfico de respuestas
Comentario

El dolor en el brazo derecho nos debe hacer sospechar una fractura de húmero, cuya complicación
más frecuente es la lesión del nervio radial.(R3)

 
 
 
 

 
 
 
 

 
 
 
 

 
 
 
 

 
 
 
 

 
 
 
 

 
 
 
 

 
 
 
 

 
 
 
 

 
 
 
 

 
 
 
 

 
 
 
 

 
 
 
 

 
 
 
 

 
 

También podría gustarte